Sei sulla pagina 1di 208

Essential Questions in Paediatrics

for MRCPCI-J
~ Volume 2,
.
'"

~.

z_ ...

j'

Edited by

'

Dr R M Beattie ,BSc MBBS MRCP FRCPCH


. Consultant Paediatric Gastro~nterologist
Paediatric Medical Unit'
. SOuthampton General Hospit~l
Southampton .
and

Dr M P Champion BSc MBBS MRCP MRCPCH


Consultant in Paediatric Metabolic Medicine
Evelina Children's Hospital.
Guy's andSt: Thomas' Hospital NHS Trust

r . .1 , . .London

..

.ttl
PASTEST
Dedicated to your success

..

2004 PasTest Ltd


Egerton court
Parkgate Estate
Kimtsford
Cheshire, WA 16 BOX

I
I
I

TelepMn~: 01565 7520_00

All rights reserved. No part of this publication may be reproduced, stored in?
retrieval system. or transmitted. in any form or by any means, electronic.
mechanical, photocopying, recording or otherwise without the prior permission
of the copyright owner.
First edition 2004
ISBN: I 904627 33 I
A catalogue record for this boOk is available from the British Library.
.,

. .

The mformation contained \\ithin this book was obtained by the authors from
reliable source~ However, while every effort has been made to ensure its
accuracy no resp<-nsibility for loss. damage dr injury occasioned to any person
acting or reframing from action as a result of information contained herein can
be accepted by the publisher or the authors.

PasTest Revision Books and Intensive Courses


..
.
)
. P.J;,Test has been establfst.ed in thefield ofpostgraduate medical educatiqn
since 1972, providing re\tision books and intensive study courses for doctors
preparingfor the IT professional examinations. Books'and courses are available
for thefollo\\mg specialties:
,, - t
.1.
MRCPPartland Part 2, MRC:PCH Part rand Part2. MRCOG, DRCOG; MRCGP..
MRCPsych, DCH, FRCA, t,:RCS and PI.AB.
For further details contact:

Pas'R!st Ltd, Freepost, Knutsford, Cheshire, WAJ6 7BR


Tel: Oi565 752000

Email: enquiries@pastest.co.uk

f'ax: tH565 650264

o/eb site: W'WW.pastest.co.uk

Contents

iii

Contributors List

Dr R M Beattie BSc MBBS MRCP FRCPCH


Consultant Paediatric Gastroenterologist
Paediatric Medical Unit
Southampton General Hospital
Southampton

"

Mike Champion BSc MBBS MRCP MRCPCH


Consultant in Paediatric Metabolic Medicine
Evelina Children's Hospital
Guy's and St. Thomas' Hospital NHS nust
London
Andrew Clark MBBS MRCP MD
Consultant in Paediatric Allergy
Department of Allergy
Addenbrookes NHS lhlst
; '.Cambridge

~-

\o

.'

Katy Fidler BSc MBBS ~RCPCH


Clinical Research Fellow
Department of Infectious Diseases and Microbiology
Institute of Child Health
London

Grenville F Fox MBChB MRCP FRCPCH


Consultant Neonatologist
Neonatal i.Jhit
Guy's and StThomas' Hospital NHS nust
Lon9on

.''

.
'

...

]ames W Hart MBBS BSc (Hons) MRCP (UK) MRCPCH


Specialist Registrar
Paediatric Medical Unit
Southampton General Hospital
Southampton

iv

<.

CONTRIBUTORS LIST

N Hasson MBChB FRCPCH'.


Consultant in Paediatric Rheumatology
Great Ormond Street Hospital For Children

London
Tammy Hedderly MRCPCH, MBBS, BSc (Hons)
SpR Paediatric Neurology
-'
Department of Paediatric Neurology
Guy's and StThomas' Hospital NHS irust
London

...

~.

EmmaJ Lim BMBS MSc MRCP


Specialist Registrar in Paediatric Infectious Diseases
Department of Paediatric Infectious Disease

StMary's Hospital
London
Chris Reid MRCP (UK) FRCPCH
Consultant Paediatric Nephrologist
Department of Paediatric Nephrology and Urology
Guy's and St. Thomas' Hospital NHS lhlst
London
. Angie M Wade MSr PhD CStat ILTM
Senior Lecturer in Medical Statistics
Centre for PGtedjatric Epidemiology and Biostatistics

Institute of Child 'Health


London

"'

Introduction

These revision texts have been written to accompany Essential Revision Notes
in Paediatrics for the MRCPCH but are also relevant for part one of the DCH
examination. The questions, in the new format are designed to help facilitate
revision for the MRCPCH part one examination. The books are split by subject
to aid revision planning. Each question has.a detailed explamition and so the

text can be used as a stand alone revision aid.


The candidate is advised to consult the RCPCH website for up to' date
information regarding the exam. The information below is taken from the
website and is correct at the time of going to press.
The MRCPCH Part One examination consists of two papers.
Paper One A (I}asic Child Health) will focus on the areas of child health that
are relevant to those who will be working with children in their mediCal
careers. not just those entering mainstream hospital-based paediatrics. The
areas to be tested will be those conditions likely tope seen in 6 to 12 months
of hospital, community or primary care practice.
Paper One B (Extended Paediatrics) will focus on the more complex paediatric
problem-solving skills not tested in Papef One A. and on the scientific
knowledge underpinning paediatrics. This is equivalent to the current
MRCPCH Part One paper.
candidates for MRCPCH must successfully complete both Paper One A and
One B before being allowed to enter MRCPCH Part 1\vo..
Paper One A (Basic Child Health) \viii replace the current Diploma in Child
Health (DCH) written papers.
The papers will consist of

Multiple true-false questions used to test knowledge when there is an


absolute Yes/No answer.
Best of Five questions used to test judgement and experience. A simple
statement' or short clinical scenario leads into ti\e options. All could be
possible but only one is completely or the most correct. The candidate has to
choose the best option.
Extended matching questions (EMQ~) are used in the same way as Best of
Five questions. In this case a Jist of I 0 possible answers is offered with three
statements or clinical scenarios. The candid.3te choosesthe best option from

vi

- - - - - - - _r ,; ___;,.

the introductory list. Agairi: e~il could be.possiblebut only one is completely or
the most correct.
.. fJ.J,1rther~9et~ls<;>n t!le mal<~I!IP ofthethree.types ofqtae~ions a,re oznhe
cOllege Website. There ateaiso:doWnloadable sample:papers;whi~li strc>UlO:be
reviewed.

We are indebted in the production of this book: to the many authors who have

enthusia?tically contributed chapters and to Kirsten Baxter at Pastest for her


enthusiasm and eXpertise in helping pull the book: together.
Mark .Beattie
Mike Champion
]une2004

vii

l't

Katy Fidler and Emrl)a Lim


~~!"'~~!l.t~!l:Tl!!1@lll!O!~;r;<qq111Slllll<lil:::lt<B~re>il~~~,a;~~tlli-II!E~Vil!l"'ac~~~7,;V.!':<:"'~

t..._.~,

"'1

i~~ ..

''

Multiple Choice Question_~. ; ;,~ ,: ::~ .


-

~!!""Y.:O;

.'~~,.-,_

,.

~-

I. The following infections can cause

0: A: ad~no:0rus, ._. ;, ,(~ . ,., .,.


0 B Epstein-Barr virus (EBVJ . .. ' ~
0 C respiratory syncytial virus (RSV) "
0 D rhinovirus
-' 11 .
0 E myxovirus
'

:~

(.;

\~

d'

'

l\

h.t;~~~hi~tius i~.!;.f-0t~'
'

,.,

"!
l

'
,_

...

<;

i ::)~

'

l. .. ' I

'r

;_.~;t

2. lYPical features of typhoid fever include

0
0
0
0
0

A persistent pyrexia

B sp!en~m~galy,; 1

C bradycardia
D conjunctivitis

.; .

''

E cough

..... ''. ~- f'.

't

3. The foUowing are characteristic features of ~easl~'s' : '

0
0
0
0
0

A diarrhoea

B fine maculopapul~rxash starling,on the face.


C purulent conjunctivitis

D convulsions
E coryza

... ~ :
,,
,,

'

4. The following organisms are recognised causes of diarrhoea

0 A
0 B

-o
0
0

nterobacter
rotavirus
c Salmonella . ' ..
D enterovirus
E Campy/obacter

'-

~.,

.r... ;

'

lj I

''

,.

'

ESSENTIAL QUESTIONS FOR MRCPCH

5. The following skin lesions are caused by viral infections


0 A whitlow

0
0

B verruca
C impetigo
D tinea capitis
E molluscum contagiosum

6. The characteristic rash of chickenpox is described as having


0 A spots mainly on the hands and feet
0 B spots that coalesce
0 C spots that are itchy
0 D spo,ts appearing in crops
0 E stiots starting as vesicles
'

7. Children in the same classroom are at a high risk of catching the


following infections from eachot~er

.0

A. human immunodeficiency virus

. 0 B hepatitis C
0 c. atypical Mycobacterium
0 D Lyme disease

E erythema infectiosum

8. The following infections are transmitted by the faeco-oral route

0 A
0 B
0 . C
0 D

rubella
polio
Pneumococcus
Salmonella
E Norwalk v1rus

9. First-line management of infectious diarrhoea in a toddler


includes

0
0

0
0

A oral rehydration salts


B merronidazole
C loperamide
D kaolin syrup
E hyoscine butylbromide

~-

..,

''v)

10. A 9-month-old child with croup, who has marked stridor at rest
and a barking cough, should be treated Y.ith
,.

0
0

0
O
0
2

A nebulised adrenaline (epinephrine)


B nebtilised salbutamol
c intravenous antibiotics
D oral dexamethasone or nebulised budesonide
E intramuscular adrenaline

-'

.1~

--

INFECTIOUS DISEASES

11: Which ortpe'follo~g ~mplications is a 12_yearold boy with


orbital cellulitis at risk of developing?
.

0
0

0
0
0

'

A meningitis
B cavernous sinus thrombosis
C cerebral abscess
P cerebral artery embolus
E epistaxis

'

'.

:~'tO

12. A 1-year-old child presenting with the following condition~


would _need further investigation of their immune function .

A herpes stomatitis

:S'

0 B cytomegalovirus (Ct.fV) pneumonitis .


. ,
~
.
0 c. two previous admissions- one with meningitis, one with pneumonia
0 o' six viral upper respiratory infection-s' in I year
.,
.
O E an atypical mycobacterial skin lesion

(/)

13. The following are n.otifiable disea;oes

0
0
0

pertussis
malaria
measles
viral meningitis
E menil')gococcal meningitis

A
B
C
D

r, .

14. The following statements about human immunodeficiency virus


(HIV) infection in children are correct

0
0

0
0

A most babies are infected by horizontal transmission from their


mothers
B a baby who is HIV-antibody::positive at 3 months of age' is probably
infected withHIV

C a baby who is HIV-PCR-positive at 3 months of age is probably infected


. With HIV
. " ~
:
'
D most babies who are infected acquire their infection prior to delivery
E anti-retroviraltherapyshould be given to babies of HIV-positive
mothers within 12 hours of birth

r : "
:-.

. "

:>'

(/)

s:::
0

..

ESSENTIAL QUESTIONS FOR MRCPCH

15. A 2-year-old presents with a mass itt the right side of the neck,
thought to be an enlarged cervical lymph node. The following
would be included in the differential diagnosis

0
0
0
0
0

A atypical tuberculosis
B cat scratch fever
c lymphoma
D Rubella infection
E Toxopla~mosis infection

16. The following is true regarding meningitis

0
0
0
0
0

A hearing impairment is more.common after meningococcal than


pneumococcal meningitis
. B neck stiffness is a common presenting sign iri neonates
c steroids may reduce the morbidity of Haemophilus injluenzae type B .
(Hib) meningitis

D glucose level in cerebrospinal fluid (CSF) is not usually lowered in


tuberculous meningitis
E Listeria monocytogenes is the most common cause of neonatal
meningitis

I 7. Children with defects in their cellular immune system are


particularly at risk of infections with

0
0
0
0
0

A mycobacteria
B adenovirus
C Staphylococcus au reus
0 PneumocysUs c:arinii
E Neisseria meningitidis

18. The following is Jrue

0
0
0
0

ot childhood exanthema ,

19. The following are proinflammatory cytokines

0 .
0
0
0
0
4

A the rash of chickenpox characteristically starts behind the ears and


. ..
along the hairline
.
. .
B the rash of measles characteristically starts behind the ears and along
. the hairline
~
~
,.
c the rash of roseola infantum characteristically gives very
erythematous cheeks
, ,, .
, . ,.
, . _. ..
0 the rash of roseola infantum characteristically appears after the fever
E the rash of coxsackie virus characteristically gives vesicular lesions on
the hands and feet

A
B
C
0
E

interleukin-1
interleukin-6
interleukin-8
interleukin- 10
tumour necrosis factor (TN F)-a

.I

INFECTIOUS DISEASES

2.0. The foll~~ng organisms are Gram stain negative

0
0
0

0-

A Bordete/Ja pertussis
B Bacillus anthracis

c. Pseudony,qnas aemgino_sa
D Neisseria meningitidis 1
E Haemophilus influenzae

,,

21. The following drugs used in the treatment oi tuberculosis may


cause the side effects described

0 A rifampicin and hepatitis


0 B ethambutol and deafness
.0 C )~9niazid and peripheral n~u_rop~thy _
0 D streptomyci!' an~ deaf!1ess , . .
0 E pyrazinamide and loss of colour vision
o..)

..'.
0

.--.'

:l

'U):

ESSENTIAL QUESTIONSfO_R MRCPCH

: Best Of Five Questions


22. A 3-year-old girl with vertically acquired HIVhas,rece,ntly started anti-retroviral therapy. Which of the following
investigations would be BEST to monitor efficacy of treatment?

0
0
0
0
0

A
B
C
D
E

CDS count
Full,blood cpunt , _1
HIV antibody
HIV viral load
Immunoglobulins

. ' '
1

'

... ;t"'

.,.

23. A 6-month-old girl has a proven Escherichia coli urine Infection ..


The MOST useful investigation to identify any'renal scarring
would be?
' ':t, ., "
. ._,_, "
1

A
B
C
D
E

0
0
0
0

DMSA (dimercaptosuccinic acid) scan


Intravenous pyelogram (IVP)
MAG 3 renogram
Micturating cystourethrogram (MCUG)
Renal ultrasound

24. A 2-year-old girl presents to casualty with a 24-hour history of


being febrile and miserable and has developed a rapidly
spreading non-blanching rash. She has cool peripheries and is
drowsy. The LEAST useful investigation to do immediately is
which of the following?

0
0

0
-0

A
B
C
D
-E

Blood culture
Full blood count
Lumbar puncture
Meningococcal PCR (polymerase chain reaction)
Rapid antigenscreen

25. Which one of the following children does NOT require specific
varicella zoster immunoglobulin (VZIG) after .being exposed to a
case of chickenpox?

0
0

A A child currently on chemotherapy


B A child who had a bone marrow transplant 1 month ago
C A child with a kidney transplant who is on immunosuppressive
treatment
D A child who has had prednisolone I mg/kg for the last 6 weeks
E A child who had prednisolone 2 mg/kg for 3 days 1 week ago

0
0

-.

.6

I
I

INFECTIOUS DISEASES

26. An 11-year-old boy presents Wtth a 1o-d~y history of headache,


sore throat, chest pain and dry cough. His GP gave him a course
of Amoxil 4,$iays,ago but he ~~ no better. and ts.now 'coughing up
yellow sputum and is wheezy. He also complains of abdominal
pain and painful swollen knees. Which of the following
organisms is MOST likely to tie the cause of his illness?

0
0

0
0

A Streptococcus pneumoniae

B Mycoplasma pneumoniae
C Mycobacterium tuberculosis
D Chlamydia trachoma lis
E Haemophilus-injluenzae

27. Which of the following is the MOST likely congenital infection


in a baby born With a 'blueberry muffin r~sh', cataracts, and a
heart murmur?

0
0

0 .

0
0

A
B
C
D
E

Cytomegalovirus (CMV) infection


Toxoplasmosis infection
Parvovinis infection
HJV infection
Rubella infection

'-'

i
'!I

II
!

ESSENTIAL QUESTIONS FO"\ t1RCPCH

. Extended Matching questions


28. Theme: Differentia) diagnosis of neo~atal infecti~n

-v/
.

""-';:

Clf

:s

tr

A
B
C
D
E
F
G
H

I,isteria monocytogenes infection


Group A streptococcal infection
Group B streptococcal CGBS) infection
Disseminated herpes simplex virus infection
Escherichia coli infection
Congenital heart disease
Methylmalonic aciduria
. Congenital infection
HIV infection

For each of the following case scenarios select the most likely diagnosis from
the list above. Each option may be used once, more than once, or not at all.

I. A baby boy presents on day 1 of life with tachypnoea, poor feeding and
delayed capillary refill time of 3 seconds: He had been born by normal
vaginal delivery (NVD) at 37 weeks, after prolonged rupture _of membranes
(PROM) for 4 days. Further examination reveals a bulging fontanelle. Initial
investigations are as foliows (normal ranges in brackets): haemoglobin
14 gtdL (1 0.5-14 gldL); white blood cells (\1\'BC} 18 x 109/L (6-1 5 x
109/L); urea 8 mmoi/L (2.5-6.6 mmoi/L); creatinine 68 ll-moi!L (20-80
J.LrrlOI/L); pH 7.29 (7.32-'7.37); bicarbonate (HCO;l 20 mmoi!L; carbon
dioxide (C02) 4.0 kPa; base excess (BE) -6 mmoi/L; alanine
aminotransferase (ALT) 33 lUlL (I D-40 lUlL); prothrombin time (PT) .
normal; APPT (activated partial Pf) normal; (-reactive protein (CRP) 22
(< 5 mgldL); Oz saturations in air 97%.
2. A baby girl presents at day 8 of life with tachypnoea, poor feeding and
delayed capillary refill time of 4 seconds. She had been born by NVD at
3 7 weeks after prolonged rupture of membranes for 3 days. Further
examination revealed an enlarged liver, easy bruising and a vesicular
. rash. Initial investigations are as follows: haemoglobin 14 .Wc!b ~- . .
(I 0.5-14 g!dL); WBC 8 x I 09/L (~iS xTb9Jil; ureii8-mmol!L (2.5-6.6
nimol!L}; creatinine 68!l-moi!L (20-80 J.LmOIJL); pH 7.28 (7.32-7.37);
HCOj 19 mmol/L; CO, ~-0 kPa; BE -9 mmoiJL: ALT 787IU!L (10-40
lUlL); PT grossly abnormal; APPT grossly abnormal; CRP 4 ( < 5
mgldL); 0 2 saturations in air 97%.
3. A baby boy pres'!nts at day 3 of life with tachypnoea, poor feeding,
vomiting and delaY.ed capillary refill time of 4 seconds. He had been
born by NVD at 37 weeks with no PROM. Further examination revealed
an enlarged liver and lethargy. Initial investigations are as follows:
haemoglobin 14 g!dL (I 0.5-14 g!dL); white blood cells 8 x 109fL (6-15
x 109/L); urea 8 mmoi!L (2.5-6.6 mmol/L); creatinine 68f!.mOI/L
(2(}-80 fl-mOI/L); pH 7.25 (7 .32-? .3 7i: HCO; 16 mmoi!L; COl 3.5 kPa; BE
-14 mmoi/L; ALT 81U/L (10-40 IU/LJ; PT normal; APPT normal; CRP 4
(< 5 mgldL); 0 2 saturations in air 979&.

INFECTIOUS DISEASES

29. Theme: Non-imported UK infectious diseases


A 1\.lberculosis (TB)
B Hepatitis B
C Juvenile idiopathic arthritis (JIA)
D Brucellosis
E Epstein-Barr virus (EB\')
F Bacterial osteomyelitis
G Infective endocarditis
.
H Lyme disease
Acute lymphoblastic leukaemia (ALL)

:s
U)

For each of the following case scenarios select the most likely diagnosisfrom
the Jist above. Each optionmay be used. once, more than once, or not at all.

I. A 4-year-old boy presents with a 2-week history of fever and being


miserable: His mother says he complains of painful joints and muscles
and general malaise. She has noticed conjunctivitis and a distinctive
rash with large annular erythematous lesions. He has recently been
camping in woodland. Initial investigations are as follows (normal
ranges in brackets): haemoglobin II gldL (I 0.5-14 g!dL); WBC 10 x
109/L (6-15 x 109/L); lymphocytes 4.5 x 109/L; neutrophils 6.0 x 109/L;
eosinophils 0.2 x I 0;/L; urea 3 mmol/L (2:5-6.6 mmoVL); creatinine
45 ,...moVL (20-80JJ.moL'L); ALT 30 IU/L (I 0-40 IU/L); PT normal; APPT
normal; CRP 2 ( < 5 mgldL); erythrocyte sedimentation rate (ESR) 30
(< 20 mrnJhour).
2. A 4-year-old boy presents with a 3-week history of fever and being
miserable. His mother says he has been complaining of generalised
aches and pains and malaise especially in the right leg and has nO\v
developed a limp. He has recently been on holiday in London. X-ray of
the tibia shows periosteal elevation. Initial investigations are as
follows: haemoglobin 1o gldL ( 10.5-14 gldL); WBC 17 x 109/L (6-15 x
109/L); lymphocytes 4.0 x 109/L; neutrophils 13.0 x I 09/L; eosinophils
. 0.2 x 109/L; urea 3 mmoVL (2.5-6.6 mmol/L) ; creatinine 45 ,...inol/L
. - (20-BO,...mol/L); ALT 30 lUlL (I 0-40 IU!L); PT normal; APPT no~mal;
CRP 25 ( < 5 mg/dL); ESR 60 ( < 20 mm/hour).
,
3. A 4~year-old boy presents with 3-week history of fever andbeing
miserable. His mother says he is complaining of painful joints and
muscles and general malaise. She has noticed an intemiittent rash
and intermittent arthritis of his knees. Examination also reveals
splenomegaly. He has just returned from holiday in Spain. Initial
investigations are as iollows: haemoglobin 9 gJdL (I 0.5-14 g/dL); WBC
II x 109/L (6-15 x 1oJL) ; lymphocytes 4.0 x l 09/L; neutrophils 6.0 x
109/L; eosinophils 0.2 x to9JL; platelets 445 x 109/L ( 150-450 x
106/L); urea 3 mmol/L 12.5-6.6 mmoVL); creatinine 45 ,...moi/L (20-80
JJ.mOVl.); ALT 30 lUlL I I o-40 IU/L); PT normal; APPT normal; CRP 20
(< 5mgldL); ESR 60 !< 20 mm/hotir).

ESSENTIAL QUESTIONS FOR MRCPCH

30. Theme: Tropical infectious diseases


A
B

c
D
E
F
G
H
I

Giardia
Yellow fever
l)'phoid
Malaria
1\.lberculosis
Diphtheria
Tetanus
Ebola
Dengue fever
Human immunodeficiency virus (Hrv)

For each of the following case scenarios select the most likely diagnosis from
the lis_t above. 'Each option may be used once, more than once, or not, at all.

10

l. A 4-year-old girl, returned from holiday in India, presents with a 4-week


history of swinging fevers. diarrhoea and general malaise, and a rash.
Temperature 38C; pulse 80 beats/min; respiratory rate 15 per min; blood
pressure (BP) I 00/60 mmHg; ; haemoglobin I _o g!dL (normal range
10.5-14 gtdL);white blood cells (WBCJ6 x .109/L (6-15 x J09JL); platelets
80 x 109/L; neutrophils 2.0 x I 09/L; lymphocytes 2.5 x 109/L; eosinophils
0.3 x I 09/L (15()-,.400); bilirubin 20 f..LmOVL (I. 7-26f..LmOI/L); aspartate
aminotransferase (AST) 80 lUlL (IQ-,-45IU}L); serum sodium 129 mmo]JL
(135-145 mmoVL); serum potassium 3.7mmo]JL (3.5-5.6 mmol!Ll:
serum urea 6.5 mmoVL (2.5-6.6 mmoVL).
2. A 3-year-old girl, recently arrived from Uganda, presents with a 2week history of intermittent pyrexia and a 5-week history of persistent
diarrhoea, oral thrush and weight loss. Temperature 37.8C; pulse 70
beats/min; respiratory rate 13 breaths per min; BP 90/56 mmHg;
haemoglobin9.2g/dL(l0.5-14g!dL);WBC 12 X 109/L(6-15 X 10~/L);
platelets 100 x 109/L (150-450 x 109/L); neutrophils 9.5 x 109/L;
lymphoCytes 2.5 x 109/L; eosinophils 0.3 x 10 9/L; bilirubin 15 fl.mo]JL
(i. 7.:26fl.mol!L); AST 40 lUlL (I 0-45 IU/L); serum sodium 136 mmol!L
(135-145 mmol!L); serum potassium 3.9 mmoi/L (3.5-5.6mmoi!L);
serum urea 4.5 mmol/L (2.5-6.6 mmoi!L).
.
3. A 5-year-old who returned from Ghana last week presents with a 2week history of spiking fevers, vomiting and a cough. Temperature
39 C; pulse 120 beats/min; respiratory rate 20 per min; BP 105/58
mmHg; haemoglobin 7.6 g!dL (l 0.5-l~ g/dL): WBC 10 x 109/L (6-15 x
109/L); platelets 80 x 109/L (15()-,.450 x 10~/L ); neutrophils 4.0.0 x
109/L; lymphocytes 4.5 x I 09/L; eosinophils 0.2 x I 09/L; bilirubin 45
fl.mO]JL (I. 7-26fl.mOl/L ); AST 39 JU/L. (I Q-,-45 lUlL); serum sodium 131
mmoi!L.(I35-145 mmoi!LJ; serum potassium 4.2 mmol!L (3.5-5.6
mmol!L); serum urea 7.5 mmoi!L (2.5-6.6 mmol/L).

--~:L._

>

. .

I.

"''

1.

'

Bronchiolitis in infants
Answers: A CD E

Bronchiolitis is a common cause of lower respiratory infections in i)1fancy. It


cause;5 significant symptoms including cough, wheeze, shortness of breath,
and difficulty feeding. About 2%of all infants require hospital admission. The
majoritY of cases (upio 75%) are cauSed by respiratory syncytial
<R5v>.
althoug.).adenovirus, rhin?virus and myxovirus can also s:ause it. AdenoVirus
can cause a particularlysevere illress.' Primary Epstein.:..Barr virus (EBV)
infection does not 'cause a lower respiratory infection but 'can present as fever.
sore throat and cervical lymphadenopathy.,"
_, . _. , i, c ,
-,

Virus

2. 1)'pical features of typhoid fever


ArlS\vets:'A B

cE

-- :. . '"r..

a;

lj'phoicfjs known' the great mimic arid can presenl'in rnany ways.lypically
children have persistent' swinging pjrexia, headache, diarrhoea, vomiting.
abdoll)inal pain, aJ1d a cough. Characteristic features of typhoid include rose
spots (pink mactiles) and a rel~ltive bradycardia when pjrexiaL they may also.
have signs of heart failure, meningism or shock. Splenomegaly is preSent in
around 25% of cases:

3. Features of measles
Answers: BE" '

M~~sl~scharacteristically present~' With a prodromal illness: which includes

-- _.. _coT:Yzai symptoms cn:inny n<:>se),-norFpurulent conjunctiVItis and-cough

. -- --'
followed by a rash. Koplik spots appear, no~ (gre?' ~~i,te les!qns on_ the ~uFca!
mucosa). The typical rash is maculopapular startmg on the face ~md working
down the trunk. The lesions tend to coalesce and fade to a dusky colour. '
Children often have a sore mouth, however diarrhoea is not common.
Convulsions are not a charaCteristic feature of measles and if they occur they
are invariably triggered by the fever, or ~eflect encephalitis.' ' , " I
t ..~.

4. Organisms causing diarrhoea


Answers:BCDE'

'; ,.~

.,

'
-

. .,
"

"
r

All can cause diarrhoea except for Enterobacter, which is a part of the normal
gut flora.

II

l.

[~-

~
~
l

ESSENTIAL QUESTIONS FQR MRCPCH

------------------------------------

-5. Skin lesions caused by vira1infections


Answers: A B E

Herpes simplex virus cause whitloW: Human papill6rl)a virus causes verrucae,
and molluscum is caused by a virus from the pox family. Impetigo is a
bacterial infection, often Staphylococcus or Streptococcus, and tinea is a
fungal infection.
I , .
'.-;

IJ

., "'

6. Chickenpox rash

cD

Answers:
t

"' "- ..
,

..

,{

: __

The characteristic rash of chickenpox is a crop of macules which quickly


become p_apular. vesicular. and finally pustular before forming a crust The
rash evoh'es in a series of crops and lesions of different ages can be seen. The
rashis'i:rlainly on ~he trunk but can be seen on the face and limbs: The spots
do not coalesce and once they are vesicles they becpme very itchy.

.. '

7. Risk of disease in the classroom


Answer: E

.~-

*.L'-~

Erythema infectiosum, or slapped cheek or Fifths disease. ~ccurs in outbreaks


in primary school children, typically during th~ wint~r or spriryg .. HUI'T)an .
immunod~ficiencyvirus (H fV) is transmitted ver~ically, sexually orVia blood
products. Hepatitis C is also vertically and blood acquired. Atypical'
Mycobacterium has a very low infectivity rate ~etween ,children,. Lyme disease
is spread by tick bites.

. ,

'

"""

.,

8. Faeco-oraJ route in disease transmission


Answers: B DE

Polio, Salmonella and Norwalk virus are all transmitted by the faeco-oral
route and therefore infe<;tions are exacerbated by poor hygiene- espe,cially
lack of hand washing.

9. Infectious diarrhoea in toddlers


Answer: A

,,

The first-line treatment of diarrhoea in infants is the prevention and treatment


of fluid and electrolyte loss by encouraging them to drink. This can be done
with an oral rehydration salts, such as Dioralyte. Anti-motility drugs .
(loperamide), anti-spasmodics (hyoscine) and absorbents (kaolin) .are not
recommended in children. Simple diarrhoea does not require an antibiotic
because it will usually resolve without treatment and is most often caused by
viral infection.
.

12

'.

"'

'I
INFECTIOUS DISEASES

10. Croup
Answers: A D

Mild croup (barking cough VJith no stridor at all) does not req~ire specific
treatment. Severe croup (stridor at rest- as opposed to on crying or when
distressed) should be treated with oral dexamethasone; if the child refuses or
cannot take oral medication this can be given as nebulised budesonide. For
severe croup, adrenaline is given in nebulised form rather than
intramuscularly. Croup is a viral infection and does not require antibiotics.
Moderate croup (stridor on exertion) should be treated with oral or nebulised
steroids.

11. Orbital cellulitis


Ar..-wen;: A B C

There are many serious potential complications from orbital cellulitis. These
include involvement of the optic nerve, leading to loss of vision, and
intracranial extension ofinfection. leading to meningitis and intracranial'
abscesses. Cavernous sinus thrombosis is another well-recognised
complication because the venous drainage around the orbit and face is back
into the cavernous sinus. Finally. it is always worth imaging the sinuses
because orbital cellulitis is often associated with underlying sinusitis.

12. Investigation of immune function


Answen;: B C

Herpes stomatitis and atypical mycobacterial skin infections do not require

further investigation if they are the only infections a child has had.
Cytomegalovirus (CMV} pneumonitis is unusual in children with normal
immune function, but children with T cell dysfunction such as HN can present
with this. Any child with two serious bacterial infections needs investigation
of their immune system. Finally the occurrence of more than ten to fifteen
upper respiratory tr;:u;::till{e_ctiQI}~_in_a year may indicate a minor
immunodeficiency.

- --

13

ESSENTIAL QUESTI<;JNS FOR MRCPCH

13. Notifiable diseases


Answer: All true

Doctors have a statutory duty to notify a large number of infectious diseases.


This helps ihe implementation of measures to control outbreaks, as. well as to
continue disease surveillance. Notifiable diseases include:

14

Acute encephalitis
Acute poliomyelitis
Anthrax

Cholera
Diphtheria
Dysentery
Food poisoning
Leptospirosis
Malaria
. '
Measles
.
.
.
..
Meningitis (meningococcal; pneumococcal; Haemophi/us injluenzae; viral;
other specified and UJ1Specified)

Mepingococcal septicae!Jlia
Mumps
Ophthalmia neonatorum

Paratyphoid fever
Plague
. [
I.
Rabies
Relapsing fever
Rubella
Scarlet fever
.,.
Smallpox
,
Tetanus
'
lllberculosis
l)'phoid fever
l)'phus fever
,,
Viral haemorrhagic fever
Viral hepatitis (types A, B, c, and other)
Whooping cough
Yellow fever.
l;

.,

..

INFECTIOUS DISEASES

14. HIV infection in children


Answers: CE

Most babies are infected by vertical transmission from an HN-positive


mother. Infection usually occurs intrapartum (at delivery), althoughit may
occur prior to delivery or postnatally, especially if breastfeeding. Mother-to.child transmission (MTCf) can be reduced from approximately 30% to less
than 1% with the following measures:
An.ti-reiroViral therapy (ART) to mother in pregnancy to get viral load < 50
copies/mL
ART to baby for the first 4-6 weeks of life
Delivery by lower segment caesarian section (LSCS), ensuring no
prolonged rupture of membranes (if possible)

Avoiunce ofbreas~feeding.
Babies should be diagnosed by polymerase chaih reaction (PCR), orviral
culture if under 18 months of age, as 'maternal antibody may persist this long
and so give false-positive results, It is important to give anti-retroviraltherapy
as saon as possible after birth ... .

1s. Enlarged cervieallymph node


Answers: A B C E

A, B, C and E may present with cervical lymphadenopathy. Atypical


tuberculosis is usually unilateral and painless, and often long-standing. Cat
scratch fever is caused by the organism Bartonella henselae, a Gram~negative
bacterium that is transmitted by cats (usually kittens) to humans by a scratch
or a bite. Of those infected, 30% experience a fever and mild systemic
symptoms but the predominant sign is regional lymphadenopathy involving
nodes that drain the site of inoculation. Most disease is self-limiting but if the
child is very unwell consider antibiotics such as ciprofloxacin. Rubella
characteristically causes post-auricular and sub:occipital lymphadenopathy.
Malignancy should alwa~s be suspected.
~

,.-' .--..--,;,.

.J

,. I

16. Meningitis
Answer: C

..

Hearing impairment occurs in < 5% of cases of meningitis, overall, and is '


most common after pneumococcal infection. Neck stiffness occurs in older .
children only. Neonates present with non-specific signs of infection but may
have fullness of their fontanelle. Studies have shO\'Irn a re,duction of deafness
. and neurological' morbid it} after Hib meningitis if steroids were given early in
the disease course (before or with antibiotics). CSF glucose is usually very low
in tuberculous meningiti~. Although Listeria is a cause of neonatal meningitis,

the commonest cause is group B Streptococcus.

IS

ESSENTIAL QUESTIONS FOR MRCPC!-1

17. Infection risk in cellular immune system defects


Answers: A B D
Children with defects in their cellular immune system are particularly at risk
from intracellular bacteria. such as mycobacteria and Listeria, viruses, fungi
and opportunistic pathogens such as Pneumocystis. Those with antibody;.
defects are more at risk of Gram-positive bacteria, such as Staphylococcus
auieus ahd the streptococcal species, as well as some viruses (eg enterovirus).
Neutrophil defects are associated with Gram-positive bacterial and fungal
infections. Recurrent neis~rial in~ectio,ns are classically seen vyith .


complement deficiencies.

18. Childhood
,, ..
"' exanthemas
,;.

. .',

~'

Answers: B DE

The rash of chickenpox characteristically starts on the trunk: The rash oL


measles does characteristically start behif~d the ears and along the hairline as
a macular rash. lt then t?ecomes maculopapular and ,spreads sequentially t .
downthe body.-.to the face. the upper arn;ts. t~n!< ~nd.then the legs~ u is the
rash ofparvovirus Bl9 (en1hema infectiosum) that causes the :slapped cheek'
appearance. The rash of r~seola infantum is caused
human herpes
(HHV)-6 and usually appears abruptly when the. feyer s~ops. Cqxsacki~ yir:t:s:
especially type AI6, causes 'hand, foot and mouth' disease.with intra-oral
ulcerative lesions, and vesicular lesions on the hands and feet. '

-'

""'

virus

by

lf'
~' ....

t"

tt,,1r

#'

"

. .

'

19: Proinflammatory cytokines


. ' . '1

..

.
.

Ansvvers: A BE
>

.....

'

'

'

'I

'

f ~ ~

'

,;,J ~

'

Cytokines are soluble small proteins that mediate signalling bet:Ween cells.
They usually act in an autocrine (on the cell that pro_9u~~ thl=m) ,or P.aracrine
(on cells close by) manner. They act by bin<;fing to specific r~ceptors at the cell
membrane which causes intracellular signalling and the ~ubsequent
induction, enhancement, or .inhibition of cytokine-regulated genes in t~e
...... nucleus: Cytokines released at the earliest stage~ of.ar infection help to
determine the nature of the subsequent immune response and may be
described as 'pro' or 'a.nti' inflammatory.

20. Gram stain negative organisms


Ansvvers:.CDE
t

' '
.. ,,

'.

...,;

,_
f

....

'
'

The Gram stain is a standard method for visualising bacteria by microscopy.


The purple stain is retained in the cell v,all of Gram-positive organis~s. ~U,~h
as staphylococci and strep~ococci, but is washed out during the staining
. procedure to. a pink colour in Gram-negative bacteria such Escherichia coli
and other coliforms.

.t

INFECTIOUS DISEASES

21. Side effects of tubetculosis dr_ugs . ."


Answers: A CD

Standard therapy for tuberculosis now usually consists o((our drugs:


rifampicin, isoniazid, pyrazinamide and ethambutol (in older children).
Rifampicin is associated with hepatitis; isoniazid w:ith a peripheral neuropathy
preventable by concurrent pylid,oxine therapy; pyrazinamide causes gastritis;
and ethambutol causes impairment of visual acui'tY and colour vision. Eye
tests are routinely recommended on starting ethprpbutol therapy, although
this is difficult in younger children. .
, .

,
' f:

~l .:"!}

22. D: HIV viral load


. ., .

..,

. '"-

t'

1/)

' ~
'

-.

Anti-retroviral treatment is monitored by looking at the CD4 count and the


HlV vira. load. H!V antibodies may be used in diagnosin_g HIV infection,
however children under J 8 months may still ha\'e maternally acquired HIV antibodies. High immunoglobu!in levels are a non-specific sign that occur in
HIV infection. Viral load is noUhe same as HIV antigen as the latter only
.
detects if antigen'is present or not; viral load looks at the quantitative amount
ofHIVvirus in the blood and ismeasured in copies/mL Less than 50 .
copieS/mL is also known as undetectable' and is a sign thai the HIV.virus is,
being successfully suppressed by medication (not eradicated, as anti- ,
retroviral therapy is only able to suppress viral replication).

!:

23. A: DMSA scan


DMSA scan is'the'best test for looking at renal scatring. Intravenous. t<
pyelograms (IVPs) are used to look for stones. Renal ultrasound. is used to .
look at the kidney structure, for example, to identify horseshoe or duplc:x
kidneys. Micturating cystourethrogram (MCUG) looks for vesicoureteric , .
reflux. A MAG _3 renogram looks at renal perfu~io11, not scarring.
24. C: Lumbar puncture

.,

.'tD

..

In a shocked child with a history and clinical picture of menh1gococ~aemia


the correct management would be to get intravenous access, collect blood for
culture: PCR, serology and rapid antigen screen, FBC:clottin-i, urea and.~-.
electrolytes (U&Es), and glucose- ifinsufficient blood is available then the
blood cultures and BM stick (glucose) and blood gases and electrolytes are
most importanUnitially. Immediate treatme.nt is with oxygen. fluid
..
resuscitation and high-dose intravenous antibiotics. Jn this scenario, doing a
lumbar puncture is very dangerous and could pre~Ipi_tate circulatory collapse.
For confirmed meningococcal disease a lumbar puncture js not necessary as
it does not alter treatment (7 days of appropria~e 9ntibiotics, usually
_,.
ceftriaxo~e) for sepsis and/or meningitis. Foro,ther causes of-meningitis
pneumococcal), an abnormal lumbar, puncture would lengthen treatment and
therefore should be done \.o,hen the patient is clinically stable. Remember, cell
'
changes persist even after antibiotics have been given. .

(eg.

17

ESSENTIAL QUE~TIONS FOR MRCPCH


',

:t~:,;
[. ,,t.i;
i~G).

25. E:

,,

'

)f!'

-~-'

....

__ .._

~~

A chil;1\vha.;had prednisolone 2

mglkglfor 3:;dayst

we~k.:"'ago

The following children need VZJG:

;,;

~'
;;;:>-~~

- - -'

Tl;~ose on~c~emotlterapy ra?iotliefapy, ~n\ithin 6 months of treatment


\\ith these

Those within months of having had' a bone marrow transplant


Thpse who have had an organ transplant and are on immunosuppressive

or

~JJ ~=:~to have r~ci:ived ,{.;thin thel~t ;mont~s'oral prednisolo~ at

;...
\.V

more than 1 inglkg/day for 1 month or 2 mgtkgtday for more'thi:m 1 week~


Those who are on steroids in combination with cytotoxic drugs.
.
Note that this means that a child receiving the .standard 3iday C()UfSe of
steroids for an acute exacerbation of asthma would riot requireVZIG after
exposure to chickenpox .. , . 1 .
. '
'
'
.
'
..

'

M'

''

t" .

26. B: .Mycoplasma pneumoniae

'

This is a typical description of Mycoplasma,pne,umoniae. The history is too


short for tuberculosis. streptococcus and Haemophjlus do n()t give t~e .extra-:.
pulmonary symptoms and Chlamydia caus~s a chest i.nfecti()n ~n. babies age!j
4-6weeks ..

21. E: Rubella infection


I
Rubella is the most likely diagnosis with this constellation of sighs.
Toxoplasmosis occasionally may produce cataracts. but is not likely to.resql~
in heart defects and murmurs. Symptomatic parvo\irus infection usually
results in hydrops, anaemia and cardiac failure with a murmur, but nqv
cataracts or blueberry muffin (petechia)) rash.

.'

"

28. DIFFERENTIAL DIAGNOSIS OF NEONATAL INFECTION

t.

c- Group B streptococcal (GBS) infection

-:.

This scenario 'describes the clasSical early presentation ofGBS septicaemia


with meningitis. This disease usually present early I< 12 hours) or late,
(> .8.,.,28 days). There is often a history ofPROM. as with many neonatal. f ..n
infections. Blood gas analysis revealsa mild, partially compensated:' , ..
metabQlic acidosis consistent withsepsis. Liver function tests (Lffi) aild
clotting are normaL CRP rais~d in this case although this may not occur in
neonates with infection. Other diagnoses in order of likelihood are as follows:
Escherichia coli infection :(may present \vith early or late disease); Listeria .
monocytogenes infection (may present with early otlate disease); Group A :
streptococcalinfection (veryrare these daysf'. ' - ,. , .; . , ..~;r .i .,
2." D .:. Disseminated herpes simplex virus infection . , . ; .
~ Again, as usual in clinical practiCe, the neonate presentswith nor,~,-specific ..
' signs and symptoms of sepsis: In this 1case the baby is a little older, has an .
enlarged liver and grossly abnormal LFTs ahd Clotting: White blood count and

is

18

INFECTIOUS DISEASES

CRP are normal suggesting a non-bacterial cause, however the prolonged


rupture of membranes (PROM> gives a clue to an infective cause. The rash is
actually vesicular and the! baby has disseminated herpes simplex virus (HSV)
infection. This classically presents around this time, usually in a mother who
has acquired HSV late in the third trimester (often an asymptomatic infection
in the mother). Approximately 85% of neonatal HSV infections are acquired at
delivery, 5% pre-natally and I 0% post-natally (eg from cold sores, herpetic
whitlow). Without early treatment with intravenous aciclovir the mortality is
extremely high. Other diagnoses. in order of likelihood, are as follows:
metabolic disease; overwhelming bacterial sepsis (still the most common
neonatal diagnosis even if not the most likely in this case)_; congenital
infection. Congenital heart disease may give poor feeding and enlarged liver
but not grossly abnormal LFTs or deranged clotting.

3.

G - Methylmalonic aciduria

There is no history of PROM. a normal WBC and CRP suggesting noq.-infective


cause. The blood gas reveals severe metabolic acidosis, with mildly raised
LFTs making metabolic disease the most likely. This child needs a metabolic
work-up although initial management. should also cover initially for sepsis
with antibiotics. Other diagnoses in order of likelihood in thisscenario are as
follows: HSV infection; overwhelming bacterial sepsis (still the most common
neonatal diagnosis even if not the most likely in this case).
29. NON-IMPORTED UK INFECfiOUS DISEASES

1.

H - Lyme disease

This is the classical description of Lyme disease, with erythema chronicum


migrans rash, especially with a travel history of an area known to haVe ticks.
Lyme disease is caused by the organism Borrelia burgdorferi which is
transmitted to man by the Ixodes tick. If not treated at this primary stage Lyme
disease \\rill progress to stages 2 and 3 with further complications. The clinical
manifestations of the .three stages are:
Stage 1 (early localised): distinctive rash; erythema migrans; red
to a
macule/papule at site of tick bite, which expands over days or
large annular erythematous lesion with partial clearing, approxil)ijitely. 15
em diameter; associated \'t.ith fever, malaise, headache; neck stiffness '
Stage 2 {early disseminated): 3-5 weeks post-bite; multiple erythema
migrans; cranial nerve palsies especially VUth (Bell's); meningitis;
conjunctivitis; arthralgia; myalgia; headache; malaise; rarely carditis
Stage 3 (late disease): recurrent arthritis; pauciarticular, large joints;
neuropathy; encephalopathy.

weeks

The second option is juvenile idiopathic arthritis (JlA). Systemic onset JIA
presents with prolonged fe\'er and malaise but a very different salmon pink .
rash. The rash of infective endocarditis (lE) (third choice) is also very different
with Janeway lesions. splinter haemorrhages and Osler's nodes.

ESSENTIAL QUESTIONS FOR MRCPCH

2.

F- Bacterial osteomyelitis

High CRP, white blood cells an.d ESR With characteristic X-ray changes make
this diagnosis the most likely answer- the most common culprit, in an
immunologically normal host. is Staphylococcus au reus. Osteomyelitis is two
to fou.r times more common in boys. 1\.lberculosis (TB) is the second choice,
which can cause osteomyelitis. It is always important to think of malignancy
in patients 'A'ith bony pain, either localized osteosarcoma or acute
lymphocytic leukaemia (ALL). Evidence of these would usually be seen \Vith
cytopenias on the FBC (full blood count). The third possibility is JIA.
3. C- juvenile idiopathic arthritis OIA)

It is always important to remember non-infectious diseases that present to

the infectious diseases department with prolonged fever. This is a classical


case of systemic onset juvenile arthritis, with a salmon pink' intermittent
rash. Splenomegaly can also occur in other diseases such as ALL (but no
cytopenia). brucellosis (does not give this rash; no night sweats) and infective
endocarditis (no murmur in this case). Cases of Brucella can be found in
Spain. but of cour~ a tr.avel history is often just incidentaL The second
possibility is brucellosis, and the third is infective endocarditis.
30. TROPICAL INFECTIOUS DISEASES
1.

C -l)'phoid

1)'phoid typically presents with a relative bradycardia to th5!pyrexia. Often the


eosinophils and platelets are low. AST can be raised and hyponatraemia is
common. With this long history, HIV has to be considered (second choice).
Malaria must be excluded but does not usually cause diarrhoea.
2. , J - Human immunodeficiency virus {HIV)
HIV is most likely. Second choice is Giardia .infection; and third, typhoid. HIV
has a longer course and persistent diarrhoea can be a presenting feature.
These children can be lymphopenic and thrombocytopenic. Chronic giardiasis
can present with offensive stools and a low-grade pyrexia. and weight loss;
however the blood results do not fit. 1Jphoid is in the differential, but the
patient is not bradycardic and has a normal sodium.

3.

D - Malaria

A child with malaria can present acutely unwell, often with a fever and cough.
Severe diarrhoea is rare. They are often anaemic wit~ a raised bilirubin but
normal liver function tests. TUberculosis is the second possibility- although
less likely as the.history is relatively short. 1Jphoid is a great mimic of other
diseases, and is the third choice:

.,

20

'

'

Mike Champiorj
t

..

'
.

'

T.~~,f;t:~::..c~:~~:.:-r:~-:.c.-;;;.cl}';.t.,i,:-~Ol"..~~':l:~-;

'
'

I.

Multiple Choice Questions

1. The following inborn errors of metabolism have x:Jinked ,


recessive inheritance

0
0
0

A
B
C
D
E

Hurler syndrome
ornithine transcarbamylase deficieJ"!g
biotinidase deficiency
Lesch-Nyhan syndrome
acute intermittentporphyria

...
,,

2. Hyperlactataemia is a recognised feature of

0
0

0
0
0

A
B
C
D
E

~j

... ~.:

isovaleric acidaemia
very long chain acyl-co-A dehydrogenase deficiency (VLCAD)
venepuncture with tourniquet

' 1'

Smith-Lemli-Opitz syndrome

hypovolaemic shock

.1
1
';

3. causes of metabolic aCidosis include

0
0
0
0
0

~ hyperammonaemia .

B ~hloridorrhoea ' "''


C maple syr:upurine disease
D non-ketotic hyperglycina~mia
E cystinosis

4. Hypoglycaemia

0
0
0
0

1 :

'

..

l .

...

.;

'

,I,

A can be diagnosed by Glucostix


...r- . .
B is defined as a glucose concentration of less than 2.0 mmol!L
C V.rith the presence of ketones in the urine excludes medium chain acylCoA dehydrogenase 1 (~1CAD) deficiency a~ the cause
.
D in the absence of ketones ahd glucose requirement gre_ater than
8 mgtkgtmin to maintain normoglycaehlia, is indicative~;>[ .
hyperinsulinism
'

E secondary to glibenclamide poisoning is. associated with a high ratio of


insulin to C-peptide

21
j '

rr .
!tCil
,.

ESSENTIAL QUESTIONS FOR MRCPCH

i:'

to

5. The following treatnients and conditions are correctly pairec.P.

0 A
0 B
0 c
0 D
0 E

methylmalonic acidaemia and glycine


tyrosinaemia and betaine
Gaucher's disease and bone marrow transplantation
ornithine transcarbamylase deficiency and sodium phenylbutyrate
non-ketotic hyperglycinaemia and dextromethorphan

6. The following features are common to both Marfan syndrome


and classical homocystinuria

0
0
0
0
0

A
B
C
D
E

osteoporosis ,
t
span is greater than height
thrombotic tendency.
hypermobile joints
ectopia Jentis

'',

7. Recognised features of cystinuria include

0
0
0
0

A
B
C
D
E

photophobia
grO\'Ith faltering
renal calculi
seizures
rickets

'

I'

.I

8. Phenylketonuria (PKU)

O
0
0

0
0

;~

'

A breastfeeding is contraindicated in the ne"Ytiorn infant with


phenylketonuria
B the diet is phenylanine-free
C the concentration of brain phenylalanine direcUy correlates with that in
the plasma
D 'diet' drinks (With artificial sweeteners) impro\e phenylalanine control
E phenylala~ine is teratogenic
'

9. Causes ofhyperammonaemia include

0
0
0

A urinary tract infection


B argininosuccinic.aciduria
C methylmalonic acidaemia
D patent ductus venosus
E liver failure

I 0.

Th~

0
0
O
0

A
B
C
D
E

' 22

.
'

'

'

following are true of mitochondrial DNA

replication is inhibited by zidovudine


.
.
rearrangements have a low risk of recurrence in siblings
encodes subunits for all the respiratory chain complexes
has no histone coat
is inherited from the mother

METABOLIC MEDICINE

. I I. Conjugated hyperbilirubinaemia is a feature of

0 A Gilbert syndrome
0 B galactosaemia .
0 C Crigler-Najjar syndrome
0 D mitochondrial DNA depletion syndrome
0 E tyrosinaemia
12. Metabolic disorders presenting with dysmorphic features in the

0
0
0
0
0

neonatal period indude


A congenital disorders of glycosylation (CDG) Ia
B medium chain acyl-coA dehydrogenase (MCAD) deficiency
C Hurler syndrome
D ::ellweger syndrome
. '
E Smith-Lemli-OpilZ syndrome

13. The following are true concerning cholesterol

0
0
0
0
0

A those at risk of familial hypercholesterolaemia should be screened at


birth
'

B cholesterol-lowering margarines are effective in the paediatric age


group
,,
C statins are contraindicated in children
D a low level of high-density lipoprotein (HDL) cholesterol is a risk
factor for premature ischaemic heart disease
E girls are at lower risk than boys for developing premature ischaemic
heart disease

14. The following metabolic processes occur within the .peroxisome

0 A urea cycle
0 B fatty acid oxidation
,.
0 C glycosylation ofglycoproteins
0 u- bile~aciifsyiltnesis - - _...
0 E phytanate oxidation
15. 11te following are CQnsistent with a' diagnosis of propionic

addaemia

0 A hyperglycaemia
0 B neutropenia
0 c hypercalcaemia
0 D- ketonuria
0 E raised lactate

23

ESSENTIAL QUES)IONS FOR MRCPCH

.Best of Five Que5tlons


"'-"'',~:.

.,

,:
"'~

16. A 10-month-old girl presents with pneumoiua. She was noted to


have marked hepatomegaly. Before starting intravenous fluids
the laboratory glucose was 2.1 mlllol/L. Further investigations
revealed lactate of 6.1 mmol/L (norlT!al < 2.0), triglycerides of
17 mmol/L (normal< 2:3),'and urate of 3.6 mmol/L (normal<
0.4 7). The MOST likely diagnosis is which of the. following?

0
0
0
0
0

A
B
C
D

Phosphofructokinase deficiency (GSD V)


Glucose-6-phosphatase deficiency (GSD I).
Branching enzyme deficiency (GSD IV)
Debrancher enzyrrie deficiency (GSD III)
E Myophosphorylase deficiency (GSD VII) '

I 7. An 11-month-old girl was admitted following an afebrile

seizure. Urine was negative for ketones on dipstick. Blood


glucose was noted to be ).8 mmol/L. A repeat glucose was 2.6
mmoiJL 30 minutes later following 10% dextrose bolus,,and
an infusion of 12.5% dextrose was required to maintain
nonnoglycaemia. Further investigations included lactate 1.8,
J.UtlOIJL (normal < 2), and ammonia'92 j.LmOlJL (normal< 50).
What is the NEXT treatment
of choice?
.

0 A
0 B
0 c
0 D
0 E

.'

Hydrocortisone
Ottreotide
Glucagon
Sub-total pancreatectomy
Diazoxide

18. A 48-hoU:r-old infant on the post-natal ward is noted to be


tachypnoeic, sleepy and feeding poorly. ~lood gas analysis
shows: pH 7 .48; Pao2 12.3; PaCo2 3.6; standard bica~bonate
20.0; base excess 1.6. What is the NEXT ~ey inve5tigation?
0 A Ammonia
' .

0
0

B Lactate
C Chest X-ray
o Amino acids
E Blood culture

_;.

.,;

'

'

24

METABOLIC MEDICINE

19. AriS-month-old infant,was noted to"'have a metabolicr-act~o:Sis


when being investigated for tailure to thrive despite dietary
supplementation ana no eVidence of malabsorption. Ain.monia
36 J.LmoVL (nonnaf< SO), lactate 1.3 J.LIDOVL (normal < 2), allion
zap 19. Urinary ketones negative:.,.The MOST likely diagnosis is
which of the following?

0
0
0
0
0

A Propionic acidaemia
B Citrullinaemi:t
C Renal tubular acidosis
D Protein-losing enteropathy
E Medium chain acyl-CoA dehydrogenase (MCAD) deficiency

. 20. Ar infant with microcephaly born to a 36-year-old woman Is

cu.-rently under investigation for potential causes. To exclude


maternal phenylketonuria (PKU) what is the investigation of
choice?

0
0
0
0
0

A l~fant plasma;:imin~acids
B Maternal plasma amino acids
.C Infant urinary amino acids
D Maternal urinary organic adds
E Maternal urinary amino acids

21. An 11-month-old boy presented with an early morning seizure. He


flad been unwell with gastroenteritis and poor oral intake for'36
liours. His glucose concentration on admission was 1.1 riimoVL.
Urinalysis was negative for glucose and ketones. Subsequent
organic acid analysis revealed a dicarboxylic aciduria. Which' of
&he following would be the MOST useful investigapon?

0
0
0

0
0 .

...

A Fat oxidation studies on fibroblasts


B Acylcarnitines
C Genotyping for medium chain acyl-CoA dehydrogenase (~1CAD)
deficiency

- - :'"----- ~ ...,_ ...... ~ --~.


D Amino acids

E Very long chain fatty acids ;

22. A 12-month-old girl was referred for investigation for


,
developmental regression having lost the ability to sit .
unsupported . .EXaminai:im\ revealed acherry red spot. iher~
were no audible murmurs and the abdomen was soft with no
palpable organomegaly. What is the MOST likely diag'i\osis?

0 .A
0 B
0 C
0 D
0 E

'\

~ '\

,.

,.

Fabry disease
Sand hoff disease
Niemann-Pkk type C
Gaucher's disease
Tay-Sachs disease

lS

:(;{.";~{::~
ESSENTIAL QUESTIONS FOR MRCPOi

. ~!?(tended, Matching QU'e.sti6n_s:.. ..

\.....
-;r;l;.:..,~.~"'l'A,.'T'

r'o

......

~~
'
.

n.
Cl)

:::J

cr

~ ~-~

23.. Theme: Investigations of inb_om erro:rs.: ~-' (


A . Very long chain fatty acids
B
Urinary sulphite dipstick
C
Urinary amino acids

D Plasma amino acids


E
Organic acids
F
Acylcarn(tines :.
G Urate
H
Lactate
. '
I
Ammonia
J .7-Dehy<:Irocholesterol.

i ~.-

:1

t'"

. ,,

.
-~

'II

. .~

_i\; .. "'

,..~~ ...~ ,:').,~~ .~...... ~TJ{:..... ~~~.J.1'..';

'

--:-

For each of the following inborn errors, select the most appropriate initial
investigation from the list above. Each option'may be used once, more than

"'
'
'
once, or not at all.

0
0
0

..... ~,

I. Zellweger syndrome.
.
1
2. Maple syrup urine disease (MSUD) ..
3. Lesch-Nyhan syndrome.

'

-t '.)

24. Theme:' Eye signs in inborn errors .


A
B
C
E
F
G
H
I

Ectopia lentis
Corneal clouding
Stellate iris
Pigmentary retinopathy
Cherry red spot
Cataract
Arcus juvenilis
Kayser-Fleischer ring
Glaucoma

...

J
For each of the following inborn errors, select from t~e Jist above the eye signs
common{y associatt;d with each condition.. Each option may be used once, .
more than Once, Or nOt at Olf.
.
" ' ''
' I , '
.

0
0
0

26

j.

;.

1. Long chain hydroxy.acyl-CoA qehydr~gen,c(~ c!<:!ficiency. , ..


2. Galactos9eq1ia.
,
, ~, , '._1t
~
'3. Morquio syndrome.
_
,

METABOLIC MEDICINE

25. Theme: Uver failure


A
B
C
D
E
F
G
H

Isovaleric acidaemia
Fructosuria
~rrosinaemia

\o\'ilson disease
Galactosaemia
Zellweger syndrome
Medium chairi acyl-CoA dehydrogenase (MCAD) deficiency
Ornithine transcarbamylase (OTC) deficiency
Hereditary fructose intolerance

' '

For each of the following case scenarios select the most likely diagnosis from
the list above. Each optidn mC?)! be used once, more than once, or not at alL

I. r\ 5-day-old breast-fed infant with deepening.jaundice, gross


hepatomegaly and se\'erebruising. Red reflexes arepresent. Glucose
3.1 mmol!L. Ammonia 97 J.Lmoi!L (normal < 50). INR 5.1. Septic
screen: positive blood culture for Escherichia coli. Organic acids>
,elevated phenyllactate and 4-hydroxyphenyllactate .. Orotic G!cid 3.4
mmol/mol creatinine (normal < 5).
.
2. A 2-day-old formula-fed infant with jaundice, gross hepatomegaly and
bruising. Red reflexes are present. Glucose 3.0mmoi/L Ammonia 117
~mol!L (normal< 50). INR 4.1. Septic screen: negative. Organic acids:
increased phenyllactate. 4-hydroxyphenyllactate and su~cinylacetone.
Orotic acid 4.6 mmol/mol creatinine (normal< 5).
. .
3. A 5-~onth-old .infant fed on soya-based formula 'lnd t\rst,weaning ,
foods, with recent onset of poor feeding alld recurrent vomiting, finn
hepatomegaly and marked tachypnoea. Red reflexes are prese11t
Glucose 3.3 mmol/L. Ammonia 82 !Lmol!L (normal < 50). INR 4.6.
Septic screen: negative. Organic acids: increased phenyllactate and
4-hydroxyphenyllactate. Orotic add 4.9 mmol!mol creatinine (nm:__m<!l

< 5).

1. Inborn errors of metabolism


Answers:BD
Inborn errors of metabolism are most commonly inherited in an autosomal
recessive fashion. X-linked reCt!SSive exceptions include ornithine
transcarbamylase (OTC) deficiency. Lesch-Nyhan syndrome, Fabry disease,
Hunter syndrome. and adrenoleukodystrophy. Female carriers may have
symptoms with OTC due to the effects oflyortisation. Acute intermittent
porphyria is inherited in a dominant fashion.

2. Hyperlactataemia

..

Answers: A B C E

Elevated lactate is a feature of a number of inborn errors including organic


acidaemias (A). fat oxidation defects (B), respiratory chain disorders. pyruvate
disorders. disorders of gluconeogenesis, hereditary fructose intolerance,
biotinidase deficiency. and glycogen storage disorders (1. Ill, Vl and IX). In
practice. hypoperfusion and hypoxia need to be excluded because secondary
caus~s of lactic addaemia are more common. Spurious elevations of lactate
commonly arise due to a squeezed blood sample. An arterial sample should be
taken if a free-flowing venous sample' cannot be obtained. Smith-Lemli-opitz
syndrome is a disorder of cholesterol synthesis resulting in dysmorphism and
mental retardation.

3. Metabolic acidosis .

..1'

,.,

. Answers: c E
'
.
Maple :Syfup urine disease (MSUD) produces a metaboliC acidosis due to the
build~up of a number of organic acids. Cystinosis results in damage to the
proximal tubule due to cystine storage resulting in a renal tubular acidosis.
Non-ketotic hyperglycinaemia (NKH) produces a respiratory acidosis as
apnoeas de\elop. Ammonia is a respiratory stimulant and therefore produces
a respiratory alkalosis. Chloridorrhoea produces a metabolic alkalosis
secondary to chloride loss in the stooL
4~

Hypoglycaemia
Answer: D

Stick tests are used as screens fo.r hypoglycaemia. but may be unreliable at-low
glucose concentrations. Diagnosis requires formal laboratory confirmation.
Hypoglycaemia is defined as a laboratory glucose of :S2.6 mmoi/L Fat oxidation

28

.'

METABOLICMEDICINE.

defects such as MCAD are causes ofhypoketotic hypoglycaemia, however the


ketone production is inappropriately low for the degree of hypoglycaemia,
rather than absent. Hyperinsulinism is a further cause ofhypoketotk
hypoglycaemia characterised by a persistently elevated glucose requirement (>
8 mg!kgtmin). Poisoning with exogenous insulin is characterised by a high
insulin to C-peptide ratio as the C-peptide is inappropriately low. However, antidiabetics such as glibenclamide promote endogenous insulin release and so the

ratio is normal.

s.

Pairing of treatments and conditions


Answers: DE

Organic acids conjugate with glycine and carnitine. speeding their excretion
via the kidney. Glycine is used'to treat isovaleric acidaemia, whereas carnitine
is use.~ in methyl malonic and propionic acidciemias. 1)rrosinaemia is treated
.with 'NTBC which effectively blocks the production of damaging metabolites;in
the tyrosine catabolic pathway. The path\vay remains blocked, and therefore a
low-tyrosine diet is still required. but the hepatic and renal complications are
abolished. Betaine is used tore-methylate homocysteine to methionine in
classicaLhomocystinuria and in other metabolic blocks with associated
elevated homocysteine. eg cobalamin (vitamin 8 12) defects. Gaucher's disease
is a sphingolipidosis characterised by the storage of cerebroside in liver.
spleen and nervous system, secondary to glucocerebrosidase deficiency.
Previously bone-marrow transplantation has been used. but has been
superseded by enzyme replacement therapy. Sodium phenylbutyrate and
sodium benzoate are used in urea cycle defects, such as ornithine
transcarbamylase deficiency, as an alternate pathway for nitrogen clearance.
Conjugation with glutamine and glycine produces water-soluble products
that can be excreted directly by the kidney, avoiding the need for processing
by the urea cycle, and thus reducing the load on the pathway.
Dextromethorphan is a partial antagonist of the NMDA receptor to which
glycine binds in the central nervous system. This blocks the excitatory effect
of glycine and therefore helps reduce seizures in non-ketotic
hyperglyCinaemia (NKH). Long-term neurological development remains

severely affected. .

6. Marfan syndrome and homocystinuria


Answers: BE

Both conditions appear physically similar with a marfanoid habitus, and


dislocated lenses. There are differences in the joints. which are stiff in
homocystinuria but hypermobile in Marfan syndrome. Intelligence is reduced
in homocystinuria. Cardiovascular compromise in Marfan syndrome includes
aortic root dilatation, whereas homocystinuria is complicated by thromboses.
Osteoporosis is common in homocystinuria, but not in Marfan syndrome.

29

r
,.

t
r

ESSENTtf..l.. QUESTIONS FOR MRCPCH

7. Cyst!nuria
AnS11'er: C

Cystinuria is a transport defect resulting in the excessive loss of COAL


(cystine, ornithine, arginine and lysine) in the urine. The only clinical
consequence is renal stone formation. All the listed features are seen in
cystinosis. This is a lysosomal defect with excess cystine being stored, \\ith
adverse effects on kidneys, liver, thyroid, pancreas and brain.

8. Phenylketonuria
Answer: E

PKU results from a block in phenylalanine metabolism resulting in elevated


phenylalanine levels and neurological impairment. It is managed with a
phenylalanine-restricted diet. Phenylalanine is an essential amino acid and
therefore cannot be totally removed from the diet. In the neonate this can be
given as formula or breast milk. The level ofphenylalal!ine in the brain cannot
be predicted from. the plasma concentration which explains why some
patients ha\e good outcomes despite 'poor control and Vice versa. A reduction
in transport across,the blood brain barrier is protective. 'Diet' 'drinks are
contraindicated because aspartame contains phenylalanine, thus the full
sugar varieties are preferred. Phenylalanine is teratogenic, and therefore even
though the chance of a PKU mother having a PKU affected child is only 1 in
100: the fetus can still be damaged by maternal PKU. The condition must be
managed very carefully in pregnancy, by rriC?re frequent blood monitoring .and
stricter control of phenylalanine concentrations.

9. Hyperammonaemia
.
.
AnSiver: All true
Hyperammonaemia is the hallmark of primary urea cycle defects such as
argininosuccinic aciduria. The liver is the home of the urea cycle, and
therefore liver failure can result in severehyperammonaemia. Secondary
inhibition of-the urea cycle occursin the-organic acida~ITiias:Transierit --hyperammonaemia of the newborn is thought to occur when there is delayed
closure of the ductus venosus, whereby blood is shunted away from the liver,
thus bypassing the urea cycle. Urinary tract infections with urea-splitting
organisms (such as proteus) result in the release of ammonia from urea. and
are a cause ofhyperammonaemia.

10. Mitochondrial DNA


AnSI\er: A B DE

~Mitochondrial DNA (mtDNA) is inherited from the maternal egg. The paternal
mtDNA is locatedin the tail of the sperm(which does not _enter the egg at
fertili;;ation). Point m1;1tations are inherited through the female line, however
rearrangements such as deletions and otiplications tend to be sporadic \\ith
" lO

METABOLIC MEDICINE

low risk ofrecum~nce. The mutation rate in mtDNA is higher than nuclear
DNA (nONA), because there is neither a protective histone coat nor the same
repair mechanisms enjoyed by DNA within the nucleus. Mitochondria are the
. product of two g'enomes, mitochondrial and nuclear. mtDNA does not encode
for all the subunits of the respiratory chain complexes. Complex II is entirely
nONA encoded. mtDNA replication is inhibited by nucleoside analogues such
as zidovudine. There is some concern about its use in treating pregnant
mothers to block the vertical transmission of human immunodeficiency virus
(HIV).

11. Conjugated hyperbilirubinaemia


Answers: B DE

ConjuQated hyperbilirubinaemia occurs in most inborn errors that adversely


affect ; he liver including ga lactosaemia and tyrosinaemia. mtDNA depletion
syndrome is. a nONA encoded respiratory.chain disorder that is associated
with severe liver. disease in some cases. The mtDNA is qualitatively normal,
but reduced in amount. Causes ofunconjugated hyperbilirubinaemia include
haemolytic anaemia, Crigler-:-1ajjar syndrome and Gilbert syndrome.

12. Dysmorphic features


Answers: A D E

. Dysmorphic features occu in inborn errors associated with the making and
breaking of complex molecules. Inability to synthesise structural molecules
interferes with embryogenesis (eg plasmalogens in Zellweger syndrome,
glycoproteins in carbohydrate-deficient glycoprotein syndromes (COG), and
cholesterol in Smith-Lemli-Qpitz syndrome). Inability to break down complex
molecules (as in lysosomal storage disorders like Hurler syndrome). results in
coarse dysmorphic features which develop over time beca.use of accumulated
storage. Initially the neonate appears normal. Most patients with inborn
errors are not dysmorphic.

13. Cholesterol metabolism


AnSVI!ers: B D E

At-risk family members should be screened, but ideally not before 5 years of
age. This practice is based on the unreliability of neonatal values in predicting
adult values, and the lack of evidence to support infant treatment. 'Lifestyle' is
the treatment of choice for heterozygotes aged less than 1oyears. involving a
healthy diet and exercise_ The cholesterol-lowering margarines are effective in
children and are more palatable than binding resins. Statins are used in
children who are homozygotes, and in heterozygotes aged more than 10
years. The age at which to start statins is controversial, and is based on an
assessment of c;holesterolle\eJ. gender, and family history. Girls are at less
risk, tending to develop heart disease 10 years later than males. Family

ll

ESSENTIAL QUESTIONS ;;OR MRCPCH

history is a very important independent risk factor for prerhature heart


disease, and is more significant than the degree of hypercholesterolaemia.
Low HDL cholesterol is a risk factor for premature ischaemic heart,diseas~.

14. Peroxisome metabolism


Answers: B D E
'

'

'

Peroxisomes are present in all cells except mature red cells. They have many
synthetic and catabolic functions. They are the site of biosynthesis of
plasmalogens, bile acids. and cholesteroL They are the site of[3-oxidation of
very long chain and branched chain fatty acids. Other oxidative processes
include those of phytanate (Vitamin A), glutaric acid. and pipecolic acid. They
are also the site for glyoxylate metabolism. Peroxisomal disorders may result
from a complete absence of function (Zellweger syndrome), from Joss of a few
processes, or from blockage of a single pathway, such as phytanate in Refsum
disease, and VLCFA oxidation in adrenoleukodystrophy. The urea cycle occurs
in the C}1oplasm and mitochondrion. and glycosylation of glycoproteins
occurs in the endoplasmic reticulum and Golgi. Disorders of this pathway

lead to the congenital disorders of glycosylation (CDG).

15. Propionic addaemia

Answers: A B

DE

..

Propionic acidaemia is an organic acidaemia caused by a block in the .


catabolic pathway of isoleucine and valine {propionyi-CoA carboxylase). l.t
usually presents in the first days of life with a marked metabolic acidosis~ with
ketosis and raised lactate. The severity of the decompensation may switch off
the bone marrow resulting in neutropenia, or even pancytopenia, and
therefore may be confused with sepsis. Moderate hypocalcaemia is common.
Hyperammonaemia may occur due to secondary impairment of the urea cycle.
Normoglycaemia or hyperglycaemia are seen, rathc;r than hypoglycaemia.
Other acute complications include globus pallid necrosis, cardiomyopathy

and pancreatitis.

us

16. B: Glucose-6-phosphatase deficiency (GSD I)


These are all glycogen storage disorders (GSD). Hypoglycaemia is not a
feature of the muscle GSDs v {phosphofructoki.nase) and,GSD Vli
(myophosphorylase). Later onset of muscle fatigue and myalgia is their usual
presentation. Branching enzyme deficiency produces long glycogen chains.
that act like amylopectin to produce cirrhosis and ultimately liver failure.
Hypoglycaemia is a feature of the liver GSDs: GSD l (a or b), GSD III, GSD VJ
and GSD IX, although it is usually mild (or absent) .in the last two conditions.''
The results of the investigations identify glucose-6-phosphatase deficiencv as
the cause. Th~ bioc.hemicalsignature of GSD I is hypoglycaemia, with raised
lactate, urate and_ triglycerides. Hypoglycaemia occurs once the exogenous .
sources of glucose are exra~:~sted as glycogenolysis and gluconeogenesis are

- _L

METABOLIC.MEDICINE

blocked. Presentation may be delayed'bY:nany months as long,as regular and


frequent feeds are taken (eg prop-feeaing or demand feeding overnignt).

Breakdown of glycogen to pyruvate is in tad;. however, and lactate is therefore


produced as a secondary fuel for the brain. Conversion of lactate and pyruvate
to fatty acids aAd cholesterol results in h)'i)erlipidaemia: Uric acid production
is increased from glucose-6-phosphate.

1 7. E: Diazoxide
Hypoketotic hypoglycaemia has a limited'differential diagnosis, and the
.
failure of rapid reversal following a bolus and the requirement of a continued
dextrose infusion is suggestive of hyperinsulinism. The coexistence of
hyperammonaemia suggests HIHA (hyperinsulinism hyperammonaemia) .
syndrome. This is due to upregulation of glutamate dehydrogenase. Diagno.sis
may b~ delayed by several months if the child has access to regular feeds, for.
example by continued breast or bottle feeding every few hours throughout ihe
night and day. HIHA is exquisitely responsive to diazoxide to control the
hyperinsulinism and it is the treatment of choice. The hyperarumonaemia is'
controlled with mild-to-moderate dietary protein restriction.

18.

. ,.

A: Ammonia

:.

.
.

The finding ofa respiratory alkalosis in a sick neonate is a'n uni.t5ual finding and
is not consistent with acidosis or a primary lung pathology causing the
tachypnoea. Ammonia is a respiratory stimulant which acts directly on the
brainstem and shouid be me.asu:red in any child with encephalopathy and a '
respiratory alkalosis. .
.
.
I

19. C: Renal tubular addosis


'

Metabolic acidosis in tt1e presence of a normal anion gap indicates


.
bicarbonate loss, rather than the presence of excess acids. The two potentiill
routes for loss are the gut and kidney with the former less likely in the
absence ofloose stool. Renal tubular acidosis is therefore the most likely
.. d_iag!l()Sis: Inborn errors of metabolism are unlikely to be the cause' of an
acidosis irrthe absence of eithera raised ladate or l<etosis.
-

'-

20.. B: Matemal plasma amino adds


Excess phenylalanine is teratogenic..uncontrolled maternal PKU can therefore
damage the unborn child with common adverse features including growth
retardation, microcephaly. mental retardation, and cardiac anomalies. The child
will be a carrier for PKU- but is unlikely to have PKU- because the risk of the
father being a carrier is I in 50 (unless he is related). Excluding maternal PKU as
the cause requires plasma phenylalanine measurement the mother. Urinary
amino acids are unreliable as they only reflect renal threshold, and therefore it
is possible to have levels sufticiently high in the blood to damage a fetus, but
normal in the urine.
'

in

ESSENTIAL QUESTIONS FOR MRCPCH .

21. B: Acylcamitines
MCAD is suggested by the hypo ketotic hypoglycaemia and dicarboxylic aciduria.
Ketone production is severely limited as ketones are the ultimate product of the
blocked fat .oxidation pathway. Dicarboxylic acids are formed when fatty adds
undergo 0-oxidation in the microsomes. Intercurrent illness, especially
involving \'Omiting and diarrhoea, is a common precipitant of decompensation.
Diagnosis is made on plasma acylcarnitine analysis by an elevated
octanoylcarnitine (C8) level. Diagnosis would then be confirmed on genotyping
. for the common G985 mutation. Fibroblast studies are used to confirm longchain fat oxidation defects and are not usually necessary in MCAD diagnosis.

22. E:

~y-Sach~

disease

A cherry red spot is seen in Niemann-Pick type c (!'-:PC), Tay-sachs and


Sandhoff disease, all of which may present with developmental delay and
regression. Tay-Sachs is not associated with organomegaly, unlike NPC and
Sand hoff disease. Hyperacusis may be noted as being startled by sound from
an early age.

23. INVESTIGATIONS OF INBORN ERRORS

1.

A- Very long chain fatty acids .

Zellweger syndrome is a disorder of peroxisomal biogenesis. Elevated very


long chain fatty acids (VLCFAs) are indicative of peroxisomal disorders
because very long chain fat oxidation occurs within peroxisomes.
2. D - Plasma amino acids
MSUD is diagnosed by the elevation of the branched-chain amino acids.
(BCAAs) leucine, isoleucine, and valine: Plasma amino acids are preferred
because urinary amino acids reflect the renal threshold. and therefore the
abnormality ,will be first detected in the blood.
3. G- ilrate
Lesch-Nyhan syndrome is a. disorder of purine metabolism. Urate is grossly.
elevated due to the block in purine salvage, resulting in increased purine
... synthesis: Urate measurement a key initial investigation in purine disorders.

is

24. EYE SIGNS IN INBORN ERRORS


1. E- Pigmentary retinopathy . .
.
.
.
Pigmental)' retinopathy is associated with long chain hydroxy acyi-CoA
dehydrogenase (LCHAD) deficiency. It develops with tim~ despite good
control. The underlying mechanism is still unclear.
2.

G -

Cataract

. .

Cataracts are usually present at birth in galactosaemia. The cataracts are


transparent to begin with (oil drop cataracts) and are therefore easily missed
because the red reflex is preserved. If the condition remains undiagnosed, the
cataracts mature and become denser.' The cataracts can completely resol\'e if i'l
diet free from lactQSe and galactose is started early.
34

METABOLIC MEDICINE

3.

B- Corneal douding

Corneal. clouding is associated with all the mucopolysaccharidoses, with the:


exception of Hunter syndrome (MPS type II). The degree of clouding may be
very s_u~,tle and only r~vealedon-slit-lainp'examination.t.Morquio synd!ome
(MPS t)tpe IV) is typified by severe skelet~ll involvement (dysostosis multiplex),
with sparing of the central nervous system.

25. LIVER FAILURE

1. E - Galactosacmia
Galactosaemia classically presents towards the end of the first week. Milder
forms may present at a few months of age \"'ith renal tubular acidosis secondary
to proximal tubular damage. Hepatomegaly may be marked, even reaching as far
as the right iliac fossa: Clotting is always prolonged and bruising on present~tio"n
comnon. jaundice is present within the first days and deepens with time as liver
funct.on deteriorates. Cataracts are usually present from birth, but because of.
their initial transparent nature (oil drop) before they mature, they are easily
missed as the red reflex is visible through them. If the diagnosis is missed they
will become more obvious with time. Iftreatment is started early they will
resolve completely. Sepsis is very common in galactosaemia at presentation,
with special susceptibility to E. coli. Reducing substances in the urine may be
positive in liver failure, no matter what the cause is, and has led to misdiagnosis. Phenyllactate and 4-hydroxyphenyllactate are non-specifically raised
in liver impairment, and the normal orotic acid rules out OTC as the cause (it
does so in all three of these scenarios). The ammonia is slightly raised and is a
non-specific finding in a sick infant, particularly \vith liver impairment (the liver
being the home of the urea cycle). Fat oxidation defects can present with a Reye
syndrome-.like picture; however, one would expect hypoglycaemia to be a major
feature.
2.

C -l}'rosinaemia

This history would also be compatible with galactosaemia. However the


presence of succinylacetone.is pathognomqnic for classical (type I)
tyrosinaemia.

3.

I- Hereditary fructose intolerance

The clue to the diagnosis of hereditary fructose intolerance is the timing of


the presentation. Infants do not present prior to weaning when they are first
exposed to fructose. Infants classically vomit and may prove very difficult to
wean. lf fructose exposure persists, hepatomegaly develops with deteriorating
liver function and marked acidosis. Diagnosis is usually based on history and
when fulminant liver failure has developed, genotyping is the safest way of
securing the diagnosis because liver biopsy for enzymology is contraindicated
due to the coagulopathy. Fructosuria is a benign disorder characterised by the
excretion of fructose in the urine. Wilson disease is not a clinical entity in
children under the age of 5 years.

35

--

"'

I -

Grenville Fox

:I

. ...

~.<r:.t?;&f.,; .

:,": .;.

..-)7. . ,~;:;.:_~i:m

Multiple Choice Questions


,.

I.

The following are recognised causes of seizures' in the first week'


of life

A congenital myotonic dystrophy


trisomy21
conjugated hyperbilirubinaemia
non-ketotic hyperglycinaemia
maple syrup urine disease

0 B
0 c
0 D
0 E

'

'-

: 1-

2: The fol1owing are associated with pulmonary hypoP,Jasia

0
0
0
0
0

A
B
C
D
E

autosomal recessive polycystic kidney disease


unilateral renal agenesis

rupture of membranes at 20 weeks' gestation


tracheal atresia
polyhydramnios

. -.

3. The riskof surfactant deficiency is increased in

0
0
0

A
B
C
D
E

the second of twins


maternal opiate use
,.,
babies born at 38 weeks compared.to 40week:s
meconium aspiration syndrome
pulm~nary haemorrhage

4. The fol1owing statements are true of,reproductive function

0
0
0

A ovaries develop from the primitive gonad due to the influence of the X
chromosome
B mullerian inhibition factor is produced by the testis _
C the uterus and fallopian tubes develop in the absence of mul!eriim
inhibition factor

p male pseudohermaphrodites have XX karyotype .


E 21-hydroxylase deficiency causes female \1rilisation

37

.-.'

. j

ESSENT:.:..L Q...:ESTIONS FOR i"lRCPCH

5. Congenital diaphragmatic h~mia

!aJ

1 ...

{/)

OJ

:s

:o-

0
0
0
0

..

A occurs most commonly due to failure of the retrosternal part of the


septum transversum to form
B is more common on the left side
,
,
C is associated with karyotype abnormalities in approximately 60% of
cases
D is associated with malrotation of the gut
E causes biiateral pulmonary hypoplasia

6. Neonatal iisterios!s ,

0 A is caused by Gram-negative cocci


0- 8-is usually a nosocomial infection'
0

0
0

.. ~
'
C is associated with maternal contact with cats
D should be treated with third generation cephalosporins .
E has a good outcome if treated with appropriat~ antibiotics ,

7. Polyhydramnios is associated with

0 A
0 B
0 c
0

placental insufficiency
pre-term birth
cleft lip an(fpalate ' '
D Mobius syndrome
E duodenal atresia

'
}

'

.,

;: .~

:,.
l

'

,,

~1

8. Haemorrhagic disease of the newborn (vitamin K.deficlency

bleeding)

.. ,

0
0

A is more common in formula-fed infants


B is more likely to occur in breast-fed infants if the mother is being' T
treated \\ith heparin
0 C usually presents with petechiae
0 D lead:; ~6 prolongation of the activated partial thromboplastin time
0 E can be p~evented by treating moJhe(Swith \itamin K supplements ..
- - ..... --antenataily

9. The following are recognised causes of conjugated


hyperbilirubinaemia in the newborn

0
0
0
0
0

38

A
B
C
D
E

intrahepatic biliary hypoplasia


polycythaemia
galactosaemia
tyrosinaemia
~-thalassaemia

,'

'

-,-----
..
''t

NEONATOLOGY

'

_,

t:;;

10. The following are risk factors for periventricular leucomalacla

0
0
0
0
0

A
B
C
D
E

(I)

A
B
C
D
E

consists of two a and t\vo -y chains


is approximately 80% of the total haemoglobin at term
causes the oxyhaemoglobin dissociation curve to be shifted to the right
is undetectable in adults
is unaffected by sickle cell disease

12., lhe following increase the risk of vertical transmission of


human immunodeficiency virus

0
0
0
0
0

A
B
C
D
E

high maternal CD4 count


twin pregnancy
prolonged rupture of _membranes
pre-term birth
'

Caesarian section

13. The following are recognised causes of symmetrical


intrauterine growth restriction

0
0
0
0
0

A
B
C
D
E

trisomy 18
altitude
maternal smoking
fetal alcohol syndrome
maternal antiphospholipid syndrome

14. The following is true ofErb's palsy

0 A
0 B
0 C
0 D
0 E

the C8 toTl nerve roots are usually involved


the grasp reflex is usually preserved
the Mora reflex is reduced
denervation of the deltoid muscle occurs
full recovery occurs in the majority of cases within 6 months

15. The following are recognised causes of hyperchloraemia in the


newborn

0
0
0
0
0

11. Fetal haemoglobin

0
0
0
0
0

i.C"

!c
:. m

maternal opiate use


maternal cocaine use
chorioamnionitis
antepartum haemorrhage
hyperbilirubinaemia_

A
B
C
D
E

total parenteral nutrition


treatment with loop diuretics
fluid resuscitation \Vit~ normal saline
proximal renal tubular acidosis
nephrogenic diabetes insipidus

39

ESSENTIAL QUESTIONS FOR MRCPCH

.en

IC

io

16. The following are recognised causes of hypocalcaemia in the


newborn

0
0
0
0
0

A
B
C
D
E

maternal hypoparathyroidism
maternal diabetes
Di George syndrome
hypermagnesaemia
hypophosphataemia

,.

1 7. The following statements about twin pregnancy are tru~'!

0
0
0
0
0

A twins with different blood groups are dizygous . . f


B dichorionic t:vvins are always dizygous

C dichorionic placentas always haye two amnions


D assisted conception .has led to a recent increased rate of monozygous
twin pregnancy
E perinatal mortality is higher in dizygotic than in monozygotic t\vin
pregnancy

18. The following statements about nutrition in thenew~om are

true

0
0
0
0
0

.;,

A energy requirements are higher in infants on parenteral nutrition than

'
infants fed enterally
B the predominant protein of human milk is casein
C approximately 50% of energy ir. human milk is fat-derived
D glucose is the predominant carbohydrate source in human milk
E the sodium content of human milk is inadequate forvery-lowbirthweight infants

..
t..,.

'

'.

40

NEONATOLOGY

"Best of Five Questions


19. A baby is born by spontaneous, vaginal delivery at 35+ 6 weeks'
gestation following rupture of membranes 30 hours before
birth. The mother was well and not treated with antibiotics ~'
during labour. A vaginal swab was taken on admission. The
babywaswell and the mother wishes to be.discharged home as
soon as possible. The MOST appropriate course of action for
the baby at this time is which of the following?

0
0
0
0

A
B
C
D

Discharge home
Ob$erve in hospital for 48 hours
Await result of vaginal swab and'treat the baby according. to this
Perform an infection screen and treat with intravenous antibiotics if
results suggestive of sepsis
E Perform an infection screen and treat with intravenous antibiotics for
at least 48 hours pending results

20. A well 3-week-'old baby boJ;l');Jat term is referred to hospital with


a discharging,um])ilk\2-s. The,co:r:.d separated at 10 days_ and.
thereis,no.,perl-umbilical swelling:or erythema. There is~a
small red. mass at the site of cord separation, which is
dischar:g,ing;asmall amount ofyellow;fluid. The GP hacttakena.
swab; of tbis wtrich grew Staphylococcus. eyidermldis. The MOST
appropriate c;Qt.!!~e ~! ;;;ct!c:n is whi'dlt of the. fOllowing?
0 1). Reassure parents and review in l-2 weei'

0
0

B neat with oral antibiotics


neat with intravenous antibiotics
D Arrange an abdominal ultrasound scan
E Refer to a paediatric surgeon

21. A l_,C::I"Il}_b_a_Eyp~_s~_!l_!~!-_!_with_y.!Y-~r~ re~piratory failure, which


. . did not respond to initial resuscitative efforts, resuh:ing-in the
baby's death at 20 minutes of age. Antenatal ultrasound scans
had shown anhydramnios and I;uge fetal kidneys with
numerous large fluid-filled cysts. The liver appeared normal
and no other anomalies were seen. Both parents had a normal
renal ultrasound scan. What is the MOST likely diagnosis?

0
0

0
0
0

A
B
C
D
E

Autosomal recessive polycystic kidney disease


Autosomal dominant polycystic kidney disease
Posterior urethral valves
Bilateral multicystic dysplastic kidneys
Meckel-Gruber syndrome

41

.o
:::1
fl)

>
~\

~ '

rten
i.
t

c .'
.

~0

--...,en
I

CIJ

::s
tT

ESSENTIAL QUESTIONS FOR MRCPCH

22. A well, breast-feeding term infant presented with jaundice at 36


-hours of age.'The serum bilirubin was 286 !J.mo)JL, direct
Coombs test negative, blood film showed spherocytes and
reticulocytes, the baby's blood" group was A rhesus negative and
<mother's blood group 0 Rhesus negative. Which of the

following is the MOST likely diagnosis? ' .

0
0
0
0

A
B
c
D

Rhesus haemolytic disease


ABO incompatibility
Hereditary spherocytosis
Physiological jaundice
E Breast milk-associated jaundice

23. A woman develops chickenpox 2 days before giving birth to a


healthy term baby. Which ofthe following isthe.treatment of
choice?

A No treatment necessary for mother or baby

B Treat mother with aciclovir


Treat mother with varicella zoster immune globulin only
D Treat baby with varicella zoster immune globulin only
E Treat baby with varicella zoster immune globulin and aciclovir if
chickenpox develops

0 .c

24. A term baby born after spontaneous vaginal delivery is noted to


be pale. Full blood count shows hemoglobin 5.4 g!dL; white cell
count IE. I x 109/L; platelets 286 x 10'~/L. The b!c:Y.! !i!m !s
normal. The baby remalns well and is transfused with packed
red blood cells. A cra.~tial ultrasound scan is normal. In order to
establish the ca'!6e of the anaemia, the NExT investigation
should be ~;hich one of the following?

A ~.i,ciominal ultrasound scan

B coagulation screen
Apt's test

Kleihauer test
Bone marrow biopsy

0 c
0 D
0 E

..

..

25. A girl infant has profound hypotonia and required intubation


and positive pressure ventilation immediately after birth. She
wasbom at term following a pregnancy complicated by
polyhydramnios andreduced fetal movements. The' chest X-ray
shows small, but clear, lung fields. What is the MOST likely
diagnosis?
. .

0
0
0
42

A Trisomy 21
B Neonatal encephalopathy
c Congenital myotonic dystrophy

NEONATOLOGY

0
0

D Cervical spinal injury


E Spinal muscular atrophy '

26. A 7-day-old tenn boy is admitted to hospital with bruising and


bleeding from the umbilical stump. He had been born at home
and was not given vitamin K after birth. The full blood count is
normal, but prothrombin time and activated partiai .
thromboplastin time are elevated. What is the BEST .treatment
for the baby? .

.
;( ,r
.'

0 A
0 B
0 C
0 D
0 .E

Ho treatment
Intramuscular vitamin Konly
. ,
Intravenous vitamin K only
Intravenous vitamin K plus fresh frozen plasma
Intramuscular
vitamin
K plus fresh froieh plasma
' '
_,.
'

'

'

~'

:s

r,
....

'

27~

A pre-tenn iruant born at 25 weeks' gestation is now 36 weeks


corrected gestational age and is screened for retinopathy of
prematurity. Both eyei's are found to have;stage 2 retinopathy of
prematurity. Which ONE of the following should you tell the
pare~ts about the baby?

A NO-further screening
treatment is required and the visual prognosis
is good
B No further screening or treatment is required but spectacles are likely
to be required later
C No further treatment is required at this stage buHurther screening is
necessary and the visual prognosis is likely to be good
D neatment is required with laser therapy or cryotherapy
E There are retinal detachments and visual prognosis is pooP .

0
0
0
0

or

'.

43

ESSENTIAL QUESTIONS FOR MRCPCH

Extended Matching Questions

tt c~. :

28. Theme: Patterns of dysmorphology and congenital malformations

I> "olif'

o~t.
' ';'

-:::. "B

c
D
E
F
G
H

<

V~CTERL ~ssoci~tion

CHARGE association
1risomy 1i '
Trisomy 18
22q deletion
Noonan syn'drome
Fetal alcohol syndrome
Goldenhar syndrome
Rubinstein-laybi syndrome
Congenital rubella

For each of the following case scenarios select the mosr likely diagnosis from
the list abOle. Each option 'nwy be used once, more: than once, or not at all. .

0
0
0

1. Term baby, birth weight 3.2 kg, facial dysmorphism with cleft lip and
palate, heart murmur.
.. .
2. -Term baby, birth weight 3. 9 kg, facial asymmetry with a~normafleft
pinna. Hemivertebrae in cervical and thor'!cicspine. Heart murmur.
3. Baby born at 3~ ~eeks, bir~l) :veight [OQ g .. J"!li.f~~S~P.~aly,p~.f!JRiex
,, congenital heart .disease. congenital,diaphragmatic hernia, facial

dysmorphism, abnormal fingers.

29. Theme: Neonatal jaundice


A
B
C
D
E
F
G.
H
I

Physiological jaundice
G6PD deficiency
ABO incompatibility
Crigler-Najjar syndrome
Congenital hypothyroidism
Hereditary spherocytosis
Rhesus incompatibility
Gilbert syndrome
Biliary atresia
Galactosaemia

J
For each of the following case scenarios select the most likely diagnosis from
the list above. Each option may be used once, more than once, or not at all.

44

1. A term, boy baby presented at 5 days of age, unwell with jaundice,


vomiting, poor feeding and hypotonia. Initial investigation results
were as follows: total bilirubin 660 1-Lmoi/L (conjugated 181-LmoL'l);
baby's blood group 0 Rhesus negative; mother's blood group 0
Rhesus positive; direct Coombs test negative; haemoglobin 8. 7 gldL;
blood film: reticulocytes + +; thyroid stimufating hormone 4.1 mUlL.

NEONATOLOGY

2. A well, 4-day-old term baby girl presents with jaundice at 4 days of


age. Initial investigation results were as follows: total bilirubin 310
JLmOIJL'(i::onjugated 8 JLmOlJL);,baby's blood group A Rhesus negative;
mother's blood group 0 Rhesus positive; direct Coombs test negative;
haemoglobin 14,7 gtdL; blood film: few spherocytes and reticulocytes;
thyroid stimulating hormone 2.3 mUlL.
3. A well 16-day-old girl. born at 35 weeks' gestation, is investigated for
jaundice. Initial investigation results were as follows: total bilirubin
180 Jl.moi/L (conjugated 6 f.Lmol/L); baby's blood group 0 Rhesus
positive; mother's blood group B rhesus positive; direct Coombs test
negative; haemoglobin 13.5 gJdL; blood film: normal; thyroid,
stimulating hormone 4.9 tnU/L

30. Theme: Neonatal sepsis


A
B
C
D
E
F
G
H
I

Flucloxacillin
Vancomycin.
Amoxicillin
Benzylpenicillin
Gentamicin
Erythromycin
Pyrimethamine
Cefotaxime
Amoxidllin and gentamicin
Benzylpenicillin and gentamicin

For each of the following cases select the most appropriate antibiotic. or
antibiotic combination from the list above. Each option may be used once,.
more. than once, or not at all. .

0
0

1. A pre-term infant. unwell with increasing incidence of apnoea and


brady:cardia. Blood culture grows Staphylococcus epidennidis.
2. A baby born at 34 weeks' gestation, following spontaneous rupture of
membranes with discoloured amniotic fluid, in poor condition \vith a

pustu'lar rash.
3. A 26-week gestation baby ventilated for chronic lung disease is
investigated for in(.Teasing ventilatory requirement and increasing
quantities of yellow secretions frorri the endotracheal tube. Culture of
these grows Chlamydia spp.

45

I. Seizures ,in Ute first week of life

Answers: DE

,,

{'

Congehitat inyoionic dystroptiy typically presents with hypotonia and


sometimes with respiratory failure. Trisomy 21 presentswith typical
.
dysmorphic features and hypotonia. Profound unconjug~ted
hyperbilirubinaemia may cause kernicterus, of which seizures may be~
feature, whereas conj':,lgated hjperbilirubinaemia is no(a'tat.ise of seizures.
Non-ketotic hyperglycinaemia may present with increased fetal movements,
which are actually in-utero seizures and neonatal encephalopathy presenting
as hypotonia and seizures from birth. Maple syrup urine disease (MSUD).'in
its most severe form, presents as collapse v.'ith encephalopathy, "including
.. :f:
'
seizures. usually from the third day of life.
.,

2. Pulmonary hypoplasia

Answers: A c E

.i'

r~

Primary pulmonary hypoplasia is rare. Secondary hypoplasia may be due to


decreased amniotic fluid volume during the second trimester,' which is most
corriinonly due to rupture of membranes prior to 24 \veeks' gestation or
de::reased fetal urine production. Decreased fetal urine production resuits
from bilateral renal.disease, including bilateral renal agenesis ('classical'
Potter syndrome), bilateral cystic dysplastic kidneys, polycy~.tic kidney disease
.(usually autosomal recessive but rarely autosomal dominant) or obstructive
~ . uropathy affecting both kidneys (eg posterior urethral valves). Neuromuscular
disease such as congenital myotonic dystrophy and congenital structural
myopathies lead to polyhydramnios due to decreasea swalloWing. and also
pulmonary hypoplasia due to decreased fetal breathing movements which are
essentialfornormallung development. nacheal atresia.prevents fluid leaving
. the fetal aim;ays and enhances lung growth. Fetal surgery with tracheal
plugging has been used successfully to promote lung growth in congenital
diaphragmatic hernia, which along with other thoracic space occupying .
lesions is al~ associated with pulmonary hypoplasia.

46

NEONATOLOGY

3. Surfactant deficiency
Answers: A c DE
The risk of surfactant deficiency is increased by prematurity. with an incidence
of approximately 9096 at 26 weeks' gestation, 55% at 32 weeks, 396 at 38 weeks
and < 196 at 40 weeks. The incidence and severity is reduced by maternal
antenatal steroids (dexamethasone and betamethasone), maternal opiates. in
females and small for gestational age babies. The incidence and severity is
increased in males. sepsis, babies of mothers with diabetes, those delivered by
elective Caesarian section and \\1th a strong family history of surfactant
deficiency. Meconium aspiration syndrome and pulmonary haemorrhage has
been shown to inactivate the action of endogenous surfactant and exogenous
surfactant therapy has been shown to improve outcome in both conditions.

4. Reproductive function
Ans11;ers: 8 C E
The SR'Y gene on theY chromosome influences the primitive gonads to form
testes. The testes secrete' 11.1ullerian inhibition factor (MIF) which results in
regression of the uterus and fallopian tubes. Male pseudohermaphrodites have
male karyotype (XY) but are under-virilised, most commonly due to androgen
insensitivity. The most common cause of female pseudohermaphroditism (XX
karyotype) is congenital adrenal hyperplasia due to 21-hydroxylase deficiency,
which leads to increased levels of J7a-hydrox:yprogesterone.

5. Congenital diaphragmatic hernia


Answers: 8 D E
The Bochdalek type of congenital diaphragmatic hernia is the most common.
This arises due to failure of closure of the posterolateral canals within the
septum transversum. Rarer anterior or Morgagni diaphragmatic hernia arises
when the retrosternal part of the septum transversum fails to form.
Congenital diaphragmatic hernia is more common on the left side
(iipproximafely 8596-of
K.aryotyPe'anomalies occur in approximately
30% of cases. Other congenital malformations include. congenital heart
disease and malrotation of the intestine.

case's).

6. Neonatal listeriosis
Answer: Allfalse
Listeriosis is caused by the Gram-positive coccobacillus Listeria
monocytogenes. Neonatal infection is usually acquired in utero after the mother
has become infected from food. often from un-pasteurised dairy products or
pate. Maternal contact \'-1th iarm animals is thought to be a risk factor also.
Listeria !soften resistant to cephalosporins and amoxicillin is the antibiotic
treatment of choice. Outcome after neonatal listeriosis is variable with a
significant mortality (approximately I 096) and risk of long-term neuro-disability.
47

-.,.I

ESSENTIAL 9UESTIONS FOR MRCPCH

7. Polyhydramnios
Answers: B DE

Polyhydramnios may.be caused by maternal diabetes, karyotype


abnormalities, tWin-to-tvvin transfusion syndrome (polyhydramnios
oligohydramnios syndrome), neuromuscular disorders (congenital myotonic
dystrophy. congenital myopathies. spinal muscular atrophy and Mobius

syndrome), oesophageal atresia (and other, high intestinal atresias) and


.congenital diaphragmatic hernia. Some caseS;' particularly mild
polyhydramnios, may be unexplained.
8~

Haemoirhagic disease of the newborn


Answers: DE

Haemorrhagic disease of the newborn is caused by vitamin K deficiency.


Vitamin K crosses the placenta poorly and breast milk contains low levels.
Maternal liver disease and anticonvulsants (phenytoin and phenobarbitone)
increase the risk, which can be reduced by supplementing mothers with
vitamiJ1 K antenatally. Early-onset haemorfhagic disease of the newborn
presents within th6dirsl!48 hours. usually with gastrointestinal bleeding.
Classic disease presents in the first week of life \vith bruising, bleeding from
the umbilicat stump. and gastrointestinal tract bleeding. Intracranial
haemorrhage may al~o.()ccur but this is more common in late-onset
haemorrhagic disease of the newborn, which occurs up to 6 months of age.
The prothrombin time and activated partial thromboplastin time are
prolonged as both the intrinsic and extrinsic pathw<iys are affected. Maternal
heparin therapy does not contraindicate breast-feeding.
9. Conjugated.,hyperbilirubinaetnia in the
. newborn
Answers: A c D
Causes of conjugated hyperbilirubinaemia in the newborn are:
ChoJestasis secondary to total parenteral nutrition
!.Jntrautedne infection (hepatitis B, toxoplasmosis, rubella, cytomegalovirus,
herpes simplex. coxsackie virus. adenovirus, syphilis)
Bacterial sepsis
e Severe haemolysis
Biliary atresia
Intrahepatic biliary hypoplasia
Choledochal cyst
. ,.
Metabolic causes (ex !-antitrypsin deficiency, galactosaemia, tyrosinaemia,
Gaucher's disease and other storage diseases, .Rotor syndrome.
Dubin-:Johnson syndrome)
. Cystic fibrosis.

48

NEONATOLOGY

1o. Periventricular leucomalacia


Answe.r:s:BCD

Periventricular leucomalacia is caused by ischaemia or infarction, but


inflammation may also be part of the pathophysiological process. Events
leading to periventricular leucomalacia may be antenatal, perinatal, or
postnatal, and include maternal trauma. maternal severe illness. antepartum
haemorrhage, chorioamnionitis, cord prolapse; hypotension, sever'ehypoxia
. and hypocarbia due to hyperventilation. Maternal cocaine use and pre-term
birth are other risk factors. Maternal opiate use is associated with symptoms
of withdrawal in neonates but this is not a risk factor for periventricular .
leucomalacia or other intracranial pathology. Severe hyperbilirubinaemiamay
lead to kernicterus. which predominantly affects the ba$al.ganglia and
leucomalacia.
auditory
pathways. It is not. associated with periventricular
,
.
~

11. Fetal haemoglobin


Answe.r:s:ABE

Fetal haemoglobin consists of two a and two -y chains. whereas adult


haemoglobin has two a and two ~ chains. The-y chains cause fetal
haemoglobin to have reduced binding to 2,3-diphosphoglycerate, which shifts
the oxyhaemoglobin dissociatipn curve to the left. Fetal red blood cells
therefore have a higher affinity for oxygen. Approximately 80% of .
haemoglobin is fetal haemoglobin at term. This falls to less than 10% by I
year of age and to less than 1-2% in adults. Sickle cell disease is caused by a:
mutation resulting in a substitution of valine for glutamic acid on the f) chain.
Therefore fetal haemoglobin is unaffected by sickle cell disease. '

12. Vertical transmission of human

immunodefici~ncy

virus

Answers: CD

Approximatelyone-third of vertically transmitted cases of human


immunodeficiency virus are transmitted across the placenta and two-thirds
during birth. The risk of vertical transmission is increased by low m?terl}al
CD4 count, high maternal viral load, presence ofp24 antigen in the mother.
rupture o( membranes greater than 4 hours before birth. pre-term birth,
vaginal delivery, and no maternal anti-retroviral treatment. With maternal
anti-retroviral treatment during the third trimester and to the baby for the first
I? weeks of life, without these risk factors. vertical transmission can be
reduced to less than 1-.2%.

13. Intrauterine growth restriction


Answers: AD

Causes of symmetrical intrauterine growth restriction include chromosomal


anomalies (typically trisomy 18). intrauterine infections {cytomegalovirus,
toxoplasmosis. rubella} and fetal alcohol syndrome. Asymmetrical intrauterine
49

ESSENTIAL QUESTIONS FOR MRCPCH

growth restriction is usually due '.o placental insuffidency and is much more
common. Maternal conditions such as hypertension. pre-eclampsia, lupus.
antiphospholipid syndrome/maternal renal disease and opiate use are
associated With symmetrical intrauterine growth restriction: The 'donor tv,rin
in twin-to-tvvin transfusion syndrome is also at risk. Maternal smoking leads
to a mean 10% reduction in birth weight. Babies of mothers living at high
altitude also follow a similar pattern of asymmetrically reduced fetal gro\Vth.

14. Erb's palsy .


Answers: B CD

Erb's palsy occ~rs as a result of peripartum brachiai plexus injury in


approximately 1 in i000-2000 births.Risk factors include macrosomia,
instrumental delivery and shoulder dystocia. The C5 and C6 nerve roots are
most commonly damaged but, rarely, more extensi:e injury may involve nerve
roots from C4 to Tl. If C4 is involved, diaphragmati-:: palsy may occur, and if
Tl is involved, the small muscles of the hand are afiected, leading to loss of
the grasp reflex. The Moro reflex is reduced on the affected side. Most cases of
Erb's palsy have decreased abduction at the shoulder with internal rotation,
decreased elbow fleXion, and loss ofwrist extension. Only about20% fully
recover, although there is usually adequate function of the affected arm ..
Recovery is rapid during the first 6 months but maycontinue until IS months.
15. Hyperchloraemia in the newborn
Ansvvers: A CD E

TOt.;tl parenteral nutrition (TPN} may lead to hyperchloraemic acidosis in preterm infants. which can be reduced by substituting the sodium chloride in
TPN with sodium acetate. Use of normal saline for t1uid resuscitation, flushes,
and in arterial lines may also lead to hyperchloraemia and metabolic acidosis.
Other causes of hyperchloraemia include proximal or distal renal tubular
acidosis and nephrogenic diabetes insipidus. The latter may be inherited as an
X-Jinked condition and leads to insensitivity of the distal renal tubule to
antidiuretic hormone (ADH). This leads to polyuria with dilute urine which
may cause polyhydramnios antenatally and dehydration postnatally, with
raised plasm~ sodium, chloride and creatinine.

16. Hypocalcaemia in the newborn


Answers: BC
Early-onset neonatal hypocalcaemia occurs seconcary to prematurity,
maternal diabetes and birth depression. Late-onset (after .3-5 days)
hypocalcaemia may be due to hypoparathyroidism ttransient idiopathic or
congenital, as in Di George syndrome, maternal h~?erparathyroidism and '
hypomagne.saemia), vitamin D deficiency (maternal vitamin D deficiency, liver
and renal disease), rapid postnatal growth, hyperptosphataemia (impairs

so

NEONATOLOGY

conversion of vitamin D to active form), loop diuretics, alkalosis and


hypoalbuminaemia.

17. 1\vin pregnancy


Answers: A c
Monozygotic twin pregnancy occurs due to single ovulation with subsequent
splitting. Zygosity may be determined after birth according to the sex and
blood group of the twins, 1Nhich are the same in monozygous (identical) twins.
Chorionicity can be determined antenatally by ultrasound examination of the
placenta and membranes. Monochorionic placentas are only found in
monozy~ous twins, but approximately 30% ofmonozygous twin pregnancies
are dichvrionic. Only 4% of monozygotic t\vin pregnancies are monochorionic
monoamniotic; Dizygotic t\vins always have dichorionic, diamniotic placentas.
Techniques of assisted conception have increased the incidence of dizygotic
twin pregnancy, whereas the incidence of monozygotic twin pregnancy has
remained almost constant over recent years. Perinatal morta:lity is almost three
times higher in monozygotic pregnancy than in dizygotic twin pregnancy, due
to the increased risk of twin-to-twin transfusion syndrome.

18. Nutrition in the newborn


Answers: C E
Enteral feeding requires energy, therefore infants on parenteral nutrition
generally have lower energy requirements. Human milk is whey-predominant
(whey to casein ratio of approximately 3 : 2) whereas cow's milk is caseinpredominant (whey to casein ratio of approximately 1 : 4). This is important
for the provision of essential amino acids. Energy provision in human milk is
by fat (approximately 50%), carbohydrate, predominantly as lactose
(approximately 400--6), and protein (approximately 10%). SOdium requirement of
pre-term infants is as high as 5-6 mmol/kg,'day. Human milk contains 0. 7
mmol/1 00 ml of sodium and additional supplements are therefore required.

-- -1-9.-E: Perform an infectionscreen and treat-with intravenous

antibiotics for at least 48 hours pending results


Risk factors for neonatal sepsis include rupture of membranes greater than
12-2.4 hours. intrapartum.maternal pyrexia (> 38~C), fetal tachycardia.
chorioamnionitis. pre~ term birth and maternal colonisation with group B
Streptococclls. In this case there are two risk factors: prolonged rupture of
membranes. and pre-term birth. Each risk factor represents approximately
I -2% risk of infection to the baby. Ideally the mother should have received
intra-partum antibiotics and. if given greater than 4 hours prior to birth. the
baby could have been considered adequately treated in the absence.of other
risk factors. In the absence ofthis; most guidelines suggest. investigating well
babies for sepsis and treating if there are two or more risk factors.

Sl

.-,

.;ii
!

ESSENTI;?-.L QUESTIONS :=OR i"1KCPCH

'!

20. A: Reassure pc:trents


aHd review
,
. ; in 1-2 weeks
;

'

The most likely diagnosis is umbilical granuloma. This is much more common
than other possibilities. such as patent urachus and patent
omphalomesenteric duct which may present with discharge of urine or
meconium. Umbilical granulomas are usually red and may discharge serous
fluid or pus. Treatment with antibiotics is only necessary if omphalitis is a
possibility, with peri-umbilical erythema in an unwell baby. Normal skin !lora,
such as Staphylococcus epidermidis, do not require antibiotic treatment.
Cauterisation with silver nitrate may be considert>d but most umbilical
granulomata are self-resolving within a few weeks.
'

21. D: Bilateral multicystic dysplastickidneys


'

The pathophysiology in this case is severefetal renal dysfunction, from as


early as the second trimester, leading to anuria and subsequent
anhydramnios. resulting in severe pulmonary hypoplasia. All of thelisted
diagnoses may do this. Autos~mal recessive (infantile) polycystic kidney
disease usually leads to large echo-bright kidneys on antenatal ultrasound
scan, rather than macrocysts, and congenital hepatic fibrosis may be found. _
Autosomal dominant (adult) "polycystic kidney disease usually presents later
in life but may rarely-be a cause of microcystic changes within the feti:ll
kidneys. Posterior urethrai valves are associated with bladdt;rand re,_nal tract
dilatation. Meckel-Gruber syndrome is another cause of fetal renalcysts, but
other abnormalities including encephalocoele, polydactyly, cardiac disease
and liver disease are also found on antenG~tal ultrasound scan.
22. B: ABO incompatibility

The bilirubin level at this age is too high for physiological or breast milkassociated jaundice, which both tend to present later. Rhesus haemolytic
disease is not possible as the baby' is rhesus negati\e. Spherocytes and
- reticulocytes are seen commonly on the blood film in ABO incompatibility.
The direct Coombs test is sometimes negative in ABO incompatibility,
reflecting a low concentration of antibody on red blood celis.
23. E: Treat baby with varicella.-:zoster immune globulin and

aciclovir if chickenpox develops


__ Perinatal chickenpox is a serious and potentially life-threatening disease.
Approximately 25% of babies will become infected if their mother develops
chickenpox in the peri-partum period. Maternal chickenpox within 5 days of
birth presents the greatest risk. as there is insufficient time for transplacental
transmission of antibodies. Administration of varicella zoster immune
. globulin {VZIG) within 72 hours of exposure has been shown to prevent and
attenuate chickenpox. Development of symptoms in. the baby maybe as early
as 3 days after the onset of the maternal rash, but may be up to 16 days.
Mortality of untreated cases ofperinatal chickenpox has been reported to be
52 .

.
I

_ _I

NEONATOLOGY

as high as 30%, therefore treatment with aciclovir is recommended if signs of.


this develop.

24. D: Kleihauer test


The most likely diagnosis is fetomaternal bleed leading to anaemia. This can
be acute or chronic. The Kleihauer (Kleihauer-Betke) test is on maternal blood
and detects the presence of fetal red blood cells, thus confirming or refuting
the diagnosis of a fetomaterna! bleed. The Apt's test is used ifthere is
gastrointestinal bleeding to distinguish between maternal and baby's blood
as the cause of this.

25. C: Congenital myotonic dystrophy


All options may cause neonatal hypotonia. Reduced fetal movements and
polyhydramnios (due to reduced fetal swallowing) suggest a severe
neuromuscular disorder. The S:-;)aJllung fields and \'entilatory requirement
suggest pulmonary hypoplasia which may be secondary to severe
neuromuscular disorders, which have an early in-utero onset (due to reduced
fetal breathing movements). This makes neonatal encephalopathy and
cervical spine injury unlikely be-=ause both of these nearly always occur
secondary to an insult close to the time of birth. Although cases of spinal
muscular atrophy with fetal onset have been described, they are extremely
rare. Congenital myotonic dystrophy is therefore the best answer.

26. D: Intravenous vitamin K plus fresh frozen plasma


The diagnosis is haemorrhagic disease of the newborn (vitamin K-deficiency
bleeding). This usually presents with gastrointestinal bleeding, bleeding from
the umbilical stump and bruising after 2-7 days. Breast-fed babies not
receiving vitamin K prophylaxis after birth are most at risk. The prothrombin
time is elevated, as is the acti\ated partial thromboplastin time to a lesser
extent. With active bleeding it is important to give intravenous vitamin K .
because this will correct the deficiency quicker than the intramuscular route.
Fresh frozen plasma should also be given because.even intravenous vitamin K
does not correct the clotting times quickly enough.

2.7. C: No further treatment is required at this stage but further


screening is necessary and the yisual prognosis is likely to be
good

The main risk factors for retino;:>athy of prematurity (ROP) are prematurity
and hyperoxia. All pre-term infants born before 32 weeks' gestation and/or
1500 g birth weight should be screened from 6 weeks of age. Staging
determines treatment and prognosis and reflects the degree of
neovascularlsation. ROP stages 1 and 2 usually regress but continued
screening is required until \'as.:ularisation is complete. Visual prognosis is
good but there is an increased risk of refractive errors and squint. Stage 3 ROP
53

Esc::c:NTIAL QUESTIONS FOR MRCPCH

usually' regresses but requires close observation until vascularisation is


complete as treatment with laser or cryotherapy may be required. There is a
high incidence of refractive errors and squint. ROP stages 4 and 5 involve subJ
total
and total
retinal .detachment, usually with. poor Visual prognosis.

.
.
1

28. PATrERNS OF DYSMORPHOLOGY AND CONGENITAL


Miu.FORMATIONS .

l. c- Trisomy 13
Features of trlsomy13 include holoprosencephaly, scalp defects,
microophthalmia, cleft lip and palate, polydactyly, renal'atinormalities. and
congenital heart disease.

2.

H - Goldenhar syndrome

Features of Goldenhar syndrome include facial asymmetry with abnormal


ears. deafness, epibulbar dermoid cysts, vertebral abnormalities. arid
congenital heart disease.
'
3. D - Trisomy 18
Features of trisomy 18 include symmetrical intrauterine growth restriction,
facial dysmotphism, clinodactyly, nail hypoplasia, rockerbottom feet,
congenital heart disease. diaphragmatic hernia, and renal abnormalities.
VACTERL association has the follo\Ving features: vertebral abnormalities,
anal atresia with or without fistula, cardiac malformations, tracheooesophageal fistula, renal abnormalities, and limb abnormalities. CHARGE
association typically has the following features: colobomas, heart
malformations. atresia of choanae, retardation of growth and development,
genital hypoplasia in the male, and ear abnormalities. The 22q deletion (Di
George or velocardiofacial syndrome) includes hypocalcaemia due to
parathyrbid gland hypoplasia, thymus hypoplasia (leading to increased
susceptibility to infection due toT lymphocyte deficiency), congenital heart
disease. and deft palate. Noonan syndrome consists of facial dysmorphism
(low-set ears, flat nasal bridge), a short webbed neck, and congenital heart
disease (pulmonary valve stenosis arid cardiomyo-pathy-commonest).

The features of fetal alcohol syndrome include prenatal and postnatal


growth failure, mid-face hypoplasia, flat nasal bridge, thin upper lip,
microcephaly, and congenital heart disease. Rubinstein-Thybi syndrome
consists of facial dysmorphism with hypertelorism and abnormal nose, with
broad medially deviated thumbs and big toes, and microcephaly.'ln congenital
rubella there is symmetrical intrauterine growth restriction, microphthalmia,
cataracts. hepatosplenomegaly, and congenital heart disease.

54

i'

NEONATOLOGY

29. NEONATAL JAUNDICE

I.

B - G6PD deficiency

The presenting signs suggest early kernicterus, made more likely by the
extremely high unconjugated hyperbilirubinaemia.The low haemoglobin and
presence of reticulocytes suggest severe haemolysis. This excludes
Crigler-Najjar syndrome; and Gilbert syndrome only leads to mild
unconjugated hyperbilirubinaemia. There are no apparent blood group
incompatibilities, thus making G6PD deficiency the most likely diagnosis,
which can be confirmed by biochemical assay.
2. C-ABO incompatibility
There is a moderately elevated unconjugated bilirubin and a set-up for ABO
incompatibility. The direct Coombs test !Tlay be positive or negative in ABO
incompatibility and the blood film often shows spherocytes and reticulocytes.
3. A- Physiological jaundice
Apart from the moderately raised unconjugated bilirubin, all investigations
are normal. It is not unusual for pre-term infants to remain jaundiced for 3
weeks.

30. NEONATAL SEPSIS


1. B - Vancomycin
Vancomycin is the antibiotic of choice for either methicillin-resistant
Staphylococcus aureus (MRSA} or Staphylococcus epidermidis infection.
Although someS. epidermidis is sensitive to flucloxacillin, treatment should
start before sensitivities are a\'ailable.
2. I - Amoxicillin and gentamicin
Congenital listeriosis is a strong possibility as a diagnosis. Li;;teria is often not
sensitive to cephalosporins. Amoxicillin may be 1;110re effective than benzyl
penicillin. To provide broad-spectrum antibiotic cover until results of blood
culture and sensitivity is available, gentamicin should also be given initially.
3. F - Erythromycin

Intravenous erythromycin is the treatment of choice for Chlamydia


pneumoiliaorconjunctivitis in the newborn: --

55

.,ro

C/)

''
-~.r

...
.

,. .':
~

..

,._

'.

'.

.. ll"

.,

,,
... ,

~" ..

,.,

Jim Hart and Chris Reid

. ~l~~~~~.~~:.::~-.-~'~1""7:'.\

~~t~~::n-.::~~~$~1-1~~-~~.r~;:~~~.

Multiple Choice Questions


1. Complications of nephrotic syndrome include

0 A
0 B
0 C
0 o
0 ,E

hyperlipjd~emia

malnutrition
primary pne.umococcal perito[litis
prolonged biet;ding
pulmonary embolism -

2. Features of glomerulonephritis include

0
0
0
0
0

A
B
C
D
E

hypotension
peripheral oedema
haematuria
hydronephrosis on ultrasound
dysuria

3. The following is true of acute post-streptococcal


glomerulonephritis

0
. 0-0
0
0

A it is the commonest cause of nephritis in children


B.. it is usual for-C3-to remain1owfor many months after the initial
nephritic illness
c it rarely occurs in children less than 5 years old
D hypertension, if present at presentation, is usually persistent
E the incidence of chronic renal insufficiency after acute post. streptococcal glomerulonephritisis less-than J%

4. With regard to immunoglobulin A (IgA) nephropathy r.

0
0
0
0

A
B
C
D
E

"

the serum IgA is usually low


it usually presents \\ith recurrent macroscopic haematuria
if it presents during childhood 40% develop end-stage renal failure
angiotensin-converting enzyme (ACE) inhibitors are contraindicated
it can be differentiated from Henoch-Schonlein purJ?ura nephritis on
the basis of the renal biopsy findings
57

ESSENTIAL QUESTIONS ;:"QR MRCPCH.

5. The followiri.g is true .of proteinuria


0 A. it requires no further investigation if it occurs during an acute febrile
illness
0 B oliguria increases the degree of proteinuria on dipstick testing
0 C urinary protein:creatinine ratio depends on how dilute or
concentrated the urine is
,
0 .D children with orthostatic proteinuria require long-term follow-up
0 E persistent proteinuria may precede chronic renal impairment in
patients with renal disease

6. With regard to proximal renal tubular acidosis


A it should be suspected when the urinary pH is < 5.3 in the face

0
0

B
C
D
E

of

systemic acidosis
urine pH can be acid ( < pH 5.3)
it can be caused by galactosaemia
it can be caused by cystinuria
it can occur as pan of Fanconi syndrome

7. The following is true of distal renal tubular acidosis

0
0

A nephrocalcinosis is a recognised complication


B there is impaired excretion of hydrogen ions
c it is associated with hyperkalaemia
D it can be secondary to obstructive uropathy
E it can present with failure to thrive

.;

..

.,.

8. The following is true of neonat~l renal function

0
0
0

A premature infants have the same glomerular filtration rate (in


1 L

m1Jkgtmin/ 1. 73m 2) as term infcimts


B glomerular filtration rate increases by 50-1 00% during the first ,.,eek
of life in well term babies
.. '"; . .
C. nephrogeriesis-is .complete .by. 36 weeks~.gestation..:. . ~~- .
D fractional excretion of sodium (the ratio of excreted to filtered sodium
expressed as a percentage) is commonly inappropriately high(> 1%)
in very low birth weight infants.

"
'
E plasma bicarbonate is lower in newborns than in older children
/
because the bicarbonate thresholdis lower in:newborns .

.9. The followiri.g suggest pre-renal, rather than intrinsic,. renal failure.
0 A urea of> 15mmoVL
.,~ ;'
0 B urinarysodium:<JOmmoi/L
~.
0 c urine output less than 1 inl/k.gthour
0 D hypotension
0 E' urinary osmolality of> 500 mosmoVL .
'{

58

NEPHROLOGY
.

1o. The following are associated With renal cysts '


0 A nephronophthisis

,. '
0 B chronic renal failure
0 c tuberous sclerosis'
0 o asphyxiating thoracic dystrophy
0 E renal dysplasia '

(""

r'~!

_ .

',

!1

. ~ ;j-'7;'\ -,.,.'"J

1~.

-.

:""-'

;,

..

#"

,.

'

"-

'-~-

"".

)..

l1. The following is true of autosomal recessive polycystic kidney.


1
~.;.
disease
.
.,
,.
,.
..J
0. 'A the gene for autosom<ll rec~ssive polycystic k.i<fney'disease is:found on
'
.
chromosome 6
..., 1. '
.... -- .,
(
.
0 B large renal cysts (> 2.mm) on aritenatal.l'enal ultrasOund Scilnning are
charactbrisiic' .''J . ' .' ,.
f
.... ,,
-' . l:
'
0 .~C, .. it-......
is always
associated
with some tsdegree
of,_ hepatic
fibrosis
,,.,
\_ ..
-,

.... " ' . '


,,_rl
.-'!
0 0 it is associated with cerebral aneurysms

'
'
0 E hypertension is an uncornmon'coinpiication .... '. I ' a ..
~~

'~--

.,

'

._

'

l.J~'"-~

)
12. The following is true of diuretics
~- -~.
A spironolactol!e exerts its potassium sparing effect b}/iricreaslng
.
PP,~~~!.~rn reabsorption in the proximal convoluted tubule
" l :
0 B loop diuretics cause hvpokalaemia by increasing distal tubular sodium
delivery
r ';
. r..., " ' '' .. : ;. I '"'
''i. f
0 c thiazide diuretics act on antidiuretic hormone receptors in the
collecting ducts
.,
0 0 loop d.iuretics can cause a metabolic alkalosis
)
0 E loop diuretics may potentially be nephrotoxic
.
--,_ ~:!
,4<,1
.. I .1
13. The following is true of renal homeostasis
P.: in c;~r, 'synd~ome re~i~iMels J;e~igh
~Jf~' "''J- :m ::
0 B in Conn syndrome excess aldosterone leads to hyperkalaerriia
0 --c renin increases blood pressure by converting angiotensin Ito .:,
angiotensin II
~. ~ .: '
; J ..~ "
0
patients
with
renal
artery
stenosis
typically
have
a
hypokalaerriic
0
alkalosis
. ' .,. .... t ;
0 E typically patients \Vith Addison's disease nave hyperkalaemia and
hyponatraemia

'

/>.

o' .

14. Characteristic-features of-Bartter' syndrome indude.


0 A increased urinaiy chloride
0 B hypokalaemia
. ' . !
"~0 C hyperchloraemia
~:
0 D metabolic acidosis
.: -,~, '1 >
' -.,. ,,, ..
0 E recurrent epi_s~des of<iehydration.

'
'

, .

59

ESSENTIAL QUESTIONS FOR MRCPCH

15. The following is true of vesicoureteric reflux

0
0
0
0
0

A inheritance is autosomal dominant

B can be excluded on renal ultrasound


does not cause nephropathy after S years of age
D 70% of grade II reflux will resolve by S years of age
E it can be demonstrated using a MAG 3 renogram .

16. The following are indications for dialysis in acute renal failure

0
0
0
0
0

A creatinine of> 700 Jl.moVL . .

.
'

.
B increasing hyperkalaemia not responsive to conservative treatment
c hyperammonaemia


D urine outputless than ~.SmVkg.thour
' '

E metabolic acidosis not controllable with sodium bicarbonate

17. Nephroealcihosis is a recognised complication of the following


0 A William syndrome
0 B steroids
0 c prematurity
0 D frusemide .
.,
0 E. captopril . ,
;t..

\,!

i:

'

\{

..

18. Hypertension can present with

0
0
0
0
0

A failure to thrive

,
B Vllth cranial nerve palsy
C petechiae
D proteinuria
E retinal haemorrhages
'

..

~ .. -

. !

i ... ,.

'

a:

,-

19. The following are recognised complications of chronic renal,


1
failure
. .
. .1
~.

'
"
.

.....

.,/ 'r

:t

.. ""'

O. A hypercalcaeJl1ia ,
0 B. iron deficiency anaemia
0 C ~secondary eJ;lU res is.
0 D delayed puberty
0 : E hypophosphataemia . , .

.,. : _ ~- -:.--:~- -::---..-- . -:::------------ . ----.. ~ ..

..

.. .,
,

' ... ~

,.

20. Posterior urethral valves

0
0
0
0
0
60

A are excluded by a normal renal. ultrasound on day J of life

~ ,_
B are rarely associated with chronic renal failure if sorrested .surgically
immediately after birth
C occur in both sexes
D are associated \\'ith polyhydramnios
E an intravenous urogram is the.investigation,of shoice

'

'

NEPHROLOGY

21. Glbmentlar filtration rate .,


0 A is expressed as a function of weight in children
0 B11 is overestiJ!1ated
by crea.tinine clearance because of tu.bular
.
-.
, .,..

o-

0
0

reabsq~p.tion

f.,

0
0
0

'

.~ ~.

-f'

,. __

...

the need for urine samples . ' ' '' '


I '
.
D may be reduced bY, a~ngioten?in converting enzyme inhibitorsbecause
they'cin.ise constriction of the efferent glomerular. arteriole ~
E reache~ the 'l?r~.a} matUre va}ue' '!Urin~ t~~ ,second year o~l:;e
>t<

"'

,.

.....

22. Di<ibetes insipidus

c dm be measured with a radioisotope usLngblood sampling without

'

o( cre~tinine

*" ' -

.,

. I""

.
" ., '
'
'

'-,
.
_,_ ...
A is always associated \liith abnormally low levels of antidiuretic
hormone
'

.
,
''
B is characterised by low plasma osmol~lity ~th inappropriately high
urinary osmolality
...
Ill .,
c is associate with'hyponatraemia
.
' p :~~~~estpe~ted,~fiit~fi!IY.pj ~yid r~str!eticirl'"'>'.
E 1s a cause of polyh)'dramntos
:~

.Y

#"

,.,.,..

~~.....

1,__.

.J~

..,...~-~#(11

... ~\

J_

..,

' .

'

..

Cl);,:i>

"*

...

'1

'
I'

. t
'-t

~-.

_...

'~'-.n

''
J

..

\.

. .. : . n ..c..

61

ESSENTIAL QUESTIONS FOR MRCPCH

Best of Five Questions


23. A 7-year-old boy present~; with a 10-day his~ory of purpura on
his arms and legs.. He ha!; had 3 days of pain and swelling of his
ankles and left knee but is ambulant. On examination he is
apyr~al,.his capillary reflll is 1 second, pulse 100 beats per
minute, blood pressure 105/80 mmHg. Urine dipstick shows
blood + + and protein + + +. Bloods on admis.slon revealed.
haemoglobin 132 g!L, white count 6.7 x 109 /L, platelets 273 x
10 9/L, sodium 140 mmol/L, potassium 3. 7 mmol/L, urea 3.0
mmol/L, creatinine 42 ,.unol/L and c reactive protein < 2.
Which of the following actions would be the BEST?
.

0
0
0
0

A Commence intravenous cefotaxime


B Commence oral prednisolone
C Advise bed rest
D Measure his proteinJcre~ti~ine ratio on early and morning urine for a
week

E Measure his serum creatinine monthly for 6 months

24. A 7-year-old presents with a 3-month history of severe


intermittent left loin pain. There is no past medical history of
urinary tract infection. On examination he is apyrexial, his
blood pressure is 1 70/90 mmHg, pulse 90 beats per minute and
he has a palpable renal mass on the left side. Urine dipstick is
negative to blood and protein. Urine culture is negative. Renal
ultrasound reveals significant left renal pelvis dilatation
(anterio-posterior diameter 20 mm) with normal renal
parenchyma and a normal right kidney and renal tract. Which is
the MOST useful investigation to perform next?

A Micturating cysto-urethrogram

--0~1fo!V!sAscan.

0
0
0

C MAG-3 dynamic renogram


D Intravenous urogram
E Plain abdominal radiograph

NEPHROLOGY

25. A 5-year-old presents with a 2-day history of fight-sided .(adal


'weakness. Hehashad headaches for 6 months. On examination
he is drowsy~ His bladd pressure is 180/95 mmHg, pulse 10,0~
beats per minute and_he is' apyrexial. He has a right.:;sided\'llth
cranial nerve palsy and bihiter~ll papillo'-oedema. The rest of his
cranial nerves are intact. Examination of his deep tendon .
reflexes reveals SY:JIUnetricalllYPer-reflexia in his.Iower limbs,
with down-going plantars. He'Has a gallop rhythm, normaHirst
and second heart/sounds, an apex displaced to Ute mid-axillary
line. There are no murmurs'and noradiofemoral delay. BloOd
tests reveal the following: haemoglobin 83 giL; White count 7 .o
x 109/L; platelets 2 93 x 10 9/L; blood film: hyppdtroiM,s: ~;: ,; :normocytic anaemia only; C reactive protein <.2L; so(lium 1)35 ,
mmol/L; potassium 4.9 mmol/L; urea 11.5 mmol/L; creatinine ~
231 J.Lmol/L; corrected calcium 11. 79 mmol/L (2.2-2.7 mm6Jru); t
phosphate'"'2.9Jn~o1/L (1.i.:.i 1mmol/L); bilirubin 11 J.LIDoiiL
( < 1 7 tJ.mol/L); urine pjpstick: prot;_ein + , blood not detected;
renal ultrasound: 2 small kidneys with cortical thinning. The
MOST likely underlying cause of these findings is which of the
foll~~!!g~l ~ i

0 A
0 B
0 c
0 D
0 E

Brain tumour
Coarctation of the aorta
Encephalitis
Haemolytic-uraemic syndrome
Reflux nephropathy

26. A 14-:year-old girl presents with a 2-day history of painless


macroscopic haematuria. There is no family history of renal or
hearing problems. Physical examination is unremarkable. Blood
pressure: 110/70,mmHg; temperature: 37 C; urine culture: no
growth; renal ultrasound:.normal;J'ullbl.Q.Qd CO}lf!t and clotting:
normal; sodium 140 mmol/L; potassium 4 inmol/L; urea 5 -
mmol/L; creatinine 79 tJ.mol/L; C3 and C4 levels normal. In
clinic, 2 months later, the urine dipstick shows blood++ and
protein +. A hearing test is normal. What is the MOST likely
diagnosis?

0
0
0
0
0

A Alport syndrome
B lgA nephropathy
c Nephrolithiasis
D Mesangiocapillary glomerulonephritis
E Acute post-streptococcal glomerulonephritis

63..

ESSENTIAL QUESTIONS FOR MRCPCH

-..,

{/}

GJ

::s

tr

21. A 2-year-old boy presents with a history of chronic diarrhoea.


His parents describe him as having a good appetite. On

examination his weight is on the 0.4th centile qnd his height on


the 25th centile. His blood pressure is 90/60 mmHg and pulse is
90 beats/min. Further examination is otherwise unremarkable.
He is mildly dehydrated. Blood tests show: pH 7.47; pC02 5.3
kPa; p02 10 kPa; bicarbonate 30 mmol/L (normal range 20..28);
urinary sodium 3 mmol/L; urinary chloride 4 mmol/L; serum'
sodium 135 mmol/L; potassium 2.7 mmoiJL; Urea 3.2 mmol/L;
creatinine 40 J.LlllOl/L; full blood count: normal; liver function
tests: normal; plasma renin: raised. What is the MOST likely
underlyingdiagnosis?

0 A. Bartter syndrome
0 B' Conn syndrome
0 c .Cystic fibro'sis
I

.
0 D 'Miinchhausen by proxy byadministration of diuretics
0 E Renal artery stenosis ..

.'

..

'

'

..

.
64

. "',.

'

. NEPHROLOGY

Extended Matching Questions


28. Theme: Renal syndromes
A Acute post-streptococcal glomerulonephritis
B Autosomal dominant polycystic kidney ~isease .
C Fanconi syndrome
.
'""D Focal segmental glomerulosclerosis
E Minimal change glomerulonephritis . .,
F IgA nephropathy
'
. ,
G . systemic lupus erythematosus (SLE) nephritis.
H Wilms' tumour

For each of the ca;e scenarios 's~Iect the mosi likely diagiiosi;from 0"e list
above. Each optkm.may be used once, more than once, or hot at all. .

0
,.

:!

"

I. A 2-year-old boy presents with a 3-week history offacial swelling. He


has ankle and sacral oedema. Blood pressure is ,90/60 mmHg. Urine
' - dipstick shows protein +;.f.+' blood +. S,odiu'rn .130 mmoi/L; : "
' p6tasslum3.5 mmol/L: urea 7.3 mmoi/L; creatinine60 J.Lmoi/L;
albumin r5 gtdi; 6 and C4 levels are normaL
.
2. A 15-ye~:;old girl.presen,..t,;;;,with 2-week history of facial swelling. Sh~
has,~<;.q~
~JQ~l~p,d pre~~~;l 7,Df80.mmHg. Urine ~ipstick
- shows prote
. +; olooC:! + + +. Sochum 131 mmol!L; potassmm 5.2
mmoVL;utea 10 3 mmOl/L; creatinine 163 J.LmOI/L; albumin 15 g/dJ;
anti-nuclear antibody screen negative; C3 level low; and C4level
normal. There is no remission of proteinuria after sweeks of
prednisolone at 60 mg/m2
.
3. A I 0-year-old boy presents 2 weeks after a sore thre>at with painiess
haematuria. He has mild pre-tibial oedema. Urine dipstick shows .
protein++, bl.ood++++. Blood pressure 170/100 mmHg. He is well
perfused and has a gallop rhythm. 'Bloods show: sodium 130 mmol/L;
potassium 6.5 mmoi/L; urea 15 mmolfL; creatinine 230 J.LmOI/L;
albumin 25 gtdl; c:Hevellow. C4 level normaL .
j'\.

. I

65

ESSENTIAL QUESTIONS FOR MRCPCH

29. Theme: Tubular dysfunction


A
B
C
D
.E
F
G
H

Bartter syndrome
Congenital toxoplasmosis
Cystinosis
Lowe syndrome
Nephrogenic diabetes insipidus
Insulin-dependent diabetes mellitus

Reflux nephropathy
Pseudohypoparathyroidism

For each of the follovving scenarios choose the most likely diagnosis from the
list above. Each option may be used once, more than once, or not at all..

66

J. A 6-month-old girl presents with recurrent episodes of dehydration.

There is a history of polyhydramnios during the pregnancy. She is on


the 0.4thcentile for weight, the IOthcentile for length, and she is
clinically 10% dehydrated. Blood pressure is 80/60 mmHg.
Investigations show: pH 7.47: pC02 4.5kPa; bicarbonate 32 mmoVL;
sodium 135 minoVL; potassium 2.1 mmoVL; chloride 80 mmo).!L;- urea
5 mmoVL; creatinine 52 JJ.moVL: serum renin raised; urinary sodium
40 mmoVL; urinary potassium 50 mmoiJL
2. An 18-month-old boy presents with a 6-month history of polyuria and
polydypsia. Examination reveals 2 em hepatomegaly. He has swollen
wrists and knees consistent with rickets. He is photophobic. Blood
pressure 100/80 mmHg. Blood tests show: pH 7.25; pC02 3.5kPa;
bicarbonate 18 mmoVL; sodium 130 mmolJL; potassium 2. 7 mmol/1.;
Urea 7.2 mmoi/L; creatinine 120 mmoi/L; glucose 4.3 mmoi/L; calcium
2.2 mmoVL; phosphate 0. 7 mmoi/L; alkaline phosphatase 2030
mmo]JL; thyroid stimulating hormone (fSH) raised; thyroxine low;
urine dipstick shows glycosuria and proteinuria ..++; urine pH 6.2.
3. A2-year-old boy 'presents with failure to thrive; There is no history of
vomiting or diarrhoea. He had cataracts detected in the neonatal period .
. He has global developmental delay and hypotonia. Investigations show:
sodium 135 mmolJL; potassium 3.2 mmolll..; iiie;fj mirioliL; creatinine
60 JJ.ffiOI/L; glucose 4.2 mmoi/L; phosphate J .5 mmol/L; corrected
caldum 2.35 mmoVL: serum pH 7.29; bicarbonate 15 mmoiJL; chloride
108 mmoiJL: urine pH 5.5; urine dipstick glucose+.

_l

NEPHROLOGY

A Chronic rena !'failure.


B Congenftal adrenal hyperplasia
C Craniopharyngioma .
D Cystic fibrosis .
E Diabetic ketoacidosis
F
Excessive intravenous administration of salt-poor fluid
G S!MJH ~is,Yndro!l'e oC:.inat:mr<;?rriate secretion. of anti-diu~etif no~~one)
H .Urea cycle defect .,
..
Viral encephalitis .
. .

For each of the following scenarios choose the mostlikelycause of the child's
condition from the Jist abi:>l'e:J.Each option may be used once, more.thanonce,
or not ai all.
0 '1: A 2;.~9,nth~old~boy presents V:rith tachypnoea and i>oor feeding. He .

,_,:"
'

30. Th~me: Hyponatraemia

.1

.was born in' good conaition at 36 weeks' gestation by spontaneous


vagih'i:il delivery. on admission he weighed 5.0 kg. His respiratory rate
was,80 breaths per min and he had marked intercostal reeession. He
f'" ,.,t ~" r
-'"" ...."" -.,.
Jil
t#
t
f
;
l
was clinicaUy,diagrtosedas haVing bronchiolitis and started Of!
1
' maintenarke' fluids of O.l8%soditim chloride and 4% deXt'rose at 31
ml/hour; 12 hours later he developed profound apnoeas requiring
.. .,.,,
intubation and ventilation,pp,i_ntensivecare. At this pointhe~i.we}l
perfused and he is not oedema tous. Investigations on admission to.
intensive care shO\\'ed: serum sodium 115 mmol/L; serum potassium
4.2 mmoi/L; serum urea 2 mmol/L; serum creatinine 41 J.Lmol/L; serum
pH 7.19; serum pC0 2 8 kPa; serum p02 8 kPa; serum bicarbonate 20
mmo1!L; serum glucose 4 mmol/L; urine osmolality 400 mosmol; urine
sodium 50 mmol/L; urine potassium 40 mmol/L.
2. A 2-week-old Caucasian boy presents with a 3-day history of poor
feeding. He was born at term weighing 4.0 kg. On admission he
weighs 3.6 kg, has dry mucous membranes and cool peripheries;
. temperature 3 7.5 cc; respiratory rate 60 breaths/min and pulse 170
beatstmlri. the-rest orthe examination is unremarkable. Initial
investigations show: serum sodium 125 mmol/L; serum potassium 6. 9
mmol!L; serum urea 6 mmoi/L; serum creatinine 60 ,.~.moi!L; serum

bicarbonate JB mmol!L; serum glucose 2. 7 mmol/L; serum


haemoglobin 170 giL; serum white cell count I B x I 09/L; serum
platelets 370 x 10"/L; serum C reactive protein (CRP) 5 mg!L;
abdominal ultrasound: normal; urine sodium 50 mmoi!L; urine
microscopy: no white or red cells seen.
3. A previously healthy 5-year-old girl was admitted to hospital following
a4-day history of profuse vomiting and diarrhoea. There is no past
medical history of note. She is uacking along the 5th centile for height
and the 50th centile for weight. J;ler weight I month prior to admission
was .,6 kg. On admission she is lethargic. her capillary refill time is
67

I..Cl
!C
I

tl).

en.

-~

ESSENTIAL QUESTIONS FOR MRCPCH

6 sec, her temperature 38 oc and pulse 160 beats/min, and she weighs
15 kg (9th centile). Initial tests show: serum pH 7.31; serumpC02
3.5kPa; serum bicarbonate 18 mmol!L; serum sodium 137 mmol!L;
serum potassium 3.2 mmol/L; serum urea 6 mmol/I..i serum creatinine
80 1-Lmol!L; serum alanine aminotransferase 50 lUlL (normal 28-44);
serum albumin 27 giL (normal 30-45); serum glucose 3.8 rrimoi/L;
urine sodium 4 mmoi!L; \.!Tine potassium I 02 mmoi!L; urine
osmolalitY 900 mosmol. She is given four 150-ml boluses of 0.18%
sodium chloride 4% dextrose over the first 3 hours of resuscitation
until she becomes well perfused. She is then started on maintenance
fluids plus 10% correction over 24 hours (0.18% sodium chloride 4%
dextrose. with 20 mmoi!L of potassium chloride at 120 ml/hour). After
12 hours she becomes drowsy, so she is re-examined. At this point her
capillary refill is 1 sec, pulse 120 beats/min, .blood pressure, 130/70
mmHg, temperature 3 7 oc. She then has a prolonged generalised
, . seizure. Investigations at the time of the seizure show: serum sodium
115 mmoi!L; serum potassium 4.2 mmoi!L; serum urea 4 mmoi/L;
serum glucose 5 mmoi!L: serum lactate 7 mmoi!L; serum pH 7.2;
serum base excess -7 mmoi!L; serum pC02 6 kPa; urine osmolality I 00
. mosmol; urine sodium 20 mmol!L.

,,.

,,

'.1

".

1. Complications of nephrotic syndrome.


Answers: A B C E

The mechanism of ele\(ation of cholesterol and low-density lipoprotein (LOL)


is unknown. Chronic proteinuria and steroid therapy can lead to muscle
wasting, which can be masked by oedema. Loss of immunoglobulin in the
urine and ,immunosuppression vvith steroids preqisposes to infection.
particularly pneumococcus. Loss of anti-thrombin JJI and protein c and
proteinS, dehydration with venousstasis, and increased blood viscosity
cause an increased risk i::>r't11rombos_is. Hypovolaemla q';le to shift of water
from the intravasclllar to the interstitial space is common. Symptoms include
abdominal pa~n and anorexia. J(hypovolaemia is severe acute tubular
necrosis can ensue.
~

2.

Glomeri1l'O~:ii:Itrs '
Answers: BC

Any of the glomerulonephritidies can present with any of the renal


syndromes: nephritic syndrome (hypertension, haematuria, renal
impairment); nephrotic syndrome (oedema, hypoalbuminaemia, proteinuria);
haematuria and/or proteinuria: or any combination of these. The renal
syndromes are not specific for particular conditions and the same conditio;:
can present with different clinical features in different patients.
3. Acute post-strepto.coccal
glomerulonephritis
.
, ..__ ,
(
-~-.<

-~--

Answers: A C E

Acute post-streptococcal_glomerulonephritis typically presents 10-14 days


after a streptococcal throat or skin infection with nephritic syndrome but can
present with any of the ten a! syndromes described above. Hypertension due to
oliguria and fluid overload is common at presentation. Persistent clinical

abnormalities (hypertension, proteinuria and microscopic haematuria) occur


in less than 5% of children followed up long term. C3 is low with a normal C4.
Complement levels should be checked 3 months after the acute illness,
because if C3 is persistently low this may indicate other glomerulonephritidies,
such as systemic lupus el)1hematosus or mesangiocapillary
glomerulonephritis.

II
\
l

69

~Yiij-]
:,

;..
~

,~J.

u)'.J!
~~~
(1).,~;

{It

c:

nS

ESSENTIAL QUESTIONS FOR MRCPCH

4. lgA nephropathy
Answet:B
serum JgA is normal. Jt can present with any of the renal syndromes. H is a
cause of persistent microscopit haematuria without any other renal
symptoms. It has a good prognosis with on1y I 0% developing hypertension,
proteinuria and renal impairment during childhood. ACE inhibitors can
control hypertension and.reduce proteinuria. Henoch-SchOnlein purpura and
IgA nephropathy are histologically indistinct and may be different spectrums
of the same disorder.
'
5. Proteinuria

Answers: B 'E
Although proteinuria is common during acute febrile illnesses follow-up
sh<:)l.i'ld be ensured to check'theptoteinuria resolves. Dipstick testing is a good
screening test forproteinuria although there are pitfalls. The gold standard
for quantification of proteinuria is measurement on a 24-hour urine
collei:tion;;howe:Ver, this is difficult to dci in chfldreri. Estimation of ,
protein:creatinine ratio on an early morning urine is~ wdrihwhii~' .
compromise. Orthostatic proteinuria is a benign condition and does not
usually require renal biopsy. In orthostatic proteinuria. proteinuria is absent
when the patient is recumbent. Regardless of how dih.ite or concentrated a
urine sample is, if the concentration of the substance being measured in the
urine is factored by the concentration of creatinine in the urine, then the
effect of alterations in urine concentration is eliminated.

6. Proximal renal tubular acidosis.


Answers: B C E

be

...

is

:_;~ ~

Renal tubular acidosis should


suspected when there alkali urine (pH >
5.8) in the presence of systef!!ic acidosis a~d. a normalal)iOn gap. In proximal
renal tubular acidosis there' is failure to reabsorb bicarbonate. However, distal
tubularhydrogen ion secretion is normatsoifplasma bitaroonafe fallsfo iow
levels (16 mmoJJL) acid urine ca!l be produced. ln distal renal tubular acidosis,
hydrogen ion excretion is impaired so there is never acid urfne. Proximal ~enal
tubular acidosis can be part of Fanconi syndrome. of\vhich there are a.
.
number of causes: cystinosis; tyrosinaemia; galactosaemia; Lowe syndrome;

Wilson's disease; heavy metal poisoning; idiopathic. Cysti.nuria causes .


increased urinary excretion of cysteine leading to nephrolithiasis and it is
distinct from cystinosis.
, ,,
'
,
'' '"

70

NEPHROLOGY

7. Distal renal tubular acidosis .

'..... I{

AnS\vers: A B DE


Distal - and very rarely proximal - renai tubular acidosis is cause of
nephrocalcinosis. Distal renal tubular acidosis can be primary isolated or
secondary to obstructive uropathy, amphotericin or cydosporin. Hypokalaerriia
is a feature, rather than h~'Perkalaemia, due to increased urinary losses. . .

8. Neonatal renal'furiction
AnS\vers: B C DE
""5o

-_ '""

-- ' ~*

,..

At 36 weeks' gestation nephrogenesis is complete. However. glomerular filtration


rate (ml/kg/min/1. 73m2) is less than 5% of ad1;11t values. Glomerular filtration
rate rises \Vith age and reaches adult values by age 2 years. The glomerular '
.filtration rate. of premature infants is markedly less than term infantS/hence
their susceptibility to renal impairment. The ability to reabs6rb filtered sodium
. and conserve sodium;in a salt- or water-depleted state is impaired in babies.
Older children and adults who
salt-depleted,have a urinary sodium of less
than 10 mmoVL and fractional excretion of sodium of Ie5s'thanU %, whereas
premature infants tend to have inappropriately high urinary sodium
concentrationin this state. Water concentrating ability is not matu~re until 2
!... '
years ot;,_age. The plasma bicarbonate concentration at.which filte;~d.
bicarbonate appe;:~rs in the llrine.(
rbonate threshold) is low in the newborn
' '-:t:~ ~-~~~
. ~ ~,-_--' ~if~'-~_.., --
t.~

(19-21 mmol/l), h1ereasing to rna


values of24-:26 mmoiJI by 4 years.

are

"

9. Pre-renal renal failure

,.,._.

.,;

AnS\Vers: B D E

In pre-renal failure the kidney tries to conserve water and sodium, hence
urinary sodium is low.(~ 1o mmoVL). During intrinsic renal failure urine
concentrating ability is impaired so urinary osmolality is usually < 300
mosmoVL. Urea increases in renal failure of both types. haemoconcerltration
and during starvation so it is. an unreliable measure of dehydration. Oliguria
(< 2 ml!kg/h in a baby and < 1 ml/kg/h in an older child) could be due.to .
either reason:-A raised blood pressure, gallop rhythm, raised)ugu1ar venous
pressure/pulse OVPJ and good peripheral perfusion suggest intravascular
overload associated with intrinsic renal failure .. Low blood pressure and poor
capillary refill suggest a pre renal cause.

10. Renal cysts


. A.nSH1er: All true
Renal cysts found coincidentally on ultrasoundscannin'g should not be
dismissed. Nephronophthisis isa rare condition but the commonest genetic
cause of chronic renal failure in children. It usually presents with gradual
onset of polyuria and anaemia before progressing to end-stage renal failure
(average age of end-stage renal failure is 13 years). Around 80% of children
71

ESSENTIAL QUESTIONS FOR MRCPCH

receiving prolonged haemodialysis for whatever cause have multipl" renal


cysts. Renal cysts are one of the diagnostic criteria for tuberous sclerosis.
Patients with tuberous sclerosis may also have renal angiomyolipomas.
Patients with cystic fibrosis may develop interstitial nephritis as a result of an
allergic reaction to drugs or infection. They may also develop acute tubular
necrosis as a result of nephrotoxic drugs such as gentamicin.

11; Autosomal recessive polycystic kidney disease


AnSlNers: A

. Large echo bright kidneys, 'A'ith or without cysts that are small(< 2 mm) are
characteristic of autosomal recessive kidney disease. The severity of hepatic
fibrosis varies from sub-clinical to overt liver disease, which is the dominant
clinical feature. There are two gene loci for autosomal dominant polycystic
kidney disease; the commonest of which is on chromosome 16, adjacent to
the tuberous sclerosis gene. Autosomal dominant polycystic kidney disease is
associatedwith cerebral aneurysms, subarachnoid haemorrhage, mitral valve
prolapse, and hepa.tic cysts. Hypertension is a major clinical feature and it
may be very difficult to treat.

12. Diuretics
AnSlNers: B D

Spironolactone blocks aldosterone-sensitive channels in the collecting ducts,


which reabsorb sodium in exchange for potassium and hydrogen ions. Loop
diuretics block the N-K-2CI transporter in the loop of Henle, increasing
sodium and chloride delivery to the distal convoluted tubule. Sodium is then
re-absorbed in exchange for potassium and hydrogen ions resulting-in a
hypokalaemic metabolic alkalosis, Potassium secretion is proportional to
distal tubular urine flow, distal tubular sodium delivery and aldosterone levd:
Thiazide diuretics block the sodium potassium co- transporter in the early
distal c()n\oluted tubule. t i

.;, '

13. Renal homeostasis

..

'

Answers: DE

In Conn syndrome there is primary hyperaldosteronism. High aldosterone


leads to extracellular fluid expansion, hypertension, hypokalaemia, alkalosis
and renin suppression. Renin is produced by the juxtaglomerular apparatus in
response to reduced renal perfusion (shock, renal artery stenosis and renal
scars). Renin converts angiotensinogen into angiotensin 1. Angiotensin. converting f!nzyme (ACE) comerts a,ngiotens!n I to angiotensin 11. Angiotensin
II increases blood pressure by causing vasoconstriction and increasing
aldosterone secretion frorri the adrenaJs. In Addison's there is aldosterone
deficiency.

...

72

NEPHROLOGY

14. Bartter syndro~e


Answers: A B E

This inborn defect in the Na-ci-2CI co-transporter in the loop of Henle results .
in urinal) sodium. chloride and water loss. It is autosomal recessively
inherited. Markedly increased urinary chloride and sodium with normal
serum creatinine are typical. lncreased delivery of sodium to the distal
convoluted tubule result in reabsorption of sodium in exchange for_ potassium
and hydrogen ions, resulting in metabolic alka~?sis:

15. Vesicoureteric reflux


AnS\-ver.s: A D E

Vesicoureteric reflux (VUR) is 20-50 times more common in children with a


family history ofVUR. Grade 1 or II reflux cannot be excluded on ultrasound.
The investigation of choice is micturating cystocurethrogram. Reflux
nephropathy can occur at any age. especially if there is associated detrusor
dysfunction, as in patients with spina bifida. Vesicoureteric rt'!flux predisposes
to urinary tract infection.

'
A MAG 3 renogram can be used to demonstrate VUR by scan.ning over the
kidneys while the child voids after all the isotope has drained from the upper
tracts into the bladder. For thistest to be reliable the child must be old enough
to have bladder control and to. \'oid on command:

16. Dialysis in acute renal failure


AnsWers: B C E

'

'

Creatinine itself is not toxic and takes days to rise. even when there is no renai
function, so there is no absolute level at which acute dialysis is indicated.
Haemodialysis can aid removal of low molecular weight toxins, such as. .
ammonia or salicylate. Acute renal failure results in impaired excretion of
hydrogen and potassium ions. H,Yperkalaemia can result in ventrlcular
'
tachycardia and ventricular fibrillation. Oliguria is not in itself an indication
for dialysis;naweveTvoluii1e~ overloa'da-rrctresrrltant hypertension

unresponsive to diuretics are: Acidosis may be refractory to sodium


bicarbonate: Side effects of sodium bicarbonate are hypematraemia and
hypertension,
'
f:l1
4.

.. '

17. Nephrocakinosis
AnS\-vers: A B c D

Nephrocalcinosis is usually part of a metabolic disorder. Nephrocalcinosis is


common in distal renal tubular acidosis but rare in proximal renal tubular
acidosis. Hypercalciuria can cause nephrocalcinosis. Hyperparathyroidism;
vitamin D intoxication. Bartter syndrome, William syndrome, corticosteroids
and frusemide cause hypercalciuria. At follow up, 27% of. infants born at less
than 32 weeks and 62% of infants with chronic lung disease have
73

, ESSENTIAL QUESTIONS FOR MRCPCH

nephrocalcinosis. Low birth weight, diuretic therapy, l1,1adequate dietary


phosphate, and duration of oxygen therapy are risk factors.

18. Hypertension

:.

'

Answers: A B DE

'

'

The deCision to,treat hyp~rt!nsion is determined by the level.ofblood


pressure and .the presen~e. of end-organ damage. Hyp~rtensive ..
encephalopathy may cause headach~. irrita?i!ity, ~e~res and hyper-reflexia.
Findings in h}1>ertensive retinopathy include cotton-wool spots and flameshaped haemorrhages. Ollly rarely will there be retinal or macular oedema ..
Hypertension can cause renal impairment, left ventricular enlargement, and
heart failure.
'

19. Complications ofchronic renal failure

Answeis: B c D

:l .,

The main causes 'of chronic' r~nal failure in childhood' a;e r<:nal, dysplasia, ,
reflux nephropathy. glomerulonephritis, genetically il'}perited diseases. such as
Alport syndrome or nephronophthisis and systemic diseases, sucf! as systemic
lupus erythematosus (SLE) and Henoch~Schbnleinpurpura (tfSP). Phosphate
retentiol! leads to hJP?.calcaemia a~d ;;~condary h~rp~~a.!~Yr,9~l!!.ciDP. ~'.
bone r~sorption~ Reduced synthesis or 1,25,hydr~xy-\_:na,min D 3 !=?~.trib~tes to
hypocalcaemia and leads to rickets. Anaemia is secondary to dietary iron
defidency, reduced red blood cell survi~al, an.d erythropoietin deficiency.
Polyuria is usual until glomerular filtration rate falls to end-stage renal failure
levels (less than 10 mVminJL73m2).

'

20. Posterior urethral valves

Answer: All false


A normal ultrasound on day 1 of life-does not exclude posterior urethral
valves because hydronephrosis may ~ot be apparen!-until urinary flow has
been established. Posterior urethral valVes are always associated with some.
degree of rena] dysplasia and arol,lnd 2~.96 ..yill end up with chronic renal .
failure or end-stage renal failure. Renal dysplasia is known to occur even if
the valves are operated on in utero. The. classic presentation
oliguria and
pulmonary hypoplasia. They are associated with maternal oligohydramnios.
They are best diagnosed on micturating cysto-urethrogram. To visualise the
valves, films must be taken without the urinary catheter in ~itu; . ., , .. :....

is of.

;,

; .

74

,,

---

.l.

NEPHROLOGY

21. Glomerular filtration rate


Answers: C E

GFR is dpressed as a functiOn of surface area(SA). Absolute values for GFR in


mVmin are corrected for SA by the formula: Corrected GFR(mVminJ1.73m2 ) =
absolute GFR (mltmin) x 1. 73/SA (where 1, 73m2 is the SA of an average adult
male). Creatinine clearance overestimates GFR because there is tubular
secretion of creatinine. The standard way o( measuring GFR involves the
single injection plasma disappearance curve method, where inulin, or an
isotope like 51 -chromium labelled EDTA. is injected and serial blood samples
are taken to allow a plot of the rate of clearance of inulin or tne isotope from
the plasma. Urine samples are not needed. ACE inhibitors cause dilatation of
the efferent glomerular arteriole, thus lowering the hydrostatic pressure in the
glomerular capillary bed, and hence the GFR may be lowered. The normal
mature GFR value of so-120 mVmin/1. 73m2 is reached during the second _year.

22. Diabetes insipidus


Answer: E

Diabetes in~sipidus may be central due to reduced ~cretio~ of ADH when


plasma levels,will be low, o'r peripheral due to end organ resistance of ADH
when plasma levels are usually around or just above the top end of normal
range. The fundamental diagnostic feature,is raised plasma osmolality with
inappropriately low uril!e osmolality. With complete absence of action of ADH
humans may produce urine osmolality of< 1OOmOsm/L Patients with ..
diabetes insipldtis commonly have urine osmolallty higher than this, because
there is not complete failure of secretion or complete end-organ resistance,
but in comparison to the raised plasma osmolality the urine osmolality is stiil
inappropriately
Hypematraemia is the characterisitic plasma electrolyte
change. 1teatment of acute hypernatraemic dehydration .includes replacement
of water deficit With extra intravenous fluid. However many patien,ts manage
to a:I[QlcJJ.hese epi~Qde~ .a~r)9ng as_!}le_x~}l.,a__y_c:J!e~~~cce"?-~.~9 ~a~::_r~wJ!en ~~-~L .
feel thirsty. The diagnosis is sometimes made when a patient is unable to have
normal free access to fluid eg after routine surgery when nil by mouth, when .
they develop hypernatraemic dehydration despite what would normally be
adequate N repl~cement fluid.

low:

,.

\,.,

..."."'
,.

~..,

U)

ESSENTIAL QUESTIONS

FC~

MRCPCH

Best of five
23. D: Measure protein/creatinine ratio on early-morning urine for
a week
The scenario suggests Henoch7Sch6nlein purpura (HSP). Around 70% of
children \\'ith HSP have some degree of renal involvement, usually just
microscopic haeinaturia with or without mild proteinuria. Patients can
develop nephritic syndrome. nephrotic syndrome, hypertension. or chronic
renal failure, sometimes years after the original rash. Persistent nephrotic
range proteinuria (proteiJVcreatinine.ratio > 200m!Vmmol) warrants referral
to a nephrologist for consideration of a biopsy. Arthritis ti?ually settles.
without treatment. Bed rest .does not alter the course of the condition. Serum
creatinine is an insensitive measure of renal involvement; although it is of
concern if raised. Steroids are rarely used in HSP but have been used for
severe abdominal pain and severe glomerulonephritis.
.
.'
24. C: MAG-3 dynamic renogram
The pati.e~t is m()St likely .t~ have pelvi-ureteric juncti()ry.obst~uction fro~,~e
scena'rio presented. 'The investigation of choice is a lv'lAG-3 (djnamk .
.
renogram))o determine ifthere_is obstruction tourimi'ry flow. The pat!eni'Is
hypertensive (95th centile for systolic BP is 106+' [age X 2)). Hypertension of
renal origin can occur by a numbed::>f mechanisms. It can occur when there is
obstruction to urinary flow, as in this case. It can also occur because Of

reduced renal blood flow. either 'generally (as in renal artery stenosis) or
locally (which occurs in renaJ scarring). Vesicoureteric flux with renal scarring
from previous urinary tract infection is also a possibility, so DMSA and
micturating cysto-urethrogram should be considered, However, vesicoureteric
reflux without urinary tract infection would not cause loin pain. AbdominaJ xray and intravenous urogram would be indicated if nephrolithiasis were
.
. thought to.be the cause ofhis pain. Nephrolithiasis is rare in childhood and in
the absence of microscopic haematuria.

25. E: Reflux nephropathy

..

Vllth nerve palsy is a recognised complication ofhypertension in childhood.


The child presents with symptoms consistent with hypertensive

encephalopathy. such as drowsiness, papilloedema and hyper-reflexia. The


anaemia. low calcium, proteinuria. and high urea, creatinine and phosphate
are consistent with chronic renal impairment. Renal impairment can cause
hypertension. or be as a result of hypertension. The cardiovascular signs are
consistent with heart failure. which could be as a result of hypertension. The
absence of murmurs and radiofemoral delay makes coarctation less likely.
Raised intracranial pressure could cause hypertension but would not explain
. the other findings. Recurrent urinary tract infections and vesicoureteric reflux
are thought to cause renal scarring and reflux nephropathy.
76

-~--

NEPHROLOGY

Sometimes there ::; no history of urinary tract infection. This may be


because the urinary tract infections have gone undiagnosed or because the
renal abnormalities are congenital dysplastic renal malformations, which are
indistinguishable from renal scars on DMSA and ultrasound scanning.
Children with renal scars should have yearly blood pressure checks for life.
26. B: IgA nephropathy

IgAnephropathy can present as an incidental finding of persistent


microscopic haematuria or: with any of the renal syndromes. Prognosis is
good although I 0% will develop hypertension, proteinuria +/-renal failure
with long-term follow-up. Alpert syndrome, a hereditary nephritis, could
present with a similar clinical picture. There are X-linked and autosomal
recessive forms. Deafness around the age of 10 years, hypertension in midteenage years, and end-stage renal failure in the third decade is usual. Family
members of children presenting with haematuria should be screened for
microscopic haematuria and proteinuria. Nephronophthisis usually presents
with polyuria/polydypsia, often has normal urinalysis and progresses to endstage renal failurein the first decade. Acute post-streptococcal
glomerulonephritis and mesangiocapillary glomerulonephritis could present
in a similar way, however C3 is usually reduced. l)'pically, mesangiocapillary
glomerulonephritis has persistently low C3, nephrotic range proteinuria
(protein/creatinine ratio > 200mgtmmol), hypertension and progressive renal
impairment.

27. C: cystic fibrosis


The patient has a metabolic alkalosis with a very low serum potassium, the
causes of which are: Bartter syndrome; pseudo-Bartter syndrome;
aldosterone excess; renin excess; or diuretics. Pseudo-Bartter syndrome
occurs when sodium and chloride are lost from extra-renal sites leading to a
secondary hypei-re~inaemia. causes include cystic fibrosis, congenital
chloride diarrhoea, laxative abuse and cyclical vomiting. The serum
biochemistry is the same as Bartter syndrome (hypochloraemic, hypokalaemic
. all<alosis) but 'there are appropriately" low levels of urinary chloride and- -.-. .
sodium(< IOmmoi/L). Renin is released from the juxtaglomerular apparatus
in response .to reduced renal perfusion, leading to increased angiotensin 11,
which causes vasoconstriction and aldosterone release from the adrenals.
Aldosterone causes extracellular, fluid expansion by distal tubular sodium and
water conservation. Sodium is reabsorbed in exchange for hydrogen ions and
potassium. Hyper-reninaemic states, such as renal artery stenosis, can
produce a similar biochemical picture to the scenario, however th~ patient is
not hypertensive. Conn syndrome, primary hyperaldosteronism, also causes
hypertension but there is renin suppression. Loop diuretics can produce a
similar biochemical"picture to the scenarjo, although there would be a high
urinary sodium.
i'

77

ESSENTIAL QUESTIONS i=Ofl: MRCPCH

28. RENAL S_YND:ROMES

1.

E - Minimal change glomerulonephritis

'

'"'ill

85% of toddlers presen'ting with nephrotic sYndrome


have minimal change

disease. This is usually steroid sensitive, renal function is normal,


hypertension'not SUStained and J!liCroSCOpic haem?tUria mild and ir"ltermittent.
2. D - Focal'segmental glomerulosClerosis
Steroid-resistant nephrotic syndrome is defined as no remission after 4. weeks
of 60 mgtm 2 prednisolone. The usual histol9gy is focal segmental
glomerulosclerosis or membranoproliferative glomerulonephritis. The age of
presentatlo~ is typically greater than 8 years or less than I year. The long-term
prognosis is poor. Hypertension and microscopic haematuria are common.

3.

A- Acute post-:-streptococcal glomerulonephritis

This child has nephritic syndrome (renal impairment and oliguria leading to
hypertension and fluid overload). Ma'ny glomerulonephritidies can present
with nephritic syndromebut it is reaso~able to assume that the cause is acute
. post-streptococcal glomerulonephritis
and not perform ..a renal biopsy if there
t'
.

is complete recovery of r.enal function and C3 levels return to normal after 3


months.
' '
'
-'
.>

-~.

'

")f.;

29.

TUBULAR DYSFUNCfiON

1.

A- Bartter syndrome .

1,t

'

.~

,c'' "

,,

,,

,.

The metabolic alkalosis and \'eryJow potassium suggests Bartter syndrome. .


1
This is due .to an inborn: defect. in the N..:.K-2ci t~ansporter'in the thick loop or'
Henle. leading to salt and water wasting. The resultant extracellular fluid
(ECF) volume contraction leads to secondary hyperaldosteronism and fvi~ ,
sodium and water reabsorption in the distal convoluted tubule, and reciproc~l
potassium and hydrogen ion excretion into the urine. Crucial to the diagnosis
is the finding ofinappropriately high levels of urinary chloride and sodium.
2. c - Cystinosis

'
.
In this scenario there glycosuria with normal serum glucose and metabolic
acidosis \vith an inappropriately alkaline urine. This suggests there is a
proximal tubl,llarleak consistent with Fanconi syndrome. There are a number
of causes of Fanconi syndrome such as galactosaemia, mitochondrial '
disorders. tyrosinaemia, frudosaemia,Lowe syndrome. Wilson's disease.
cystinosis, and heavy metal poisoning. Cystinosis is an autosomal recessive'
defect in the transport out of lysosomes. It leads to a multisystem disorder..
Early featu;es inch.ide _Fanconi syrldrome, photophobia' due io cysteine : '
crystals in the cornea, and hypothyroidism, Renal failure occurs around 1o ':
years of age if untreated. Diagnosis is confirmed by raised peripheral blood ' .
white cell cysteine levels. .

'

is

3.

D- Lowe syndrome

..

tn

. The occulocerebrorenal svndrome of Lowe is an X-linkea disorder . '\''


characterised by congenital catara,cts, hypotonia. intelledual .impairment. and.
renal Fanconi syri~rome. Hypokalaemia' is not marked. There is no specific 1'.
}

78
'.

NEPHROLOGY>

treatment. Patients develop chronic renalfailure in the second to fourth

decades.
30. THEME: HYPONATRA.EMIA

G - SIADH (syndrome of inappropriate secretion of anti-diuretic


hormone) secondary to bronchiolitis
The apnoeas are likely to be due to hyponatraemic seizures rather than
1.

impending respiratory failure. This patient has inappropriately concentrated


urine in the face of very dilute plasma (2:39 mosmol). The plasma osmolality is

is given by:
2 x (Na

+ K) + (urea + glucose)

The normal range is 275-295 mmoi!L. To diagnose SIADH there must be


evidenct of fluid overload as well as signs of volume depletion. As water
distributes freely bel:\veen the ECf and ICF oedema is not usually present. In
SIADH an increase in ECF volume and reduction in plasma osmolality .fails to
suppress anti-diuretic hormone (ADH) secretion and, if normal fluid intake is
maintained, hyponatraemia develops. It Is caused by surgery. or by pulmonary
disorders, such as infections or positive pressure :ventilation, or by disorders
of the central nervous system, such as meningitis or hypoxic-ischaemic
encephalopathy. Treatment is to restrict water intake.

2.

B - Congenital adrenal hyperplasia

The inappropriately high urinary sodium in the face ofhyponatraemia and


hypovolaemia suggests an inability to reabsorb filtered sodium, broadly the
causes of which are renal failure. aldosterone deficiency. and administration
of diuretics. The relatively normal serum creatinine makes renal failure
unlikely. although creatinine takes hours to rise in acute renal failure. With the
exception of spironolactone most diuretics cause hypokalaemia. The
combination of low sodium and glucose and high potassium suggests adrenal
insufficiency, the causes of which are primary (such as congenital adrenal
hyperplasia or Addison's disease) or secondary (such as panhypopituitarism
or tumours affecting the pituitary, eg craniopharyngioma). The lack of cortisol
and aigost~rQ~ji}_C.Q!)g~..!Ji~l. a.Q!~!!al hypen>Iasia .~eads to productio_n_of _
adrenocorti~;:otrophic hormone (ACTH). ACTH acts on melanocyte-stimulating
hormone receptors to cause pigmentation in skin folds. There are a number of
forms of congenital adrenal hyperplasia, which can present at various ages in
various ways. In classical congenital adrenal hyperplasia there is deficiency of
one of the enzymes in the biosynthetic pathway of the adrenal cortex (21hydroxylase deficiency). The continuing ACTH drive leads to the precursors
being directed along the androgen biosynthetic pathway, causing virilisation.
This presents with ambiguous genitalia in girls and a salt-losing crisis or
precocious puberty in boys. Investigation involves karyotyping. serum
17-hydroxyprogesterone (the precursor of 21-hydroxylase), urinary steroid
profile. and adrenal androgen levels. Acute treatment is with hydrocortisone
and salt replacement.
79

ESSENTIAL QUESTIONS FOR MRCPCH

. 3. F- Excessive intravenous ad~lnistration of salt-poor fluid ~


The initial history and investigations are consistent with gastroenteritis \'>.rith
I 0% dehydration. Although serum sodium is normal there will be a total body
sodium defi\it, as diarrhoea and vomit contain large amounts of sodium (7~
mmolJL). The slightly low initial potassium indicates that the distal tubule is
reabsorbing sodium at the e>..-pense of potassium. The girl is resuscitated \~rith
the equivalent of half her circulating ~load volume (40 ml!kg) of salt-poor
fluid and she then receives further salt-poo~ maintenance fluid. ;This will,
correct her water deficit but not the ~alt deficit: resulting in dilutional
hyponatraemia.

...

,,

r.,_,

'

.
j

. I

.t

..

.
, I

\'

'.

'

_,-r
'l'

:1:
80

Tammy Hedderly

Multiple Choice Questions


1. In n.::urofibromatosis type l

0
0

0
0

A
B
C
D
E

25%of cases have no previous family history of neurofibromatosis


bony dysplasia is a diagnostic criterion
brain magnetic resonance imaging is a routine diagnostic test
there is a 10% risk of malignant change in plexiform neurofibromas
optic gliomas occur predominately in children over 7 years

2. Friedreich's ataxia

0 A
0 B
0 c

0
0

is caused by an expansion of GM trinucleotide repeat


motor nerve conduction velocities are usually normal
usually presents in the first 6 years of life
D cardiomyopathy is the commonest cause of death
E requin;s p;s cavus for the diagnosis
'

3. The following conditions are correctly paired with expected


cerebrospinal fluid findings

a
0
0

A acute disseminated encephalomyelitis and CSF lymphocytosis with


increased protein
B 'hibefclildusmeningit1s a-no-high glucose with raisedprotein. c Guillain Barre syndrome and normal cell count with reduced protein
D fungal meningitis and lymphocytic CSF wi th.Iow glucose
E subarachnoid haemorrhage and xanthochromia after centrifugation

4. 1\Jberose sclerosis

0
0
0
0

A is an autosomal recessive condition


B is associated with renal angiomyolipomas in approximately 80% of
people
c is excluded if the brain magnetic resonance imaging is normal
0 is associated with retinal hamartomas
E is associated with autism

ESSENTtA;_ QUESTIONS =oR MRCPCH

5. In acute disseminated encephalomyelitis

0
0
0
0

A unilateral optic neuritis is seen more commonly than in multiple


sclerosis
B peri\'entricularwhite matter lesions occur less commonly than in
multiple sclerosis on magnetic resonance imaging
c a relapsing course is more common than in multiple sclerosis
0 'the EEG demonstrates excessive slow wave activity E headache, fever and meningism are less common than in multiple
sclerosis

6. Children with Rett syndrome

0 A
0 B
0 c
0 0
0 E

have normal prenatal and perinatal development


have a small head circumference at birth
are usually female
often fail to thrive
show a mutation in MECP-2
'j..

_,;.

! ""

.J

7. The following neuro.muscular disorders are


their mode of inheritance

OA
0 B
0 c
O 0
O E

J'

'

corre~tly

paired with
. '

. (~

myotonia congenita is autosomal dominant


congenital mus<:ul.ar dys~rophy is autosom~l recessive
facioscapulohumeral dystropJ1y is.autosomal recessive
Ouchenne muscular dystrophy is X- linked dominant
familial hyperkalaemic. periodic paralysis is a';ltosomal recessive

8. The following conditions are considered to be ion chanilelopathies

0 A
0 B
0 c
0 o

episodic ataxia type 1


autosomal dominant nocturnal frontal lobe epilepsy
Lamberts-Eaton myasthenia
, ._
hyperel<plexia
E generalised epilepsy with febri~~eil,!Ie.~_(_9U_S_tl_ _

9. Neonatal and childhoo?


. stroke
ischaemic stroke results in a motor deficit in 75% of cases
is as common as brain tumour
is commonly treated by thrombolysis
has a recurrence risk > l 0%
.
_
.
risk is increased _in children with sickle cell anaemia

0 A
0 B
0 c
O 0
0 E

10. Febrile seizures

0
0

82

'

::"

,"'

A the recurrence rate is greater than SO% ..


B ha\e increased risk ofrecurrence if there is a family history of febrlfe ~
com'Uisions

_;

NEUROLOGY

0
0
0

C are associated with mesial temporal sclerosis


D occur more often in children with neurodevelopmental abnormalities
E should be investigated with EEG

II. Peripheral neuropathy is associated with


A abetalipoproteinaemia
B metachromatic leukodystrophy
C amyloidosis
D porphyria
0 E ataxia telangiectasia

0
0
0
0

:s
U)

12. The following side effects of anti-epileptic drugs are well .


recognised
0 A carbamazepimi and hyponatraemia
0 B valproate and weight loss
0 C phenytoin and ataxia
0 D lamotrigine and insomnia
0 E topiramate and psycJ:10motor slO\-:ing
13. In Sturge-Weber syndrome
A there is a naevus in the distribution of the facial nerve
0 B angiomas of the cortex occur in the occipital region 1
0 C clinical seizures are focal and contralateral to the side of cerebral
lesion
0 D there is an association with hypocalcaemia
0 E glaucoma is common

"1-,~

''

..

.83.

ESSENTIAL QUESTiONS ~::>R. MRCPCH

--------~------------------------

Best of F!v~ Questions


~ 1

::;lr;

:. ~ .... t,"lfrf"-<~"'

14. An 8-year-old boy is found at night making salivatory and


gurgling noises. He is unable_ to spe~k.but seems conscious. His
face is twitching on the left side. Which investigation is MOST
likely to confirm the diagnosis?
'

0 A
0 B
0 C
0 .D
0 E

Brain CTwith contrast


BrainMRI
EEG
Thrombophilia screen
Lu,mbar puncture
, . . . . '"

.1 ,

., ,

15. /!;.newborn pre;;;~nts with a w~ak suck and weak

cry. There is a .
fluctuating ptosis and hypotonia. The baby has attacks of

apnoea. The motber'has"no "medical hi.story of note. The'tests
for anti-acetylcholine receptor antibodies a're negative and
.
there is a decremen~l re~ponse on r!!pe'titive nerire sti.mulation'
electromyography. The MOST likely diagnosis is?' '
~-

0
0
0

o.
0

"""

,.

,.

"'

-.

'

A Transient neonatal myasthenia


B Autoimmune myasthenia
c Congenital myasthenia
D Mobius syndrome
E Spinal-muscular atrophy type 1

11'

j; .

'

"'

'.-<

'!"'>,.

-~

J' .

16..An overweight 14-year-oldgirlhas complained ofheadache on"'


awakening for ~veral weeks. She has papilloedema. Her blood
pressure is normaL Brain MRI is normal. Which investigation is
MOST likely to confirm the diagnosis?

0
0
0
0

A SleepEEG
B CT brain with contrast
c Magnetic resonance spectroscopy
D Lumbar puncture with manometry
E Visual evoked responses and electroretinogram

17. The following signs are seen in a child with an ischaemic


stroke: contralateral hemiparesis with face and arm more
affected than the legs; horizontal gaze palsy and hemisensory
deficits; language and cognitive deficits including aphasia and
apraxia; homonymous hemianopia. Which artery is the MOST
likely to be involved?

0
0
0
0
0
84f 3

A Cominon carotid
B Middle cerebral.
c Ariterior cerebral
D Posterior cerebral
E Posterior cerebellar

NEUROLOGY

18. A child presents with a facial palsy several weeks after a flulike illness she had in the school holidays. She also has
musculoskeletal pains and headache on direct questioning. On
examination there is also splenomegaly and generalized
lymphadenopathy. Which of the following investigations will be
the MOST useful to aid diagnosis?
,

0 A
. 0 B
0 c
0 D

Borrelia antibody serology


Magnetic resonance imaging scan
Anti-basal ganglia antibodies
Stool culture for viruses
E Serum treponema! serologic tests

I 9. A 3-year-'old boy presents with frequent falling. There is '


e\ idence of delay in motor milestones; He walked at 22 months.
On examination he has a shuffling gait and foot drop. He has
difficulty climbing stairs. Serum enzymes are norina~ Nerve
conductionstudies arenormal. CSF exam is normal.. EMG
reveals denervation and paucity of movement. The MOST likely
diagnosis is which of the fo1lowing?

0
0
0
0

A
B
C.
D
E

1)rpe 2 spinal muscular atrophy


Parkinson's disease
Werdnig-Hoffman disease
Emery-Dreifuss muscular dystrophy
Kugelberg-Welander disease

20. An infant presents at.age 4 months with hypotonia, weakness


and constipation. On examination the pupillary responses are
poorly reactive. Investigation reveals incremental response on
rapid (2o-so Hz) repetitive nerve stimulation and abundant
small motor unit potentials on electromyography. Tensilon test
is negative.,.-What is the MOST likely diagrtosis?

0
0
0
0

A Spinal muscular atrophy

B
C
D
E

Congenital myotonic dystrophy


Infantile botulism
Congenital myopathy
Myasthenia gravis

ESSENTIAL QUESTIONS FOR MRCPCH


'

''

....

21. A IO~year'-ol~girl presents ~th h~adache. It is 'l!nilateral,


f', pulsating and'fates 6/10 on the pain scale. The headache is
worse on physical exercise and helped .by sleep. Som'etimes she '
sees photopsias. There is e>ften nausea. ~at the MOST likely
diagnosis?.
r.
0 A Cluster headache
0 B Tension headache
0 C Hydrocephalus
0 D Chronic paroxysmal hemicrania

0 E Classical migraine

is

22. A child presents:in a coma. There is central hyperventilation. .


,; Pupils are mid-position andirregular 3-5
and fixed. The
"" vestibulo~orular reflex is intact. He shows bilateral deCerebrate
postu,r:ing to noxious stimuli. What is MOST likely ,to be the
posith~n/location ofthe.transtentor:ial herniation? , . ,
. '
,. .. ' ' '
j
0 A Reticular formation.

..
0 B Diencephalon
. : (,
0 c Mid-brain-upper pons
I . . '
r~
0 D Low pons-upper medulla
0 E Medulla

mm

I~

'\ ~ ~

. ..

~)

1 ' ~

...~

'

'

{"
-

- !

86

'

.1.l,~f.!

. ~ (L
:.

2i-l'

....-_of!

NEUROLOGY

Extended Matching questio11s


.,

23.
A
B
C
D
E
F
G
H
J

'"''"~;~;-,>_.,

;:.;.,'~~,U:. ,':'...1.-~:.-:-...-,.,;:t~~

, .

.. :K,-;,;r..:~k!..;':-~

Theme: Anti-epileptic medication


Gabapentin
Sodium valproate
Phenytoin
Clonazepam
Carbamazepine
Lamofrigine
Levetiracetam
Phenobarbitone
Vigabatrin
l.o. .azepam

.. . ~

.-

''

-, '

Select the most apprC?prfate a!'li"epileptic medication for each of the following
children. Each option may be used onaf, more than once, or not at all.

24.
A
B
C
D
E
F
G
H
f

.. r-_""

., .

1.. A 15-year-old boy seen in clinic who has had three witnessed
generalised tonic'-Clonic seizures in'the last 3 months: -
2. A 4-year-old girl seen in.dinicwho has had'severaf complex partial
seizures.
.~


. ,.
3. A 6-month-old with infantile spasms and a chaotic 'hjpsarrhythmic
electroencephalogram. Woods lightexaniinatibn reveals several
hypopismented patches. ' .
.
,., :
' '

;>:!

~--~1 ,-

-. .,!: .;'
' '
1
~- ,._. ' p~ttel'l!s in epilepSy syndromes

Theme: F;EG
,. .Hypsarrhythmia
4-5 Hz poly-spike and wave discharges
Diffuse slow and spike wave discharges
Burst suppression pattern
ltace pointu alternans
Occipital spikes- Ct;ntrotemporal spikes
No abnormalities
Electrical status epilepticus in sleep
Unilateral temporal spikes

'

i :

,.__

rrr, :. ~

For the following epilepsy syndromes choose the most likely EEG pattern from
those listed above. Each option may be used once, more than once, or not at
all.

0 I. Juvenile absence epilepsy.


0 2. Lennox-Gastaut syndrome.
0 3. Landau-Kleffner syndrome.

81

ESSEf\jTIAL QUESTIONS FOR

MRCPCH

25. Theme: Neurodegenerative disorders


A Segawa disease
B Hallervorden-Spatz syndrome
C Huntington's chorea
',
D Rasmussen's encephalitis
E . Sydenham's chorea
F Ataxia telangiectasia
..
G
Wilson's disease
H Subacute sclerosing panencephalitis
I
Lesch-Nyhan syndrome
J
Batten's disease
Match each ofthefollow_{ng clinical sceparios with the mostlikely'diagnosis.
Eachoppon may be used once! moie than once. or notal all.

"

:"

..

~...

...

'

'

I. A child presents \1\'ith a difficult ~o control ~ocal $eiiu~e. On .


. examination there is a hemianqpia and hemiplegia. An EEGreveals
diff~se parox}rsrrlal activity with sloW bac'kground and a brain MRI
. shows abnormalities in one hemisphere.
2. A 6-year-oid boy presents follo~ihg a perioa.ofno~mal development
.. 'ith rapid d~generat~cm. f?gidity occu;,s if!_l}is !c:>!Ver l_im~s w_i~h
,, equinovarus deformity in his feet. Cnoreoathetosis occurs in the , ,
musdes 'of the fac~; tongue and pal~'ite. Copper studies are normaL
Ophthalmology is unremarkable.

3. A 6-year-old presents with intermittent toe walking and disturbed gait.


'
There is diurnal variation in symptoms With improvement on
awakening.

'

'

.., 4

n.

:>

c'
\

...

'

'ss

. .t

,,

'j

1. Neurofibromatosis type 1

'AnS\ver:s: B D
'1.
'
Neurofibromatosist.ype 1 (NF 1) is an. autosomal dominant condition with an
incidence of J in 2500 to i in 3000. About half the cases are the first in the
family. ForCiiagnosis, two or more ofthe following are required:

}'

Six or more cafe-au-lait m<kules (> 0.5 em in children and E5 em in


adu 1ts)
.,
Two..-;},~. more ~ta.neou71subcut~neous neurofibromas. or one plexiform
neurofibroma

.
~~~ary,.g~.groil) fr~cklil)g
~. qptic;patnwayglioma

l' ~
~
'
I
....~ lWo oqnore ~i~.ch nodules
sony dysplasia

~: firsfaegre~ relative witfi NF 1


.
"

..... '

,;

~~-.

;,

..... ,

..

Brai~~~}~F~nt)y ~~ecomme,Atd.,~,<l,as a ro.titin<;_diagnostictest c;~ the


diagnosis is made by clinital assessment: MRl is currently not used for .
sereehil1'g foroptic gliorfia ir{iheabsence of progressive visual disturbance.
Unidentified bright objects (UBOs) caribe seen on MRI in childrett with NF l,
especially in children aged 8-16years. These can help to confirm diagnosis in
some cases. Plexiform neurofibromas undergo malignant change most
commonly in the second and third decade in about 10% of cases. Optic
gliomas are pilocytic astrocytomas that occur predominately in children under
7 years and are often indolent.. Manifestations include optic atrophy,
abnor'mal colour vision, squint. proptosis and hJP,Othalamic disturbance.

2. Friedrekh~s-ataxia- .. ------.-:. .. -- --.-~--~~--~-. -- - - - - - - - - - - AnS\1er:s: A B D

Friedreich's ataxia is the most common hereditary ata~ia. The triplet repeat
GM is located in the first intron of the frataxin gene on chromosome 9ql3.
The frataxin protein is a mitochondrial protein that plays a part in iron
homeostasis. Onset usually occurs around puberty with tlumsiness of gait.
.
Clinical features are: .autosomal recessive onset prior to age 25 years,
progressive limb and gait ataxia, absent tendon reflexes in the lower limbs,
and evidence of axonal and sensory neuropathy on electrophysiological
investigation~ The motor nerve conduction velocities are usually normal.
There is progression to dysarthria, areflexia, and loss of proprioception or
distal joirits. extensor plan tars. and pyramidal weakness of the legs.

89

ESSENTIAL QUESTIONS FC :::. MRCPCH

3. Cerebrosplnarnuid f"mdings
Answel5: A D E
CSF is produced mainly by the choroid plexus. It' should have a clear and

colourless appearance. Normal values in a child are as follows: protein 5-40


mgtdL; glucose 40-80 mgldL \\ith a CSF to blood ratio of0.6; cell count< 5 x
109/mL. In tuberculous meningitis the glucose concentration will be
decreased, with a high protein and increased cell count. rn Guillain'Barre. the
protein can be normal or raised. Values peak between days 4 and 18. There
are often oligoCional bands that indicate local synthesis of immunoglobulin
within the central nervous system. Oligoclonal bands can occur in multiple
sclerosis. in sub,acute sclerosing panencephaHtis and in Lyme disease.
Xanthochromia occurs With cell counts above 500tinm 3 This is a red-orange
colouration of the CSF that occurs after breakdown of red blood cells.
Oxyhaemoglobin is released and, can t?e detected 2-4 hours after a
subarachnoid bleed. It remains for 7- J0 days. Xanthochromic CSF also occurs
in hyperbilirubinaemia, hyperproteinaemia, hypercarotenaemia anq with
some drugs.

4: 1\lberose sclerosis
Answers: B D E

1\lberose sclerosis is a COt:Jlmon multisystem disorder affecting 1 in 5000 to 1


in 10, 000 newborns. Hamartomas or tumours mosfi:ommonly affect brain,
kidneys and skin. It is a dominantly inherited condition' but 6o-7o% of cases
are sporadic and represent new murations. 1\vo genes have been implicated,
TSCJ on chromosome 9 and TSC2 on chromosome 16. Renal

angiomyolipomas are benign hamartomas containing smooth muscle cells,


blood vessels, and fat They are often asymptomatic in children. By adulthood,
.complications can include renal failure~ haemorrhage and mechanical

obstruction. Normal imaging does notexcluae the diagnosis and MRI is


preferable to CT; although CT caD be helpful to detect calcification. The
predominant neurological manifestations include seizures,' learning , .
. , _. _
dtffic:;ulties ali.cll::l_~h~~9_1:1ral problems.

5. Acute disseminated encephalomyelitis


. -.

. .

Answ~r:

i ;

D . , .r

' '

~-

;, .

-~

Acute disseminated encepha}pmyelhis (ADEM) is the term given to a process


in which there is disseminated inflammation at multiple sites ~vithin the
central nervous system thought to occur at one point in time. Infections are
often i'l!?SOciated, especialJytQ,OSe <:f the, upper respirat?ry tract. Rel~pses 'can
occur, arid these are thought to represen~ part of the same acute immune .
proceSS; the term multiphasic ?CUt~ disseminated ensephaJomyeiitiS is used
(MDEM). If, .however, there appears tobe.a chronic imr!)une process with
relapses occurring indifferent sites at different tin;teT, a diagnosis of multiple

;'<.

90

-,., ..

. NEUROLOGY

sclerosis (MS) is made. There are many pathological and clinical similarities
between ADEM and MS. In children; ADEM 'is diagnos~d more freque.n.tly tK:im
MS. Some features help point tmitards ADEM as a diagnosis. Unilateral optic
neuritis occurs less frequent!}; in 'ADEM than in MS; for e~mple: MRI can,b~
useful. In ADEM there is relative sparing of peTi\;entricular white matter and
follow up scans can show partial or cor:nplete resolution in contrast toMS
where there may be new lesions. The EEG is frequently abnormal in both
groups 'With slow-wave abnormalities being a non-specific' sign of the
encephalopathic process. Thecorrect diagnosis is important as the risk
relapse and prognosis is very different bet\veen ADEM and MS and the best"
way to differentiate the cortditions is the subject of ongoing research.

of

6. , Rett syndrome

y ' ''

Am .ver.s: A CD E

Since the onset of genetic screening for th.e MECP-2 ge'ne on chromosome .
t.x92~ ~e p~7~o~e}or ~ett s{;:~;p!)le ,~~,b::n noted t~. be bn?~d~r:d ~on- .
speclfic.The mhentance 1s presumed tOpe x:lmked dommant With early male
J~thaJ}ty: ~upportive criter!2 f?r~;qi.~gnosis)nclude los~ o.f ~cqu!red.?.~il!s.
acqmred rrucrocephaly, de\"elopmental delay, and aut1st1c features with ...
stereo typic himd movements. The' head growth usually slows between 2-4
months of age but is often unrecognised. Failure to thrive is common in
children with Rett syndrome. Dysfunctional swallowing may necessitate
gastrostomy tube placement. Fluid intake needs to be increased to
compensate for drooling and hyperventilation.
1. Inheritance of neuromuscular disorders
Answer.s: A B

Facioscapulohumeral dystrophy (FSH) is autosomal dominant and shows


marked clinical variation. It must be carefully differentiated from some of the
forms of congenital myopathy. Duchenne muscular dystrophy is the most
common and one of most severe forms of muscular dystrophy and is an X linked-recessive condition due to a mu_tation on chromosome Xp21. The
mutation results-in a lack of dystrophin protein. About a third of cases arise as
new mutations. Familial periodic paralysis is autosomal dominant.' .
.

8. Ion channelopathies
Answer.s: All true

Neurons. nerves and muscles communicate through electrical impulses


mediated by rapid transit of ions through channels. Many diSorders have their
basis in dysfunction of ion channels. Channelopathies are sub-divided
accOrding to the ion channel involved in the molecular defect. The symptoms .
are often paroxysmal but are often associated \\1th progressive dysfunction
and disability. Sodium channelopathles include the autosomal dominant

91

'ESSENTIAL Q'-'::STIONS. F:Ji'l. ~-'IRCPCH


>

(.

~..

conditions hyperkalaemic periodic paralysis and paramyotonia congenita: ,


F'Otassium cl"!annelopathh~s include Ai1derse!' syndrome (dysmorphism,
.
ventricular arrhythmia and periqdic paralysis). Chloride and calcium channel
disorders are also well described.
.
.

9. Neonatal and childhood stroke

Answers: A B D
Childhood stroke is as common as brain tumour, occurring in 2.5 to 13 per
I 00, 000 children and in 40 per l 00, 000 neonates. Around half the children
presenting with ischaeniic arterial stroke'will have sickle cell anaemia or
congenital cardiac disease. Other predisposing conditions include
immunodeficiency, homocystinuria and bacterial meningitis. In the neonate'
the pathology usually suggests an embolic aetiology. Venous sinus
thrombosis is particular,ly, c~mmon in the neonate altryough it can ocqu. at all
ag~s. Haemorihagic stroke may be caused by coagulopathies or structt~ral
lesions such as arteriovenous malformation. There has been <m argument to
give thrombolysis for ischaemic st.rokfi within the fit st 3 hours. very.few;
children will present within this recommended time to a stroke centre for
accurateassessment and management. Around 75% of children with .
ischaemic stroke h~ve a residual.motor deficit. They may.~lso present with ,
cognitive and behavfoural difficulties. M~ifiable risk f?ctors need to be: ..
identifiea to. weveht r~cu'rrence: .~:. . .
'
'';

I 0. Febrile seizures

Answers: B CD
Febrile seizures are one of the commonest neurological disorders of.
childhood. The diagnosis is clinical and an EEG is nolindicated.,The
outcome is usually good. The recurrence risk for febrile conwlsioris is ..
increased by betvveen 20-40% in younger age$ and if there is a familv
history of febrile conwlsions. The risk qf epilepsy following febrile S'eizures
.....Js .multiplied by a factor of four .when.coJ11pared.to:the-population. Risk .
factors for epilepsy include posit.ive family history, and prolonged or : ,.
atypical febnle sc;iwre~. Remer;nber, if!.children with e;pilepsy fever may .

.
lower the seizure threshold. ,.
t ?'' ...

11. Neuropathy

Answer: All true


Hereditary neurop"athies co~stitute a complex heterogeneous group of
disorders. The); freqliently ha\'e insidious onset and slow indolent
progression:T~ere are sey~ral_ main groups;- afew exa~ples are: hereditary
motor and sensory neuropathies, such as Ch;ucot-Mane-Tooth;
, ,
~
predominantly sensory:':'europathies (~.I IV1: ata~ic neuropathies such
as giant axonal neuropathy: metabolic~disbrders; and degenerative disorders

i?

92

. ,..

.l

.L

with peripherarher:~ 'involvement: The question includes examples of' ' .


conditions associated with neuropathy
.
'

12. Side effects of anti-epileptic drugs


Answers: A CD E
There are many, side effects listed for each of the antH~pileptic drugs but.these
are some of the more commonly reported ones. Valproate is associated w.ith
weight gain, hepatic ::r;zyme e:evation. alopecia and tremor; it can also cause
thrombocytopenia and hyperammonaemia. Carbamazepine can cau2e the
syndr9me of inappropriate anti~diuretic hormone (SIAD~) and therefore low
sodium; other side effects are diplopia. vertigo. sedation, headache and
stomatitis. Phenytoin can cause ~rowsiness. ataxia. diplopia, hypotension,
hirsuti~ m and gingival hyperplasia to 1\st just a few. Lamotrigine may cause
rashes. dizziness. and ataxia. as well as headache. vomiting and insomnia.
Speech disorders. renal stones and sedation are side effects of topiramate, as'
wel(as psychomotor slowing. . . .
:
.

13. In Sturge-Weber syndrome


_. ,AJ;lswe,rs: B C E
,

sturge;;::v~Jeb,~r is a,S)'Ji<:J!on;e;,.,~.hjiJ~.r...~!b!~es f~cial naevus in the trigeminal~


,
distrib~tion, hemiparesis, unilateral seizures and in more than half,ofcases .
learning difficulties. Calcium deposition occurs adjacent to abnorf[,lal vessels'
often te~med tram line calcification on X-ray.

14. C: Electroencephalogram (EEG)


This is the typical presentation of a child with rolandic epilepsy, also te~med
benign childhood epilepsy with centrotemporal spikes. It is also common t<?
have brief daytime hemi-facial twitching with increased salivation. This is a
common condition accounting for about 15% of children with afebrile
seizures. It can be recognised on clinical grounds and hasa distinctive .EEG.

15. C: Congenital myasthenia


The clues to the diagnosi!> of myasthenia are the fluctuation of weakness and
the decremental response with repetitive nen'e stimulation EMG. There are
three main groups of myasthenic syndromes: the autoimmune form (often
known as juvenile myasthenia). congenital myasthenia gravis (sometimes
called genetic), and tran~ient neonatal myasthenia. The latter is due to :
transplacental p;:~ssage of antibody to the acetylcholine receptor in mothers
with myasthenia gravis. Clea.rance of antibody can take 5 months after birth.
Congenital myasthenic syndromes are a heterogeneous group and the defect
can be pre-synaptic, synaptic and post-synaptic. It is important to remember
these, in the differential diagnosis of apnoea in the newborn. Spinal muscular
atrophy type 1 involves degen.:'ration of motor neurons. There is symmetrical

ESSENTIAL QUESTIONS FC !'\ MRCPCH

weakness caused by denen-ation. with associated muscle atrophy. The


neonate. often \Viii present \,ith hypotonia, but the face is usually strong with
full movement. There is no loss of sensation. The EMG demonstrates a
neuropathic reduced recruitment of voluntary muscle unit potentials.
Molecular analysis is now t:sed to make the diagnosis and over 95% of cases
have deletions in the sunri\at motor neuron (SMN) gene. Mobius syndrome is
charaCterized by bilateral facial weakness. There is often paralysis of the.

'
abducens nerve associated \\ith iL

16. D: Lumbar puncture with manometry


Benign intracranial hypertension is a headache syndrome showing raised
cerebrospinal fluid pressure in absence of mass lesion or dilated ventricles.
There ar~ usually normaltT:.dings on examination except for papilloedema
and occasion1a,I Vlth nerve palsy. The condition requires close monitoring of ,
visual acuity as pptic nervej.amage can occur as a result chronically raised
pressure. Treatment can imolve repeat iumbar punctures to remove
cerebrospinal fluid, acetazolamide and analgesics. ; '

of

1 7. B: Middle cerebral

Common carotid occlusion may be asymptomatic. The anterior cerebral artery


occlusionspresent with contralateral hemiparesis involving the leg to a
greater extent-tha!ithe arm and face. There can be a contralateral grasp reflex
and gait disorders. Urinary incontinence is a feature. Posterior' cerebral artery
occlusions involve contrala.teral homonymous hemianopia. !here may be
memory loss and dyslexia.

18. A:. Borrelia antibody serology


. This clinical presentation is consistent with the diagnosis of Lyme disease.
This is a m'uttisystem disorder caused by a tick-tranSJilitted spirochete,
Borrelia burgdorferi. Localized infection occurs folloviing a tick bite, which is
often forgotten about or not noticed. This is foll<?~ed b_x d_i~seminated
infection for up to 9 months. Late or persistent infection can occur for months
or years. Neurological manifestations include progressive encephalomyelitis,
focal encephalitis, dementia, seizures and polyneuropathy.

19. E: Kugelberg-Welander disease ,


The usual onset ofKugelberg-Welander disease is after 2 years and children
have often \valked late. There is slow deterioration that re'sults in scoliosis .
and wheelchair dependence. Type 2 spinalmuscular atrophy,
Werdnig-Hoffman disease. and Kugelberg-Welander disease are all forms of
spinal muscular atrophy, each having degeneration in the anterior horn celi of
the spfnal cord, and in some cases the motor nuclei of the brainstem. Type 1 is
the mos~ severe form and can present with reduced fetal movements or
neonatal hypotonia. l)rpe 2 is intermediate severity but the children are
94

NEUROLOGY.

usually unable to walk unaided. Muscle biopsy ancl.molecujar.genetic studies '


. for~,he?MN gene c9nfirms, the diagno?i~. Parkinson's disease,rarely.pre?ents ,
in childhood. " .
. ~- ,
: ., .
'~-

. .

-t,}J1'

' ' .," :

\.

'-~

l,_

-~'~~

20~ C: Infa~til~~otuHsm ,1 . , ,
., ;. 1 >:<
.' .
,. .. ,_. ""'r
"'
The condition's listed above can all present with peripheral hypotonia; Infant' .
botulism most commonly:presents at 2-;6 months.with the s_yrt.ptoms' ' . ~
described above. Clo~<ridium botulinum is' a Gram-positive spore-forming
organism found in soil, agricultural products, and honey. Respiratory failure .
can occur. The toxin.is released and binds irreversibly1to J:!re~synaptic, ,. -l~t . ,
cholinergic'nerve'terminals and' dis~;upts the exocytosis oi.acetylcholine, .f ' . ~
Positivestool. culture o_rJsolation ofioxin.fs d.ifticult because of constipation. n
Manar ~~e~~is supportiv~., PupiIl~ry respo-~se.:i sh?uld-not_be affe,c'ted ln the .11
other c. ~nditions. A positive.Tensil_ot;qest \y-ould ~t;._expected wit)l,myasthenia n'
syndromes. The EMG in congenital myotonic dystrophy may.de;monstrate;J . ;
myotonic pot~nt~al~.qril)g,at highratesJhatwax a_I;ld warye in frequency-an.c!
ampliJude.
~-- -1 . .
_. ~ ...
,
.
"~- _,, ...,'11 :1

, :.,

...... .

-'

"

. , .- .r,.

"

21. 'E: ClassiCal migraine -

.<

1;'t.( _

'"

'

,I

,.; .

,..; t. ,

t1

~-~~, ,._.

-~~

.,.< , '- :

,,

Headache disorders are common in childhood,. This is a description of classic .:


migtaine;~.,tith,'aut~ o(11a~QinglighJ~:;Giuster headaches are more ,cotpmon iii -

males, most often occurring in those in their late 20s and rarely in children.
"
Periodicity is the predomiqanHeature with chronic episodes of headaches ,
lasting for 2-3 months;-there is usually no aura; and the pain is excruciating '
often in the trigeminal distribution and may be associated with lacrimation,
sweating. ptosis and nasal .congestion. Tension headaches are diffjJse and :
bilateral often described as pressing or a being like a tight band; auras are rare.
Chronic paroxysmal hemicrania is similar-to cluster headache but the paih is " '
usually in or around the orbit; headaches last for. a short time,(S minutes) but
occur many tim~s ~ day. Hyd~ocephalus can present as headache, This is 1
usually wors,e in t~e morning. There may be associat~d gai~ abnormalities and .
crantalnerv~ ~igns n<?tably,VIthnerve palsy and qiplopia; There is often .loss of ;:J
fine motorcoor~ination. Papilloedema is frequently present i; .:
..
,,,

22. C:

Mid'brain-uppe~

pons

,, ..... ~ .,

<!~

~-

There is a rostral-;e~mdal progression of signs seen with ooth Iateralana


central trahst~ntorial;herniation,,indicating worsening of the hernia'tion. This
begi!'S With <:Jit~.nceph'!JiC iO\'O)Vement followed by mesencephalic; pOntine, ''
and finally med1,1llary involvement The signs described above are seen with.
involvement ofthe mid-brain and upper pons. With .herniation auhe level of:- 1
the reticular formation there is altered consc-iousness. With invohement of
the diencephalon there is drm\sin~ss oragitation with Cheyne:..stokes . - I
respirations. Pupils are small but brisk. Eye movements can be roving. the
vestibulo-ocular, reflex is weak or brisk. There is loss of vertical movement.

.;.

95

~:~;tl)>

. .. ftiJt.

--<

.,
(I)
(I)

.~~

.. ;;:, :.

. :~t~:~~

Ut

ft!.

ESSENTIAL QUESTlONS F8~ MRCPCH

Decorticate posturing to stimuli occurs. Plan tars are extensor. With


involvement of the low pons-upper medulla the patient will' be in a coma with .
tachypnoea. Small mid-position fixed pupils are present. Vestibulo-ocular
reflex is absent. There is fla.:cid flexor response to noxious .stimu~i. With
.
involvement of the medulla the patient will be in coma. Breathing will become
apnoeic; then stop. Pupils are fixed and dilated. There is novestibulo-ocular
reflex. Limbs are flaccid 1.vith no deep tendon reflexes. .,
., - '
,.

23. ANTI-EPILEPTIC MEDICATION

The identification or a seizure type and or an epilepsy syndrome nrovides


information on prognosis and guides the choice of drug treatment when this
is deemed appropriate. It is always .important to consider the safety profile of
the treatment arid available formulation. Monotherapy at the lowest dosage
to achieve control is the ideal with any drug treatment.

1 . B - Sodium valproate
.
For idiopathic generalised epilepsies-sodium valproate is usually first-line
choice of medication. The exception is for teenaged girls because it is
necessary to consider the nsk of polycystic ovaries as well as teratogenicity.
Lamotrigine would be preferred in this group.

'
2." E - Carbamazepine
Carbamazepine is first-line therapy for partial seizure disorders.
3. I - Vigabatrin
West syndrome is characterized by infantile Spasms, hypsarrhythmia and
developmental delay. The first-line medications for this group are
prednisolone. adrenocorticotrophic hormone. or vigabatrin. With an
underlying diagnosis of tuberose sclerosis vigabatrin is preferred.

24. EEG PATTERNS. IN EPILEPSY SYNDROMES


1. B- 4..:5 Hz poly-spike and wave discharges
';
Juvenile absence epilepsy begins in the early part of the second decade.
Absences are less frequent but may be longer than in childhood absence
epilepsy, Generalized tonic-clonic seizures (GJC) often occur. The frequency
afthe spike wave discharges is often fastenhan the 3Hz seen in childhood
absence epilepsy.
2. c -Diffuse slow and spike wave discharges
.
Lennox-Gastaut is the best-known example of symptomatic generalized
epilepsy. Multiple aetiologies have been identified including tuberose
sclerosis. cerebral dysgenesis and Batten's disease: Seizure types include GTC,
absences. drop attacks, and myoclonic jerks. Characteristic EEG is general'
irregular 1.5-2.5 Hz sharp and slow-wave activity with slow background.
During sleep, bursts of sharp discharges around I 0 Hz can occur:' ' ~ :. '
3. I - Electrical status epilepticus in sleep
Landau-Kleffner is an acqt.:ired epileptic aphasia in which regression of .' "
language skills occurs. The EEG abnormalities can show multifocal spike or
96

'

Spik~andwaVe in'thetempo~<;>parietaf region Whilst aWaKe, and frequent


:generalized, spike waye discharges (ESES, electrical. sta_tus epil~pticus) in
sleep. :.

. 25: .NEURODEGENERATIV DISORDERS

1. D.-:- Rasmus~e~'s encephalitis


:Rasmussen'{enc~pnaiitis typkally presents with focal seizures which can be
frequent or even continuous. Rasmussen.(jescribed aS):Vdrorne ofseizure;:;,
spastic paraly~is an? Jear)iing diffifulties,asso~iated with chronic enceP.haliti~:;
The brain irm\ging ean be~nbrtnal eaT!y in the disease. rater in the course.
cerebiai sWeliing canbeseen with higli:int~psity,.Iesis>ns in the basaisanglia
and periventricui~X whit~'hlatteron T2-weighted,j~aging. The diagn(}sis. .
would i eed to beconfirined onb'r~Hn biopsy in a clinically suspected case,
2. . B ,->}lallervorden-Spatz syndrome ,
Halie.Vdrden.:sj)'atz ls.a' rare degenerative disorder inherited as a recessive .
trait. Thereis usually progressive dystonia, rigidity.and chor,~oa!hetpsis.
peat~ us~~a.lly ocfurs Y earJx adultho?d I~aging shows.iesidns of the ~globus
4
pallidu,s>Ne~ropathology rev~als.excess accumulation or iron-contajn,ing
pigments in the gi:Rbus pallidus ana substantia nigra.
3. A.7" Segaw;:t disease ,
.,., 1
.
,
.
s~ga\~~,~~pj~~;i~.2J,~~,twise k.
dop~;responsive dystonia due to u1e
clmical response. It is more c . . 1h females and typically. presents around
the age oi 6 years with dysto~k postuth1g of the lower limb: It improves with
a smalldose oflevodopa. .

97

'

,,

'

... ,

..

j..

~.

..
'

'

'I

' ~.

,,

..

'
i

'

_)' 1

'

,,.

.l

'
'

'
A'

,,

,.

'oft.

,,

cJ"T ..

. ..

.~

- - - 1_

~Mike

Champion

b1'iSI.'"m"~L:..-;z~.....~::~~~---="""-'~~--~~,!;.S=.::,~, . lr

'

,1., Tlf~Jpllowing:is.true:of etiniti~;pigm_entosa"


0 A the qjsease process predominantly affects theco11es -~.. .
0 B+ is a~recognisedfeature oftaJrenc<:!...:Moon-Bi~dl syndrome
o C night blindness is a presenting feature .
,
'
0 D oteritral scotorha' ls the characteristic resultant visual field defect
0 E disease progression is significantly retarded by vitamin C
'J
~up~!em;ntatj:9n
'
yf '

-~ .

'.

..

..... '

2. The following present with a painful'red eye;


0 A juvenile rheumatoid arthrHis
B Toxocara canis infection '
0 .c glaucoma
0 b herpes simplex
0 E toxoplasmosis

'.\

'

3. The features of acquired Homer syndrome in a 3:.year:.old child'


following cardiac surgery include

0 A mydriasis
i
0 B ptosis
,0 c ipsilateral anhydrosis
0 D normal PtiRiHary_r~flexes_
l
0 E hetero!=hromia iridis .
4. Glaucoma is a recognised complication of the following

0 A Wilson disease

B Horner syndrome

.,

0 C retinopathy of prematurity
0 D Marfan syndrome
0 E neurofibromatosis type I

ESSENTIAL

FOR t1RCPCH

..
5. The following are causes of ~on genital cataracts

0
0
0
0
.0

A maternal diabetes
B galactosaemia
C varicella zoster
D hypothyroidism
E ~maternal ~teroids.

'I

6. The following is true of retinoblastoma

0 A
0 B
0 c
0 D
0 E

leukocoria is the most common presenting sign


the affected eye is usually painful
most cases result from a sporadic mutation in the retinoblastoma gene
the risk of recurrence in unilateral disease in the unaffected eye is 10%
patients are at risk of developing secondary malignant tumours at
distant sites in later life

- .

7. Blue sclerae are a re~ogriised f~atureof the following

0 A
0 B
0 c
0 D
0 E

osteogenesis imperfecta
trisomy 21
Ehlers-Danlos syndrome ,

_, ..

~-thalassaemia

ataxia telangiectasia

>, .

8. The following are inherited in an autosomal recessive fashion

0 .A Leber's hereditary optic neuropathy


0 B keratoconus
0 .c J Waardenburg syndrome
... ..
0 D Leber's amaurosis
0 E Joubert syndrome
--,

'
~.

9. In congenital rubella syndrome

0
0
0

"

,J:

'

.- ._

A infection in the first trimester is associated with the gr'eatest likelihood


of an affected fetus

r '

B .pregnant seronegativewol'!1en-sh0tHdbe-immedtatelyiinrrtuntzed
C cataract is the commonest ocular feature
D the presence of rubella igG antit)odies in the iftfant~c~nfirffi~ the
diagnosis
'
E the electroretinogram is usually normal

1 o. In orbital cellulitis

0
0
0
0
0

...100

A
B
C
D
E

blood cultures are often negative

' '

Staphylococcus aureusis the most common cause in infancy


incidence is greatest in children over 5 years
investigation includes sinus X-ray
,
intravenous antibiotics alone are sufficient to clear the majority of
cases

.......

OPHTHALMOLOGY

f,

11. In chlamydial conj!lrtctiviti'S) , -' i, . ~:.,


oral erythromycin is the treatment of choice
cytopl<!s,ruC'inclusion botlies ad!charattetistic .t:::~
it usually presents within 48 hours of birth
asymptomatic par~nts require treatment ,
concurrent pneumonitis is common

0 A
0 .B
0 C
0 D
0 E

":

' ...

l.

r . '.. .

;" ~ "' .

12. Ptosi$is afeat.ure..._of


A M6biussyndrome
B Kearns-Sayre syndrome
C .j<abduc~Jl~P~Jsy
~t. tr;, ~
_,.) 'li"
Q D. coHodio!1 baby~
: \'" . .,~ . ..: ,
0 E Marcus-dunn jaw winking syndrome

0
0
0

13. Vernalconjunctivitis

,-_.

0
0
.0
0
0

is commoner in girls
is assoeiatecl,with atopy
is ~eas8nal "
i J
requires long-term corticosteroid use
E has photophobia
a common feature.q ..

0
0
0
0
0

A birth weightless thanl 000 g


B female
c blood transfusions
D pre-term gestation
E intracranial haemorrhage

111!111

,.

,-;,


:r
t.

A
B
C
D

"

*' jJ~iJ~r!i#~}j.~
A ~"''<'
... , .
14. Ris~ factor~ for rejinopathy of prematurity include

15. _Leukocoria is a recognised feature of


A cataract, ,
-r0 _J3 . to:x;ocariasis
0 i: retinoblastoma
..
0 D Hurler syndrome
0 E persistent hyperplastic primary vitreous

_,
!I:

,,

O,.

"

.,. .

101

ESSENTIAL QUESTIONS FOR MRCPCH

Best of Five Questions

--

16. What is the MOST likely visual field defect in a child diagnosed.
with craniopharyngioma?

0
0
0
0

A Blindness in one eye

B
C
D
E

''

Homonymous hemianopia ;
Bitemporal hemianopia
Peripheral field defect
Central scotoma

..

17. A 2-day-old neonate develops a copious purulent discharge


with associated lid swelling. What is the MOST likely causative.
organism?

0
.0
0

A Chlamydia

t:.

Neisseria gonorrhoeae
C Haemophilus lnjluenzae
D Staphylococcus aureus
B

t:

E Herpes simplex

18. Examination of the pupillary light reflex in the left eye of a '
7-year-oid girl reveals an absent direct reflex and normal
consensual reflex. These findings are MOST consistent with
which one of the following?
r "' _,;.;:;_

0
0
0
0
0

A Left oculomotor nerve palsy

B
C
D
E

Right oculomotor nerve palsy


Left Horner syndrome
Left optic nerve lesion
Right optic nerve lesion

~c

~..

..

19. What is the treatment of choice for Toxocara canis infection in a


3-year-old boy presenting with leukocoria. Fundoscopy reveals '
a large lesion close to the. macula with marked inflammatioil . - ..

A-Cryotherapy.. -- . -

0
0

B
C
D
E

Corticosteroids
Laser therapy
Observation
Antihelminthic drugs

"

..

. . . .

. "

.r

. _.L -

- ...L

OPHTHALMOLOGY

20. Viral conjunctivi_ti~ is ',MOST commonly plU~f!d by which of the


following? . _
'

0
0
0
0
0

A
B
C
D
E

Adenovirus
Herpes simplex
Patamyxovirus
Corona virus
coxsackie virus

2.1. A 5-year-old child presents With acute onset ophthalmoplegia


ofthe right eye with associated proptosis ari'd ptosis.foliowing a
heavy head cold. The MOST likely diagnosis is which of the '
following?

i"f

~I

A Orbital cellulitis
0 >s Mastoiditis' ' ~
0' C Cavernous sfm.is'thtomoosis' .
0 D Frontal sinusitis
0 'Cerebral ab:SCess
(~

-,~,:,. ,1~,

...

-...

1~j.

. "..\'

22. The MOST appropriate test for assessing a.cui,ty_in a


child is which of the folJowing?

0
0
0

A
B
c
D
E

Sheridan Gardner test


Standard Snellen chart
Catford drum
Preferential looking test
Graded rolling ball test

3.~year-old

t.

.J.

,,
t~

. }. ...

.
I

~.

'
~

\ . , j .

'.

1-03

. r:
_ _ ;,_'-~."[
'

-..

. ESSENTIAL QUESTIONS FOR. MfKPCH

tu)

Extended Matching QueStions


,~<::~-:'

~c

' -.,.,_..

,,

23. Theme: Eye movements

t'O

A
B

~-- ~.

c
D

, F
G

Abduction of the right eye


Adduction of the right eye
Abduction of the right eye on upgaze
Adduction of the right eye on upgaze
Abduction of the right eye on d~wngaze
Adduction
of the right eve
on dovvngaze
.
""'
Upgaze
Downgaze
All directions

..

--

.;~

...

!.

..
.;

i '" "

...
~~

For the three causes of disconjugate eye movemencs described below, select the
position of the eye where diplopia is maximal from lhos~ listed above. Each;
option may be used once, more than once, or not m all.
tJ

0
0
0

I. Right-sided Brown's syndrome.


2. Right-sided abducens nerve palsy.
3. Right-'sided trochlear nerve palsy. -- ' ~ t
f

t ... rt.

,{;,

-.

24. Theme: Masses around the eye


A
B
C

D
E
F
G

Rhabdomyoma
Hordeolum
Neuroblastoma
Cyst of Moll
Meibomian cyst
Dermoid
Obstructed tear duct
Capillary haemangioma
Cavernous sinus thrombosis

- ' t, .

Select .the best option from the Jist above to match the followingdesr:;riptions of
masses around the eye. Each option may be used once, more than once, or not
at all.

0
0
0

; 104

I. A painless hard nodule in ari 8-year-old boy near the upper-lid margin

that has been present for some weeks. but has recently become red and
inflamed.
2. A tender red superficial swelling on the outer eyelid margin in a 3-yearold girl.
3. A 4-day-old boy presents with a bluish mass inferior to the medial
canthus.

OPHTHALMOLOGY

25. Theme: Drug adverse effects


Phenytoin .
Amiodarone
Paracetamol
lbuprofen
Ethambutol
F
Rifampicin
G
Vigabat.rin
.
H
Corticosteroids _,.
insulin

Match the following adverse effects to 1he dhigs listed above. Each option may
be used once, more than once. or not at all.

U')

A
B
C
D
E

0 1.' \ isual'field defect.


0 2. Colour blindness:
0 :3. corneal deposits.

. . I

./

'.

'1,

..

! "

,,) .

lOS

::s

'

"

-.

'

ansWers
.

'.

"''

1. Retinitis pigmentosa
Ansvvers: '8 C

.. '

t'

Retinitis pigmentosa is a progressive disorder predominantly affecting rod :,


cells. and therefore poor night vision with abnormal-dark'adaptation}s~ '
common presenting symptom. Peripheral vision is affected first, where the '
rods ~rp concentr~~ed,_resultingin sparing ()f the central vision and,the',,l\.
development of tunnel vision. Afou.nd Sp% of ca:;es are sporadic, 20% are ."
recessive. 20% are dominant and 10% are X-linked. Associations include ~-
abetalipoproteinaemia, Refsum syndrome, Laurence-Moon::-Bied!'syndrome;
mitochondrial disorders, Friedreich's ataxia, neuronal ceroid iipofuscindsis, .
Usher s)IJldrorne. and Chediak-Higashi di,sease. The retinitis-pigmeritosa ~(
abetalipbproteinaemia may respond to vitamin E supplementation.

2. Painful red eye


Answ'ers: A

cD

The uveal tract consists of the iris. ciliary body, and choroid. Anterior uveitis
(anterior chamber and iris) and pan-uveitis are painful. whereas posterior .
uveitis (choroid and retina) is painless. and therefore disease progression may
be more marked prior to diagnosis in young children. Causes of posterior
uveitis include infection by Toxocara and Toxoplasma.
3. Homer syndrome following cardiac surgery
Answers: B CD

Horner syndrome results from a lack of sympathetic innervation of the pupiL


The pupil is constricted (miosis) with ipsilateral ptosis. Heterochromia is a
feature of congenital Horl)er syndrome or early surgery involving the eye in
the first year. Pupillary reflexes are normaL

4. Glaucoma
Answers: CD E

Glaucoma (raised intra-ocular pressure) results in expansion of the eye size in


infants under 3 years of age, and damage of the optic nerve head. Congenital
glaucoma results from abnormal drainage of the anterior chamber due to
abnormal angle structures. Secondary causes include eye diseases that affect
the anatomy making outflow obstruction more likely, and these include
aniridia. Sturge-Weber syndrome, Lowe syndrome. neurofibromatosis type I.
Mar fan syndrome, retinopathy of prematurity, and rubella syndrome.

106

OPHTHALMOLOGY ..

5. Congenital cataracts
Answers: All true .
50% of congenital cataracts are idiopathic; The most cominon identifiable.,, .
cause of congenital cataract is autosomal dominant inherited cataract, and
therefore the parental history and examination need to be thorough. Other
causes include maternal disease such as diabetes, and the maternal ingestion
of exogenous steroids. Intra-uterine infeCtions (3%)'and systemic disease (5%)
account for oniy a small number of cases.
' \ .

6. Retinoblastoma
Answers: A C

Retinoblastoma arises frnm the l'etinausuallypresenting with leukocoria ..


(White .lUpil)'or squint. Usually the disease is painless, unless tumour necrosis
results in inflammation. Calcification is common and can be detected by t;:r
. scan of theorbits. The retinoblastoma gene is located ori chromosome I 3 an'd
is a growth suppressor gene. Disease resulrs from a double hiL In hereditary
cases, one affected gene is inherited and the second undergoes mutation after
conception. sporadic cases result from spontaneous mutation of both gen~s:
Cure rates are 90%in uni~ocular disease. Large tumours require enucleation.
Smaller tumours can be treated with radiotherapy or chemotherapy. The risk'of
recurrence in the unaffected eye .is 20%, and falls markedly after 2 years.
Patients are at risk of developing late secondary tumours, even at distant sites.

7. Blu~ scie~ae
Answers: A

Blue sclerae are not the soulpre$erve of osteogenesis imperfecta, but are also
seen in other conditions with connective tissue involvement. These include
Ehlers-Danlos syndrome; h,Ypophosphatasia, Marfan syndrome, 1\lrner
syndrome, arid trisomy 18.'
.

'
~Blue sclerae are seen in the most severe osteogenesis imperfecta (types
and Il)atid arenot a feature-of the milder dominantly inherited formscThe
bluish appearance results from thinning of the sclera, which allows the black
choroid to be slightly visible. fj-thalassaemia is associated with icteric sclerae
secondary to haemolysis.

.'

'

I~ -.

. ,{_

'I

8. Aut()somal recessive inheritance


, Answers: D

Leber;samaurosis is a'n autosomal recessive, childhood onset, retinal


degeneration that results in blindneSs from birth or early childhood. Joubert
syndrome is similarly inherited; this consists of retinal dystrophy, cerebellar
vermis hypoplasia, nystagmus-and episodic tachypnoea. Leber s hereditary
optic neuropathy is a mitochondrial disorder resulting from a mtDNA'point
mutation. It is characterised by painless acute or subacute bilateral central
107

' .:-t~ ~.q~~.~


'. :'
. .I:1..~i
.-.

__

.,_,

ESSENTIA:.. QUESTIONS FOr. M.RCPCH

_-

vision loss occurring in the second to fourth decade. It is more common in


males. Keratoconus is sporadic, or dominant in < 1o% of cases: There is
corneal thinning, which results in localised protrusion, proaucing irregular
astigmatism and myopia. ~\'aardenburg syndromeis an autosomal dominimt
COn9ition with sensorineural hearing lOSS; f~cial dysmorphism, and abnormal
pigmentati.~n_ o,f eyes. hair, ind skin:
,, .
~.

c
td

.;

- ""

-,_'

., . j

'

""

, 9. Congenital rubella. syndrome


;;

........

,11

".

,4

'

Ji .

\'

Answers: A E

,..

"m,

The risk of congenital rubella syndrome diminishes as the pregnancy


progresses. and the full syndrome is rare if the infection 'occurs(after the
fourth month of the pregnancy. Seronegative pregnant womeri should be
immUf!i_seq:post-partum. Pigmentary retinopathy;is the most common ocular
marlif~station, ~owever the el.ectroretinogram is l.!_?ually, n9!mal, .indicating
that the pigmentary mottling does not affec~ the ret in~! pigmentary
~pit~elium .. f?i~gnosis is confirmed on isolation of tl:e virus from. i
nasopharyngeal aspirates,
from 1.,1rine,
m cerebrospinal
tl)Jid.lgG
antibodies
+-
,f
.-'j-.,.
;t,
''II'
may be maternal in origin. The presence ofboth.lg(J and IgM antibodies
sugg~st.s.recent infe.ction. catal]lct occursin 20-30% of patients.
.

..

~.

1"'

~~

>:

"

,. , . . .

I 0. Orbital cellulitis
.

-- '""

'

AnSlvers:A CD E

Orbital cellulitis occurs mos(commonly in children aged over 5 years:


compared to pre-septal cellulitis (cellulitis involving the external soft tissues
and eyelid) which is more common in younger infants (mean age 8 rrionths).
90% of cases are secondary to sinusitis and therefore sinus X- rays are
essential. Some advocate computerised tomographic imaging, which sho\vs
sub-periosteal and orbital abscesses that are not apparent on plain films.
Haemoph/lus is the,comJ'!'Ionest organism in infants. and high-dose
intravenous cephalosporin is the treatment of choic~. curing over 60% of
cases. Fungal orbital cellulitis is associated with immunodeficiency and
.. -diabetes.

.
'.

'.

11. Chlamydial oonjunCtivitis ..


~

'

'

Answers: A B D

Chlamydia! conjunctivitis typically presents after,48 hours, unlike gonOC!JCC~I


conjunctivitis, and may present as late as 14 days or more: Diagnosis is '
.
confirmed on Giemsa staining of conjunctival scrapings to reveal cytoplasmic.
inclusion bodies. Oral erythromycin is the treatment of choice having the ,
advantage overrtopical treatmentof clearing na.sop!1aryngeal carriage,
reduci~g the.risk o~rec~rrence. Parents.,. even if as)mptomatic- require .
treatment. Pneumonitis occurs in approximately I 0-20% of cases, and usually
presentsf much)ater,
between 3 and II weeks..
.
.
., , ,
;
t.

~'

108

.;~l

r'

j.d

'

,.

.)

OPHTHALMOLOGY

12. Ptosis
Answers: A B E
Mobius syndrome results from a combination of facial nerve (Vllth) and
.
abducens (Vlth) nerve palsy and is often associated with an oculomotor (IIJrd)
nerve palsy. An isolated Vlth nerve palsy causes failure of abduction of the
eye, but does not affect the lid. Kearns-Sayre is a mitochondrial dis?rder with
progressive external ophthalmoplegia (reduction of gaze in all directions),
ptosis, pigmentary retinopathy; and heart block. A collodion baby is covered
with a tense cellophane-like membrane resulting in ectropion as the tissues
around the eyes are held in traction. The ptosis ofMarcus-Gunn jaw winking
syndrome resolves on mouth opening and lateral mO\ement ofthe jaw. It
results from aberrant innervation of the levator muscle of the eyelid from the
'
trigeminal (Vth) nerve.

13. Vernal conjunctivitis


Answers: B c E
Vernal conjunctivitis is a seve!e allergic condition presenting with intense
itching, tearing. mucus production, and giant papillae on the upper tarsa!'
conjunctiva. Secondary keratitis during exacerbations causes photophobia.'.
The condition is seasonal, often starting in spring and is commoner in boys, '
usually starting in those under 10 years of age. 90% of cases have other
allergic diseases in addition to an atopic family history. n'eatment is based on
mast cell stabilisers (cromoglycat~; drops) and topical antihistamines. Severe
exacerbations may' necessitate topical steroids but long-term use is to be
avoided to prevent secondary glaucoma or cataracts.

14. Retinopathy of prematurity


Answers: A CD E
Risk factors for retinopathy of prematurity include low birth weight,
hyperoxia, respiratory distress syndrome, intracranial haemorrhage, pre" term
gestation.:and bloqd transfusions.,The .incidenceJncreases dramatically. with ' .
birth weights of less than J OOOg. Adult haemoglobin dissociates oxygen more
readily than fetal haemoglobin. therefore transfusi!>ns increase retinal oxygen
dose. Gender is not a risk factor.
.

15. Leukocoria
Answers: A B c E
Leukbcoria is defined as a white pupil imd is indicative of an opacity at or
beh'ind tne. pupil JOfthe lens, vitreous or retina). The differential diagnosis
include~ cataract. retinoblastoma. toxocariasis, persist~nt hyperplastic .
prim~ry Vitreous. mye!in;'lted nerve' fibres, and retinopathy or prematurity.
Hurler syndrome and the"mucopolysaccharidoses- with .the exception of
Hunter sjndrorru'i- are associated with corneal clouding.
.

109

ESSENTIAL QUESTIONS FOR MRCPCH

16. C: Bitemporal hemianc;>pia


Craniopharyngioma may be suprasellar. or supra- and intrasellar, and
therefore <IS the tumour increases in size, pressure is applied to the optic .
chiasm. This'is the site of &!cussatfon of the temporal field fibres and

therefore' vision in the temporal ~elds is lost bilaterally. '

1 7. B: Neisseria"'gonorih.oeae
The very early onset makes Neisseria gonorrhoeae the most likely organism,
usually presenting on day 2-4 of life. Staphylococcus and Streptococcus cause
purulent neonatal conjunctiyitis, but present lat~r (day 4-7). Chlqmydia
produces a serous or purulent discharge (day 4-1 O).Haemophilus tends to be
a serous discharge (onset 5-l 0 days) as does!hetpessimplex, but the onset is
much later 16 days to 2 weeks).

18. D: Left optic nerve lesion

l"'

The findings are consistent \'lith damage to the optic n~rve in the affected eye.
In lllrq nel'Y,~ pa}syt~~ pupU,~s fi~~<;ld.iJ4!_~d C!nd q?e~ ~pt r,~?~~ toc::Ur,ect or .
consensuallight. In Horner syndrome the pupils ar:e smaH due to the . .
interrupted s.Ympathetic supply to the pupill~ry dilator'inllsc:le, but are able to
constrictto lig~t. .
.. j
' .
'
'
'~. 1 '
;, .
'

19. B: cOrticosteroids
.

'
'!>

The siting of the lesion requires active m;:magement. Steroids are the
.....
preferred choice, either systemic or periocular. Small peripheral lesions may
be observed. The use of antihelminthic drugs is controversial as death of the
larva can exacerbate the inflammation, and steroids would still be prescribed.
Laser therapy has been advocated but it has the sarne inCreaSed rfsk of '
inflammation following death of the larva.

20. A: Adenovirus

Viral conjunctivitis is usually caused by adenovirus ~nd 1s v_e,ry c6ntagious.


First one eye is involved, and this then spreads to the other..Signs and .
symptomsinclude tearing, redness. and the sensation of having a foreign body
in the eye.lfthe cornea becomes involved, then photophobia can de\elop.

21 . C: cavernous sinus thrombosis

, ,.

The main differential is between orbital cellulitis and cavem()_Us sinus ,


thrombosis. However, the latter is suggested by the acutepnset of reduction
in eye movements. The ophthalmoplegia is secondary to)llrd, IVth and \1th
nerve involvement, all of which pass through the cavernous sinus. Eye
mo\,ementS may be slightly reduced secondary to. pain in orbital cellt]litis ..
neatment consists of intravenous antibiotics and/or drainage of sinuses if
they are the source of infection.
Fron'tal sinusitis
presents
with tenderness
'
,,.
' ;
..
t
110

. _ _J ---

OPHTHALMOLOGY

over the forehead and may be the source of the infection along withsphenoi~
and the ethmoid sinuses. Mastoiditis presentswith swelling over the. mastoid
air cells behind the ear. A c;erebral abscess is more likely to present with
.
' altered consciousne_ss, v~imiting and pyryxia:

; .. . ' .
.

\.

-~

'

22. A: Sheridan Gardner test


In the Sheridan Gardner test, the child has a key,card

' f':

with.tfv~ lett~rs. The

exC~miner stands,(:) m away ariq holds "up letter~ ofdiff~rent size which ~he .

.(

child. has to match with those on the card. A standard Snellen chart can
usually be used in c;hildren aged 7 years upwards. The other.three tests are .
used for testing in pre-verbal children. The Catford.drum (verticalstripes)
relies on n'!rmal ~CIJity to produce optokineticnystagm~s.

'.

23. EYE MOVEMEi.,hs

l.

D -Adduction of the right eye on upgaze

Brown's syndrome results from a congenitally tight superior oblique muscle


-tendon complex. This means that the eye fails to elevate in adduction. Patients
may develop a compensatory chin lift and may slightly turn their-face away
from the affected eye.

2. A- Abduction of the right eye


The abducens (Vlth) nerve innervates the lateral rectus muscle which abducts
the eye.
3. E- Abduction of the right eye on downgaze
The trochlear (IVth) nerve innervates the superior oblique muscle, which acts
to depress and in tort (twist nasally) the eye. Patients usually present \-Vith
head tilt to the opposite side. This is common congenitally or from trauma.
24. MASSES AROUND THE EYE
1. E- Meibomian cyst
Me\bomian glands are sebaceous glands with their opening at the lid margin.
Their function is to reduce evaporation of tears by covering the tear film with
sterofesiers and waxes. Blockage of the gland results in swelling (chalazion).
If it fails to resolve after several weeks, drainage may be necessary.

2.

B - Hordeolum

Hordeolum (stye) is an abscess of an eyelash follicle which therefore appears


as a tender swefling at the eyelid margin. A cyst of Moll lies on the anterior
eyelid margin, but is painless and translucent.

3.

G - Obstructed tear duct

Infants v.'ith nasolacrimal duct obstruction present with tearing and increased
mucus accumulating at the inner canthus. Spontaneous resolution occurs in
h<ilf of all cases by 6 months. An amniotocele may appear as a swelling in the
medial canthal area as a result of fluid sequestered in the nasolacrimal sac. If
not infected, local massage may affect a cure. Infection requires intravenous
antibiotics and probing.

'
Ill

:
~:'

'

'

..

..

. .

ESSENTIAL QUESTIONS F:.OR MRCPCH

25. DRUG ADVERSE'EFFE<;TS

1.

G ~ Vigabatrin

Vigabatrin may cause permanent visual field loss, and 'therefore an attempt
should be made to assess ba.seline perimetry prior to"'- or SOon after- starting

treatment, and every 6 months thereafter.


2.

E- Ethambutol

3.

B- AmJodarone

..

Ethambutol is used to treat tuberculosis. Retrob~lbar neuritis is a dosedependent ~ide effect that presents with colour vision defects, central and
paracentrai scotoma, and reduced acuity. Recovery may take weeks to months
after ethambutol is stopped.

Amiodarone deposits are seEm in both lens and cornea, and are related both
to dose and duration of treatment The deposits are reversible .. _

.'

-~

.( 4iV

I i2

';.

Andrew Clark

......

'

;.;;.;:::c..f.::-::.!!:"'::::::o. .'~:c:z~~~~:.--x,;c;_.,_.llfi',.;

~w~e~.;.~--t.:-;.~"'-.-~.:::-:.~-'"':::;:~i--.~ :_~.;1-:::::,:;:xr.....

, ~-

. t,

.Multipl~ C~?oice.Questions
1. Erythema nodosu.m is a recognised f~ature of

0 A
0 B
0 c
0 D
0 E

sarcoidosis '

"

!.

,.

cystic fibrosis
'~ .~
pneumococcal pneumonia
pulmonary tuberculosis
Kawasaki's disease

.v,r

. , .,

':"\:

2. The oxyhaemoglobin dissociation curve i~ shifted to the right in


the following situation(s)

'

0
0
0
0
0

A
B
C
D
E

increase i.n temperature


hypocarbia
alkalosis
increase in o-2,3-diphosphoglyc;erate
carbon monoxide poisoning
'

3. In transfer of gas across the alveolar capillary membnine

0 A
0 B
0 c
0 D
0 E

oxygen diffuses more rapidly than carbon dioxide,


rate of gas transfer is proportional to solubility
rate of gas transfer is proportional to capillary area
gases with a higher molecular weight diffuseacross more rapidly .
type I pneumocytes comprise 95% of the alveolar surface

4. Digital clubbing is

0
0
0

'

J'

'

a chatacteristic feature of

A cystic fibrosis
~,
. , .
B pulmonary sequestration
., ,. ,
.... .
c bronchopulmonary dysplasia '" . ' '""-~ t*' ''
D primary ciliary dysplasia
E biliary cirrhosis
. n ''

J '

,.

'i 13

ESSENTIAL QUESTIONS

MRCPCH

5 Nasal polyps are a recognised feature of

0
0

0
0

'A }Wegener's granulomatosis


B chronic rhinitis
c Aspirin sensitivity
D cystic fibrosis
E hypothyroidism

6. Percentage ofpredictttd FEV,

0
0
0
0
0

A is a suitable measure of disease activity in cystic fibrosis


B is used to 'assess lung function of pre-school children
C is increased in asthma
\
_. r. 1
D represents the volume of air expired in 1'second during a forced
expiratory manoeuvre
E can indicate large and ine'dium'air.Vay obstructioh

7. An enlarged tongue is a typical feature of

0
0
0
0

A hypothyroidism
B trisomy 18
C Pierre Robin sequence
D GMJ gangliosidosis
E. neurofibromatosis type

...

')

'

I'

8. Causes of bronchiectasis include

0
0
0
0
0

A sarcoidosis
B primary ciliary dyskinesia
c gastro-oesophageal reflux
D pertussis infection
E measles infection _

'

9. In early lung development

0
0
0
0
0

A the airways and parenchym~ a~ed_e.r!y~d Jl-Pm.mesuderm .


B pulmonarysurfaciant production begins at 24/40
c alveolar development is complete at term
D surfactant reduces alveolar surface tension
E thyroid hormone inhibits surfactant production , . , ..
''

'

'

10. Adrenaline (epinephrine)

0
0
0
0
0

A is secreted from the adrenal cortex


B stimulates ~2 receptors

c should be used by the intravenous route in anaphylaxis


D causes hypoglycaemia
E causes dilatation of renal vasculature

. ~~~
I

,.

~~.'I
rm .

II The following is true of (ood allergy


A peanuts are the commonest cause of fatal food-inducecfreactions
B allergy to hen's egg is commonly outgrown
,
'
0 C other anergies are uncommon
...
0 D the diagnosis can be made on the history alone
0 E peanut allergy usually resolves

0
0

12. The following is true of of tuberculosis'.

0
0
0

A
B
C
D
E

~,,;_,.,..
~~~::.~~

~. (/)

.......

..

the incidence is falling worldwide


notification to public health authorities is required .
a tuberculin test wheal > 5 mm is significanUn any child.
isoniazid cancause peripheral neuropathy

children are less p1 one to extra-pulmonary complications than' adults

13. In cystic fibrosis

0
0
0

A life expectancy depends upon nutritional status


B life expectancy is. on average, 20 years
C eczema can cause a false-positive sweat test
D cystic fibrosis transmembrane conductance regulator (CFTR) fails .to
absorb chloride ions the lungs
E raised Aspergillus IgG lilre is characteristic of a1lergic
bronchopulmonary aspergillosis

in

14. The following cause vasoconstriction of the pulmonary


circulation

0
0
0
0

A
B
C
D
E

hypoxia
nitric oxide
prostaglandin 12 ..
platelet activatingfactor
histamine

15. Lung compliaricnninfants

A is defined as the volume change. per unit of pressure

0
0
0
0

B
C
D
E

increases with age


is increased in respiratory distress syndrome
is independent of lung volume
is dependent on type II pneumocytes

16. Recognised extrapulmonary complications of cystic fibrosis


inClude

0 A
0 B
0 C
0 D
0 E

hypertrophic osteoarthopathy
rectal prolapse
glotl;lerulonephritis
azoospermia
diabetes mellitus

'

'. ' i

115

ESSENTIAL 91,J,ESTIOJ;IS FOR MRCPC::H

-.

"'

c'
..rJ:
.....

17. Recognised assodations 1of.primary cili~ ()ysldnesia includer


~ Jr
, :1
,
0 ..~. 1 ,!1Jfe~ility . .

0
0
0
0

B
C
b
E

nasal polyps
..,
hydrocephalus
bronchiectasis , .1
malabsorption

."

,,

. ~:- l
. . ,

''
I

18. Differential diagnoses of bronchiolitis in infants includes .

A total anomalous pulmonary venous drainage


.:
B pertussis infection
c gastro'"'oesophageal reflux

D interstitial pneumonitis
.. E pneumococcal pneumonia

0
0
0
0
0

.,

.,.

': '

'

''

,,.

~.

~(

.. :
' i

.Tr

. '....
..,

...116
~

.,~

..

'

RESPIRATORY MEDICINE

B,est of Five Questions


"'"-""

.--.~.r-

.:::; .

~ -1'~4\'i:::"..t.~~.:-.;..-~n.t.-);.

... '-

19. Parent~ rome to you for ~ntenatal counselling; the mother is a


carrier of a cystic fibrosis mutation and the father has never
been tested and has no family history. Which ONE ofthe.
following is 'the chance that their offspring may have cystic
fibrosis?
.

0
0
0
0
0

A l ~n 4
B I in 100
c 1 in 160
D 1 in 200
E 1.in 2500

20.

0
0
0

A. 'Arterial blood gas .


B Electrocardiogram (ECG)
c Microlaryngobronchoscopy (MLB)
D Overnight oxygen saturation recording
E Lat~rall.!P.per airways X-ray. , ,, ,

'

I;

...

'

2 "year-old boy has a history of lethargy and falling asleep


during the day. His mother reports that he snoresJoudly. Which
is t.he MOST useful investigation?

.

..

.. ,. !

.~

'

..

f _;; '

'

,,

21. A 14-m.onth-old girl with a history of eczema develops


_. g~neralisecfurticciuia, w_heeze and severe dyspnoea shortly after
eating some peanut butter for the first time. What is the MOST
appropriate initial treatment?

0
0
0

. o .
0

A
B
C
D
E

Adrenaline intramuscu,larly
Adrenaline intravenously
Hydrocortisone intravenously
.Chlorpheniramineintravenously
Chlorpheniramine orally
'

'"

'

22. ln. an 8-year-old boy wlth inspiratory stridor due to


. c_ompression of;the trachea, what would you MOST expect the
spirometry flow-volume loop to show limitation of? .
.
e

0
0
0
0
0

A
B
C
D
E

Peak expiratory flow rate (PEF)


Forced expiratory flow rate (FEF 25_75 )
Expiratory flow only
Inspiratoryflow only
Inspiratory and expiratory flow

117

E~ENTIAL QUESTIO~S

0
0
0
0
0

FOR MRCPCH

A I 0-y~ar-old boy,who was previously wep presents. with a .


2-week history of malaise and headache: with pleuritic chest
pain, cough and fever over the past 3 days. Five days of '
treatment With oral penidllin has J!lade'_ no improvement. On
;examination there is a small area of stony dullness at the right
lung base. The MOST likely diagnosis is whiCh' ofthe following?
A' Lymphoma '
'
B Mycoplasma pneumoniae pneumonia
c Pneumococcal pneumonia
D Pneumocystis carinii pneumonia
E Staphylococcal pneumonia

24. In a child who pre~ents with ascending paralysis of the legs and

0
0
0
0
0

areflexia, what is the MOST useful method of respiratory


momtoring?
A Respiratory rate
,1
B Partial pressure of carbon dioxide in arterial blood' (Pco2 ) ,
C Oxygen saturation

D Vital capacity
E Peak expiratory flow rate
.
.

,.

25. A 3-month-old baby has a history of wheeze and coughing

0
0
0
0
0

usually after feeds, since birth. Her weight is normal ~d on


examination she has a Harrison sulcus and a hyper-expanded
chest. Which of the following investigations would be the MOST
useful?
A Flexible bronchoscopy
B Computed tomography (CT) scan of the chest
C Ciliary brushing for motility
D Sweat electrolytes
E Upper gastrointestinal (GI)cohtrast sh.iay '-- - -----

26. An I 8-month-old girl presents with a 6-month history.ofweight


loss and cough.' Examination reveals a clear ch'est, cerVical
.
"
't
'
...

' .
lymphadenopathy and red, tender lesions on her shins. Which is
the MOST useful diagnostic test?

0 A Erythrocyte sedimentation rate (ESR}


'
0 B Bronchoalveolar lavage .
,;
0 C Sputum culture
0 D Gastric aspirate
0 E Skin biopsy
~

118

J.

'"'"-'"
.

RESPIRATORY MEDICINE

A Total lung capacity (TLC)


B Serial peak expiratory flow (PEF) rates
c transfer factor
D FEF 2 H 5
.
E Residual volume

i .

Fe
c
.

Ltt)
'u)
~.:.

;i

,.,...

'

. ..

..

!.

r.c:
,.._,

27. A 10-year-old boy_presents to yollr general paediatric clinic


with a long history of asthma. It is poorly controlled and he
suffers frequent exacerbations. Which one of the following
measures of lung function would be MOST useful?

0
0
0
0
0

119
"'

ESSENT.IAL QUESTIONS FORMRCP,CH

'

28. Theme: Causes of chest:disea;se

-,

........
U) ..

..,;.

C1)

:J

c-

A
B

c
D
E
F
G
H..
1

Bronchopulmonary dysplasia
Acute lymphoblastic leukaemia
Asthma
Interstitial pneumonitis
Cystic fibrosis
Rheumatoid lung disease
Pulmonary tuberculosis
Allergic bronchopulmpnary aspergillosis .
Primary ciliary dyskinesia

......

!
.J

For each of the following case st,enarios select the most likely diagnosis from
the list above. Each option''may be used ohce;~rriore than once, or not at all.
I. A6~year-old boyhasa historyoffailure to thrive"and multiple chest
infe,ctions. On exfimin_.ation_,h; ha~ cpgi_tal club~ing .~nd ~oar~~ c;,:f~~'
throughout his chest. Haemoglobin: 1.1.2 gldL. Forced expiratory
volume in I second (FEVJ 86% of expected; Chest X-ray (CXR):
-Bilateral streaky shadowing.

2. An.rii-year-old girl presents-with a,, persistent cough.pver the past 6.


weeks her weight centile has fallen from the 9th to < 0.4th. On
examination she has digital clubbing and unilateral monophonic
wheeze. Haemoglobin: 8.2 g!dL FEV1: 90% of expected. CXR:
. imilateral peri-hilar shadowing.
.
:3. A 2-year-old boy has a history of failure to thrive, multiple chest
infectiors. perennial rhinitis and chronic secretory otitis media. On
examination he has a blocked nose, digital clubbing. and bilateral
coarse crackles are audible throughout his chest. Haemoglobin: 12.5
g!dl. CXR: Bilateral streaky shadowing, dextrocaraia.

29. Theme: Investigation of cllrttfie-nWiiil respfi~tory dis~ase


A
B

Sputum bacteriology
. Nasal brushing
c Bronchoalveolar lavage
D Lateral neck X-ray
E Barium swallow
F
Rigid bronchoscopy
G Serum Aspergillus specific JgE .
H Flow-volume loop
Serum viral studies

For each of the following case scenarios select the most useful inl'estigation
from the list abo\'e. Each option may be used once, more than once, or not at
all.
. I

120

__,
RESPIRATORY MEDICINE

0
0

L A 10-year-old boy.With cystic fibrosis has a history of persistent


wheeze and breathlessness. His symptoms require daily oral
prednisolone for control. Chest X-ray (CXR): bilateral patchy upper
lobe shadows. Forced~X(Jiratoryvolume in 1 second (FEV1):.65% of
expected. Serum JgE: > 5000 IU/L.
2. A 2-year-old boy presents with a 2-day history of a persistent dry
examination he has right-sided rrionophonic.wheeze,O<R:
cough.

hyper-luscent right lung. liver displaced downwards. ,


3. A 5-month-old girl with_HJV infection presents with a 1-weel< history
of fever, cough and dysp!loea. On .exatnination there are cr~ckles"
throughout her chest. Oxygen.satur.ation: 85%. CXR:_b~lateral di[fuse
, shad(}wing. Nasopharyngeal aspirate for bacteria, viruses andfu~gi:
normal.

. .
.. / .

on

30. Thenie: Outpatient manag~ment of childrenwith asthma


A
B
C
D

E.

Referral to a respiratory paediatrician


.Inhaied beclometasone dipropionate400 Jl.g per day
. .
*- . .
. ....
Leul<otnene receptor antagomst
Inhaled short-acting 132 -?gonist as required
No chanoe . ~. _~.> . '-t/_i"'"':t"'::~
.

)phaled beClom~tasoneaipropionate 800 J.t.g per day


Slow-release oral theophylline
Long-acting.l32 -agonist (lABA)
~

,'<

F
G
H

";,;

.,_

..
r

. }

For each of the following scenarios select the most appropriate possible
addition or change co outpatient management from the list above. Each option
may be used once, more than once, or not at all. .

I. A 5-year-old boy with asthma regularly uses inhaled beclometasone


dipropionate at a dose of 200 J.ig per day. He finds that that he still
wakes up with wheeze every morning and needs to use his 132 -agonist
inhaler daily.
.

'
-0 ... 2. A 9"':ear-old girl with asthma is reviewed after 3 months.-She has been
taking 800 J.t.g of inhaled,b~clometasone dipropionate per day, a longacting 132-agonist and a short:ai::ting 132-agonist as required. She
reports that her symptoms have improved greatly and she now hardly
.. uses her short-acting 132-agonist.
. .
0 3. A 3-year-old boy with asthma has persistently po_or control despite.
regular use of inhaled beclometasone dipropionate ~00 ,...g /day.

' 121

'"'J

1: Erythema nodosum
Answers: A CD

Erythema nodosum are tender, nodular erythematous lesions which occur


mosi commonly on the shins followed by thighs arid arms. They last for 1~2
months and evolve into blue bruise-like lesions. causes include infections
(Streptococcus: Salmonella. Ye'rsinia, Campylobacter. tuberculosis; Chlamydia,
hepatitis B. and Epstein~ Barr virus), inflammatory disorders (such as
sarcoidosis, ulcerative coliti~ and Crohn's disease), malignancy and drugs,
sulphonamidesand the oral contraceptive' pill.

2. Oxyhaemoglobin .dissociation curve


Answers: A D

,.
'

.
.
Haemoglobin (Hb) affinity for oxygen increases as the partial pressure of
.

....

oxygen in arterial blood increases. This is shown as a sigmoid-'shaped plot of


Hb saturation against partial pressure of oxygen (the oxy-Hb di~sociation
curve). The flat upper portion allows arterial oxygen content to stay high
despite variations in Por The steep middle portion of the curve describes how
peripheral tissues '(with a lower partial pressure of oxygen) can withdraw
large amounts of oxygen fro in Hb for only a small drop in capillary Po2 . A ..
variety of conditions can alter the binding affih.ity of Hb for oxygen, rhus.
shifting the curve left or right. Increases in temperature, carbon dioxide
partial Nessure, hydrogeniOl) concentration. and o-2,3-diphosphoglycerate
all shift the curve to the right. facilitating oxygen removal frpm the blood by
peripheral tissues, as the affinity of Hb for oxygen decreases. Opp9site
changes and fetaiHb result in a left-shifted curv~: n . ,

3.

Gas transfer across


a'Iveolar
capillary membrane
.
.
.
'
"

Anshers: B

,)

cE

The exchange of gas at the alveolar-capillary interface is achieved by passive


diffusion. Fkk's Uiw states that the diffusion of gas is proportional to the .
tissue area'of the membrane and the difference between the pa'rtial pressures
of the gases on opposite sides. Diffusion is inversely proportional to the
thickness of the membrane. Properties of the gas are also important, diffusion
is proportional to gas solubility and inversely proportional to the square root
of its molecular weight. Carbon dioxide diffuses through tissues 20 times

122

RESPIRATORY MEDICINE

faster than oxygen, because it is more soluble. According to Fick's Law the
volume of gas transferred is gi\en by:
r. .

{A X

\;u-

DX
T

(P 1-:-P2)}

'

where A= membrane.area, T = membrane thickness, D = diffusion constant,


P1-P2 =difference in partial pressures:

4. Digital clubbing
Answers: A DE .

Digital clubbing is an important clinical sign. The first change'is loss of the
nail-fold angle and a fluctuant bagginess of the nail bed. Increased curvature
of the nail bed and enlargement of the distal phalanx occur later. Causes.
inciude any cause of bronchiectasis or cyanotic heart dlse~se, uberculosis,
empyema, malignancy, bacterial endocarditis. biliary cirrhosis, chronic active
hepatitis and inflammatory bowel disease.

5. Nasal pc;>lYP?
.. ..,. .
~

Answers:B CD .

Nasal polyps present with nasal obstruction, symptoms of rhinitis (decreased


smell and rhinorrhoea).and.an enlarged nasal bridge. Very often theY:are
ignored; as they can presentinsidiously. Cystic fibrosis must be considered in
any child with nasal polyps. but they also occur in chronic rhinitis and the
triad of nasal polyps. asthma and aspirin hypersensitivity (Sampler's triad),
more commonly seen in older children and adults. Wegener's granulomatosis
is a necrotising vasculitis found in the lungs, joints, eyes, kidneys, sinuses arv:!
nasopharynx. It causes ulceration and bony destruction in the nose, with
symptoms of rhinorrhoea, nasal congestion and sinus pain.

6. Percentage ol predicted FEV1


Answers: A DE

Lung function testing inpre:Sthool children is limited to research centres.


From 5 years of age'most children can perform spirometry. with practice. FEV 1
represents the volume of air expired in I second during a forced expiratory
manoeuvre and gives an indication of large and medium airway diameter.
Results are calculated as a percentage of expected results for a healthy Child
of similar age, height, age and sex. It is the best measure aflung function for
children with cystic fibrosis and when measured over time it can indicate
prognosis. In obstructive ain\'ay diseases such as asthma, the FEV1 and ratio
of FEY: to forced vital capacit:y(FVC) are reduced, showing baseline airway
obstruction. An improvement in FEV1 of at least 15% after administration of
an inhaled bronchodilator is indicative of asthma. Bronchial challenge tests
qn also be used in the diagnosis o( asthma; administration of methacholine
causes a 20% fall in FEV1 for 3 hours.

I
!

123

.;

ESSENTIAL QUESTIONS FOR :iRCPCH


i

'!"

7. Enlarged tongue

>-JA. :<' ~

Answers:Aco'-

'l
.

'

.... , ~~.. ~ .
~,_
An enlarged tongue can obstruct the airway and lead to an emergency,
'"
,.,.
'h ' .
...
particularly in infants who ha~e small nasal passages. Tongue-reduction
surgery is successful. It occurs in hypothyroidism, mucopolysaccharidoses,
trisomy 21 and Beckwith:-\\:iedemann syndrome: Pierre Robin sequence is ,
caused by a posterior attachment of the tongue tpgether with a small - '
mandible resulting in pseudo- macroglossia and the tongue commonly falls
.
:... .J a ~
' ' ~'
back to obstruct the airway.
.

8. Causes of bronchiectasis
., ,Answer: A]/ cme
, I ->~ I . >,!
i;- ~

' ... -'..

"

.-,

'

'

't

'

I''. .

..;

'

'.

Bronchiectasis arises where there is permanent dilatation of the sub':..''


segmental airways associated w:ith'inflammation:Accumulation of exudative
material in the dilated air\\'ays leads to the clinical manifestationof copious~
muco-purulent sputum production. Poor appetite and weight gain are alSo''
common. Examination reveals clubbing and coarse crackles throughout the .
chest. There are many causes including cystic fibrosis, immun'oglobi.llin '
deficiency, and primary ciliary dyskinesia. Pertussis and measles were \..
~istorically,important causes, reduced in incidence. byimmunisation.
i
Treatment involves elimination ofrespiratory infection (vigorous treatment of
ac.ute inf~ctions and colonisation). effe~tive mucous clearance (physiotherapy
with <;>r.\\1thou.t mucolytic drugs) and increased calorie intake. ,

1-

,.

--

r<!o:

9 .. Early lung development"


~~~AnS\1lers:

BD

'

,.!;

...,

Early lung development starts With a ventral out pouching of the fore-gut,
lined by endodermarepithelium. This givesrise to right and left lung buds
which by 6 weeks contain segmental bronchi. By 16 weeks all the conducting,
ainvays are present. as blind tubes lined by cuboidal epitfielium (pseudoglandular stage). Cilia and cartilage begin to develop. From 17-'.24 weeks .
(canidkuiar stage) distal airway epithelium thins out in preparation for gas':
exchange. The lungs become.\ascularised and surfactant synthesis begins. In
the alveolar sac stage (25-:40 weeks) thin~walled saccules at the end of
respiratory bronchioles develop into alveoli. After birth, the majorityof lung
development occurs in the respiratory unit. and alveoli continue to be formed.
until approximately 3-Syears. when the number of alveo!Lreaches that of an
adult. Surfactant is a mixture of surface~active phospholipids which reduce . ..
alv~qlar,membrane surface tension, reducing the negative pressure the infant
needs to generate to expand al\'eoiL Surfactantproduction is increased by

'
glucocorticoids andthyroid:hormone: "
:.i ,...
'

124

"I

~r
RESPIRATORY MEDICINE

r-- --

r~

I 0. Adrenaline

".
'
1l .

Answer: B

Adrenaline is an endogenous catecholamine secreted by the adrenalmedulla.


With increasing dose it stimulates~~ (increased heart rate and contrac~ility)
and ~2 receptors (bronchodiiatation, peripheral vasodilatation and reflex
tachycardia). At higher doses o:-receptor stimulation dominates, causing
peripheral vasoconstriction and elevation of systolic blood pressure.
Adrenaline inhibits insuiL\ relea~e. causing hyperglycaemia. Adrenaline is the
treatment of choice for severe allergic reactions (anaphylaxis) and should only
be administered intramuscu:arly, except in extreme circumstances. Children
who have.had a previous allergic reaction involving airway narrowing or who
also have asthma should be ?TOVided with pre-loaded adrenaline syringes.

11. fZood allergy


Answers: A B

The perceived prevalence of food allergy is high (20%) but the prevalence of
Confirmed food allefgy is more likely to be1 2-3% in ch-ildfen. Nearly all food
allergic children have sufferej from some other allergy. such as eczema or
asthma or le_ss commonly rhinitis. Egg and milk allergies are usually outgrown
by school age, with more se\ere cases persisting. Peanut allergy on the other
hand is longer lasting anp it v.ill resolve in only 20% of those with mild allergy.
Again, more severe cases are likely to be long Jived. Diagnosis is made on the
basis of a typical history ofan allergic reaction occurring soon after ingestion of
a likely allergen. This is confirmed by detection of specific IgE to the allergen
either by skin prick test, or in the serum (CAP-RAST test)_ In equivo~l cases oral
challenge may be used. Treatment involves providing detailed written and verbal
avoidance advice, together \.ith a written emergency treatment plan detailing
how and when to give oral antihistamine and/or intramuscular adrenaline.
12. Tuberculosis
Answers: B D

The incidence of tuberculosis worldwide is rising reflecting increasing Hfv


(human immunodeficiency\-:rus) infection, poverty and overcrowding. All cases
must be reported topublic health authorities. Thelungs are the commonest site
of infection and source of spread, although extra pulmonary complications, like
~uberculous meningitis, are more.common among children than adults.
Symptoms are highly variable so one must always consider the diagnosis in any
child who presents with fever. anorexia, .weight loss and cough. Therapy is with
isoniazid and rifampicin for 6 months and pyrazinamide for 2 months.
Rifampicin turns secretions. such as tears (warn patients not to wear contact
lenses) and urine orange. Iso:-~iazid and rifampicin can cause deranged liver
function tests and isoniazid can cause peripheral neuropathy. Ethambutol (risk
of visual disturbance) is usee in resistant or high-risk cases.

125

::s
~. fl)
-~

tD
..,,
.,

V'l-:'Yc~
"'' "' '
r .

-ESSENTIAL QUESTIONS FCO' MRGPGH

) 3. cystic fibrosis
Answers: A C

cystic fibrosis is an inherited recessive disoraer of chloride secretion in the


lungs and exocrine glands. The gene on chromosome 7 encodes CFTR.
Deficiencies in CFTR lead to failure of chloride ion secretion and excessive
sodium ion absorption, leading to "dehydration of ainvay secretions~ The ions
are transported In the opposite direction in sweat glani:ls and this forms the
basis for the sweat test; a sodiuin or chloride ion concentration > 60 inEq!L is
diagnostic. Median life expectancy is now 40 years. Allergic
'
. bronchopulmohary aspergiiiosis (ABPA) occurs in cystic fibrosis arid asthma.
It results-from an overzealo_us lgE host response to colonisation with
Aspergillus. Typical findings are wheeze and a dry cough. Patchy infiltrates on
CXR appear in different sites over time; a high total and Aspergillus specific
lgE together with Aspergillus precipitins help with the diagnosis. ireaiment is
with oral corticosteroids and anti-fungal agents teg itraconazole). "'
,

14~

l,

'i''

. ,. 1 .)

o/""

vasocon,striction of pt_JimonaiY, ci[CUI~tion

., Ans\-vers: A DE

. ,

1 , ,

1,

. The pulmonary circulation delivers blood to the ah~eoli for gas exchange: The
relatively low mean pulmonary arterialpressure ( 15 mmHg) is maintained
t everi though the pulmonary atteries receive 'the entire i:a'rdiac output, because
of the extremely low vascular resistance: If the cardiac output increases (eg in
:> exercise) then the low resistance system will dilate and recruit previously
closed vessels, to accommodate. However, pulmonary vessels can also' be
,, made to constrict, and one of the most powerful stimuli is hyPoxia (di\erting
, ccirculation away from under-'ventilated areas thus avoiding
'
ventilation/perfusion mismatth):'Nitric oxide is a po\\ierful vasodilator which
can be administered via inspired gas to treat pulmonary hypertension. Platelet
aggregating factor and histamine are released during inflammatory and

allergic reactions respectively causing vasoconstriction.

..

IS. "i.ung complfarice


Answer.s: A BE

Lung compliance is defined as the change in lung \'olume per iihit of pressure .
. It is also represented by the slop~ of the pressure-\ioJume curve. This is a
sigmoid shape, so compliance also depends on the initial lung volume, from
which the change in volume was measured (eg at large lung volumeswhere
the lung is near its elastic limit, the pressure.:.volume curve is less steep .and
hence compliance is reduced). Pulmonary surfactant produced by type 11 ' -
pneumocytes in newborns reduces surface tension and increases compliance .
. In respiratory distress syndrome there is a deficiency of surfaCtant (most
commonly due to~prematurityj and compliance is reduced.
t,._,.-..,~,

126

'

16. Complications of cystic fibrosis

Answers: A B D. E
There are many extrapulmonarycomplications of cystic fibrosis (CF).
Gastrointestinal complications include pancreatic insufficiency (causing
steatorrhoea and weight loss) and diabetes mellitus (due to pancreatic
fibrosis). Diabetes in.CF is often straightforward to control and ketoacidosis is
uncommon. Diabetes gets more common with age affecting 7% of adults with
CF. Meconium ileus in the neonatal period occurs ip. 15%ofpatients with CF
and dist<:H intestinal obstruction syndrome (DIOS) tn 10%. The diagnosis
should be considered in chilcren who present with rectal prolapse, although
constipation is a more common cause of this condith;m. Approximately 98% of
men with CF are infertile due to vas deferens obstruction. Sex hormone
functk n is normal but puberty is commonly delayed due to malnutrition.
n-eatmcnt of infections 'with aminoglycoside antibiotics over time can damage
renal tubules.

17.. Prlmaryciliary dyskinesia

Answers: ABc D
Primary ciliary dyskinesia occurs in one in 15, 000 births, with autosomal.
recessive inheritance. It results in abnormalities of cilia such as missing
dynein arms, absence of radial spokes or aplasia. In the respiratory tract this
results in failure of the mucociliary elevator and clearance of secretions,
lading to bronchiectasiS; chronic rhinitis and glue ear. Fifty percent also have
dextrocardia and situs inversus (~rtagener's syndrome). infertilitY through
.impaired sperm motility and hydrocephalus, presumably due to impaired
ventricular ependymal cilia, also occur. Diagnosis is suggested by low exhaled
nitric oxide and confirmed by nasal brushing for ciliary beat frequency and
electron microscopy. n-eatment is with physiotherapy and antibiotics with
specialist Ear Nose and Throat (ENT) assessment.

18. Bronchiolitis in infants

Answer: All true


Bronchiolitis is an acute respiratory infection, most commonly caused by a
respiratory syncytial virus (RSV}. The symptoms and signs are respiratory
distress and coryza, hyperinflation, occasionally apnoea and widespread
crackles on auscultation. Many conditions can mimic bronchiolitis and one
should be particularly aware of the possibility of cardiac lesion. Treatment is
supportive witrypxygen, clearance of secretions and hydration as appropriate.
InfantS witr bronchopulmonary dysplasia may benefit from a humanised
monoclonal anti-RSV antibody (palivizumab).

127

ESSENTIAL QUESTIONS FC=<. MRCPCH


l

19. B: 1 in 100

2.

This question illustrates the commonly examined topic of inheritance. You will
be expected to know the carrier frequency (l/25) and the prevalence in the
general population of cystic fibrosis (l/2500) and other common conditions.
We know the mother is a carrier and the father has a 1/25 chance of being a
'carrier so the calculation
1/25 x 1/4 = 11100 gives the odds per
pregnancy of pro'dticing a child With cystic fibrosis.The abnormal gene codes
for'cystic fibrosis transmembrane conductance regulator (CFTR); whose main
role is as an ATP-depehdent chloride ion channel. Defects in CFTR in the lung
.. result'in reduced'chloride secretion and hyperabsorption of sodium ions ..
leading to \l}scid secretions. Remember that CFTR works in reverse in the.skin
leading to 'failure to reabsorb sweat and hence high sweat electrolytes the
basis of the sweat. .test.
'
,

rx

i'

20. D: Overnight oxygen saturation recording


This child has a history suggestive of sleep apnoe~. The presentation can be
subtle and families will often not report pauses hi:breathing whilst asleep'
unless specifically questioned. Other features which are suggestive are early
morning headaches (due to.high arterial C02 partial P.[essure (Pco2 ) an9 an
'adenoidal' voice.ltis most often cause4 by adenotonsillar hypertrophy but
neurom.uscular conditions must also be considered. Arterial blood gas 111ay
show high Pco;and upper aitwayXR or MLB may_~eveaUarge adenoids; but
most information will be obtained from overnight oxygen satur~tion .
monitoring. This will detect periods of desaturation.and apn~a. and linked to
analysis of chest and abdominal movements can help distinguish b~tv:een
obstructive causes (tonsils) and central causes (eg brain stem tumour).

21. ,A: Adrenaline intramuscularly


The history clearly describes an episode of anaphylaxis. Adrenaline is the
most important single drug in anaphylaxis. It reverses upper and lower ainvay
.. oedema. causes bronchodilation, increases blood pressure and causes. :
peripheral vasoconstriction. reducing capillary leak. It should be given by the
intramu_scular route)n most ca.ses. The intravenous route should only, be used
by those experienced in,this met~od and at a concentration of no greater than
I in 10,000, with ECG monitoring. The prevalence of peanut allergy in
childhood .has tripied over the past decade anq it no\v affects 1.5%. Up to80%
of children with peanut 'allergy react on thr first~pparent ingestion, .
suggesting prior occult sensitisation: The m~dian age. at onset is 2 years and
only 20% 9fyoung children with mild allergy can expect to grow out of it. A
management plan for nut-a!leigi~ children which includes avoidance advice
and a tailored patient-held self-treatment plan, together with regular follo\v
up. can reduce the number of further nut-induced reactions. Future potential
treatments include use ofanti-lgE and desensitisation witn modified peanut

allergen.
128

---L

Rf:SPIRATORYMEDICINE

22. ,D: Insp!ratory flow only.


Examination of the spirometry trace is important, in addition to interpretalion
of values such as FEV1 We can deduce tha~ the lesLon is caush:g.~JIYJ)afuic .
obstruction as there are only symptoms on inspiration; fixed lesions cause
obstruction in both inspiration and expiration. A dynamic extra-thoracic
obstruction causes limitation of inspiratory Oow only. Dyna'mic intra;thoracic
airway obstruction. as occurs in asthma, causes reduction of expiratoryJlow
and therefore FEV 1 and FEF2s.:w.
23. B: Mycoplasmapneinnoliiae pneumonia
MJicoplasma pneumoniae is a common cause of atypical pneumonia among
school-age chi!dren. The clues in the h!;;tory are.,malaise and headachf more
comm n in Mycoplasma infection, together with the fige of.the child and lack
of respJnsiveness to penicillin (My~op/asma lack a cell wall). Macrolide
antibiotics are the treatment of choice. Pneurriocyslis carinii'infection~usually
only affects chiidren ;tth' severe underlying immune defi.ciency. such as HIY.
severe ,combined immun<;>,?~ficiency syndr~me (SCIJ.?l. di George syndrome,
and post bone-marrow transplantation (BMn.

. .
'

'

'

i ~

.,

'

..,;,

.'

, -

;~'

. ~

. .

; ... _

'

'."_... !lif>t;_-

,.

',t-,.

24 . D: Vital capacity
.
.
.
The case sc~nario describes 6Mn.ain-B~nre syndrome; a post-inflammatory
infectious poiyneuropathy. However, regardless of the cause all neurological
conditions which affect ,breathing need careful mon}toring b'ecau
deterioration.and respi~atory failpre can b~ clinicai1Y silent. Peak eXRiratory
flow rate will be normal until':'ery late; the best measure is vital capacity. AlSo
pay attention to bulbi:u'functionbecause failure of gag and cough reflexes
leave' the patient vulnerable ~o aspiration and possible asphyxiation.

25. E: Upper gastrointestinal (GI) c~~trast study


This child presents with a pittureoflower airway. obstruction that is
associated with f~ds, implying the presence of gastro-oesophageal reflux
disease, an H-type. tracpeo-oesopliageal tfstula (fOF)or Viral~induced
wheeze. The most useful. information would come from an upper Gl contrast
study which should inc!lide a tube oesophogram to exclude aTOF. Flexible
. bronc;hoscopy !s poor for excluding TOF; and ciliary brushing is used only for
the diagnosis of primary ciliary dyskinesia. It would be_sensible to screen for
cystic fibrosis ~t a later :Stage.

'
26. D: Gastric aspirate
The scenario describes a child with tuberculosis, the red lesions suggesting
erythema nodosum. The presentation can be subtle with few chest signs. The
classic features of fever;:lethargy and weight loss can be absent in children.
Acid-fast bacilli are best obtained by gastric washings, usually on three
successive mornings; bronchoalveolar lavage has a lower yield. The diagnosis
129

'--

'

ESSENTIAL QUESTIONS FOR MRCPCH

a.r;:"
~
rn

L .. :.
. :

..-:

c:

<

can be confirmed by int!adermal tu9erculin testing. l~itialtreat~~~t' consists


. of isoniazid (for 6 months); rifampiCin (6 months) and pyrazinamide (2
month.s). Monitor liverfuncti~~tests'and examine for peripheral neuropathy.
. . - . -:

;. .

"

., "" ,

'

r,,

...J:

.;-,; ~-:-

.'

- it

, ) .. ~

. ..

~- i(

- - ,

21. B~,Peak expir;:atory flow (PEF) rate.



.
-f
~
~.
~
Lung function testing can be useful in. asthma. Serial measurements of peak
expiratory flow rate are.easy io perform and are s~itable for home

monitoring. It is the most useful measure available to a general paediatridan.


Total lung capacity and residuaLvolume,bp~h i~cre~~e in a~t~f!l~ rew~senti,ng
air-trappingJJy obstructed airways. The ideal measure of small airway '
.
funCtion js, ~Ef, ls-15 It repfese~ts forced expiratory flow bet~een 25% and
75% of vital capacity and is ,less effort-dependen( but is only available with
spirometry.' Transfer factor meas'u'res dlffl.ision of carbon monoxide and gives
.a~ estimate opung diffusion capacity (reduced in consolidation and fibrosis
and ~ncreased in pulrrionarv haemorrhage). - -~ t:> .
f

,/."

j '

'.,'!.':

~~:')'

,. "'

-i

28. 'cAUSES OF CHEST DiSEASE

0. ,

~(.:,r.,J

.1.

"\
1

'

il

.~>~.

.J>

, ;

'.{,If

',-

I. E- Cystic fibrosis

'\

I'
.

:. . '. ' . '


This is a typical presentation of bronchiectasis with failure. to thrive, a.~istory
of chesq!"!f~ctions, and digUal Slubbing. Possibl; ~au;;~s from the_ list of,
.
options are cystic fibrosis and priritar)tciliary dyskinesia ofwhich'cystic
fibrosis [s more prevaient (I in 2500 versus I in 15, OOOj and therefore the
most likely diagnosis. Further clues that the diagnosis is' cystic fibrosis may
. include a history suggestive of malabsori>t\?n (pantp::atic_inslifflciehtyJ or
sub-acute bowei obstruction (distal intestinal obstruction syndrome),
meconium ;ileus, rectal pn;>l<:~ pse, chest in(ections with ~taphylococcus as'an
infant, or with pseudomo'nas at school age.'Uver dysfunction leading to
hepatosplenomegaly may also be app~re.11t., . ,
. ,14,) _ ,,
2. G - Pulmonary tuberculosis
.
The girl presents with anaemia, weight ioss and mondphohlc whe~ie, with a
CxR suggestive ofhilar lymph~deriop~thy,J~C>J>9~siql_e di~ign9~e5qr~.' .... ..
pu.lmonary tuberculosis trn> and haematological malignancy. The presence of
digital clubbing makes 'pulmonary TB a more likely diagnosis: . ... '
3. . I.,- :Prtrniuj;ciliary dyskinesia
. . , -
...! '
This case is very similar to case (1). However there is 'a predominance of upper
ai!Way Syrrlptoms which makes primarjdliarydy~kinesia the most l!kely
diagnosis. Other clues may include~ family histofy.of inale infertility '
(autosomal recessive inheritance), dextrocardia in 50%, and abdominal situs
inversus. lfeatrne!)t is as for cystic fibro~iswith the addition of speci~list Ear.
~ose, and Throa((ENn. ma!'agement. . .
.
,

'!' , ,; ,

.'

.,
.

130

''

,.

.'

~-

1. G - Serum A.Spergilius specific IgE


Allergic bronchopulmonary aspergillosis is a recognised compliCation of
cystic fibrosis and less commonly asthma.jt arises as an overzealous allergic
reaction to Aspergilhis colonisation within the airways, leading to mucous
impaction and airway narrowing. particularly of the upper lobes. Patients
P,resent with dry cough, wheeze and deterioration of lung function tests. It is
suspected if total serum lgE is very high and Aspergillus specific IgE and
Aspergillus precipitins are raised. Treatment is with long courses ofor;:tl
steroids arid anti-fungal agen:s such as itraconazole.

2.

F- Rigid bronchoscopy

Foreign-body aspiration must always be considered in children who present


with a monophonic wheeze and unilateral hyper-expansion on X-ray. Rigid
broncL :>scopyis suitable for removal of such objects -flexible bronchoscopes
are too narrov.i.

3.

c - Brorichoalveolar lavage

The girl is likely to have Pneumocystis carinii pneumonia. This extracellular


parasite causes a relatively common opportunistic infection in patients with T
lymphocyte immunodeficiencies (e'g HtV infection. CD40 ligand deficiency, or
Di George syndrome). Peak incidence is at age 3-6 months. Even when
hypoxia is present. chest auscultation may reveal no crackles. CXR almost
always shows bilateral diffu'se alveolar shadowing. Bronchoalveolar lavage is
'
the be~t method for isolating the organism, if it cannot be isolated from
nasopharyngealfaspirate ('NPAJ or sputum. Tt!eatment is with high-dose
trimethoprim-sulfamethoxazole,. pentamidine, or dapsone. Children at risk of
infecti~:m should receive prophylactic trimethoprim-sulfamethoxazole.
.

30. OUTPATIENT MANAGEMENT OF CHILDREN WITH ASTHMA


1. H"L. Long-acting fJ2 -;agonist'(LABA)
New gl;lidelines on the management of asthma were published ~y the British
Thoracic society (Thorax 2003 58(Suppl 1):i l-i83. websitehttp://thorax.bmjjourmils.cori1/contentNolS8/suppl_l). The aims are to control
symptoms; prevent exacerbations, achieve the best possible lung function and minimise side effects. The principles are to achieve early control and maintain
it by increasing and decreasing therapy as necessary, usually at 3-month
intervals. The next option for the boy is to add a LABA and assess his
response.
2. B -Inhaled beclometasone dipropionate 400 JLg per day
This case illustrates that stepping-down treatment should be considered
every 3 months; to minimise side effects from steroids.

3.

c- Leukotriene receptor antagonist

.A change to the updated British Thoracic Society guidelines was the inclusion
ofleukotriene receptor antagonists in children aged 2-5 years who ha\'e poor
control despite-maximal sterbid therapy.

131

!i~t:-t
t';~..,

f':~:U')~. ::~<
~
~

Ul

--c

U)

.
'

...

.. ;

_,

.,

~-~

----1

Nathan Hasson

.Multiple Chok:e Questions


;'.

n'-

::;:..e.~;..:::.

. -

~-"'t~".;:>"".l

of juvenile idiopat~c
artitrltis
'

0 A the majority of patients are positive for rheumatoid factor ' '
O~B. arthritis is presentfor a mhiirnurn"'f 3;months.
0 iYonly occurs in children l1nder;\j 6 years of age
....
0 D it occurs in 1 in 1000 children

0 E;~,i~ is mor~!~quen~n.girls.
~
1. Th ... following are characteristic features

c.

W.

. . . ...

2. The following .is true of systemic ons.et juvenile idiopathic

anhritis

...

...

A it usually occurs in children under 5 years of age


B fevers last for more than 2 weeks:
.

0 c only cervical lYmphadenopathy is seen


0 D antinuclear antibOdy is usually positi.Je

0 E; disseminated intravascular coag:uiaUon is a recognised complication


,

-~

..,

:t

...

,.._

3. In polyarticular juvenile idiopath,ic arthritisr:


0 A rheumatoid factor is. ocia$ionally positive
0 e low-grade fever is. common
0 C thrombocytosis is seen
0 D methotrexate is the treatment of <:hoice
0 E three joints or more are affected
4.. The following is true of oligoarticular juvenile idiopathic

arthritis
0 A it is more common.in females
0 B antinuclear antibody is usually p(!,sitive
0' c methotrexate is the treatment of choice
o o the affected)imb has deceleratedj:rovith'
0 E it is usually seen after the age, of8 years

133

ESSENTiAL QUESTIONS FC~ MRCPCH

5. Juvenile psoriatic artluitis is ~ssociated with

0
0
0
0
0

A
B
C
D
E

skin rash
nail pitting
. . .
antinuclear antibod) positivity
family history of psoriasis
a poor response to methotrexate

1 ,

;-:

6. lit arthritis associated with htflaminatory bowel disease

0
0
0
0
0

A disease course is usually polyarticular


B erythema nodosum is a recognised association
c human leukocyte antigen HLA 827 is present . ;;
D sulfasalazine is the rug' of chOice-~ ~~
' '
E poor outcome of the joint disease is usual
+ 4 ~-~
tu.. "1,.-~/'~"" J,
.. .;~ .:c'*
I;,,,.~

,,

7. Juvenile dermatomyositis is associated with

0
0
0
0
0

A
B
C
0
E

.(:

" ... '

a rash in a malar distribution .'"


' "
;olJ " 1 -1
arashovertheextensorsurfacesoffingers ~n,.:: ..,
HH.!'
'
nasal speech
. "':
t
.' J
antinuclear antibody positivity in the majority
'' .. :'II
raised serum levels of lactate dehydrogenase
v; -

A
B
C
D
E

oral ulceration
photosensitivity.
lymphopenia
inflammatory bowel disease
rheumatoid factor positivity
1"

,.,

r-

! .. ,l.,t

.5:

...

8. Features ofsystemic lupus erythematosus l'ndude 1 i

0
0
0
0
0

""t..

N
.l
- 1
L

J~Jl, .. rJ ~-

n:
f\

""... , "),J'

.. : ...

...rt

-...

<'t

-.

9. cfi~racteristic features of Henoch..:.schonlein purpura include


0 A onset before 3 years of age 1 '1 , ~~! ;
d, "
r.
0 B flittingarthritis
'..<. .v!1 ....
,., . ' l l ' '! .!.,'!..~'

0
0
0

C minor.abdominaLpain
--------~--~"""' -: ._,. ;
f ~-:
4
0 immunoglobulin A complexes in affected skin on biopsy' f -' 1 E good response to methotrexate
' '; 11 J .> l , : - ;

,.

10. Kawasaki disease is associated with

0
.0
0
0
0

A
B
C
D
E

fever for at least 5 days 1 " ' ' ' 1 '


purulent conjunctivitis
thrombocytopenia
coronary artery aneurysms in 30%.
generalized lymphadenopathy

, r-

t:._.~ ~

'"',

~ :;r~ "".-.-r'~
e~

J ,

1.

. . ,.

~.1
~

..

-- ~'-

~1--

RHEUMATOLO,GY

II. Recognised features of Wegener's granulomatosis are

0 A
0 .B
0 c
0 D
0 E

destructive granulomata in the si.nuses


skin rash
. . .

'
positive antineutrophilic cytoplasm' antibody'
pulmonary lesions '
.
~

goc>d'resp6nse to methotrexate'

f
.~
'1,.~ l
,.;

....

<

'>I'

12: The folloWing are features'~(,p.Qlyarteritis nodos~


A fever
'

B abdomin~l pain
,
C low white cell count
D steroids are contraindicated
E Staphylococcus is a recognlse(ft~igger '

0
0
0
0
0

..

.)

.r

~-

13. The following are associated with rheumatic fever

0 A grouifA:;trep~ococc;al i!:J~ciion 1 i'


0 ,B subcutaneous nodules ... , . . "
0 . c .antinuclear antHx)'dy poSitive ; ,_.
0 D arthralgia .. . , - '
0 E' eiythema muitiforme

.
. f

14. Causes of antinuclear af\tibody ~s.itivity in children include


0 A systemic lupus. erythematosus
, ..
0. B systemic onset juvenile idiopathic arthritis
0 C P<irvovirus infection
0 D scleroderma
.
0 l E mixed connective tissue disease~
{

~ f: ...

'

ts: The.f?llo~ngare side effects of methotrexate


0 A flausea . .. ..

0 B1 pulmonatyJibrosis ....
0 C transaminitis
0 D raised platelets
0 E rash

.'
",_!.s

.:~':~l~

135

. ~~~;;

ESSEf'~TIA:..

QUESTIONS FOR MRCPCH

~!

,,./.. ,

u)~:.

tf

c~;:

:I

0,:

.........

\.

. (I)

. (I)

::s.

tT

. , v.~ Best .of Five Questions


'

. t'f ,.

- :

16. A 3-yearold girl presents with a 3~week history of fever, with


daily spikes of 40"C, a pink rash that varies, painful muscles,
some swelling of her wrists for 7 weeks, and generali:z;ed
lymphadenopathy. Investigations revealanaemia~~!h a high
white cell count, raised platelets, very high erythrocyte .
sedimentation ra~e an~ <:;: reac~ive protein_.. f\nti_nm;:lear antib<?dy,
double-stranded DNA, and rheumatoid factor are negative~
Which of the following is the MOST likely diagnosis?

0
0

0
0

A
B
C
D
E

~ ,l

Systemic lupus erythematosus


Acute lymphoblastic leukaemia
Systemic onset juvenile idiopathic arthritis
Par\'ovirus infection

Neurob~astom~. .. ~.

"

r
f

'!

f-1

....
''
17. A Syear-old boy presents with a 2'-month history of rash
around his eyes, an inability to brush his hair, nasal spe'ech, . ~
and he uses Gower's manoeuvre to'get up from the floor. His ....
creatine kinase is normal but his lactate dehydrogeri,ase is '
raised. What is the MOST likely diagnosis'? 1 H : '" ' '

0 .I\ Be~~r muscul~r dystr<?phy


0 B Systemic lupus erythematosus

0
0
0

C Juvenile dermatomyositis
D Myasthenia gravis ''
E Viral myositis

..-~

'

.:~ .., llJ ,. ~. , "'

'

'

.~

I
\

18. An Syear-old boy presents with an s~week history of swelling


of both knees, and a dry rash over.his elbows and knees. His
father has dry
patches in his scalp. He als'o is found to .t:: '
have swelling of two of his toes. His GP has been treating him"
for a fungal infection o(his nails. What is the MOSTlikely 1 1
diagnosis?
T.
-,..~-

skin

0
0
0
0
0

A
B
C
D
E

Enthesitis-related juvenile idiopathic arthritis <:: ~~ ....


...:
.
.J.
Psoriatic arthritis

Ofigoarticular juvenile idiopathic arthritis
]uYenile dermatomyositis
Inflammatory bowel disease related arthritis

(' '

19. A 5-year-old boy presents with a history of recurr;ent right knee


pain for 4 weeks. He has had difficulty with sport, and now he
. limps. Previously he was very active; and was ver) good at
gymnastics. He had two bouts of ton'slllitis recent y, neither of
which required antibiotic treatment. There has be<:n no fever or
136
............

RH EU MATOLOG Y

0
0
0
0
0

rash. Examination reveals a compJetely normai right knee and


the only abnormality is loss of internal rotation at-the right hip,
but no pain. What is the MOST likely diagnosis?,

,
A Oligoarticular juvenile idiopathic arthritis
B Benign joint hypermobility syndrome., . .
- I.J
C Enthesitis-related arthritis.
D Perthes disease
E Reactive arthritis
ti' "

'I

f"

_,,

\.

f' .

J'"'

'

..

'{

, ..

{'

. ! ., "

'' .
'

.i-f

..

.'
'

."

:!

137

ESSE!'-::rlAL CLIEST!ONS FOR i1RCPCH

; Extended Matching Question


'

~.Jt~JI?II'!':l.~V~~ ..

:y,

(<:!1

~~~,_~

'

..-a:-~~.r .. ,.*~.'~.;.Jt:t'"'

20. Theme: Rheumatology case studies


Systemic onset juvenile idiopathi~arthritis
Polyarticular juvenile idiopathic arthritis
Oligoarticular juvenile idiopathic arthritis
Enthesitis related arthritis
PSoriatic arthritis
Systemic lupus erythematosus
Juvenile dermatomyositis
Acute lymphoblastic leukaemia
Chronic fatigue syndrome
Polyarteritis nodosa

A
~B
C
D

GJ .

. ::,_..

c-

H
I

'

"\..
:

~J '

Fot; each of the following case scentiriqs select the most likely diagnosis from

those lisled above. Each oplion.m,~Y;M.t!Sf'!d once, more lhan once, or not at
pll.
. ' .
.

I. A 4-year-old.boy presents with a !-month history of fever, rash. and


painful joints. On exam'ination he has ar'thrftis in both' knees but the
rest of his joints are clinica!fy normal. He has rash on his face.
Investigations show (normal ranges in brackets): haemoglobin 6.5 gtdL
(10.5-12.5); white cell count 5.6 (5-15) (lymphocytes 0.7); platelets 100
(150-400); eiythrocyte~~djJTient:ation rate '67 mrnJh (< 15); C reactive
protein 4 (< 7); antii}Uci~a(antibody 1 in 2560; boneinarrow blasts 1%

2. A 9-year-old boy presents with pain in both heels for 3 months. He has
also been to the GP with a painful red eye on several occasions. On
examination he has an effusion in his left knee. decreased range of
movement in his right hip, and tenderness along the~Achilles,tendons
and heel. His erythrocyte sedimentation rate was I 00 mm/h; C reactive
protein 45 (< 7): full blood count normal; human le"!lkocyte al)tigen
(HLA) B27 positive.

3. AS-year-old girl presents with arthritis in her right kne,:andleft wrist


for 2 months. She has generalised lymphadenopathy. Ophthalmological
eXamination is normal. Investigations show her antinuclear antibody is
negative. Her erythrocyte sedimentation rate is 65 mm/h; C reacti\e
protein 40 (< 7); full blood count shows a haemoglobin of9g/dL
(I 0.5-12.5); white cell count 2.3 (5-15); platelets 97 (150-400).

(Q-2%).

138.

.,

'

r-.,

..
T

1. Juvenile idiopathic arthritis

Answers: DE

The new c;lassification of juvenile idiopathic arthritis' replaces the old one of
'juvenile chronic/rheumatoid arthritis'. Most patients are negative for

rheumatoid factor; with only 2-3%. p<)sitive. For diagnosis of juvenile


~
idiopathic arthritis, arthritis has be present for 6 weekS rather than tne old
criterion of 3 months, in a patient aged less than 16 years. Systemic onset .
juvenile idiopathic arthritis also occurs in adults. h1cid~nc~ iS I in I 000 and
there art eight subtypes. Generally more girls are seen, as the oligoarticular
form is the commonest, which is more frequent in girls.

to

~ ....

tt . .__-

~~

.'{

'i:J

't

'.f'J .. ' .. t..

-'

2. Systemic onset juvenile idiopathic arthritis


Answers: A B E '

.. .

.,..

'.

'

Systemic ~nset is more corrimon in children aged lessthan 5 years old.


, ..
Incidence is equal in boys and girls. The definition is a minimum of 2 weeks of
fevers, in which daily spikes are seen. The rash is a fleeting salmon-pink
'
colour. Generalised lymphadenopathy is seen unlike ,in J<;awasaki disease,
where only cervical adenopathy is a diagnostic criteria: Antinuclear antibody
and rheumatoid factor are usually negative. Disseminated intravascular
coagulation is occasionally. seen as a complication.
'"
.,
f
)

3.

'"

Polyarticular. juvenile idiopathic arthritis

i'

Answers: A CD:

Rheumatoid factor is positive in 2-'3% ofpolyarti~ular juvenile idiopathic


arthritis patients. Low-grade fevers are only occasionally_seen ..
Thrombocytosis is a feature, as are raised erythrocyte sedimentation rate and
C reactive protein. Methotrexate is the drug of choice in both positive
rheumatoid factor and negative rheumatoid factor polyarticular juvenile '
idiopat.hit arthritis. Five joints or more are affected.
'
4. Oligoarticular Juvenileidiopathic arthritis
Answet.s: A B

Oligoarticular juvenile idiopathic arthritis is the commonest subtype of


juvenile idiopathic arthritis. It is diagnosed if four joints or less are affected in
the first 6 months of the disease. Oligoarticular juvenile idiopathic arthritis is
four times more common in females. Antinuclear antibody is positive in' .
6Q-70% of patients. Methotrexate is rarely used to treat the disease except for
139

~,~-"?~-

I
.

'

ESSENTII'.!.: QUESTIONS FC:"( MRCPCH

non-responsive disease or severe anterior uveitis. The affected limbespecially if a knee is affected::.. has acceierated growth. It is usually seen~
under the age of 6 years. It is termed persistent oligo-juvenile idiopathic
arthritis if arthritis is restricted to four joints or less after the firsf6 months of
disease, and extended oligo-juvenile idiopathic arthritis if more than four
joints are affected after the first 6 months of disease.
,.

5. Juvenile psoriatic arthritis


Answer.;: A B CD
.

'

!i

~.

In psoriatic..arthritis the rash _may .?~,rnlwnot be coincident with arthritis. Nail ..


pitting is seen il1 psoriasis.Dactylitis Of.fingers or toes is seen and is typical.
Distal interphalan~eal joint 'arthrfii~tialso typical. It can affect a few or many
joints. Soine
pat~ents
rriaJ'de\eh;)puveitiss6
slit-lamp :monitoring
is n~eded
. ~

~
~~",_ .,
.-"!-!'. -~
-.
Some patients a!e positive for antinuclear antibody, and have a family hist()ry
of psori,asis. The condition respof!dS well methotrexate.
. ' I. '' .
/""'',!--.-

.....,_.

,_...

'

-'

'

<-.

,-

'"''''

to

6. Arthritis associated with inflammatory bowel disease


Ans\-vers: BCD

. ,,

Inflammatory bowel disease-related arthritis is usually oligoarticular in . A


course. Se\~cer~ldjfferert,~!2n.Je~ion~ i_ncluping erytry~ma nodosum <~;re seen.
HLA B27 positiVity is a fe~tur~:;Su~(!l,~alazine !s the disease-modifying drug of
choiee~ Good joint outcome is usually seen.,
.
..
""

1. Juv~nue'dermat6myositis
'

.,

...

Answers: B c E

..

'

..

. . . ,

Rash is usually~around Ufe eyes. associated with oedema, and spares the .1
malar,area. Gottron's papules occur on theextensor surfaces oftfingers, ii!1d
are red raised papular lesions: Pal,atal weakness leads lo nasa! speech.
Antinuclear antibody is only positive in a small percentage. Creatine ldnase
levels can. be hormal but Iai:~~e dehydrogenase is ,often. r~J~ed~ . , , . . ..,

8. systemk lupus erytherhatosus , '.


"

:~.

"H

f~ol

".

Answers: All true,.

~('~

"

..

These are aU features of systemic lupus erythematosus.~The 1997 Americ<Ul


College of Rheumatology criteria h1clude most of these. Renal disease is fairly
common in children with systemiclupus erythematosus. A person shall be
. said to have systemic lupus erythematosus if any four ormore of the
'
following 11 criteria are present:
1. Malar rash .. .
'1
2. Discoid rash . :
'' .. I
3. Ph. otosensitivity.'.
,
;!..' ~
4..oi-afutc~rs , . .
.

5. Non-erosive arthritis of rnore than, one joiryt

...

140

.q

'!

...,

'

_L

:~~1
!
RHEUMATOLOGY

6.
7.
8.
9.

Serositis (pleuritis; or pericarditis)


>
Renal disorder (persistent proteinuria> 0.5 g!day; or cellular casts)
Neurological disorder (seizures; or psychosis) .
Haematological disorder (haemolytic anaemia; or leukopenia; or
lymphopenia; or thrombocytopenia)
10. Immunological disorder (anti-phospholipid antibodies; or anti-DNA
. antibody to native DNA; or anti~sm antibody to Sm nuclear; or falsepositive serological test for syphilis)
ll. Antinuclear antibody (abnoT1llal titre).
.

9. Henocb.:.schorilein purpura
Answers: D

Henoch-Schbnlein purpura usually occurs after the age of 3years. There is a


male predominance. The arthritis is very painful but is not flitting, as in
rheumatic fever. Abdominal pain is usually severe, and complicationssuch as
intussusception occur. Immunoglobulin A complexes are seen in the skin and .
glomeruli ifbiopsied. Renal involvement is common. Non-steroidal anti.inflammatory drugs are used, and occasionally steroids, to manage the
arthritis- but not methotrexate.

1o. Kawasaki disease


Answer.s: A

Five days of fever is necessary for the diagnosis. The conjunctiva is red but not
purulent. Thrombocytosis is seen rather than thrombocytopenia. Aneurysms
of the coronary artery and other large arteries are seen in 20%. Cervical
(rather than generalized) lymphadenopathy only is a diagnostic criteria.
Erythema of palms and soles ;with oedema and subsequent peeling are seen.
Erythema of the lips with oral inflammation are also part of the diagnostic
criteria.

11. Wegener'.s granulomatosis


'

!I

Answer.s: A B CD

Wegener's is a granulomatous \'asculitic disease. Biopsies of affected tissues


show granulomas. Destructive granulomata occur in the ears, the nose and
the sinuses. Rash, pulmonary and renal lesions are seen. Antineutrophilic
cytoplasmic antibodies are positive in wegener's. The treatment is steroids .
and cyclophosphamide for systemic disease.

12. Polyarteritis nodosa


Answer.s: A B

Polyarteritis nodosa is a vasculitis affecting small to medium sized muscular


arteries. Polyarteritis nodosa can present with fever and abdominal pain.
Leucocytosis is a feature. Treatment is usually with high-dose steroids.
14l

FY;~!Yi
~~: . .~.. ~J ~~..~ .i ~:.
- i'

.ESSENTIAl! QUESTIONS FOR MRCPCH

Streptococcal infection- ratt..;r than staphylococcalinfection-'- is alrigger.- so


antistreptolysin 0 test should be performed and penicillin prophylaxis used.
Aneurys.ms are diagnosed by angiogr~phyand are present-in sm~IHo, "~ .
medium-:sized muscular arteries such as renal,,liver, splenic end bowel. ('
'

13. Rheumatic fever-Ht


AhS\1'C'TS:.A B D ''

'

< ..J

'

'I;

"I'<

.f ,I

tt ~ -

...

~.

'

~ l ...... I

'

""'!

11' ,, ' ''


r

- ---

For the diagnosis there has to be proof of group 'A streptococcal infection.
Subcutaneous nodules are a major criterion. vmTmust'have: two major c:dt~ria
or one major and tvvo minor criteria for the dia~osi~.!-rJinuslear antib9dy .~s
1
usually negative. Arthralgia is quite often severe: and 'is mifior c!fi terion: The
classical rash is erythema marginatum. which is a major criterion: The revised
. )ones:criteria include,the following major_manifestations:wJ . 11 k , ' , '

:e
.

carditis
Polyarthritis ..... ,. ' 1 ' .,
rEi'yth:ema margiriaturn .
Subcutaneclusnodules'"
Chorea 'Ci'- ''t rr: 01 '' ,. ' '.
. .

"Jll

1 .~.l:

1_,, ..; j

:r

II

I
.f

~,l

The revised Jones criteria inClude the following minor manifestations:


Fever
Arthralgia
. ~ Pr_evious rheumatic ~ever_ or rhempatiche.cu.t disease;;
. .,.
Raised.erythrocyte sedimentation rate or C reactive protein
Prolonged PR inte~al on ECG.
:
.

..

14. Antinucle~r antibodY. positivity


. AnsweJ:S: AD E

.,
Of patients with systemic lupus erythematous, 97% are positi':'~ for
antinuclear antibody. Systemic onset juvenile idiopathic arthritis is not
associated with antinuclear antibody.Ebsrein.;.Barrvifarmrection'- hot'
parlrovirus- is associated with antinuclear antibody. In both scleroderma and
mixed connective tissue disease patients are often antinuclear antibody
positive. Causes of antintJC!ear antibody positivity in children include: , 1

Systemic lupus erythematosus


Juvenile idiopathic arthritis
Chronic active hepatitis
Scleroderma
Mixed connective tissue disease
Drugs (eg anticonvulsants)
Ebstein-Barr virus infection.

'142

.
.

r'

'

'

AnS\:relS:AC-~-"..:t r1-~.t1

t:.-"'j-"!

>t'r ,:4 n-

_.1:(,

--.t~~- ./t.~""'

~,-t_,._"' .~:

-~' "Tt

M'ethotrexate iS the most frequently used disease-modifying drug in' I


\<paediatric rheumatology: It is.n6t only usedin.juvenile idiopathic arthritis; but
also in' den'natomyositis,~scleroderma. and in some patients with luplls: It can
cause nausea:whiCh responds to anti-emeticssuch aspdansetron, or nausea
can beav6ided if administered by subcutaneous injection. It can cause mouth
ulCers:: and folic acid supplements are given to prevent this and other.sidet
effects: PUlmonary fibrosis issafd. to occur in adults but is riotseen in,, ,f'
paediatric practice>Lhrer enzymes can .rise,:but not usually resulting in liver,
fibrosis. It can lower plateletor"white cell G:ounts, and occasionally ;. 1:.: " '
haemoglobin. Rash is not usually a side effect
,
..,. --~ ~:: l ... :,_ ~.::.t - ~ ~v)~ -~ C""., 1 ; ...:Ch { v t

16. C: Systemic onset juvenileidioP.athic


1"-Jt-.'r.~
~;.(.
'* -~ arthritis
~ -'
.~.~ i j f
.

(.;

These areal I the classica!Jindingsin systemic onset juv~I}ilt; i~iopathic a[tt,lritis .


. More than a 2-week history of spiking fevers is one ofthe.<;ritefia neede~Jor._t,he
diagno~is. The rash is salmon pink anct,can ,vary in intensity. ()Ve!r.th.f! da,Y.. bei~g
prominent at times offever: Generalised lymphadenopathy ts ~ commorfeature.
ArthritiS has tO oe p'resent for more than 6 weeks in systemic_onserjUV,!!ni\~
idiopathic arthritis. Systemidupus erythematosus usually has ,a pos!!iye,,,'lantinuclearantibody and normal C reactiye protein. rlatelel$ ar~ l!~u~)ly normal
. orlow.inacute lymphoblastic leukaemia. The history istoo_prolgngedf<x
parvovirus. and rash is unusual in neuroblastoma, as is hi?h plat<:J,~t cpunt

I 7. C: Juvenile dennatomyosi tis


The features are all classical of juvenile dermatomyositis. The rash around the
eyes is the violaceous heliotrope seen in juvenile dermatomyositis. The
weakness is proximal and more often worse in the upper limbs. Palatal
weakness causes nasal speech. Quite often creatine kinase can be normal but
lactate dehydrogenase raised. ln Becker muscular dystrophy the creatine
- kinase is-very high: There are not enough features for the diagnosis_of lup.us _
erythematosus: Myasthenia gravis is not associated with rash. Viral myositis
often affects the calf muscles and has an .associated high creatine kinase.

J?'

'

18. B: Psoriatic arthritis


Psoriatic arthritis can present with few or several joints affected. It can occur
without the classical psoriatic rash but occasionally a psoriatic-like rash as
described here can be seen. Dactylitis is a feature of psoriatic arthritis and is
rarely seen in other types of juvenile idiopathic arthritis. The father's dry scalp
patches may possibly be psoriatic and family history is often seen. The nail
problem may be psoriatic and typically pitting is seen. In enthesitis-related
arthritis, enthesitis and tendonitis are features- which are not seen in this
patient. Enthesitis is inflammation at the site of tendon insertion into bones
such as the posterior heel.

143

~"E

ESSENTIAL QUESTIONS FOK: o1RCPCH

'I!

19. D: Perthes disease


Perthes disease is avascular necrosis of the femoral head. lt is more common
in boys ofthis age. Frequently it presents with referred pain to the knee,
rather than hip pain. The only sign on examination may be loss_ of internal
rotation initially. There can be an as~ociated synovitis. The history is too short
for oligoarticular juvenile idiop~thic~ar!hriti~ (6,weeks beipg necessary for the
diagn,osis) and the hip is almost never affected if.'ith this: The patient is
probably hypermobHe giv.en that he is good atgymnastics, but the range of
movement \VOuld be increased with often 90 of internal rotation, rather than
the limited movement of this patiel!t. The history is possible for reactive
arthritis. but an irritable hipwoula;_M1Jast for 4 weeks, and it is unusual in
other re~cti\'e arthritidesJor the h~_pjC);;~e affected.

~_-_-.,::

;;,r.'
UJ

rd

20. RHEUMATOLOGY CASE STlJDlES


:
'.. . ' -

1. F- systemic lupus eryuiematosus


- These are all typical of systemic lupus 'erythematosus afid there are enough
features to fulfil the diagnostic criter!a.
[
2. D- Enthesitis-rehtted arthritis
These are all the. features of enthesitis-related arthritis.
3! H -Acute lymphoblasti~ leukaemia
This' girl is unlikely to have'oligoarticuliiir'juvenile idiopathic ahhritis with
these biood results, becau~e the erYthrocyte sedimentation rate and c reactive
protein level are not usually raiseq; ant:! the fdll blood count is very abnormal.
In parti~ular the platelets are iow.- - -

144

-. ......
~

._L

l -

Angie Wade

\;.

... -

t~

.._ .:-.

~~~~';:."=';~~~;-o.."!$.-~~~
$

-~.:

< . \

'

...

Multiple Choice Questions


it.'l.rA'

.:.

-~--:-\

,-

'

1. Randomisation of patients to treatments within a triai ensures


that

A the patient is unaware of the treatment group to which they are

assigned
.
B each patient has an equal chance of being in any treatment group
C the treatment group is lcnow.n before consc:;nt is obtained ,
.
D although individuals receive:differ~nt treatments. each pati~rit will tie
allocated to the treatment most likely to benefit them
E differences between treatments will be significant

0
0
0
0

'

2. Within a clinical trial, allocation of patients to treatments


should, where possible, 'be

0
0
0
0

A blinded

B randomised
C systematic
D decided prior to obtaining consent
E performed away from the study centre

,,

3. In a trial of vitamin supplementation on reaction times among


12-year-olds, which of the following need to occur for. ~social
class' to be a confounder?

0
0
0
0
0

A reaction times are differentiA the supplemented and non-

supplemented (control) groups


B the children are not randomised to treatment groups
C the supplemented group have a different social class distribution to the
non-supplemented (control) group
D the non-supplemented (control) group are all children of professional
parents
.
.E reaction times are associated with social class

-- 145

: .... ''.?';
:..: : ,:;~~~. . . ' :~;/:: .:~--

~~
:~

0 ...

ESSENTIA_ QUESTIOI\IS FOR i1RCPCH

4. A new asthma irihaler is tested ag~inst a standard inhaler within


a double-blind ttossover trial. With this study design
0 A neither the "patient nor the assessor knows which treatme'nt (standard
or new) is being given at any time
0 B any differen2es found between the new and standard inhaler must be
~~rn~ro

0 c the order of treatments (new and standard) should be randomised


0 D fewer patients will be needed than if a parallel trial of new versus
standard inhalers had been used

E the outcome must be normally distributed

5. Observational studies
A cannot be randomi~ed
B give in ore convin~ing.~'idence of true differences than experimental
studies
' '
"

'.
0 C are always large
0 D can never be useful
0 E must be blinded

0
0

6. The following is true when age-matched and sex-matched pairs


of patients are allocated to new or standard treatments
0 A age and S~X c~mnotconfound the study results
0 B ran,domisat;ion to th:e new or standard treatment should take place
within pair_s
,
0 c the treatmen~ allocations must be blinded
0 D the pairing should be retained in the analysis
0 E di~se severity will be similar between the treatment groups
7. The following are categoric variables
0 A height
.0 B sodalclass
0 cage
0 D gender
0 E ethnicity

...

'

'

8. When data are ranked


0 A the highest rank is equal to the total number in the sample
0 B the median is the middle ranked value
0 c the lowest value has rank I
0 D any equal data values must be removed from the sample prior to
ranking
0 E the mean of the values is the middle-ranked value

146

--~-.!..

:I
STATiSTICS.

I
II

v
11. The standard error of an estimate

!':_,.

0 A
0 B
0 ,C

is smaller for larger sample sizes


.,
is a measure of the precision of that estimate
cannot be negative

.
,r> depends ~m the ave~~ge Y.aJu~ oft~e ~a~ pie .
E is used. to constiJJct confidence
intervals
,.
'
:'

li

.t

")

l "t!

,.

'"

0:

"

Apvalue .;,

12.

'

"

. : ''

0
0
0
0

'

sample(s)
' .
'

- ' :.
B lies between-! and +I
:~ 1
,..f
C is more useful than a confidence interval for.interpretingresults
D is the probabiJUy.of obtaining the current sample if the null hypothesis
is true
,.
'
E" indicates the clinical significance of any differences seen iD the sample(s)

13. A parametric
correlation
coefficient
.
'
. .
.
. .. ,
~

:,~ i[t<J).~~~{::~ ~h~~t_c~ti~!cal sJgnifi~nce of any differences seen~inthe

~.

A must be positive
.. , ,
Q, B (!fzero indicates no relationship bet\Veen the measurements ,
0 C takes the value I only if the points lie on the line of equality
0 D shows the extent to which two continuous measurements are linearly
related
0 E is negative if there is no association
d47

ESSENTIAL QUESTIONS FOR MRCPCH

.'

14. Reflex times are measured. in a group of children aged 5 to 15


years old. The correlation between reflex time and age is
calculated as 0.76 (95% confidence interval 0.7, 0.82). In this
study

0
0
0
0
0

A the correlation coefficient is significantly different to zero


B more measurements need to be made . '
c there is a linear: association bel:\veen reflex times and age
D the association' is clinically important
E older children tend to have slower reflex times

15. In a group of asthmaticchildren there is a tendency for heavier


children to have lower lung functions. It is true that
0 A losing ~eight will improve lutig function

0
0
0
0

B weight affects lung function


""
c the correlation bet:\\een weightand lung function will be negative
D sotial.class may be a confounder . '

'
. ' ' ' r '
E there needs to be a control group to corriparethese villueswith
~-

ir

~~-

~-

~~-.:.--,

'":t"',

16. The following statistics associated with screening tests are


J
directly dependent on prevalimce

0
0
0
0
0

. ,, ' .,

\,'

A sensitivity
B. specificity
C likelihood ratio
D positive predictive value
E proportion of false positives

I 7. Ah ultrasound screen is applied to 500 pregriant women at risk of


DOwn syndrome based on their age and preVious medical history.
Seventy fetuses are found to have femur lengths-in the upper
decile of the normal distribution. Of these 70 fetuses, 25 with
extreme measurements had Down syndrome- as oppc)sed to
30 of the fetuses with measurementS not in the
declie. In. .
this analysis, the following is true
0 A the specificity of the test as a screen for Dm\'n syndrome is 45/445
0 B 25/70 is the positive predictive value of the test (measurement in the

upper

. upper decile)

the sensitivity of femur length in the upper decile for 'diagnosing Down
synd_rome is 30/55

D of those with measurements in the upper decile 1110St will not have . '
Down syndrome

E the study shows that age is an important predictor of Down syndrome

0 C

0
0

,.

148

onJy

STATISTICS

18. Blood pressure is measured in two groups of people. Those


receiving some treatment have blood pressures that a:re on
average 6 mmHg lower than those in the untreated group.
At-test was applied and p = 0.02, 95% confidence interval for
the difference {4.16, 7.84) mmHg.Jt is true to say that

0
0
0
0
0

A the tr~atm~nt sho.uld be introdu~ed as it may be clinically relevant


B the treatment must have improved blood pressure by at least 4.16
mmHg on average in. the population

..
C the difference observed would have occurred oy chance I time in 20 if
there really was no treatment effect
\.
D randomisation to groups was not successful
E the Hest would not have been appropriate if the blood pressure'
. ., measurements were skew .
.l

'

"

"'

19. The number of children positive for a certain genetic defect:


were compared across groups of asthmatic chilqren arid healthy
controls of a similar age. It ca_n be said that

0
0.
0
0
0

A the study is invalid because it is not randomised


B at-test could be used .to assess the significance of the differences

between groups
"
.
'
chi-square could be,used to test the significance of the. differences in
proportions that were positive in the two groups

D age may be a confounder


''
E a confidence interval for the difference in proportions that wefe
positive would help to interpret the results

"

20. The power of a study

'

0
0
0

A varies between -1 imd + 1


B can be calculated retrospectively
C is the probabilily of correctly rejecting the null hypothesis when it is
false

0
0

D increases as the sample size is increased


E is larger the greater the difference that is to be detected

21. Within a randomised controlled trial, of 100 individuals who


received standard care, 40 had an adverse event in the .'
following year. This is in contrast to 20 of 100 Who received a
programme of intensified care. It is true that,

0
0
0
0
0

A the percentage reduction in adverse events attributable tointensified


care is20%
B the number that need to be treated (NNT) to avoid !.adverse event is 5
C relative risk (RR)'is 0.5
D a confidence intervalfor the percentage reduction wi'll depend on the
sample size
E age may be a confounder in the comparison
149

ESSENTiAL QUESTIONS FOR f1RCPCH


-J

Best of Five Questions


' Jt:

}'"

- .

',;

~ "-;~

'

/,.:_.; ....... ..:.' ..

~~:.- .. '1''1:';:;..1 "'~-~


~

.,

.,

-._.

'~
~

'

,. f

22 .. In children ~th renal failure, a study shows that vi.t~min D


levels'are found to be severely depleted (p < 0.0001). Which is
the MO'ST appropriate course of action based on this 'study?

0
0
1

0
0
0

A_ Introduce vitamin Dsupplementation as standard practice


B Consider extent of depletion, clinical implications, costs of
, supplem~ntation and make a decision basea on these
c Re:analyse the data taking into account the ages ofthe 'children'
D Carry out a further.:study of greater size . '
'

E Do nyt.hing,
'

'

' '

23. Cirrhotic children aged 6-10 years old are randomised to a new
die~ ~e~!men er,.s~andard :advice. Afte~ 2 y~ars their l\eight ..
. standarddeviation (sd) scores are compared. The group allocated
to the new diet have a higher t;nean sd score for height
(difference.0.2, 95% confidence interval (-0.8, 1.2)) but this
difference is non-significant (p 0;52). An improvement of 0.2
sd scores over a 2-year period would be considered clinically
important in this group of children. Which is the MOST
<" '
'
. .
'

.
.
appropriate course of action based on this stUdy?

0
0
0
0
0

'
'
A Do nothing further- the study has shown the new diet is riot
statistically significantly better than current practice

B Re-analyse the data using non-parametric' methods


C Follow the children for longer to try and obtain statistical as wei! as
clinical significance
D .Carry out a trial of a larger size to obtain a more precise estiiT!ate of
the effect ofthe new diet compared to standard
..
..

E Introduce the new diet as standard practice- the average'


;
improvement is clinically important
.
'
. . - .

24. Haemoglobin measurements'w~re made in small groups of


children with five different syndromes. In order to assess
:whether there are differences between the groups that are
unlikely to have occurred by chance, which one of the following
. is MOST appropriate?

0 A
0 'B
0 C
0 D
0 E

ISO

A further study ofmuch larger size


Analysis of variance comparing means between the groups
The data should be plotted according to syndromic group
Mann-Whitney u~tests between each pair of syndromic groups
Non-parametric analysis of variance comparing medians between

groups (Kruskai-Wallis)

STATISTICS

.c

25. What is the BEST re~son why co11current control groups are
useful when performing studies?

0
O
0
0

A They allow the use of statistical tests for the comparison of two groups
(eg two sample Hests)
'-

:
B They help to ensure that any differences seen are due to the treatment
or disease being studied

c They allow the study to be blinded
D They help boost the overall numbers studied
E They are better than historical con trois

26. Which of the following applies MOST to a reference range for


CD4 counts in childhood?

'
O A The study should be based on large numbers 6f children
0 B The study needs to be age-related
'
0 c It is useful for assessing children \Vith known disease
0 D It allows the CD4 counts of individual children tc be compared to what
' is expected for normal children of that age
.
0 E It does not give the sensitivity oflow CD4 count in detecting disease
'

'

27. Intelligence (IQ) assessments. and heights are measured in a


, " group of healthy 7 year-olds. To investigate w.h~ther, there is a
meaningful statistically significant relationship which .of the
foHowing is MOST.appropriate?.

0
0
0
0
0

A The correlation coefficient should be calculated


B A regression analysis should be used and the regression coeftlcient
presented with confidence interval
c Ap value must be obtained
D More assessments could be made
E Heights should be ex-pressed as sd (standard deviation):scores

IH

.c

,(I)

u,
t!t-

'0
~;::J

.. (1)

ESSENTIAL QUESTIONS
'

; '

~' ~

..

FQR,MRC~CH

Extended Matching Questi9ns


,,~:;

.., '; . . ~ :. .. l

.~,

' .

:.._~.;.\~'l...~::.JI!."'~:""~~f,;.!-~:.A.V.~ ..

., .

28. Theme: significance tests


A: , 1Wo sample t:test
B .. Paired t-test
'
C Mann-Whitney U-test
D One-way):u1alysis~of variance
E Kruskal-Wallis analysis of variance
F
Regression analysis
G r Correlation,,<;oefficient . . , " ,,, "'
i-1 Chi-square

t ... :. . . . .

For each of the fo!lowiryg stvdy scenqrios c~oose,t[le most appropriate .


statistical test from the Jist above ~oan~Jyse the data, Each option may be used
once, more than once, or no~ at alL
.

pres~ure f!leasu.r,e~en ts 'ar.e mfde in.~ grbup,~f children .with


pit)Jitary hor'mone' di:SOrditrs'a~d.age~ and :Sex-matChed control pairs.
The s:udy ai~s to investigate whether,~i.tuit~;Yh~~!:r,lo~; diso:~e~s. a.re
associated w1th altered blood pressure.
r .

0 2. ~evelopmental tes~ are applied to determine whether children who


'
were admitted to intensive .dire ih the neonatal period are more likely
to have delayed.development at age 5. than those:who were not.
0 3 .. Blood pressures (assumed normally distributed) are compared between

5-year-olds from four different clearly defined racial backgrounds

. 0'
'

'1 . Blood

:29. Theme: Interpreting trial result~


A The difference is statistically and clinically significant; the new cream
. .
should not be introduced
B The difference is statistically significant but difference is clinically small;
the new cream should be introduced

C A larger study is required to determine whether it is worth introducing~


the new cream
.
.
D The difference between the cr~ams is,botll slatlsticaliy'i:ind c'ifrilcali_ ,_
significant; the new cream should be introduced
E The difference is statistically significant but the difference is not of
clinical importance and the new cream should not be intro~uced
F The study provides enough evidence to discount the usefulness of the
new cream
H The study is invalidated by the drop-outs
The results cannot be interpreted because the analysis used was
inappropriate
l

.~

A randomised controlled trial is used to compare the effectiveness of a new


cream (T) for treating eczetna compared to the Ctlrrent alternative cream {C) for
children aged 5-10 years of age. Severity is rated or. a 0 {no rash) to 10 (severe
152

_L

STATISTICS

rash) scale. Ail average fall of2 points on the severity scale attributable. to the
new cream would be deemed ofclinical importance and worth changing io the
new cream to achieve. For the following study results choose the mosh
appropriate interpretation from the Jist above. Each option may be used once,

more than once, or not at all.

0
0
0

I. Those allocated to the new cream have ah average rating of 5:4 '
compared with 7.8 for 'those on the current aJterm3tive (95% .
confidence inten!al for the difference (-3.6, -1.2), p < 0.0005) ..
. 2. Those allocated to th_e new cream have an average rating of 5.4.
. compared with 7.8 for those on the current alternative (95%
confidence interval for the. difference (-6.0, 1.2), p = 0.23).
3. Of the 40 children allocated to the two creams, 30 who used the new
.cream had an !'IV~rage severity ;ating of 5;4. The 50 children wh? used .
. the Ctif!:en~> tre~tl!lent (40 randomised to t~is ~!ea,tme!lt plus the 10 who ,
.did not use t~e new cream but reverted ~tp'"cur;reri_t). h~~ an average rating
. of 7.8. The'95% confidence interval for the mean fall in severityrating .
(-2.4) was (-i6, -1.2). p < 0.0005.
'

\,

,%

f''

30. Them~: Diagnostic testing


A
B

Sensitivity

. Speci~city
c f'ositive predictive value
D
Negative pje.cJictive value
E
False-positive rate
F False-negative rate
G Positive.likeUhood ratio
H Negative likelihood ratio
I.
Percentage correctly:classified
J the preva!ence ofml~carri(lge '

During the first trimester of pregnancy, 400 women. at high risk ofmiscarriage
halle.ultr:asound measurements ofnuchalfold thickness. Subsequently, 80 of
these women miscarry. 100 women hadabnormalnuchalfoldvalues.
Considering abnormal nuchalfold thickness as a potential diagnostic test for
miscarriage in this group, which ofthe above items has a value of 75%?.Each
option may be used once, more than once, or not at all.

0
0
0

I. Equal numt>ers o( those that. miscarry have abnormal or normar


'
.
nuchal fold thickness.
2. Ofthosev.:hosubs~quentlymi~rry,7S have abnormal nuchal fold,;
. thickness.
..
,
..
.
. 3. Of those with abnormal nuchal fold t'hicknesses, 60 subsequently
.
miscarry.

153
I,
;}

-0
:I
C/)

1. Randomisati_on
AnS\\'t'f:'B

t;~ ~reatments
1

in a trial ,

. ,. : "'~

Particip~nts should be randomised togroups to rem'ove 'any potential bia?.


. .

'

meaning that each patient haslhe same'char'ice Or being assigned to either of


the groups; re'gardless of their personal charactefistics. Rimdomisation aims
1
to ensure that the treatrf.lent groups are similar apart froJjlthe ireat~eht
under stud)~ herice'an}"differ~nces in outcome are more easily attributable to
being causall)rrelated to treatmerit.Wis the procedure ofblindil\g- not
randomisation- that'ensures that individuals do not know which groups they
are' in. Randomisation should take place after con serif is ob~airied:
Randomisation does 'riot ensure that differences. in treatme'nts will besignificant.
''
I
t i
. '1 '

2. Allocation of patients to treatments


Ansuers: ABE

~ ;

. ..
Allocation to treatments should be random rather. than systematic to avoid
"')

.J

potential bias. Ideally randomisation should be made via telephone. so. that 1 ,
the process cannot be influenced by any known features of the patient. .
Consent should be obtained before treatment group is determined othenyise
the approach taken to gaining consent (and/or the patient's ded~io1_1 to
consent or not) may be affected by the planned allocation. lfthe patient
and/or the clinician (or assessor) know which treatment a 'patient is having.
then this may influence their recorded outcome. A study is blind Wh<7n~ither 1
the patient and/or the clinician (assessor) does not know the treatment
a!location. Single blind is the term used when one of the two (patient or
clinlciarVassessor) 'does not kitow tfie' aliocatran but'the other ooeS:"bouoleblind means that neither knows about it. '

3. Sodal class

as a confounder .

Answers: C E

A confounding factor isa background variable (something not of direct '


interest) that is distributed differently between the groups being compared,
and which affects the outcome being studied. so for social class to'be a
confounder in the comparison of reaction times beMeen th.ose supplemented
and those who are not, the social'class' distribution must differ betWeen '
supplemented and non-supplemented groups and social class must affect
reaction times.

154

l
. STATISTH:s

4. Double::. blind 'crossover trlal of asthma inhaler


AnS\-vers: A

cD

'l '

..

In a crossover study, each patient receives treatment and placebo in a random


order. Fewer. patients are needed because many between-patient confounders
may be removed. The order of treatments should be randomised. otherwise
bias may be introduced if there is an order effect. A study is double.blind if;
neither the patient horthe researcher assessing the patients (orthetreating
clinician) knowswhich treatment the patient has been randomised to receive.
The outcome may or may not be normally distributed. Choosing a particular
study design (crossover) does not make the outcome normally distributed~
Any differences foun&Jn outcome during the two treatment periods,may or
may not
be statistiCally
significant
and/or
clinically significant.,iThere
may,
be
i 'i
.
'
J " ' .
" . .
:
<
.
" '
piffer~':lces Jha.t are non-signpicant;.
, -..
~ . '-' .,
,
.
~

5. pb,~e~~:~<!~al ,~~~,e~_',
An~wer:A

.-('

'I

, , ' ;. "., ~ ',: :

' .

>; ~.

,1 1 ,

~In

obsetvationalstudiesthe groups being compared are already defined'and


the study merely observes what happens. Since groups (different diseases or
<different treatmt::,nts) ar~ ~!ready deterf!lined and known they cannot be blind
.OHanddrriiSed:,:fhe
the sainple'<that
is studied is detetminea.
by the i 1
<;:
1
<. 1i\:.; ~
1-<

researcher and may-be small.'Ifa difference is found in an observational study


.
then i~is more likely to be due to confounding f<ktors than it is'incifl
experimental
trial, because,;v the groups were determined b.efore
the .study. 'A
#
'
difference found with an experimental design is more likely to result from th_e
treat~ent than 1; is in an oJ;>servational sq.ldy. Despite the potential for.C
'
confounding, observational studies can provide useful information;;, .. ; ,;,
Interpretation ofthe resultS should.take into account the iimitations of the
desigQ,. ,
" . , .
', "
. .
.
. ,.
~ ~. ,
't .

size or

':!

j;

6 .. Age~matched cind'~ex-'inatched pairs


1''

'.

"f

'

{\hswe~?~~D

"'

,.

"

-1 .

. ,_.;

'..-

'.

Confounding may t>e'avoided by matching individuals in the groups according


to potenti~l .confounders: 'Here, individuals of the same age and sex are ~ .
allocated to new or standard treatments within pairs. Therefore the new and
standard groups wilL have the same age and sex distribution and these two'
variables cannot be confounders. Randomisation to new or standard..
treatment should take place within the pair and this pairing should be
retained in the analysis. ideally treatment allocation should be blinded, but it
may not be possible:to do so for these particular treatments. All individuals of
the sameage and sex are not necessarily similar in:their disease severity.
hence matcliingfcm.agean9 sex does not .ensure that disease severity will be .
similar between thegroups.
fl

'

~'

t{

I
Ii
l

ESSEr;JTIAL QUESTIONS

FO~

MRCPCH

7. categoric variables. .
Answers: B DE

Data may be either categoric or numeric. With categoric variables, each


indi\'iduallies in one category. Numeric data are measured on a number scale.
Height and age are both numeric. Social class falls into one of five or six
categories depending on how it is defined. Gender is only either male or
female (one of two categories) and ethnicity may be divided into a variable
number of categories.

'

8. Ranked data, , '

I.

AnS\\'ers.: A B C

Ranks give the order ofincreasing magnitudeof numeric variables.' The.


.
lowest value has rank I. The highest value will have a rank equal to the total
number in the sample (if there are I 0 values, then the ranks I to I 0 will be ,
assigned and the largest value will have rank I 0). Half of the value~ will be c..
smaller than the middle ranked value and half will be larger, hence the
middle-ranked value is the median (50th centile): If the data are skew then the
mean and median will be different, and because the median is always the
middle-ranked value the mean will not be equal to ~he middle-ranked value if
the data are skew .. Equal data values should be given equal ranks. To achieve
. this, the corresponding ranks will be averaged between the data values. Each
value in the dataset should be given a rank. Data values that are the same do
not need to {should not) be removed from the sample prior to ranking.

9. oxygenation index in children


Ans\ver.s: A c
.r
For symmetrically distributed data: the mean and median "values are the sam~

.
and wiU be mid-way between the extreme values of the distribution. When data
are skew then the mean is pulled in the direction of the ~kew, away from the ,
..~.mediaR,.SI<ew is named according-tothe-direction~ofthe outlying tail. For the
oxygenation index. the mean is larger than the median and both are much
doser to the lowest value of 2 than to the highest value of 250. Hence it is
reasonable to assu)'Tlethat most individuals have relatively low oxygenation .,
values and there are a few with high values that have a large influence on the .
mean (making it much larger than the median). The mean is influenced by a .
few large values, so it is not representative of the bulk of the values- the
median is a much better measure of what is average or representative of most
indhiduals. No measure of the precision of the estimates of mean and median
are ghen. The precision will depend on the sample size. Hence the group of 30
children may or may not give adequate information- we would need to
'
estimate the precision and see whether this is suitable for.our needs: If greater
precision is required then a larger sample would have to be taken: Reliability is
the e:x1ent to which the measurements would be replicated if taken again (eg at

tl56

STATISTICS
a different time or by a different assessor). The valu~s.given are,based on a
single measurement in each child and this gives us no informatiof! about
' reliability.

10. lntelligeiu:e tests in fragile X syndr?me


Answers: A B CD

Normally distributed data are symmetric and therefo're not skew. Since the 1
measurements are approximately normally distributed, then we would expect
about 95% of them to lie within a range mean 2 standard deviations (97
2(5) = 97 10 = 87, 107). The standard error is calculated as the standard
deviation divided by the square root of the sample size and the interval.(mean
2 standard errors) is an approximate 9S% confidence interval for the '
population mean. Hence, for this sample, standard ,error = 5/11(1 00) = 5/10 =
0.5 and a 95% confidence interval is given by (97 . 2(0.5)) = (97 l ). = (96, .
98). The confidence interVal gives the range ofpopulation values that the . ,..
sample data are compatible with. In this case the interval (96, 98) excludes the
expected meanofu)o found amongst normalchn'dren. Hence the mean IQ in
the sample is significantly different from.! oo.
,..
.'

II. Standard'etror ofan estimate


Answers: A B C E

The standard error is a measure of how precisely the sample vahie:


;I
approximates the true P?Pulation yalue. For continuous data it is calculated
as the standard deviation divided by the square root of the sample size.
,.
Confidence intervals can be constructed around the sample estimate uSing
the standard error. Precision 'Nill obviously be greater or better for larger ,
sample si.zes and we _can also see this froin t~e f?rmula for ~~culating . . ,
standard error. (The standard deViation is divided by the square root of the ..
. . sampl~ size. hence as tne sample size increases we divide by a larger number,
and the standard error will be sml:lller. indicating greater or better precision.)
Also. sii'lC~ the'standard deviation is always positive, the standard error must
also be positi'(e. Although it depe!ids on the spread of tl;l~ vaiues around t_he
average (ie the standard deyiation) it does not depend on the average itself,
being a measure tne precision of, ~at average. .
. '. . . ,, ; ., .

o(

12. p value
Answers: A D

_,

The p value is the probabilitv of obtaining the current sample if the null . ,
hypothesis were 'true. u giy~s ~measure of ~he.sta~istical significance of any
differences seen. As it is a probability it can range from o (no probability/never
happeris)to I (certainty/always happens) but cannot be negative. The p value
gives an indication ofliow likely one particular hypothesised value is to be
true, whereas the confidence interiral_gives the. range ofhypothesised values
157

~SSENTIAL QUESTIONS FOR MRCPCH


F

..

.-;,,
...

-:

tn
.C

( .

~.) \-

\Vith which the sample is compatible. Hence confidence imervals give much
more informi'!tion and enable dinical interpretation or' the'results. The pyalue
gives statistical significance. but clinical significance will depend on other
factors such as inconvenience associated With treatment level of
J
improvement, or difference and costs. '

13 . , ~rametric correlation coefficient


. ~'/f
~-
.;-~ -~

.>.:~"'-Anslver.:D

_,

The most commonly used correlation c~effi~i;rit' is Pearson'S', which. is


parametriC. It gives a' measure of the linear association between two '
"
continuous measurements. Non-parametric correlation coefficient~ n .
(Spearman or Kendall) measure the tendency for one variable.to fall or rise as
the other increases -.whether this tendency is linear or not. All cor.relation
coefficients cim take value~ ~et\-veen-1 and+ 1. Neg<!-tive values indic~te that
1
as one of the variables increases the other decreases. A value of 1 indicates a
perfect positive relationship, and if the correlation coeffiCient is'pa,rametric.
then this would mean that the points lie on a straight line (howeyer thy lin<;is
not necessarily the line of eqJality). A Pearson correlation coefficient of zero
(0) indicates that there is no linear association bet\'.'een the variables,

although there may still be a non-linear one: '

. ns

14. Reflex times

. Answers; A C

The correlation coeffideJ'!t' gives a measure of linearassodation between .


reflex times and age. Zero. indicates no linear association, the closer _the value
is to+ 1 or- I :1the stronger the linear association. For this sample, the value is
0.76, which indicates some linear association: The coefficientis positive and
this shows that age. and reflex time both increase together (that is, older .
children have longer reflex times). The confidence interval for the correlation
(0.7, 0.82) does not contain zero and the correlation.is therefore significa'ntly
different to zero. Whether or not more measurements.need to.bemade-- ..
- -depends on whether the precision obtail1ed.forth~-estimate is suitable tor
whatever. 'purpose it was
The association is statisti~ly significant,
whether it is clinically important depends on additional factors.

made.

IS. Lung function in heavier asthmatic children


Answers:

cD

If heavier children have lower hmg functions then as weight lnc~ease~ lung
function will tend to decrease, 'so thecorrelation.bet\veen these two variables ,
will be negative. Tnis is an observational study and measurements are Only
,
made once on each child. 'Hence, whi.ist there is relationship between'the. - ,,
variables; there is no evidence that.if individual children change their Weight
that this\'llill result in a change of lung function. The relationship observed

158'

STATISTICS.
bet\veen weight and ll!ng function may be due,to some oth~rfactorthai may
be caus~lly related to IJ.!rigfunctiof1 1and whkh acts as..a confo!.!,t:~det For:
example, if the children from lower social classes tend to.be.heavier/an!:lilow
social class'adversely ii1rluences lung function (maybe due"to a genetic . .
component or behaVioural faCtors) then indeed social class will be a.
confounder in the comparison: To study the relationshipbetween weigh rand .
lung function among asthmatic children, a 'Selection of asthmatic cnildren' .
with differing weights ':!!lly1is ,required and ;a filrther. comP,aris2n.groupjs'not
necessary.
. ,, .
: ,,
, ; ,,
.\

~ ~ ...: ~'

. ..

r . :...,

, V'

16 .. Preva~~nc:e,-d~pendent statistics .
.; Answers: D , ,. '

... . _

. ,;

,. '

, ,,

. , . ,

::

The pre alence bf a EIase is the proportion of the p<:>ptilation wi ttl the' ' :_~,A
disease.'Tiiesertsitfvi.ty of a screening test is the proportiOn of those. with:!. 3 1:
diseaSe who screen positive. Therefore, because it is based solely on those

wtth clisease, this~oes qot vary if the prevalence changes~The speeffitttiis !\ r


tti'e proportion ofihose\inl:hout the disease who screen negative. :r.~erefore,
becatlse iUs based:solelyon those without disease. it does notvaryifthe '",
prevalence changes. The likelihood ratio.canbe calculated from,the sensitivity :
and specificity and .because tt)~se are notprevalence dependent, the. . -~.' ''
likelihobd ratio witl not be. either. :rhe positive predi~tive value .is the<
. , .. ,
proportion ofthostfwho screefi positive who actuall,ihave the disease; ln a , ~ ;
popula!jon with low prevalenc~. most willjnot have disease anUor a givenr , ... ;
specificity the proportion q(false positives in the population will oe quiteJligh,:
and thiscwill make .~he positiv~predictive value teTid.~o be lower. (less ot;th"ose.'.
screenifig positive Clo have disease}. Conversely,. if the prevalence is high tlien .,
the proportion of false positiv&> will be relatively low7(the riumtih of non-
diseasep people ar~"lower; so ,the number}Jf misdiagt;osed people are also;
lower)kand
the pqsltive predi<::tive
value \'(ill tend to . higher. ' . . . . '
..
"'
1

'

--

"

,_,.~

'.

oe

"

,:\:

'

t . ,

17 .. scr~en. fo~.Ji:sk ,or-DoWn. syndr~me _


Answers:B D"

.:-,

'

We need to construct a2 x 2 table of screen test results against true


diagnosis. This is the information as given.
Down syndrome. ,

PPLin upper decile .. ,

.:.Yes

Yes

: 25 .

No

30

.Totat.

No

Total
....

70

500

And the restcan:be conipletedfrom"thisinformation. Note: check that,aWthe


marginal totals agi:ee.before proceeding (eg 70 + 430 = 500; 55+445 = 500).

159

ES~ENTIAL

QUESTIONS FOR MRC?CH

...

. ,..

PPL in upper dedle

~ -'

Yes

..

-~

'\

.. Down syndrome

Total. -. ~

Yes.
'

...

.,

)"

,.

45

'70 '

'

,.30 ,. t

No
...

Total

ss

t.

'

445.

"'

... '

... 500

' .,

The specificity is the proportion of those without diseage Who screen' . '
. negatively, thus 400/445 (A is false). The positive predictive value is the"'
proportion of those who screen positive !Vho actually have disease (25/70).
The sensitivity of the test is the proportion of those with DoV..rn syndrome who
test positiVe (25/55). Of the 70 fetuses with measurements in the'upper decile,
most (45) do not have Down syndrome.The s.tudy gives .no information about
the relationship between age ard DO~'ll syndrome.
., ,

' ,
'.;.

'

I .,,..

'

18. Treatment
for, blood.
pressure
~
li'.\
.....
;
.
-

'f

Y,

\.

'

;t;l::l

+ :

'

'~

.;1~

~-

, / ' .,

"t _;

;:"'1~-.-

't., ;

""':c l~

!~"1"

'~

'

{''

,.AnStver: . , ...
.
;. '' r
..to..;,, ......
Th~ difference observed is statistically significant'and would.have:cict:Urred by~';
chance Himes in 100 (since
o:021 or I timeiriSO if there really were no

p.=

treatment effect. We cannot tell from the information:given whether or not<' .


randomisation was successfuL The sample is Compatible with average fall in
blood pressure of betWeen 4.16 and 7~84.mmHg. We are 95% confident'that the
population average fa:lllieswithili this interval. however it may not.(in fad; 5%
of the time it will not). Statistical significance does not necessarily imply " t
dinical significance or relevance. The 1-test is not valid if the measurementS J:;. .. "
are not normally distributed. Skew data are not normally distributed; ' '' I . ' '"
'

.t

' \

t ..

':

19. Genetic defectin.asthmatic ant! healthy children-...:, ,;


AnSlVers:C ~;

! ___ :.. Jf'"!~"t

,:

.i

.J:..-,'-'-... lt 1 :.;~;.t'

l
,,...
1

".,.f

..

~ "".~ -t

, .. ,,
""

This is an observational study and cannot be randomised. We cannot al1ocate


asthma randomly. For a variable to be a'confounderit must diiTef'bftween the!
groups being compared and it must also affect outcome. Since the asthmatic
and healthy children are of similar ages, age cannot be a confounder in Lh~,
comparison. Presenting the difference in proportions positive in the two .'
,
groups, together with a confidenceinte.rvalto show the precision with whiCh .
this difference is estimated, would.be informative. !:Tes_ts.areappropriate for
continuous numeric outcomes. Chi-square can be used to compare

differences in proportions.

-
-
t- .,. ..
~~
.
-~

.io.. Power of a stUdy


AnS\\'ers: B C DE

--

The power o(astudyis the probability {usually expressed as apercehtage}'of.


correctly rejecting the null hypothesis when it is false.'As it is a probabilit}lit
must lie bet\veen 0 and I (or 0 and 100% when expressed as percentage). The

. l

sTATisTics
greater the difference to be detected, the greater the chance that the study will
find it; hence the power is larger for bigger differences. The larger the sample
size. the greater the power to detect a difference of a given size. Power is
usually calculated at the commencement of a study. Sample size is often
based on achieving a given power to detect differences of clinically important
.magnitude. Sample size estimation is based on unknown quantities and the
estimations of power may be made once those quantities have been
determined from the ~ample data: Hence power may be calculated
retrospectively.

21. Adverse events in randomised controlled trial


Answer: All true

.. _;,,

The per :::eritage suffering from an adverse event. fell from 40% to 20%, hence .
the reduction was 20%. With a 20% fall this means that an extra five
individuals would need to receive intensified care for one to avoid a.n adverse
event. Hence the number needed to treat (NNT) is 5. A confidence interval
around the percentage reduction (20%) will take into account the sample size .
-the greater the sample size the more precise the estimate ofthe difference
attributable to intensified care and the narrower the confidence interval. The
relative risl<~is given by: the risk in tne. !ntenslfied ~regroup divided by the
risk in the standard care group (20 ~ 40 = 0.5)~ If the groups (standard and
intensified care) differ in their age distribution ahd age affects outcome.
(adverse event yestno) then age would. be a confounder in the comparison.

22. B: Consider extent of depletion, clinical implications, costs of


supplementation and make a decision based on these
There is a statistically significant difference as shovim by the p value. We are
not told who the renal failure children are compared with to get that value.
Was this concurrent healthy control's or an established reference range? How
we interpret the results will depend on who the comparison was made with ..
The differenceseeri is statistically significant but thismay or may not be
associated with a clinically important difference- although the fact that the
question sta,tes that values are severely' depleted suggests a clinically
relevant reduction has occurred. There may or may not be other factors, such
as age, that need to be taken into account when interpreting the results. The
studywas presumablyundertal<en to answer some research question, the
answer to which would inform clinical practice. Therefore we do not expect to
do nothing after obtaining the trial results. On the other hand we do not want
to introduce (or even trial) supplementation without first considering the
.
clinical relevance and implications for the reduction found. The p value shows
thaUhe study is large enougn that the observed difference cannot be
attributed to chance: Hence. the rriost"correct answer is B.

161

:I

,ESSENTIA:. QUESTIONS ;:c::, MRCPCH

, 2.~ Dt ca~ry out ~ trial of a larger size to obtai~ a, more precise


~;,);; , ' , estimate ofthe effect ofthe ne"rdie~ compared to stan(lard
(ll"~'

~~:~t[j
W

">,

c;n

fa

.
.:

The average Improvel'l)ent seen is clinically relevant so we would not just want
to discount the information because it is statistically non-significant. The
confidence intenal for the difference is wide and shows that the data are
compatible with the nev, dietary regimen having no effect, or an adverse effect
on height .. and also with clinically relevant improvements (up to ~ .2 sd
'
scores). Because the diet could be associated with a detrimental or zero effect
. based,en the study results. it would not be reasonable to introduce it as '
standard pufely because the average effect iS gocd. The children could be
followed for longer to see whether. the effettbecomes larger and statisticall)
significant but this would not answer the question of whether an
.,

improvement can pe see'! o;er. 2 years. Sd scores are us~;~ ally normally
distributed. so it .is unlikely...: although 1;10t impossible- tha,t non-param~tric,
methods \vould be neerted. The normality of the scores should have been . "
verified prior to pa'rameti'ic testing.~A 'larger trlal would enable a more precise
estimate of the effect of the new diet over'a2-year period to be obtained and
this would be the bes'tcourse of action '(o is the 'most correct ans\~c:;r). , . .
"-

.',

'

'

24. E: Non-parametric analysis of variance comparing medians


.
betWeen gioups (KruskaJ..;.Wallis) .
_. ' .
.
1

. J

The gro~ps are small. so it is likei/t~at parametric meth.~ds ~re not .


appropriate. Before embarking on a formal analysis of the differences, the
haemoglobin measurements should be plotted according to syndrome group.
This plot will allow some assessment of the normality of the measurements.
Testing IX!t\veen pairs of g~oups will enable significant differences to be .
identified but it does rely on multiple tests and the p values obtained will not
be valid \\rithout adjustmenL It would be preferable to perform one overall.: .
test of the significance of the diff~?rences observed be,tween groups. It may be
that the study is not large enough' to identify differe~ces of clinic;~! importance,
a11g~ INger sample is required. This will become-apparent-from the plot,
.. ... significance test and.confidence inter;rals.
, ,. . r
4

25. B: They help to ensure. that


any differences
seen. are due_
to' th~ l' '~
..
'
. ;
' (
-treatment or 'disease .being stpdied .
''. .
If there is no control group then it will ~ot~bepossibie t~ s~y~hether any .
effects/outcomes seen in t}1e diseased or treated group are due:; to the disease
or tr~atment. Hence a control group is necessary. If a historical control group . _,
(a group previously measured!asse~sed) is used then we_ !=<:tnnot. be sure that' .I
any difference is not due to factors that have changed over time,(for example;
improvememt in diet or clinical care)~ Concurrent controls areJherefore
preferable. Using concurrent con~r91sC{.vm r<;rryove some of the P.otential ..
confounders. We want the controls to be similar to the treatmenVdisease
group so that any differences observed are more likely to be causally
162

__ l

.
STATISTICS,
----------------~--------------~~----attributed to the treatment or disease. If the groups ~re,blin(l to treatment .
then treatment knowledge does not differ bet\veen groups apd se this is a
similarity that we want to have (Where ethically and feasibly possiOie).'

26.

D: It allows the CD4 c~unts of iiu:lividual children to be


compared to what is e":pect~d for normal children of that age

A reference range aims to give information on the values of CD4 found amo~g
normal non-diseased children. It consists of a series of centile values. Since
. CD4 count changes throughoutchildhood the range should be age-related. In
order to construct precise .ranges, quite large groups ofchildren will be
needed as we are often interested in extreme centiles (the 5th and t>eyond).
Often the 5th centile is used as a cut-offto define abnormality or cause for .,_ .
further investigation. By definition. 5% of normal healthy.~ndividuals wili lie ;
on or below the 5th centile. Hence the specificity of a test which uses the 5th ~
centile as a cut-offwillbe 95%: thus, 9.5% of those without disease (healthy,,
normal. children like those on whom the reference was based)
have values
above the Sth centile and will test ;negative'. The sensitivity of a reference' :
range to detect disease will vary according to the disease. The purpose of the
reference range is to allow the CD4 counts of individual children to be

compared to what is expected for normal children of that age. If the aim was
to compare a group of children With disease to a group of non-diseased
children then these non-diseased'children shOuld be coneurrently measured.
Referenc~ ranges are,pot recom~ended as a substitute for control groups in
trials, rather as assessment tools for individual children, lhe most correct
answer is therefore D.

will

2 7. B: A'regression analysis should be used and the regression


coefficient presented with confidenc;e interval:Firstly the data should be plotted in a scatter;plot of IQ against height. The
... ~~!!_elation C()C:~ficient could be calculated'toestimate significance of any
observed association. The ielatiorfsftipcould be fury her quantified br
regressing IQ scores on heigh't:This will give a measure of the extent to which
IQ changes with height and is much more informative than the correlation
coefficient-alone, The extent to which the two are related is given by the
,
regression coefficient. Presenting the regression coefficient with a confidence
interval shows the range of population scenarios the current sample is
compatible with.' If the confidence interval is wide then we may decide that a
larger sample needs to be taken to obtain a more precise estimate of the
relationship; The statistical significance of the observed difference is given by
the p value. Since the sample are all 7 years old it is unlikely that expressing
their: heights as:sd scores \\ill make any difference to the results. If the
chilaten were over a wider. age range and we wanted to remove age ~sa
potential confounder in the comparison of.JQ and height, then expressing
heights as sd scores would do this. " .

161

28. SIGNIFICANCE TESTS.

I l

1. B..;; Pciir~d r:test


'
There are tWo groups of children (those with pituitary hormone disorders and
their age-sex matched control pairs). Hence a two-sample test for comparison
between groups is appropriate (two sample Hest, paired t.:.tesi,'0 ::: - J~ Mann-Whitney u-iest.or chi-square>. Outcomes are continuous nuinenc
(blood pressure) and it iS the' Within-pair difference that will be analysed ; I
(l?lood pressure 'for child v.'ith disorder, minus blood pressure for:age-and
sex-matched pair).' Hence the testmust be appropriate for continuous. "
outcome data (that is, not Chi~square). Since it is \\'ithin-pair differences.that
are to 6e 'analysed. these are likely to be normally distributed. The appropriate
testto use' is the paired't~test:

'
,,,
, .\ ,;
2. H.:..chi-square : '''
.-.
. . ; .
There are two groups ofchildren (those admitted to intensive care and those
who are not) ..Hence a two-sample test for comparison'between groups is
appropria'te (two sample Hest. paired t-test; Mann-Whitney Utest. or. chi-: ,
squar<;:): outcome' is binary.that is categoric, With-two categories. . " -.
(developmentally delayed: yes/no). :rhe proportion with developmental delay 1
is to be compared between those admitted to intensive care or nothThe. ,
appropriatetest forc?rripanng proportions between two groups is chi-square.
3. D-One-way analysis of variance.
_, .
,
.,
There are four groups of children to be compared (from different racial ,
backgrounds). Hence a test for simultaneous comparison between more than
two groups is appropriate (one-way analysis ofvariance,K:rUskai~WaHis
analysis of variance, or Chi-square). The outcome (blood pressure)'is. ;
continuous (hence chi-square is not appropriate) and normally distributed
and hence parametric 'testing should be used (eg one-way analysis of ' l
variance).
' . '-'
r '~
1 .
<'
'("

, I

'

'

'

,.

29. INTERPRETING TRIAL RESULTS


.:
For all sectionsKeahncirl:5e-t:orrecCiflnenewcream is found-io be bothr
statistically and clinically significant then this means that the difference
observed is unlikely to be dueto chance and is also large enough to make it
clinically relevant. The study shows the average difference attributable to the
new cream must be large enough to be of clinical importance after taking into
account all other factors (cost. ease of use, prob-lem-associated with,_" ""' 1;
introducing new treatment). Hence in this scenario. the new cream shoulq.be
introdu-ce'd.
, .~ _,,rr. I:'' c- A larger study is required to determine whether it is worth . '
intrOducing the neY.(cream

:1t
,.:t ~, ~-. ',
t.
The differerke is statistically significant since the p value(< 0.0005}is smalL.
The average fall of 2.4 points on the severity scale is larger than deemed sufficient to be of clinical importance. However, the confidence interval shows
that the data are compatible with a difference of between 1.2 and 3.6 points.
l

164

-. STATISTICS

An average fallofl .2 .would not be deemed clinically important enoughto


warrant changing to the neo.v cream. So the data are.compatible with
outcomes that are synonymous with differing courses of action. For a fall of
1.2 to less than 2 points on average, the cream would not be introduced,
whereas an average fall of between 2 and 3~6 would lead to introduction of
the cream. A larger study needs to be done to reducethe width of the
confidence interval and gain a more precise estimate of th~ yalue of the new
cream.
2. c - A larger study is required to determine whether it is worth
introducing the new cream

The difference is statistically non-significant since the p value (0.23) is not


small. The confidence inter\ral shows that the sample data are comp<!tible
With an average change of anywhere. between a 6 point drop in favour of the
new cream and. it making the rash 1.2 points worse on average. Hence we
cannot discount scenarios that would lead to introduction of the new tream
(ie. afall of between 6 and 2 on average on the severity scale). Neither can we
discount. the faCt that the new cream does not have a clinically important
etfect.(difference may t>eless than 2 points and thep value and confidence
interval both show data are compatible with no difference between creams).
Hence a larger study needs to be done to distinguish. between differences of
clinical relevance and not . '\
i._:;;:
. .. ' . .
'
. ,
.
\
3. . I - Th~results canl'lot be interpreted because the an~lysis used
was inappropriate ,
, ..
The differenc~, co~fidence~ntervals and,p value are t,he same as in (l) .. The _
observed difference i~ sta,tist~cally significant but may or may not be of a
clinically relevant magnitude. However, the children allocated to the new.
cream, but not using it, have been combined with those in the other allocation
group. Hence tile groups are no longer randomly sel~cted. Those:that
.
changed treatment from their allocation may differ in someway that biases
the results. Ttie data shOUld have beeri.ailalysed on an intention-to~treatbasis
(that is. outcomes compared according to allocated group, rather than
according to the treatment actually used).This flaw-in-the analysis makes it
impossible to interpret the results as we cannot assess the extent of any bias .

. 30. DIAGNOSTIC TESTING

The best way to proceed with a question such as this is to construct the 2 x 2
table of screen result against outcome. The data given is as follows:
Miscarriage
Nuchal fold abnormal

Yes

Total
No

Yes

100

No
Total

80

400

165

; ...... ,, .
. .. :.,
..
~ .

ESSENTIAL QUESTIONS FOR MRCPCH

?' .--

1. I - Percentage correctly classified


Equal -numbers of those that miscarry have 'abnormai or normal. mkhal fold
thicknesses. 1So, the information giveri is:
' ' . i,

~<

Nuchal fold abnormal

, ,,,Yes

Yes

., 40

t ,j

No.

b ... ~

'!.

..

Miscarriage
No

. ..

'f . . .

"1

"

1-.,,

400

80

'"'+

"'".5
J

Total

J . - .. Total

,~/

)._J

"

,(

And the rest of the table can be completed, thus:

..
--

Nuchal fol~ abnormal


Yes
No
Total

Yes

'

40

...

'

'

..

.,

60
260.

so.

Total

"

i_,.

No

-<

40,,

..

..

Miscarriage

,.

320

;, ...

..

300

400

100

..,

There are 40 \vho screen positlve (abnormal nucl:l~iJold'thickness) a~d. ~


subsequently miscimy. A further 260 screen negatiYe and do ttot miscarry. ,
This gives a total ofJ00,(40 + 260) out of the 400 [or 75%) Wfro are corr_ectly.
diagnosed using' the' test. '' . "'
.
. .. ..
1 ,
, ' .I
Sensitivity is 40/80 = 50% r ' : .:~
.. "'
Specificity is 260/320 = 81%
Positive.predictive value is 4011 oo = 40%Negative predictive value is 260/300 = 87%
.
False positive rate is 60/1 oo = 60%
., ;. .i
. False negative rate is 40/300 13%
Positive likelihood ratio is 50/(1 00""' specificity) = 2.63
Negative likelihood ratio is (100- 50)/specificity 0.62
Percentage correctly classified is (40 + 260)/400 = 75%
...
The prevale~c.e of mis~arriage is 80/400 = ~99(,.
1..: ..

1 "''

'

'

.. '_...
i

;'

...

'

'

' \

'""!"_- ......... -

f ".. , t} .. ~i,f

. <t,

.i~u~~

;,. l;

~- . ... .
b

l'

.... '

.r , ;\
'!"

~-:

Ct \ ,.- 1
-t

f. '
'

166

.'~

't

STATISTICS

2.

C - Positive predictive value

Of those who subsequently miscarry, 75 have abnormal nuchal fold thickness.


So, the information given is:
'

Total

Miscarriage
Nuchal fold abnormal

Yes

Yes

75
;P,-

No

Total

No

100

'
. 400 .

80

..

And the rest of the table can be.. completed, thus:


!
'>
'
Miscarriage
Nuchal fold abnormal

Yes

Yes

75'
5

No
Total

80

Total
~

No

25 ...
295
' 320

Of the lOOwomen-with abnormal nuchal foldvalues. 75 (or 75%)


subsequently miscarry. Hence th~ positive predictive value is 75%.
Sensiiiyity is 75/80 =:: 94%
Specificity is 295/320 = 92%
Positive predictive value is 75/1 oo:= 75%
Negative predictive value is 295/300 = 98%
False pOsitive rate is.25!1 00 = 25%
False negative rate is 5/300 = 18%
Positive likelihood ratio is sensitivity/(! 00- specificity) ::::: I 1.75
Negative likelihood ratio = (1 00- sensitivity)/specificity = o:o~5
Percentage correctly classified~~ (75 + 295)/400 = 93~
The prevalenct: of miscarriage is 80/400 = 20%.

'

lOO
300
. 400

ESSENTiAL
3.

QUESTIONS FOR MRCPCH

A- Sensitivity

. Of those with abnormal nu~hal foJcfthidcnesses: ~o ~l;!!>sequently miscany.


S<?, the information given is:
..
'
. ..
Miscarriage
Total

Nuchal fold abnormal

Yes

60

Yes
No

Total

..

. . 80

..

.
.

No

-..
.

..

'

'

100

' '

400

,.

And the rest of the .table can be completed, thus:

'
Nuchal fold abnormai

'
\

Yes

Total

No

Miscarriage

Yes

..

60

. Total ,,
No
t

40

20

280

80

320

..

tOO

300

..

..

400

Of the 80 who miscarry, 60 (or 75%} had an abnormal nuchai fold


measurement This is the sensitivity of the test.
. . .
. .
t;..

Sensitivity is 60/80 =75%


, \.
SpecificitY. is 280/320 = 88%
.
Positive predictive value is 60/100 =60%
Negative predictive value is 280/300 = 93% , , .,,,
. False positive rate is 40/1 00 = 40%

, , ;:
False negative rate is 20/300 = 7% .
,;
,..
Positive likelihood ratio is 75/( I 00- specifiCitY} = 6.25
Negative likelihood ratio is (I 00- l5}/specificity = g28 .
Percentage correctly classified is (60 + 280}/400 = 859{> .
The prevalence of miscarriage is 80/400 20%

I
I
I

!
\

,-

-~

'

'

..-,

This Index covers Volume I and Volume IL .The volumes are i!ldi,cated
roman numerals, 1 and
,/~ I

n:

..

,['.

:: ,

-abdominal. .' 'of" ,.


b ' ';;.
acute myeloid leukaemia (AMLl t: 150, 15~
,
mass 1:97,tl iS.' 154 ' S
acute post-streptococcal glomerulonephritis.
pain J:l 110, 112. 115. lB. I (1.1:7)
e.
(1I:57, 65. 69f 77 . .78] ~-. _ .,. .
,:'.___ ,'(
wall det.::ct5 1:104; ~
"'
acylcarnitine plasma level(u:34l'
abducens nerve t:49(n:94. 104,1 fl j'
Addison's disease t88(t!:59, 72)
.
adenoidal.voice (td28, 134, 136, 1401.
abetalipoproteinaemia 1: l25(U:l 16. I 83:106)
ABO blood groups. 149,156, 158 [:::1 48,-42,
adenoids (u:l28)
'
.,,
45, 52, 55]
.
>,)
adenosine (intravenous)..I: 10. 20, 21 .
ABO incompatibility ]n:42] -.~ ... '
adenotonsillar hypertrophy (n:i28].~ , , ,
abscess formation in orbital cellulftis(n!I08}
aden0\1rus infection [II: I, 4, 1 I, 48, .1 03, IIO)n
absence seizures [u:87, 96]' . : '{,
adoption 1:95. 101
' , .,,
.,
1
absent septum pe!lucidum 1:81 ,i "".. =.- . ,
adrenal gland 1:74,86,90
.
,...
, ,a
' 1 :<
hyperplasial:73.75,83,85:87
.t
acanthocytest:l25! ' '
acetazolamide (n:94)
'
hypoplasia 1:83

,
acetoacetate t:76
' '
. , adrenaline (epinephrine) (u:2,.12, 114,.1 17,
1
acetylcholine(i1:93;9sj. . .n~, '' '
.
125; 128)
,
: ~ ...
Achillestendon(n:l38l''t,
,. .f
'adrenalzoriagloinerulo5al:7{86 ._,
achondroplasia li73, 8<1;"128, 138;i 39, 144
adrenergic receptors (11:114, 125).,
, ' i
aciclovir therapy !:67, 178, 180 (u:J 91 yr '
adrenocorticotrophic hormone (AciHl 1: n.
acidaemia (n:21 23, 28, 29, 32, 123) :,.:. ~
174, 81;83-84, 86, &a,'S9, 90 (11:96)
acid-fast bacilli in tuberculosis {ii:l i9r'') <>'
adrenoleukodystrophy (n:28, 32]
.
aciduria 1:99 [II: 8, 19, 29, 30,:34, 122: 125) ' adverse events in clinical studies (u: 1_49, 161]
acitretin !:66
,! .
:' ~"'
i '' ... '
afebrile seizures (11:24, 93)
" .
acne t55, 65

:"'" 1 '' ' '


age of majority 1:95, ]01. 102
": .
1
acoustic neuroma 1:138 H
v: }<>' ..
aggressive behavioun2ti. .
.
,
acquired epileptic aphasia 1:96-:97 "i
airway'6bstructioi1Jn:ll( '1 18, 123,;1'2&.
acquired hypothyroidism1~88 -::
'"
. + . ----12'9; 1.30)
, : l'~ , ..
1
acrocentric chrombsomes 1:1 39; 145 : :
alanine aminotransferase 111:8, 9. 68)
acrodertnatitis enteropathica 'i:53, 62, 6s '" albinism (occulocutaneous) 1:61
'
acromegaly 1:82
' i '
albumin
,
acropustulosis (infantile) 1:60. 70'
blood !evei(J!:S 1. 6~. 68, 69, 78)
,, ,
activated charcoal:441 s!
drug binding' M3. so, ? r'
h . .
activated partial thrombOplastin tiine tAP'fn'"
alcohol consumption 1: II, 24, 31 (11:39.' 49] .
1:149. 158, 162-163(n:-8; 9;3s. 43~48;
aldosterone 1:78,86, 89.90 (11:72; 77) ,
'! ~ t'

alkaline phosphatfse (u:q6)


:'~, . ;
.53, 1531 , , , .
acute disseminated encephalomyelitis
allergic bronchopulmonary aspergillosis
... '
(A0.~1) (n:82 90;91] ,J:.
'" V
(ABPA) (H:126. 131]
..
. . . ...
acute intermittent porphyria (11:28( ' "''
allogeneic bOne-marrow transplantation
acute lymphoblastic
' ~ ~
. ,.,
1: J 4 7
. , .,
, 1
leukaemia 1:ISO, 159(n:l38, 1441
' allopurinol therapy 1:160

lympboma 1:154, 164 (n:J 43) '''' t. ' . :;.alopecia 1:53, 551 61, 64 [it93) "~ , ,
acute lymphocytic leukaemia (11:20;
>f < a-fetoprotein IAFP) 1:95, 103, 150, 160

ll.;

\'

169

~SSENTIAL

QUESTIONS Fc;::>R MRCPCH

Alpon syndrome'[n:74, 77)


aluminium,toxlcity 1:48
alveolar
capillary membrane (n:ll3, 122-123)
gas exchange [II: 126)
surface tension [11: 124. J26)
ambiguous genitalia 1:77, 86. 8.9. 129,
14()-141
"
amenorrhoe'a'1:2s. 27,33
amiloride therapy 1:74
amino acids [11:25, 30, 33. 34)
aminoglycoside antibiotics 1:48 (r:: 12 7)
amiodarone 1:20, 21 [11:105, 1121
ammonia [n:22. 24, 2S, 27, 30. 32. 33. 60, 13;
93)
amniocentesis 1: 143 .
l
amnion in nvin' pregnancy [u:SI 1
amniotic nuid [11:45, 46)

amniotocele [u: I I I)

.
.
amoxicillin therapy [11:45. 47. 55)
Amoxill:l7S [n:7)
'
amphetamines 1: 11
.
amphoteriein't:l61 (11:711
ampicillin 1:.1 78
.
amylase levels 1:33
,,.
amyloidosis (11:83)
amylppectin [u:32) ' ;
,. t
anaem1a 1:33, 164. l6S,(11:l8, 71. 76, .130,
136)
.,

Angelman syndrome (happy puppet


syndrome)r:28, 3S
angioma [11:83]
angiomyolipoma (11:7'1, 81, 90]
angiotensin [n: 72, 77)'
angiotensin-convening enzyme (ACE) [II: 721
inhibitors [II: 70, 75)
angiotensinogen.[n:72)
anhydramnios [u:41 1
anion gap [n:25, 33. 70)
aniridia 1:164 {II: 106J
ankylosing spondylitis 1: 120
anomolous pulmonary venous connection
1:2, 3. 13, 14 (II: 116]
anorectal malformations 1:9.5. 102-103
anorexia 1:2s. 27, 32-33, 35. 84 [n: 125)
anosmia 1:79, 86. 90
,
. ,.
antepartum haemorrhage [n:39, 49)
, il
anterior uveitis (u: 106) . ,
anthryKYcline cardiotoxicily 1: !50
anthrax [II: 14)
"
antibiotic
resistance t64 (u:47) .,
!,.
side effect!i 1: ltl3
therapy see specific agents
antidpation cgeneti~} 1:137, 139 'r'
anticoagulant therapy 1:12. IS, 178
antlconvulsants [n:48, 142]
antidiabetic agents [n:29)
anti(jiuretic hormone 1:74, 86 [n:SO, 7S]
aplastic i:4s. 147: ISS~ t56
,
141)

antf:'oNA antibodies
autoimmune haemolj1ic 1: 148, 156
feta'la: 139

""
'
anti-endomysia! antibody 1:117
fetomaternal bleeding [n:53)
anti-epileptic d[Ugs [u:83, 87. 93) see also
haemolytic 1:148. 162, 167, 174(11:31, 141)
specific drugs
hereditary haemolytic 1~147, ISS [n:27. 3S)
antifungal therapy [11:126, 131}
antiglobulin test 1: IS6
hypochromic mkrcicytic 1:124, 147; ISS,
'161 [i1:63)
.
.
'
antihelminthic drugs [n: II OJ
antihistamines [11:109, 12S)
. iron deficiency 1:114, l24.a'47, ISS, 161,
162 (11:28, 40. 60, 74) . .
anti-lgA antibodies 1:174,
microcytic hypochromic 1:147, ISS, 161
,, anti-lgE therapy [1i: 1281
neonatal [11:42)

anti- La antibodies 1:1 1. IS .


antimicrobial peptides 1:169. 177., ..
pernicious): I 17
severe fetal1: 139
antineutrophilic cytoplasm antibody tANCA)
(11:13S 141]
sickle cell!: 179111:39, 49, 82, 92)
sideroblastic t: 161
. antinuclearanti~ody.(ANA} [n:l34, 135:138.
139. 140, 141, 142, 143) . .
. .
anaesthetic {before cardioversion) 1:20
anal atresia (VACfERL association) [u:S4) .
antiphospholipid syndrome (materna)} {11:SOJ
analgesics [11:94)
.'
anliplatelet antibodies 1: IS7
analysis cif variance (II: ISO. IS2, 162, 164]
antiretroviral therapy, [11:3, 6. 15, 17. 491
anaphylaxis 1:124. 174 [u:l28; 142)
anti-Roantibodies 1:11, IS

Andersen syndrome (11:92) .


anti-RSV antibody (paliviZU!Ylilb} [n: 127)
androgenisation 1:88
anti-Sm antibodies [II: 141) - .
androgen
antispasmodics [11:2)
production 1:74,81, 85, 86,90
antistrepsolysin o test [n: 142]
sensitivitv 1:99. 129, l4Q-141 (1!:47)
antithrombin I (n:69J.
androstene'dione production 1:86
antithyroid therapy 1:82-83

170

II
I
\
I

I
I

Ill:

\
\

I
I

.. ,

:j""'

,..,
-<--

:~

_, _ _ _.,. .... ,

.?.~

' . ~.
.._#

~~:'~~-";r-"\.

_ .......
..

~J

<t

. : ..

,.~,.,:'"' ~

_..

~~

1'

.....

INDEX

1..

at-antitrypsin deficiency 1:i 19, 124(n:48]


astrocytoma 1:159 [n:89],
' ' ''
aonic.~
d' ' . +.... -..
" 1' 1
'atax'ial:"i8,3s,:Iso.I54,160;I64[n:83,93]
arch. interrupted 1:14;20'
I' 11
atopy,,;I62 [n:IOI)~-~ _,
.f;'t .-,_,
dissection 1:85 ,:!! ' "' 1 - .. , ' ,,,,:
atrial'''' .
.. ,_ : .J''' ;
rootdilatation[n:29)'
.. , ....
hypertrophy1:12
' ,.,. " ,:~-' 1 ,stenosi~ I: II< 14,\1'9: 20, 1!9_-14.1 -~-.'',if
isomerism 1:3. 13,.14
;, : ' .i ,! 1 ,., ...
'
septal defect 1:9, 19,20 ,;_,.,A
val\e (biCuspid) 1:4. 16
aphasia 1:28 [n:84, 96-97)
' '" :~, 'c.n
atrioventricular septal defect (ASI:l) 1:1; 2;'11,
aphthousmou,hulcersl:411ct
12. 19,132,143 ,_,
<~ .-i
-
1
apnoea [11:45.48 .. 67: 84, 93, 96; 117. 127) _
atientiondelici~ ~; . i;.
"
~
apoptosisJ:I69. 177
.- '
disordersi:I4I, 175
.;.c
appendicitis 1:94. 99-100, 105. I 06
,.,,
hyperactivity disorder (ADHD) 1:24:31 ;34
appetite 1:85, 90 (II: I24F;" ''
' 4 - ' 11
atypical tuberculosis,(n:4) ; ~" . ' ..j._j, !
apraxia (11:84]
,, ..
, ' "_.,.,
~ura (heac!aches) [11:95). ,,, ~.-. : ''
Apt'stest(n:53)
--
..;.,!;;1
auscultation1:ll
.r -,~-.- ,,,,1u
arachnooctyly 1:8S
. .,,
: ""11
autistic spectrum.disorden25, 26, 32, 34, "'
,,arginineGcliciencyl<72 [11:30) '' .,. "~''
36. 175 [11:81, 91] t. .<. "' _, '"'
arrhythmia 1:20. 21. 32, 83[11:87) ' 1
autoimmune ha~molytic anaemia (AJHA)"
arterial
;.-.
., __ ,._,,,.n
1:148,148.156, !''-~..;:
.; ;~
aneurysm 1:15 (11:141: '142] "
''
autologous bone-marrow transplantation
circulation 1:167
': \ u, ' '""
1:156
..J!S !.
duct 1:1. 2. 1 I, 12
, \<,
.-,,_ ~
autosomal dominant inheritance 1:128 [11:60,
S\\;tch operation 1:2.~12
:,? ''" .-"- .,
82]
. '
' .J- .
.
thrombosis 1:14
-' '--''
' n:
autosomal recessive inheritance [11:37, 59;'.
aJ1eriovenous malformaiion [11:92) ; '"'"'' y
.82, )00)
- :

artery occlusion [11:84, 94)
avascul<!r necrosis [11: 144] '
,.t .
arthralgia [11:19. 135, '142]' <
.. ~,.,!'
azathioprine 1:12(?.
. -'' "' 'f'. ,
arthritisJ:l29, !40, 175 [11:76)
:i'l "'
azoospermia ~:84. 85 [11:115) . :t-,:-. "''
enthesitis-related [11:138,143, 144] 1 '
AZr(zidovudine) therapy1:111 [11:22,'31)'-'.; '
flitting [11: 141)
-~
endocarditis [11: 123]
-~.
.,_
non-erosive 1: II 1. 120 [II: 140]
r
osteomyelitis 1:9. 20
OSteoarthritiS1:123
overgrowth-1:109,118 _,,,
. ~t~ ' ' ..
psoriatic 1:67 [11:99. 134, 136, 140, 1143] '
sepsis [11:48]
reactive [11:144] ._. .: -' ,

"
bacteruria ~:99, ....
recurrent[ll:l9) ' ,I ' .
\~
balancing skill1:25, 34
. :: ,-. ~
rheumatoid (11:99)
balanitis xerotica obliterans (BXO) 1:'1 05
seronegative 1:129. 140
-~"
ballooh dilatation 1:4,J 6
arthritis
band cells 1;148. 157
< ~ l ' "'
juv(!riileid!op_athi~_Ul,AU:9;:!9.:20_. ' '
_ bgr!u.mrgdiologyl:l20,123.... > _ . :: :_,,,,;
Ili:I39-I40)' ''- '

Bartonellahenselaeinfection~:168,175[11:4]
oligoarticular [11:133; 139.;,140, i44]. '' ,~o
Bartter syndrome (11:59, 66, 73, 77, 78) .
polyarticular [n:I33, 139, 142)' .,
'" .,_ . basal ganglion lesions [11:97]
' : -~"
"o''~-~'l~syst~miconset(II:.J33,136.139, 143] : baseexcess[u:8,24,68]
,:,; ~
arthropathy, psoriatic 1:54, 63
~ ., , , baseline peiinjetry;[11:112]
' . .. ,.
asparaginase 1: ISO. 160
Batten's disease [11:96)
.. ' , .;: , -~
aspartate-aminotransferase [n:IO] -' , ' ..-., :
B cell
r. F .. .
Aspergil/usinfection[n:I20-121,:126, ]'3Ir '
immunefunctionl:l77 . , : .6
asphyxiating thor;~cic dystrophy (11:59) :
lymphoma. 1:154:
' ''
Beau's linesl:64
_.,, .. .
.. ,
aspira&ion pneumonia 1:32
aspirin (salicylic acid) 1:15.68 [11;1.23).
'Becker muscular dyStrophy-1:127; 134; 13( .
assist~d ventilation [11:42, 45]
~~
136, 143. 144 (11:143].
.~~. ~
asthma [11:18, 119, 121. 123, 129. 130. 13l)'j
Beck\vith-Weidernannsyndrome 1:79,91,
allergic [n:l25,.126. 131]
"
'
128, 138.;I50. 154, 160,164.[n:I24).
in clinical stu'dies [11:146, 148, 149. 155, '
, beclometasone dipropiorlate therapy in: 121.
158-;159, 160]-.
.,,,,...
,,,,(,
131)
astigmatism [11:108) ,
Behc;et syndrome 1:lll, Ill. 120
!. \

"':a_ ;

...

171

f.

ESSE!;J"'[IJ. _ QUESTIONS ::oR MRCPCH

I
I

Bell's palsy t\11 cranial nerve) {:1:19, 60, 63,


76, 94. 109. Ill)
.
benign intracranial hypertension,Iu:84, 94)
benzyll;>enzoate ther!IPY 1:58; 6!!: I 78
benzylpeniCillin 1:43. 47;50, 51. 53{11:55)
betaine (n:291
betametasone [n:47J
beta-blocker toxicity t:49
beta-defensins 1:177
beta-thalassaemia {11: 107)
Betnovate therapy t:64. 65, 67 '
bias (statisticah [n: 145, 154, 155. 165]
bicarbonate blood le\el
'

infectious diseases (n:8J


""'
metabolic acidosis [11:241
renal disease (n:58. 64. 66, 67, 6B. 70, 71)
urine le\'el [;!: i' I J

bicuspid-aortic valve 1:4, 16


bilateral lens dislocation 1:131
bile acid synthesis'[il::i'3, 32)
bile duct obstruction 1:.122
bile salts 1: II 0
biliary
atresia (11:481 .
cirrhosis[:;: 113. 123)
fibrosis 1: 12 2
bilirubin 1:45. 48. 51. 118(11: 10. 42, 44, 52.
. . , 63) see also h};perbilirubinaemia

b1oUmdase deficiency (u:28J
birth depression (11'50]

birthmark,s !:54-55,!,64
bladder disorders 1:99. 103
Blalock.-Taussig shunt 1:2, 4, {1. 13. 16. 18
Blaschk.o's lines 1:60, 142

bleeding diathesis 1: ISO
bleeding disorders ~-:153. 158~ 163 (n~53J
bleeding time 1: 153.n 63
'
blinded clinical studies (II: 146. 154, 155)
blind loop syndrome (bacterial overgrowthi '
1:109, 118 (11:109) .
blistering disorders 1:59--60,-62; 69 ''; '
blood-brain barrier 1:40,45,46. 51. (li:J40)

blood
clotting disorders 1:139 (11: 18]
clotting fa.:: tors 1:149. 153, ISS, 163
coagulation U49. 153. 158, 162, 163, J 78
coagulation \'alues 1:153
coagulopathy {u:35J
culture 1:161 (11:100) \
gas analysis {u:l7, 18, ,19.--24~ 128)" ~~ d
group compatibility (n:42) 5e1: also ABO
blood groups .

in urine s:-e haematuria .


pH (n:8, 24. 64, 66. 67, 68) see alsi:J
meta:.olic Scidosis; met.a~lic
.alk.alo.sis ".,

172
; 'l

pressure (II: 1o, 62. 63, 64; 65, 66, 681 see
also hypertension
'in clinical studies [11:149, 152, 160, 164)
maternal 1:73, S3
raised (11:71, 125, 128!
sampling t:148, 156111:281
transfusion 1:148, 149,'149. 156, 158, 174
Bloom syndrome r:57, 59, 62. 68. 69
BM stick. (n: I 7J
Bock.dalek type diaphragmatic hernia [11:4 7]
body mass index (defined) 1:1 13
bone

,
.age ~:79.
destruction (n: 123]
disease of prematurity 1:84
marrow aspiration 1:155
marrOW blastS (It: 138)
marrow transplantation 1:32, 147. 156, 175
(11:18. 29. 129)
mass. peak r: 73, 84
metabolism 1:73; 73, 84
pain !:164, 165!11:20)
resorption (11:74) '
bony dysplasia (11:81, 89)
boot -shaped heart 1: 18
Bcrrelia burgdoiferiinfection see Lyme

disease
\;
'
BOtulism, infantilt(11:85. 95]
boVine spongifornf encephalopathy (BSE)
1:177
bowel
abnormality 1: I Q4
inflammation 1:124
wall lesions r: I I I
brachial plexus inJury (It: SO]
bradycardia t:32[u:'<.l, 11, 20, 45)
brain
biopsy (u:97)
dysgenesis 1:35
hainatoma (11:90] .
.
tumolirsd5g..:l59 (1~:82, 128)
branched"chainamino adds (u:34Jbranched-chain fatty acids (11:32)
..
branching enzyme deficiency {GSD IV) (11:24,
32)
. ;"''
breast
canc::err:l34.134, 144
development 1:8!, 88 .
feeding 1:169. 178 (n:I.S, 22. 30, 33. 48, 52,
53)
milk. (n:40. 52) .
.
breathlessness 1:6, 9, 16. 17 [11: 11. i 21 1
breath testing 1: I 16, 1 18, 125
British Thoracic Society asthma guidelines
(11:131)
broad spectrum antibiotics 1:161 (11:55]

- ----~ -- _j

INDEX

broncheolar lavage [II: 1211


bronchial challenge [11:123j eof
,p '
bronchiectasis [11:114, I i6, 123,.124, 127,
1301
.
,..
!'
bronchiolitis' r:2 [II: If i i \ 6 7.''1 i 6; 12 7)"'
bronchoalveolar lavage [II: 12G-12 1: 129, 1311
bronchodilation [n: 125, 1281
' ' ~ .Jl '
bronchodilator challenge [n: i i31 - .,.
bronchopulmonary dysplasia 'ill: 1'2 71 '
bronchoscopy (n:'J2(}.-J21-. '1291 q
Brown'ssyndrome[II:J04;Hll /"'
brucellosis (Brucel/d infection) [u:201'
bruising 1:157; 163 l11:8,'27, 43, 48;<~31'
budesonide [11:2, 121
,. '

bulbar function (11:1291 '


r, -rv":!_,
bulimea :33

PC .,
Caesaria.1 section [11:15; 47]
w
ca'fe-au-lait patches 1:87, 128, 133:' 138, 142.
143, 144 (11:891
caffeine 1:48

'
calcification 1:14 [11:90, 93J-'"'-" '
caldtrioll:84
A> "

calcium absorption i: 1r9' ' r


d_.l'
calcium levels 1:i4. 080, 159[11:63.' 66;161
" 111;~10. 93isee also- hypercalCaemia/ '
~'Pocakaemia
.
'
~-
caldu:m.'inetabolism 1:84
Caldico'tt'Guardians 1:95, 101''
= ~
calprotectin 1:124
,<- -' ;
"'
Campylobacler infection i:LJO, i l8[i1: I, 1221
cancer

' r.-.- ;:.


disease associations 1: IS{ IM
sunilalr:74. 85, ISO, 159

susceptibility 1:134, 144; 145 ' '


Candida infection [II:Hil 57

:c'

..

canthus mass [11:104, 111}

'

capacity <mental) i:IOI. 102

capillary leak reduction [II: 1281


"
capillary refill time 1:S 7[11:62, 6 7. 68. 711
Cf\P=RAST [11:125)'. -'" --'-..i..---------~-----"'----carbamazepine therapy 1:4H6. 49, 52~ 169,
178 [11:83, 87, 93, 96)
carbohvdrate

absoi-ption 1:119 . it
deficient COGs [11:31)
dig'estion abnormality .-:'125'
intolerance [11:124, I25l
ca~~~~~

n.

blood level [11:8, 24, 64, 66, 67, 68,1281


diffusion tn:I2i-I23. ,3-i>l

. carbon monoxide poisoning 1:42. 50


carbm.:ylase deficiency 1:55, '65
'
carcino-embyronicantigen {CEA,I r:f 60 r
cardiac catheterisation 1:11
'
cardiacfailure 1:83 [n:II, 18, 74. 76i
congenitaltiS

high-output 1:103
neonatalr:I7. 20
right-sided r: 12, 13

CardiaC

~f

~~

t.~.., ~,.

lesions[n:l271
output 1:13, 15 [II: 1261 . .,j
cardiologyl:l-IO,II...:21 '-':.<"11'(. .., t,
cardiomegaly a:9. II (II: 17]
q
t'
cardiomyopathy 1:11: 15 [n:32, 54;59, 81)'
cardiotoxicity {anthracycline)i:lSd 11, '
cardiovascular surgery 1:1, 2. 11; 12,16,
17, 18
: l ' c' -",'. ' ..
cardioversion 1:!2 1' o ' : ' -t~-- ' ":
carditis(n:I9, 1421 ' --
'...

Care Order (Children-Act\ 1:29.''36'


carnitine therapy [n:291
carotid artery occlusion [11:941 ' :
carotid sinus rri?lssage 1:20. 21
~
carriers, genetic disorders r: 131, 136, 131,
l43 [11:33, 117,1281 ' j
cascade streening; cystic fibrosis 1:137 - ''
caspases 1:177
'

cataracts 1':s1: 68;-88 [li:7,18, 34, 35, 54;HXl}


congenilal t:I3Hn:7B; 107} '
neonat-al [ir:661' ,. . . .. 1.
. '
_l
oil drop (n:34, 351
''
steroid-induced [u:ioo, 107; 1091 ,_
catecholamine production 1:165
.
Catford drum '[11: I 03, I (I I
cat scratch disease see Bartonella henselae
cavi!rnotis siilUs thrombosis [11:3, 13, I 03, ,
1101

co1s defect {leukocyte adhesion defiCiency


type r:I, 181
'
CD40disorderl:181 [n:I31) 1
CD4 expression {CD4 count) 1:172. iBO [11:17,
49, ISI,'I631..
. ..
.
cos expression 1: ISO
cefotaxime r:l 70, I 78
ceftriax6ne-[llif'7l "-- ~ ... '
cell-mediated immunity 1:155
cell shrinkage in apoptosis r:I69, 177
cellular casts [n: 1411 .: ~
cellulitis 1:3, 17 3 [n: I OBI see also orbital
'
cellulitis
centile charts (height) 1:83.
cephalosporin therapy 1:48 [n:47, 55, IOBJ
cerebellar ataxia 1:!'67, 175
cerebella-r vermis hypoplasia uoubert
syndrome) [n:IOO, 1071-'
cerebral

abscess [11:3. 13, II I 1


aneurysm (n: 721
.
artery occlusion [n:94l"
dysgenesis [11:961
palsy 1:30
c

,.

173

ESS.ENTIALQUESTIONS-FOR MRCPCH -.-,1.

~.

chorior'iic \1llus sampling'l~;"l43 '


swelling [u:9i]

choroid abnormalities (11:1 06, 107}


cerebroside [11:29] """ ... , ; ''" : ..;;.,t .
chromatin condensation 1: 177 '
cerebrospinal fl~Jd (CSF) i:l80 [n:IS, ,!!1:.9Q.
chromium-labelled EDTA (11:751.
94. 108]
..
f\'
.
'chromosomal sex 1:89. 135
.
chalazion (Meibomian cyst) [11:104,,111]
chromosome
chaotic hypsarrhythmia [11:87] -. , .
analysis 1:1 so
CHARGE aSsociation [u:S4] . .. ,
defects r: 129 .
Chediak-Higashi disease 1:61 [II: I 06]
chromosome 11,1:138,
chemotherapy 1:85, I 50, 159 [n: 18}
also
specific drugs
; .

chromosome 12,1:139
MMR vaccination 1:32
chromosome 15,1:138 . chromosOme 16(il:i2. 90)
sideefrects 1:151. 160,.161
chromosome 18(11:39, 44. 49 . 54, !07),
in varicella patients 1: 170; 118; .
chromosome 20, 1: 145 .

cherry red spot [n:~S. 34]


. ,,:')
chest

.. . ... .
. -t - , ,
chromosome 21. r:l39.140, 143
hyperinflation [n:I)!J,.I27.'13.1)
., .. ,
[11:46. 124] . '
..
movementscbreathing) [11:128]
, , chromosome 22, 1: 14. 129, 140, I ; 2.
.
180 [t1:54}
.
pain [u:7, 95. IQ3,118J.
X-ray [u:42. 120. 121)
. . ,
chromosome 5, 1: 140
ctie)'nes-:-Stokes respiration.s [11:951
chromosome 6,-1:138lt1:59) ,
.chickenpox see \'aricella
.,
chromosome'7,tl4. 138, 145[1i:J76)
Child'Assessment Order 1:29. 36-=-37 ..,
chromosome9.[n:89.-901 ""'"' tt : .
c:hiid development, psychiatry and
__chromosome Xp21 [u:91L , - . . ,
chronic bullous disease of childhood 1: :9,
community medicine t:?J.-.29.
3Q-37
. 59-60.69
.
. . .
. . .
.
. >
child protection.1:29. 36-37
chroni_c fatig-,le syndrome 1:27. 35, 8.8-t ~
Children Ac(1:36-3 7, [u: 129] . .
,
chronic granulomatous disease-1: 180 .
chi-square test (statistics) li!: 149. 152. 160;chroniC:paro:~;ysinal hemicrania .(n:95] , 1
.chylo~icronsl:ll9, 12, .. ; .

,. ,
- > ,164}
,, , .
e
I
cicatrix formation 1:105
Chlamydia infection [11:18, 45, ss: 101~ 108,
""
.
. ' 110, liii
ciliary beat frequencyin:l27)
chloramphenicoll:40,43, 45, 48, so
ciliary.brushing [11:129]
chloride

. ' .
..,
cimetidine ingestion 1:100
,
ciprofh:ixacin therapy 1(179 [u: t'5J.
diarrhoea [ti: 771 .

neonatal disease [11:39. 50]


,
,
c~rculat~t?' collap;;e (11:!,71. , .
renal disease [n:59. 64, 66,.73.77, 781
c1rcumos1on 1:97. 97. 105, 106
cirrhosis in clinical stu'dies' (u: 150. 162).
respiratory disorders (n:i26, 128),
i~
cisapl-ide 1:15 , ' "

chloridorrhoea (n:28J -, 1 , :
,
chlorpromazine hypersensitivity 1:119
__ .. clarithromypn therapy 1:179.
. choanae atresia In CHARGE associiition;classical migraine (11:86)
. (11:54)
.
.
.
...
cleft lip(1i:44. 54)
choledochal cyst (u:48)
cleft palate 1: 14 (11:44, 54)
. ,
climbing ability [u:85J
. , , . .
cholelithiasis 1: 122, 123
cholera 1:30 [n:I4J
. clinical governance 1:93-97. 98-107, .
cholestasis 1: 122(11:481
clinical pharmacology ~cf- .to'EiCQlogy
cholesterol
,1:39-46. 4 7-52
; \.
clinical rele\cmce of clinical studies (u: 16 I..
binding resins [n:31)

,
. 165)
. . .
..
le\els 1:125 [!1:23, 28, 31, 32, 69, 73]
also hyperch?lesterolaemia
clinical trial allocation ofgroups[u:t4.5, 14.
hn>ercholesterolaemia 1:25, 33 [n:32)
159}
lowering margarine [11:23, 31) .. , ..
clinodactyly ::55 [u:54] .
metabolism 1:116, 125 (11::23. 31-32)
clonidine pro:ocation test 1:12, 82
svnthesis [11:32, 33) ;

clonidine'toldcity 1:49
chorea [n:I42J

Clostridium t::-rulinum infection [11:85, 95)


choreoathetosis [u:88, 97}
see also :.Vtulism 1:2-infantile
CLO testing ::; 25

chorioamnionitis 1:51:[11:39, 491

see

see

'
174

_[

'
COAL (cysteine, ornithine, arginine, lysine)
loss (11:30)
coarctation of the aorta 1:2, 4, 7, 12, 16, 18,
19, 87(11:76]
cobalamin (Vitamin B 11)1: 110 [n:29J
cocaine use (maternal) [11:39, 49]
Cockayne syndrome r:57, 59, 68
cocktail party inanner 1:14, 141
coeliac disease 1:109. 112; 115, 117. 121, 125,
.
134, 144, 167
cognitive (intellectual) impairrilent r:i5, 34
(11:78, 92)
coliform bacteria [11: 16]
colitis 1:ll8; 121 '
collodion baby J:54. 54, 62{11: 109) _.
co1obon _lin CHARGE association [ri:54]
colonoscopy 1!120
colour blindness (11:105, 112]
colour recognition 1:25, 28, 31, 34
coma (11:86, 96]

common arterial trunk 1: 14


compensatory chin lift (11:111!
complement '
levels 1:169 [11:34, 65. 69, 77, 78]
in blood products i:l49, 158
immune function 1:177 '
complex partial seizures (11:'87]
computerised tomography !en Sc.an (11:90,
108]
concurrent control groups [11:151. 162-163]
confidence intervals [il:l47, 149. 157-158,
_160, 161]
confounding factors (clinical studies) {n: 145,
146, 148, 154, 155. 156, .159]
..
.
congenital
adrenal hyperplasia (CAH) 1:73. 75. 85, 87;
89[u:47, 671

disorders of glycosylation (COG! (:1:23, 32] .


external angular dermoid cyst 1:94
heart disease 1:1
heart disease 1:9, II, 20 (n:44. 54, 92]
infections '[n: 7]
malformations [11:44]
rr.yopathy [11:48, 9t]
rubella syndrome [II: 1oo. 108]
conjunctival scrapings [11:108]
conjunctival telangiectasia 1:69
conjuncti\~tisl:9, 14, 176(n:ll, 19. 55,101,
I 03, I 08, 141] see also eye discharge
connecti\e tissue disorders 1:103. 148
Conn S)ndrorne (11:59. 77]
consang1,1inous mariiage (genetic riSks)
1:131, 143
- .
.
consciousness. loss ofl:27
consent
. child 1:95. 102

- .INDEX

parental!: 106
written 1:106
constipation 1:25,33, 112. 115, 118, 121. 125

(u:85, 95, 127]


contagious diseases 1:2 .
contrast study (gastrointestinal) [II: 118]
controlled clinical studies (II: 146, 149. 155,
161)
.
.:
'
i
convulsions see seizures
coordination 1:24. 26, 32 (n:95J ,
cop)~ng skills 1:23, 25, 26, 28, 30, 31, 34
corneal

clouding (u:35, 109)


deposits [n:78. 105, 1121
thinning [11:108)
. '
coromiry artery
aneurysm 1:14, 15 (11:141]
arteritis 1:14, I 76
.
coronary heart disease {familial) 1:20 ,
correlation coefficients (statistical) (n~l47,
148. 158, 163)
'
' .
corrosive substance ingestion !:42,-50
cortical atrophy 1:32
corticosteroid therapy (II: 73, 10 102. 11 o.
126)
also steroid therapy
cortisol 1:73. 74, 79, 83, 84, 86, 89, 90
coryzal symptoms 1:56, 58 (II: I, II, 127)
co-trimoxazole t48, 178
cotton-wool spots [u:74)
_
c6ugh(n:l,2, 7,10,11,118,120,121,125,
126, 129. 131) .
..
counting skills 1:34
court decisions 1: l 02
cover test 1:25, 33
cow's milk
allergy t65 (u: 1251
free diet 1:65
protein 1:126
.
coxsackie virus infection [1i: 16, 48)
cranial nerve palsie'SI:I68, 175, 176
Ill cranial nerve (oculomotor) (11:109, II OJ
IV cranial (trochlear) nerve (u: 104, 11 o,
Ill]
v cranial (trigeminal) nerve (n: 109]
Vl cranial nerve (n:9s. 1D4, I 09, I 10, Ill)
Vll cranial nerve {11:19,60, 63, 76, 94. 109,
Ill)

craniopharyngioma 1:74, 78. 85, 86, 90


[11:102, 110)
.
.
c reactive protein (CRP) 1:88, 125 [11:8, 9, 18,
62, 63. 67, 136, 138, 144]
creatine kfnase 1:28 (u:t36, 140, 143)
creatinineblood level[n:62, 63, 64, 73, 76]
hyponatraemia [11:67-68]
liver failure )11:27]
neonatal infection [11:8]

see

i:

J' ~

175

ES~ENTI.:..L

QUESTIONS FOR. MRCPCH

non-imponed UK ihfectious diseases [11:9)


renal syndromes [11!6s] . '

tubular dysfunction [11:66)'. ..

creatinine kidney clearance [11:'75)


Creutzfeldt-Jacob disease 1:177
cri-du-chat 1:129. 140

Crigler-Najjar syndrome [11:31, SSJ


Crohn's disease 1:109,-168
differential diagnosi~ 1:35. 11 o. 110. 1j5
disease associations 1:174. 175 [n: Ill]
features f: 1 1 1. 1 18

cromogly~te therapy 'ru: 109)


crosby capsule biopsy 1:11 7
croup 1:1, 1. 12 ,
,
,
cruisi!lg ability 1:24. 32' , ... .
,
1
cryoprecipitate (blood products) 1:149, 158
cryotherap:: [11:53)
.
crypt hypenropny ,, 11 1
,.
cryptitis 1: 120
cryptocnordism i:79. 84, 89
Cryptospor:dium infections 1:181
crystal deposition (renaiJ 1: 150
cubitus valgus 1:8 7
Currarino 5\ndrome r: I 03
Cushing s},;drome !disease) 1:55. 65. 73. 83;
84,88
'T
..,
'
cyanosis 1:1, 4. 6'(11:7, II, 12. 13, 16, IS. lOJ '
'
cyanotic hean diSease (n: 1231
cyclic adenosine monophosphate (cAMP) 1:80
cyclical neutropaenia 1: 173, IS f
cyclophosphamide therapy 1:8S [n: 141)
cyclosporin therapy 1:49, 55, 56, 61, 64, 66
[11:71)
'
cyproterone acetate therapy :65'
cysteine (u:30. 78)~
cystic dysplastic kidneys [11:46)
cystic fibrosis
complications [n: 127]
cystic fibrOsis-related diabetes 1:80
differential diagnosis (11;64, 77. 120-121.
123, 130, 131)
disease associations 1:SS, 65, 112. 121, 137
[u:4s: 1241
features ::127 [11:114]
screening 1:137 (Ji:l 17, 126,'128. 129]
sweat test (11: 11,5. 126, 128)
cystic fibrosis transmembrane regula~or
(CFTR)!:I27,137(n:1l6,128)
cystinosis {:;:21, 28, 66, 10:18, 126]
cystinuria [:::22, 30, 70)
cyst of Moli [ll: 1 1 I)
cytokines. proinnammatory Ju:4. 16]
cytomegalmirus infection 1: II, 112. 180 [11:3,
48, 49!
cytomegaiQ\irus pneumonitis (n:3. 13)
cytopaenia [11:10)

176
.... , ; I

cytoplasmic inclusion bodies (11:101, 108)


cytotoxic drugs 1:31 (tt:ISJ
'
dactylitis [n: 140. 143]
dapsone therapy [u:l31)
dark adaptation [II: 106)
data
.
analysis l5catlstical) [n:l46. 147, 148, 149.
156. 157. 158, 160, i64j
distribution [n:l47, 148, ISS, 156, 15'1.
.
160. 164] . ' '
.
protectiont95, 101 ,
daunomycin 1:150, 160 ~
~
deafness see hearing impairment
debrancher enzyme deficiency (GSD Ill!
(11:14, 32]
decerebrate ;:-osturing [11:86] ,
decision-making based on clinical.study
results [:;;150, 161, 165)
decorticate posturing (11:961 .
defibrillation 1: 1o. 10. 11
dehydration ::97,97 [u:66, 71, 75]sccalso
rehydration therapy

dehydroepiandosterone sulfate 1:86, 90 .


delayed

capillary retill time [11:8]


cord separation 1:181
gastric em?tying 1:33,
deltoid muscle denervation (11:39)
demand feecing (u:33J
'
dementia (11:94)

De Morsier syndrome
septo-Qptic
dysplasia
dental disea5e 1:14,31, 57;68, 84 :,.
Deny's orash syndrome ~:164
, . . ,
deoxyribonucleoside triphgsphates 1:176.
depression ;: i. 35

dermatitisl:33, 53, 62,65,69: 117 ,.


dermatology 1:53-60, 61-70.
dermatomyositis'("' 1'4lf .. '
Dermovate therapy 1:64,.66
desferrioxamine treatment 1:41, 49. so
desquamation 1:168, 176'
development disorders
clinical stt:dies [u: 152. 164)
delayl:l4. 18. 31. 1.39(n:91]
differential diagnosis lo35-36
genetic stt:dies 1:26, 18. 35
regression !:34 [u:lS)
SIOW1:26
development ~normal) t23, 24, 15, 16, 30, 31,
3l, 34
'
'
'
development, psychhiltry, and'
community paediatrics 1:23-29.
30-39
. . .
dexamethas.::one 1:73, 83.'84 [u:l, 12, 47]
dextrocardic.t:l:l20. 127, 130)

see

- j ---

.... . 1 ...... -~~ J.-

dextromethorphan therapy [11:22. 29) ,



.:
dextrose bolus (11:24)
dextrose infusion [11:33, 67, 68) . , .
diabetes 1: 113 (11:29, 1081
' ,
insipidus t:74, 80. 86 [11:75, 39. so. 6l)c
maternal I: II, 91 [11:40, 47. 48, 50. 100.
.

. . ' .....
107}
.
mellitus 1: I L 70, 71, 80, .87. , 09. II 7, 123
[11:115127) .
" ' ' ' ...
transient neonatalt:l38
dialysis

haemodialysis J:40, 48 [n:7Z.


peritoneall:4@, 48 .

, ,
renall:41, '48 (u:60. 73]
,.
Dianette t65", _ .
. .
.
diaphra, matichernia 1:95,103 [i::-;..;,541,
Bockd<:lek type '[11:4 7)
congenitall:95 [u:38, 4~. _48],
;._.
. -,
left-sided [11:38, 4 7]

. Morgagni [11:47]
.
diaphragm~tic palsy (11:501 ,, .. t., ,.. t'
.
. diarrhoeat:2,"10,34,67, 120,121. 165[11:80)bacterial ~:110, i 12.' Uljr11: I, I ij
'
bJoOdit:l)"s. tiB~~l2t.'".
',f( ..
chronicl:lll[11:64]
, .. _,
congenital chloride [11:77]
dysentery [n:l4]
immunodeficiency 1: I 73, 181
neonataltlll
persistent [n:20l
rehydration therapy (11:2, 121
viral infections 1:172, 180 (II: I]
;.vatery 1: 1J 6
diazepam ~:40, 48

diazoxide therapy [n:24, 33]

,,.
diencephalon [11:9Sj
'diet' drinks, phenylalanine conte;,t 1~:30]
Di George syndrom~ 1:_24

disease associations i:SS, 65{r::,;O, so, 129)


diSease trarlsmissioil(rl: t31J.
features 1:3. 14

genetic factors 1_: 129, 140 [11:541


impaired immunity_l:32
, .
digital clubbing [II: 113, 120, 123. 1301
digoxin therapy 1:20, 21, 40, 41. .;,5. 46,_48,
49; 52
. .,
. '
. , .
dilated cardiomyopatny t:3, IS, 136. 141, .
142,143, 144 [!1:)34)
I.
Diorylate [u: 121

diphosphoglycerate binding [11:4'7].


diphtheria [II: 14) '
.
diphtheria-pertussis-tetanus.!D?TJ ;:23, 3o
diplopia [11:93, 95)

direct Coombs testr: 148, 156 [u:.il. 44, 45, .


52. 55)
.

.. '
disaccharidase acthity t:109. H&:. 119, 121

discoid rash [n: 140]


., :~
disease prevalance (genetic) 1: 134
disease prevalence in clinical studies [u: 159)
disseminated HSV infection (n:8, 1i- I 9! ...
disseminated intravascular coagulation (DIC)
jn:l33. t39)
distal interpharyngeal joint[n: 1401
distal intestinal obstruction syndrome ,
(DIOS) [11:127, 130]
distal obstruction syndrome 1:122
.. ',
distal renal tubular acidosis fn:58, 70, 71) !dithranol in Lasser-'s paste 1:S 7, 67
!l
1 ,,, .
diuretic ther~py 1:6, 12, 17,86 [11:13-74]., 1
IOOp1:41,49[11:51,59, 72, 77)'
,',,
side effects t41 (11:59, 77) -,.

.,.
thiazide 1:49. 74 [u:721 , ,
dizziness (drug-induceq} [11:93]
DMSA scanju:6, 17, 77)
. ,,)
DNA, ,
.; -.
cleavage 1:177:
;.;;r~ 1,..
expansionl:l76
.
.
~:.
donor compatibility 1:168, I 76 [11:42, 45; 55)
dopa-responsive dystonia see Segawa '
disease,. .

~- ... --.
Dovonex ~:66
.. ,. ,- ,.
Down syndrome 1:1. 2. 85, 130.132, 139; 140,.

142, 143 [11:46, 24, 148, 159-160]


drawing skills 1:25.29,30,31
1
dre~ing skills 1:23, 26
dropattack5 [u:96J
.. ' '
drowsiness t6. 24, 63, 68, 76. 93; 172 [1r:95) .;
drug
.. .
,,,
..,
absorption 1:39, 47, 82
acetylation (in SLE) 1:41, 49, 61
acidity 1:39
addiction 1:31
bioavailability 1:39,47,52
clearance !:51
distribution 1:4 7
excretion ,;48
formulation 1:39,47
haif-Ufe 1:40, 40f 48 "'
ionisation 1:47, 48
metabolism 1:45,51
monitoring 1:41, 45-46 (II: 17)
pharmacokinetics 1:40, 43, 45-46, 51 [u:5t,
protein binding !:40, 45, 47, 48, 49, 50,51
solubility 1:39, 45, 48
steady state ~:52
dry scalp [11:136, 143)
Dubin-Johnson syndrome (n:48]
Duchenne muscular dystrophy
differential diagnosis 1:35, 36,.120
disease associations 1:24, 31, 34, 143
features 1:130
genetic factorSJ:132. 141-142 [n:91)
4

1'77

ESSENTIA.:..
QuESTIONS . FOR- MRCPCH
'
- '".
'
..
*~

~uodenal atresia [n:38J

duodenal bioP;~Y 1:109, 117


duodenojejuri;~l flexure i:97, 106
duplex kidney [ii:17) " L '
duplication cyst 1:105
'
dural eclasia 1:130, 141
dynein a'rfns {u:l27)' ''
I;
dysarthria (n:89J

dysentery (u: 14)
dysgenesis 1:35, 82
'
dysgeririinoma 1:$6
..__k
dyslexia [u:9~1
' '
dysmorphiC features 1:34, 87[11!23. 28. 31, 44;'
46. 54)

'
dysostosis multiplex (n:35)
dyspnoea (11:117. 121)'
:J:: '
dysrh)1hJI1ia !:10. 10, 20....21 1 t
1
I ~ , " .
dystonia (n:SS. 97]
dystrophia myo~onica protein kinase !:137
dyslrophin 1:136: 143, 144 (lf:91)
'
ear disorders 1:25.34 (11:44, 54.' 1.41) see also
heaiingimpairmeri't' : ; . 1-'1:{
.
Ebstein's ancimoly 1:7, II, 18
.

echocardiography {ECG) 1:8, 15, 16.' 19 '


ectopic gastric m!Jcosa 1:98

ectropion 1:69[u:i09J
"1. "
eczema 1:59 ~ 175.'.1 78[11: 115)
allergic (u:l 11. 1251 ': ,, ,.
''
atopic 1:54. 55. 56, 57, 58, 63. 64:'.65, 66.67 t'
clinical studies (u:l52-153, 1~165]
~,
herpeticum 1oS8, 67

therapy 1:55. s6; 64. 65, 66. 67
~raJ infections r:54. 58, 63, 67l ..
egg allergy 1:32 (n: 115, 125)
. .
Ehlers-Danlos syndrome (11:100, 107)
Eisenmenger syndrome 1:12

elaslase 1:114, 12.2:124, 141; 181 "
elastic limit (lung) (u:l26)
'
elastin gene deletion r: I 41
-. -
electrical status epileptus in sleep fESES)
(n:96-91]
electrocardiogram (ECG) 1:4, 6, 8, 12; 15, 18,
19.144 (n:7. 142)
electroencephalogram (EEG)
anorexia r:32

diagnosis r:2S, 34 (11:82, 84; 93)


epilepsy S)ndromes (11:87/96-97) ,.,
neurodegenerative disorders (11:88)
speech delay 1:28, 36
'
electrol)1es. r:99. 159 (11: 17, 66)
electromyogram (EMG) [n:84, 85, 93;'94. 95)
electron microscopy (11:127]
electroretinogram (ERG) [u: 100. l 08)
Embden-Meyerho( pathway 1:155
.
.
embryology (lung} (11:1241
Emergency Prot!i>;!ion Order (EMO} r:29. 36

.' .

emollients 1:56: 6s! 66


empyema (n:l23)
encephalitis 1: I 74 [u: 14, 94)
encephalocoele (11:52) 0
encephalomyelitis (u:94l
encephalopathy (n:l9, 33, 46, 53, 76)
endocrine dysfunctiOn 1:33,74, 85 .
endocrinology 1:71.:79. ~o-91
endonuclease a.:tivation 1:177 .
end-organ resislance (11:75)
,endoscopyl:t23
, ,.
s1
Enterobacter infection (n: I 1J ~. ...
entero\~rus inieclion5 1:54, 63lu:l. 16)
enuresis (11:60)

'
'
' .
enzyme replacement .therapy (11:29) .
eosinophilSI:9. IO.IS7(1t:20J .
~ eplbulbar dermoid cyst[u:5'4J . . .
epidermolysis bullosa' 1:53: 55, 59. 62, 6-L 69
'epilepsy
:
~
.
.
"
abscence (n:S7;'96) .
' .
benign childhood (roiandic) [u:84. 93)
COmpJex partial l:ii, 35[1!:87) I
, ,
drug therap\' [u:83, 87. 93, 96;'105, I h1
epilepsy syndromes Iil:87.'96-91J ~
generalised [n:82, 96) 1
~ '
maternall:l. 11

myoclonic (1::96]
noctur!lill front lobe (u:82J '
reflex anoxic 1:35
sub-clinical r:34
tonic-clonic [u:87. 961
epistaxis 1:163

..
Epstein-Barr \irus (EBVl. infection r: 167, t74,
175 [n:J I. 122. 142)~
equinovarus deformity (11:88) .
Erb's palsy (u:39. 50)
'
erectiledJSfun-ction 1:100.
erythema (em:hematous lesions) (u:9J .
. Gar4ioifasc-Utar-diseaser:-14 ~ ... -- -'-- ,_
causative factors r:54
differential diagnosis 1:56,57,61, 62, 69
rheumatologic disease (n:I41J
'

erythemainfectiosumseeparvovlrusBI9''
infection '

erythema
marginatum 1:63 [u: 142).
migrans t54, 63 [n: 19)
multiforme r:63, 67, 69
neonatorum 1:54, 63
.
nodosum 1:54, 63, 168:175 [n: 113; 122.
.129. 140)

toxic 1:63
eiythroblastic leukaemia 1:1 so. 159
erythrOC)1e sedimentation rate {ESR)'I:9. 136,
138. 139. 142, 144

erythroderma meonatal) 1:55. 65

.j

INDEX

erythromycin
resistance 1:64
therapy 1:1-7Sfll:4s. ss;

wL

erythropoietin 1: l S6(n:74)

1081

' " _.

Escherichia coli infeciion 1:6, 16. 17. 18. 118,


178, I 79[11:27]
ethambutol therapy [11'17: 105, 112, 125]
ethinylestradiol (Dianette) 1:65 .
ethmoid sinus (u: II 11
' " '
ethnicily see ra.ce factor:;
Eumovate cream 1:66

evidence-based medicine, cllnkal


governance, surgery and consent
1:93-97,98-107
exanthem 1: 176, [n:4J
subit :m troseala infantum} 1:63
exfoliat,,e erythroderma ~:62
exomphalos 1:104
external angular dermoid cyst 1:94. 100-101 ,;
extracellular nuidvolume tECf) 1:39,47, 50

eye

'

abduction (11:104, 109. I I I)


abnormalities 1:17 , 68-69. (i1:26. 106] see
also visual function
adduction [11: 104, Ill)
contact 1:31 '
'
discharge (II: II OJ
fixation 1:76, 79. 88. 91 '
masses [n:I04. Ill)
movement disorders [n:95, 104, 110, li!J
pigmentation disorder [II: 108 :.
rash [II: 143] ,.
tests 1:25, 33

eyelid
cellulitis [II: 108)
nodule [11:104) '
swelling [11: 102, 104}.
FAB classification 1:159
Fabry disease [n:28]
facial
asymmetry [11:44, S~J
choreoathetosis (n:88] :
diplegia 1:137

dysmorphism [i1;44; 54, '108]


nervepalsy(n:l9;60.63, 76,94,109,111)
palsy [11: 19, 63, 16, SS] see also cranial
nerve palsies
swelling [n:65)
.,~
twitching [11:84, 93]
weakness [n:63, 94)
facioscapulohumeral dystrophy (n:91]
faecal mass 1:125
faecO-Qral disease tr-ansmission 1:2. 12
failure to thrive (faltering gr0\\1h) 1: Ill (n:25,
66, t:2, 91, 120. 130)
.faliopian tube development[11:37, 4 7)

false positive results in clinical studies


(11: 148, 159]
Family Law Reform Act ~:1969. 102
. family periodic paralysis [it:91J
Fanccini svndrome [n:58, 70, 78]

fat
absorption ,:119, 125
content (breast milk} [11:40)
content (neonatal body) t:47
fatigue 1:27~ 35~ 8~89. 1.14
~--.
oxidation (11:23, 32, 34r .
oxidation defects 1:28-29. (u:21, 34]
synthesis [11;33] . .
... : ..
febrile neutropenic episodes 1:151' 161
febrile selzureS/conwlsions !:32, 112; 180
[11:82-83 92] '
.
feeding ability (normal) 1:26.
feeding disorders [11:25, 129]
co<Jrctation of the aorta 1:16
differential diagnosis 1:35--:-3~ """ , .
feeding difficulty (u:ll]
hyponatraemia (n:67]
liver failure (n:27)

neonatal ~:6,.9~ 28 {1~:8, 19~ 24] ~

neonatal ja'undice [11:44]


Noonan syndrome 1: 139
ferritin tserum) 1:155

fetal
alcohol syndrome ju:39, 49. 54).
haemoglopin (11:49]
heandiseaset:l, ll,IS
lung growth (11:46]
movements 1: 137(n:42, 94i
renal disease [u:46]
seizures (u:46J
1.1rine production [n:46)
fetomaternal bleeding (u:53]
librillin gene 1:141

fibrin 1:158
fibrinogen r: 149, 153, 158, 163
fibroblast growth factor receptor gene
(FGFR3) 1:128. 138
Fick's Jaw (11:122, 123]
.
.
fifth disease see parvovirus B19 infection

finger abnormality (11:44, 113]


FIRST support group 1:57
.
fixing and following 1:16, 7.9. 88. 91
flame-shaped haemorrhage (11:74]
flashing light aura [11:95)

flat nasal bridge [11:54]


necainide 1:20. 21
flexible bronchoscopy (11: 1291
flexor response (n:96]
now cytOmetry 1: 172, ISO
flow-volume loop {spirometry} [II: II 7]
lluclo.xacillin therapy 1:1 ?9Iu:55J

..
179

f. ,-~f:',

t , :\~\'
t1:
f

f;_

-_ C::'"

ESSENTI~.:.

Q:.JESTIONS FOR MRCPCH

,: l

tluid eye signs in inborn errors [u:26. 34]


dextrose maintenance tluid (n:67J
features 1:65. 88 [11:23, 31, 38, 48. 58,70.
intake in Rett svndrome (n:91)
78)
intraven6't.is (n:'75)

hypoglycaemia J:91 '


- normal saline (u:39)
,
liver failure [11:27, 351
overload in nephritic syndrome [11:78]
_
galactose-free diet (u;34]
sodium chloride maintenance tluid [11'67]
gallbladder hydrops 1:!68, 1.76
therapy 1:99(:::24, 39, 75, 80]
,
gallop rhythm [u:63, 65, 711

tluorescence in-situ hybridisation (FISH)


gallows traction 1:98
1:129, 129. 140.. 172. 180
gamma globulin. intravenous (MG) 1:14-15
FMRI mutation 1:127. 136
ganciclo\'ifl:l74
focal segmental glomeruloscleritis [1i:65, 781
gangliosidosis (li: 1141
focal seizures in.Rasmussen's encephalitis
gastric
[11:97]
,_

absorption 139, 47
folic acid anaphylaxis [n: 1421
aspirate(~:: 118! .
follicle stimulating hormone !FSHI1:71.
dilation 1:33
,
8o-8L a2. ss. 89, 90
lavage (\.:ashingl 1:49. st (u: 1291
fontimelle. bulging [11:8, 15]
motility 139. 47
Fontan operation 1:13 1
gastritis [u:-17!food allergy (;l: 115, 125j
gastroenteritis 1:116, 126(11:25, 6 7. 68. 80]
food poisoning (11: 14, 47!
.
gastroenterology and nutrition
foot deformitY iri Halleivordim-_Spatz
, 1:109-116. 117-126 .
syndrome r'n:88]
gastrointestinal ble~ing (11:53]
foot drop in Kllgelberg-:Welander disease
gastro-oesophageal retlux (GER) 1: I 04; 1 14,
(n:85]
'
123(u:ll..:.ll6,1,29],

,.,.
forced expiratory now rate (FEF![11:129]
gastroschisis :96. 104,..
..
forced expiratory volume (FEV) [n:l 14, 120;
Gaucher disease 1:54, 62 [11:29, 48]
121,123. 129f"'

,
gazedisorders[Ji:84, 109.111] ..
forced vital capacity (n:l23]
..
gerietics 1:121-135, 136-145. , ,,, ~
forehead size in achondroplasia 1:73; s4
genital hypoplasia in CHARGE association
foreign body aspiration [11:121. 131]
.
[11:54!
foreskin disorders 1:96, 97, 1.00. 104-105,'106
genital tubercle 1:77
formula feed 1: n 6. 116. 125;126 [11:27, 30,
genito"urinar\ abn'ormalities 1: t02, 164
35]
. ' . . .
.
. genomic imprinti'ng abnormalities): 128, 138
fost~ring 1:96. 105
..- 1
,. ,
gentamicin 1:45. 46. 48, 49, 51, 5.2 [11:45 5.SJ:
fragtle site mental retardation (FMRIJ gene.
germ "cell tumours 1:103. !50, 160
,
1:127
germ-linemosaicisml:l42 ;.
fragile X smdrome ~:26, 28, 3(36, 127.. !36
giant axonal neuropathy fu:92J. ,
[11: 14 7. 157]

giant melanocytic naevi 1:64


.,
frataxin protein [n:S9J
,, giardiasis (Giardia Iamblia mfecw:::i L 1 1/.,
freckling 1:128 (11:89]
122 (u:20J
. .1.
free bilirubin ::48
Giemsa staining [11: I 08] =
, !:'i
freefattvacidsl:79, 119
. ,
, GilbertsyndromeJ:IIO,IIS(n:31,55! ...
fresh frozen plasma J:l49, 158 (n:53] .
Gillick competence !:1 02 .
.. , 1
Fried reich's ataxia 1:15 (11:81, 89, I 06]'
'
gingival hyper;:olasia [11:93]
:.
fructosaemia (;;:78]
'I
, _ gingivostomatitis (herpes) 1:57. 67 ;
fructose r:i 19, 121 (11:27, 28, 35]
r Glanzmanns thrombasthaenia 1: IS:~. ! 6:!
frusemide [u:60, 73]' '
., . 1 glaucoma (11:&3: 99, 106. 109) full blood count ;:88, 113 (u:l 7. 20: 42. 43. ' glibenclamide [n:21, 29]
.,

138, 144) '


global develo;:-mental delay 1:34.[u:66] .,,.. ,.
fungal infections:: 161 (11:12,.16. 81, t08J
globin synthe:;is (red blood celTs) 1:14_8
fusidic acid therapy 1:179 ' '~ '
globus pallidt:s
~,;>
G985 mutation (1::34]
I
lesions (11:91.]
, .. , . : ~
'GAA trinucleotide repeat [n:SI, 89]
necrosis (H32!
,
gait disorders [n:ss:
S9, 94. 95]
glomerular m:~ationr<'lte !GFR) 1:83!11:58, 61, .
galactosaemiaseea/sogalactoSe
71 . 74. 15. i61]
,. ,

as.

,r 1

180

------'-~

_I_

INDEX-

I---glomerulonephritis [11:57, 69, 74. 16]


membranoproliferative (II: 78]
mesangiocapillary (11: 77]
minimal change (11:65, 78] ,
glomerulosa cell binding 1:90
glomeruloscleritis (11:65]
glucagon provocation test 1:72. 82
glucocerebroside deficiency (11:29)
glucocorticoids 1:84. 86, 90 (II: 12; 1
gluconeogenesis disorders [H:2S)
glucorinidation 1:40, 43, 45. 50. 51
glucose 1: 155, 180 (11: I 7.. 81. 90! s~t also
hyperglycaemia; hypoglycaemia
'''
after dextrose bolus (11:24] ~"'
CSF [11:15)
hyponatraemia 1:67-68
--liver f .ilure [u:27) "
metab0lism 1:119, 148
pituitary hormone testing 1:90
tolerance 1:82. 122 . tubular dysfunction [11:66. 78)
urinary [11:25, 66]
gluco~e-6-phosphatase deficiency rGSD I)
136 [11:24, 32).
'
glucose-6-phosphate (11:3~)
glucose-6-phosphate dehydrogenase (G6PD)
defiCiency 1:14 7, 155 [11:44-45 55]
glucose galactose malabsorption 1: Ill. 121
glue ear 1:25, 34 111: 127)
glutamate dehydrogenase [11:33)
glutainit acid decarboxylase antibodies 1:71,
80

gluten intolerance 1:117


glycerol!: 119
trinitrate 1:167, 174
glycine [11:29)
glycogen breakdown [11:33)
glycogen storage disorders'(GSDSI 1:19, 79.
88, 91 [11:24, 28, 32, 33]
glycoproteins-1:80, -163 [11:31-;-i.3lJ __
.. _-,__. ___
glycosuria [II: 78]

glycosylati_on [11:23, 32)


goitre ~:83
Goldenhiu syndrome [11:44, 54)
gonadal dysgenesis 1:75,'86
gonadotrophin-releasing hormone (GnRH)
'1:71, 81,87 .
. :
gonadotrophins 1:11. 78;79, 81;.85. 87,"90
Gore~Tex tube 1:13
Goltron's papules 1:61 [11: 140] Gower's manoeuvre [11:136)
G protein 1:80, 142
abnormalities 1:71, 80
graft-versus-host disease (GVH) 1:3. 16S, 176
Gram-negative bacteria 1:5. I 6)
Gram-positive bacteria [11:16, 95)

granulocyte colony stimulating factor (GCSF)


1:176
granuloma (11:52]
granulomatous disease 1: Ill, 179
granulomatous vasculitic -di5ease [11: 1411
Graves disease 1:82, 83
'
'
great arteries 1:2; 12
grey baby syndrome 1:40, 43, 45;.48,' 50~
grid analysis (statistical) (11:138, 159-160,
165]

.
'
group A streptococcal infection [1i: 135, 142]
group B streptococcal (GBS) infection [11: 18)
growth 1:83
growth disorders see also failure to thrive;
'
. short stature; tall stature
childhood malignancy 1:168
,)
coeliac disease 1:88'
i,:'
growth failure 1:68
growth retardation 1:1 76 \
maternal PKU [11:33] -
-, .....
j,
puberty 1:81
.
trisomy 1:18 (11:54)
grO\Vth hormone ((:;Hi '
.
deficiency 1:73, 74. 79, 83; 85. 88,'144
provocation test 1:72, 82

secretion 1:71, 81, 89, 90


. GSD see glycogen storage disorders
Guillain-Barre syndrome 1:167. 175 (11:90,
118, 129)
' "
gut malrotationl:96, 97, _I 04, I 05, I 06 [11:38)
gut obstruction 1:93, 9{-98, 101, 105, 106
' c

guttate psoriasis 1:63


gynaecomastia 1:72. 81.'82, 84,89 ', "
haematology and oncology 1: 14'7-154, '
155-165
. '
haematology values (normal) I:' I 53, I 62-163
haematopoietic stein cell transplantation
1:168
,
haematuria (blood in urine) [11:62, 69]

- macroscopic [11:57, 63]


microscopic [11:76, 77, 78]"
painless [11:65)
haemoatopoiesis 1:181
haemoglobin 1:9. 39, 114
chest disease [11: 120)
clinical studies [II: 150, 162]
differential diagnosis 1:152
dissociation [11:49, 113, 122)
'fetal [11:39. 49)
hyponatraemia [11:67]
neonatal d53, 162-163 (11:42]
oxygen affinity [11: 122]
renal disease [11:63)
structure 1:148, 157 [11:39, 49)
total [11:39)
haemoglobinopathy 1:131

. '.

181

.ESS ENTIA_ QUESTIONS .FOR .M RCPCH

I
1

!
i

'cariler tes:ing 1: t43


childhood immunodeficiency 1:113
differential diagnosis 1:124, 153, 162
neonatal infection (u:8)
neOnatal jaundice (IJ:44]
,
non: imported UK infectious diseases (11:9]
rheumatoiogic disease i11: 138]
tropical infectious diseases (n: 1OJ
, haemolysis [u:48. 107}
haemolytic t:raemic syndrome 1: 153. 162
haemophilia

Al:l53, 163
B (Christmas disease) 1:135, 145, 163
Haemophilus infection (Haemophilus
injluenzaei i~23, 30, 178.(11:4. 14;' I 08)
differential diagnosis 1: 179 [11: 18. 11 OJ
Gram stain [11:5, 16)
haemorrhagic disease of the newborn t:.l63
(11:38, 4:l. 48. 53],,
.. f.
haemorrhag1c fever, \1ral (u:l4)
haerri synthesis t:148. 156:-157
hair disorders 1:86, 87.[11:93, 108) see also
alopecia
.

, ,
Ha!lervorden-Spatz syndrome [11:88.,97]
hamartoma !retinal) (11:8l)
hamar,t'2matous j)<>Jyps 1: I I 7
.
hand flaJ?ping .1:26 ,
hand. foot and mouth disease 1:54. 63, 68
{11:16]
happy puppet syndrome t:l8, 35
Harrison sulcus [11:118)
Hartnup disease 1:61 ,, . ,
headache
'!.I .,.
cluster [11:86. 95]
. ,
diarrhoea [n:63)
1
differen~ial diagnosis 1:27 (11;7, II. 74; 85.
118, 129}
drug; induced (11:93]
migraine ::118, 121 (11:86, 95)
morning [a:84. 128}
non-imported UK infectious diseases
(11:191
.
tension (t::95)
\1ral infections 1:1 72
head injury:: IS
head size 1:26. 34, 73, 84, 138 [11:911
head till[u: J I I I
hearing impairment 1:25, 34, 57. 68 (n:S. 15,
17. 77. 108}
heart
,..
.
block 1:4. II. 15--16, 134, 143, 144 [11:109]
contractiiity (11: 125}

malforma:ions 1:139 (1i:54J


murmur~ ::4~5. 6. 7, 9. 17, 18-1.9.20 (n:7,
18. 4..;J
'
.
'
septum defects !:II. 139

.
182

valve defects 1:i6, 139, 141,,143. 144'


heavy metal poisoning [u:70, 78)
Helicobacrer pylori infection 1:116, 125
hemianopia [:~:84, 88, 94,.102. 110}
Hemi-Fontan operation 1:13
hemiparesis (~::84, 93, 94]
hemiplegia [11:88)
hemisensorv deficit [11:84]
hemivenebiae [Ii:44J
Henoch-SchOnlein purpura (HSP) [11:62. 70,
74. 76. 134. 141)
'
heparin
anticoagulation 1:169, 178
therapy (maternal) (u:48]
hepatic
artery aneurysm [u: 142)
" '
cirrhosis 1: 122 (11:32)
"'
cysts [11:72]
failure jn:22. 27, 30, 32,351
fibrosis {11:52. 59, 72]
glucorinidation 1:45. 51.
. . . ,,
hypoplasia.[u:38J '
impairment i:41, 49
metabolism r:40, 43, 45, 50, 51.
. steatosis,, 1'23
hepatitjs 1:150, 160, 175 [u:S. 14, 17; 123)
A.l: I 72. 180
B 1:23:-30. Ill, 119, 179[11:48, 122)
CI:IJI,I19,171,179.(u:l2)
hepatobiliarv disea5e 1:122
nepatobla'stoma 1: 1SO.iJ6o, I 64
hepatocellular carcinoma 1:150,.160
hepatomegaly (liver enlargement) 164 [u:8
18, 24, 27. 35, 66) 1 .

'
hepatosplenomegaly 1: 7.9. 88 (u:54, I 30)
hepatotoxi<;ity 1:49 [n:93,125) ,
hereditary ,haemolytic anaemia 1:14 7; ISS ..
[11:21.' 35)
c
,.
, ,
I
.,_
herniotomv 1:94. 98
'l
herpes-infections 1:11. 59 [u:I3J see also
human herpes \1rus
herpes simplex \1rus (HSV) infection
differential diagnosis 1:57, 69. I 72 [n: 12)
disseminated HSV infection 1:18-19,48
features ~:58;63. 180 [11:99. II OJ

gingivostomatitis 1:6 7
neonatal infection [11:8}
herpes zoster 1:110
heterochromia [II: I 06]
hexose monophosph<lle shunt r: 155
high altitude. effect on uterine growth Jn:SO]
high birth weight 1:88
hilar lymphadenopathy [u: 130] .

hilar shadO\\ing [11: 120] ,


,
hip movement disorders (u:l3(, 138, 1441
hirsutism 1:86 [11:93]

L~

If:.'-

histamine (endogenous) Jesponse 1:15 7 t.


[11:126]
histamine pharmacology (II: 115]
histiocytosis 1:86
,,
histone coat {u:22, 31]
histoplasmosis 1:1 75, , ~
Hodgkin's disease 1:154, 164 .
holoprosencephaly (11:54]
,, , .
homocystinuria 1:142 (u:22, 29, 921
'''
honey cont<!minatiqn (11:95(.
,,
hooded foreskin 1: 1oo
hordeolum (stye)[ll: 104. Ill.I
horizontal gaze paisy' {ti:84J
Horner syndrome (11:99. io6. 1.\0]i
horseshoe kidney 1;81, 88\H: 17). .
HSV infection (maten1ali [11~ i.9J, ..
'human 'lerpes virus t:,63, I 72. i BO ,.
I (11:16)
. ,
,, , " ;
human in1muriodeficiency \~r_us (H 1\1 ,
infection 1:30. 169(11:10, 13, 20. 121; 125.
129. 131)
disease transmission. (n:6, 12,J 5, 39, 49]
immunisation in HIV,paiien[S 1:23: 32; 175_
maternalt:3, 15, 31..1/?9.,1 77:;-178(11:~. 49]
,
1 neonatalt:l69 (11:39: 49)
therapy 1:6;, 15,J i, 31, .177. i 78 [u:3, 491
viral load t:3, 15, 168,.1.76, 178 [u:6, 171
human leukocyte antigen (H LA) 1: I 09, I I 7 .1
(11:134, 138, 140)
human papillomaVirusi:63(n: 121
human platelet antigen 1:149. 157
Hunter syndrome 1:127, 136 (u:28. 35)
Huntington's disease 1: 139
Hurler syndrome [11:31. i 09l.
hydrocephah.is 1:73, 84, 138 ill:95, 1-16. 1271
hydrocoele 1: I 07
,
hydrogen ions (11:58. 70, 1i. 73, 78]
hydrolysate formula 1:126
hydrOp? 1;_4..!Q3,168 (n:J8) ->""
hydroxybutyrate 1:76
,
.,
17-hydroxyprogesterone' 1:89 (n: 14 71
hyoscine, (n:2]
, , ,
hyperactiVity'(behaviour) 1:31
hyperacusis (u:34]
. .
,.
hyperaldosteronism [11)2, 77]
-.
hyperammonaemii,i (11:22, 30, 32, 33. 60, 93]
hyperandrogenism 1:65
"

,
hyperbilirubinaemia 1:68, 1 i 8 (n:49. 90)
conjugated (11:23. 31, 38, 46. 481
unconjugated [11:31, 46] ;
hypercalcaemiat:l4, 141(11:73]
hypercarotenaemia 1:33 f11:901
hyperchloraemia [11:39, 50]
hypercholesterolaemia 1:25, 33 (11:32)
hyperekplexia [11:821
'
.
hyperglycaemia [11:23, 32,125) .
<

'<

,,

hyperhydration 1:150, 160 . .


hyper-lgM syndrome 1:173, .i8l. ':, . .
hyperinsulinism 1:79, 91 (u:21, 29, 33] .. ,. . ,
hyperkalaemia !11:59, 60, 731
, ,'
hyperlactataemla (u:21, 28)
. , . "''hyperlipidaemia (11:33, 571
hypernatraemia [11:73; 75]
hyperoxi~ (1!:53: 1091.
~' . . ,,
hyperparathyroidism [11:50, 73. 74)
. ,.
hyperphagia t:90
hyperphosphaiaemia [11:501
hyperproteinaemia [11:90).
hyper-reninaemia [11:771
hypertelorism. 1.: 14Ju:54). .
...,,..(,1 c: {~
hypertension [n:69, 77. 78]
benign intracranial (11:94)
. . , :>' ,
childhood malignancy 1:154. 165
differential diagnosis1:l6 ,.
drug-induced 1n) 3]
,, . . , , .
drug therapy tu:70]
-:
fe?tur_es (u:60, ,74),,. ,
~~.,
.,
1 ._,
hypothyroidism 1:88.
intracranial [u:76)
maternal (u:SO] ~
,,
r
polycystic kidney disease [11:72]
portalt:112. 122.
pulmonaryt:S, 12, 17 ..
renal {n: 76]
""" . ,.. .; .
hypertensive encephalopathy (u: 76]
. '.;
hypertensive haemorrhage [1,1:74]:
hyperthyroidism 1:35,72-73. ~2;-83seealso.,;
thyroid gland ,
hypertonia 1:79
(.,"
hypertrichosis t:61
,
, . ..
,
hypertrophic cardiomyopat_hy
8, II,
15, 17, 19. t39, 143, 150
...
hypertrophic osteoarthropathy [11: 115)
hyperventilation [11:86)
.i
1
hypQali:>!Jminaemia [11:51, 6_?,:7L,
. ..
hypocalcaemia t:l4, 15;84, 140 [11:32, 40, 50,
51. 54, 741 .
.,
h)'pocarbia [n:49J.
hypochloraemia [11:77]
hypochloraemic metabolic. alkalosis 1:33
hypoglycaemia 1:28-29, 49. 76. 79. 88 [11:321
hyper-insulinaemic 1:79, 91 [11:211
hypoketotk [11:29. 33, 34]
insulin-induced 1:72,82 . .
hypogonadotrophic hypogonadism 1:86, 90
hypokalaemia 1:15.33,49 (n:59; 71, 72, 78)
hypokalaemic alkalosis (11:59. 77)
hypoketosis(11:33)
hypomagnesaemia t:4. IS [11:50) .
., ,
hypomelanosis of Jto.l:l30. 14.2
hyponatraemia (11:20, 59, 67. 80)1

hypoparathyroidism [n:50]
~

.....

hypophosphataemi~ 1:80, 84 .,,_It


hypophosp~atasia Ili:1on

hypoplastitlefl h.tt. #ridrome ~r! ~2. 9, i i;:
13. 19, 20, 132' - .
. ' .
. ..
hypoproteinaentia':49 .

hypospadias ~:94. 100 ..


._ ..
hypotension 1:~;2- ~.8 [11:4?. S8.:?t 93] -~'tt
hypothalamic hormone secretionsl:81, 86;
90
'!
. .
:,__ .,,.
. '"'

ryypothalamus d\sfunction r:78, 79. 90 (11:891


hypothermia tiS
'
"'
,. , :.
hypothyroidism !:35, 55, 64; 7Q-7i. 87,
88-89. 161 [,;:78. I 14, 124)' '' .
hypotonia 1:28,36. 132; 137,143 [:;! 42-43,
i .
1
44;<46, 66, 8.Y. 85, 94)
t' hypovolaeli'lia [u:69]
.
" "
1
hypov<;>laemic shock:(li:211 ' :' ,
hypoXia ;,41!.so t:l:4-9. 'ii5;:126.'13 11 -
hypsarrhythmia [:1:87, 96] '
'
ice packs 1:20~21
r ..
ichthyOsis (steroidsulphatase defidency)
'1:55, 57, 65
..

.
.
ichthyosis \'Uigaris 1:62. 65'
,.
icteric sclerae (n: t 07]" .
]l:nmigrant status 1:24, 31 . ' - . ' '
immune cell rrioli!it}IJ:168'.~!; ,., ' :: ,
imm'l.me reactive 1trypsi'nogent-l: 127; 13-7
immdbe system dysfunction ~:4. i6 '
immunodeficien6vJ:I62; 173, 175: 18~181'
fi1: 13, 92. 1
~
111
innrlilno81obU1i~ '
.
.. ~~
A complexes [li: 134. 141 f''', .. '
A deliciencv !:109. 117, 167.~174 (n:63]'< ,t
Anephropathy(n:57. 63, 70h7l ' ,:-, , ' ""
E (11:12Q-1'21,"125, 126, 131)' ~-t', ~--:
Gt:156 '
~-- I '
G antibodies (r:: lOBJ
. " " ''r
Mantibodies [H: 1OSJ
' , , - ,_
synthesls..(n:90}' _:
, therapy i: 14--' 15,- I 78
immunology 1:167-173, I 74-181 1 ' i!
immunosuppression inpeplirotic S)~drorrie'
(11:69)
\ ,.f _,. ' ' e-"1
immunosuppresSive therapy 1:32 [!I: IS) +
impaired sperm motility (11: 127]
"L
impetigo [ir: 12]
impulsiveness 1:3 I.
~
inborn errors of metabolism [il:2L 26; 28,'31,
fo"

oir'

34jr).

i~ .,:J

, .:

' \

?O,

<r_.",r.~

incoritinentia P.iW:nenti 1:55,


61. 69.:.70; "~
infantile
l . _,,, -. . .: ..
acropustulosis 1:60;'?0
' :
hypertrophic pyloriC stenosis Io99
.1
spasms 1:28."35 [1t:87, 96)
< :
., -"' 1"
infectious diseases [li: 1-10, 1r:-2or z,, ~ l
iryfect~ousdiseases 1:179 ;''" .. I''

: infecuve'endocardilis (1i: f9. 201


inferior petrqsal sinus sampling 1:84'
infertilityJ:84, 85.86-88 [1i:ll6,127. IJO,
137]
.. .
' . '

'

innammatorv bowel disease


scle!rosing
. cholimgitis . . .
'"
'
inflammatorY indices in Crohn's disease .,,..
I: II I. l,l5
.
.
inflammatorY markers 1: I I 2' . "
inguinal hernia 1:93:94/97. 8;'99, 107
inhibin 1:71

innate immune System 1: I 69. '169.' Iii
'
innominatcarterj f13 .
J'

insomnia [li:83; 93) t


.;;,
instrumental deHvi;:ry_ {li:SO].
insulin ~:76. 19: 91 1(u:33] __ "
hyperinsulinismi: 79, _91 11i:2 1. 29; 33]
insulin~like
factors i~'13; 83. 90; 177
poi5911ing (11:29]

,
' ' '
' .,:
release (i!:l25J
resistance 1:86, 88
tolerance tesn:78, s2;9o '
.
. intellectual impairmenli:Z5, 34[i1:78)see'
also mentatde~eJopment~ {;--
intelligencetests (IQ) (1i:l47, 151,157, 163]
inten~ive chemoth~tapy-induced .' '
.
'i!hmunoddticilncy 1:1 70

intention' to treat (clinical studies) (n:165I'".


intercostal recession [11:67)

1
interferon-a therapy 1:179.
''
interlf:ukini ~t:84 [ti:4, 16]'
~"
interletlkin-12 1:179. :
' '
interle_ukin-6jnA, 16) . ; : '
international normalised ratio (lNRJ i:l49.
153.'158. 162-163[11:271 "'

interrqpted aortic arch 1:20 ''


inter-sex 1~94
)
''
interstitial
-
"
.nephritis.~II:12J ~"
................ ______ _
pneumonia '{11: 116)

intracranial l:; ,; ' "- ... ,'-''


bleeding 1:158 [u:48, 101, 109}
pressure (11:76] ' . : 1"
intrahepatic biliary hypoplasia[u:38,48]
intra-uterine growth 1:73, 83 ._
-'
'restrictio~ -[11:39;:49.:so.:54)
( '
infection [II: 107]

'
-' '
intravenous
antibiotic therapy 1:56, 66[11: I ()OJ "
cannulation 1:56 !

Ouid therapv.r67 ' r '
I 1.1.
. ,
gamma globulin (!VJG) therapy' 1: I 4-15
line llushil'lg 1:46,'52
:,
'
.,
..
pyelography 1~: 1iJ
intussusception 1:93,97,98-99. 105, 106, 'I''
112,122(11:141) , ' .
'
,,'

growth

_j__ -~ --~
?'. : r~ ', -. '
~

'

...

&

[.

:'

(
~-.

in-vitro fertilisation (IVF) 1:88 ,. _


kidney glucorinidation 1:40 .
Klebsiella meningitis .1:178 '-'", ; . ,
iodine therapy (radioactive) 1:83 '
KJeihauer-setke test (n:5'3J ,;
' '"'
ion channe_lopa!hi~s [n:_S2, 91-92] :. ;, ,
Klinefelter s)'lldromi:l:/1, 82; 84, 85.~89, 130.'
10n transport (11:72, 73] .._ . . , . .
142
' ,;:,
ipecacuanha 1.SI
'
' ' '' ,,.
Klippel-Trenaunay-Weber syndrome t:64 .
tpsilateral anhydrosis'[1t:99.'106)
-~ '
Koplil<spots(u:ll]
,, "
.,.,,., . ,.
iridocyclitis t: I 76 , ' . . ,L
iris [n:I06) I ,......
'

Kruskal-\\'altis
t<Est[u:l50,
162,
1641
' -1
*"'
,,
Kugelberg-Welander disease )u:85; 941-"
iron
. ,.
" ' '
' "kyphosis 1:84
absorption 1:119. 1.24 ..
laboratory imestigations 1:1 72
chelation ,a:SO : .
. ., ....
labour (prolonged) 1:55, 65 ' 1
containing pigme11ts (11:97), ""
! , .
lactasetll9
''

.. '.'
ingestion 1:41, 41 .. 49, 49-50
.
laCtate dehydrogenase (u: 134, 136, 140, 143)''
__ Jevelsl:114. 155~eealso'!naemi(, ~-;~:.:
lactate levels [u:23, 24, 25, 32, 33, 68)see
1rrad1ated b.lood transfusio!"' 1: 1,4
' .
islet cell antibodies 1:80

also hyperlactataemia
'
lactose-free diet 1:125 (11:34)'
isohaemagglutinins 1:156, IS
"
isoleucine (u:34J . ,,
..
lactose hydrolysis 1: II 0
Lamberts-Eaton myasthaeriia [i1:82)
isoniazid ~:41~ 49~ 111..171. 179 [n:s. 17,-J'rs;
lamivudine 1:179
- '
125, 130)
'" :
'.
lamotrigine (11:83, 93,:96]
'
';
isopre.nann,e t:4
..
isotretinoin therapy t:65 .
Landau-Kleffner syndrome 1:34. 36 [11:87;' '96-97) - 1>~ "
;
itraconazole (11:126, 131)
'
Ixodes (tick) 1:63, 175 (n: 12, 19, 94J
'Langerhans
histiocytosis 1:53, 62 language development 1:26,28, 34, 36(11:961
Janeway lesions [u:-191

lanugo 1:25, 33'
jaundice 1:51, 118, 1 i2, 173, 180
laser therapy 1:64 (n:53, I 1"0]'' ' 44-45t52. 55).
.

Lasser'spaster:57,67

Jervell,-Lange-Nielsen syndrome t:-i


joint
Laurence.::Moon-Biedi s}r!:tdrcime 1:85-(u:99.
106)
.
effusion [u: 138)
hyper mobility 1:8S (u:29, 144)'
laxative abuse [11:77(
laxity t: 142
lead poisoningt:SO, 161
pain [n:7, 9. 62, 136, 138, 144)
learning disorders r:35, 77, 88; 127, 131: 137,
' ' 141 [11:93, 97)
stiffness [11:291
Leber's amaurosis (hereditary optic
Joubert syndrQme [II: i oo; l07].
jugular venous pressure (II; 71) ;
neuropathyl (u:IOO, 101. IOFI08]
Jell atrial isomerism !:3 jumping ability t:26, 30. 34
lefHo-right shunt 1:11. 12, 17
juvenile dermatomyositis [II: 134, 136, 1~().
143)
.
'
left ventricular enlargement [u: 74]
legal
Kalman syndrome 1:75, 86, 90
Kaolin. [n:2]
consent 1:97. I 06
Kartagener ~s syndrome see situs imertus
protection 1;36-37
Kasabach-Merritt syndrome 1:64
recognition 1:95

Kawasaki disease 1:3, \4-15, 53. 62, 168,


Lennox-Gastaut syndrome (u:87, 961
leris
176--177(11:134, 139, 141)
deposits (u: 112] '
_
Kearns-Sa.).'Te syndrome (u:IOI, 109]
keratinisation disorders 1:62
dislocation 1:8S, 131, 142
k.eratitis [n: 109J
opacity [ttl.09) see also cataracts
.leptin le\els 1: 74. 85
keratoconus [11: 108) : .
~A- leptospirosis [u:l4]
kernicterus ~:45, 48, 51 (11:46. 491
ketoacidosis [11:127)

t:esch-Nyhan syndrome r:l27[u:21, 26, 28,


'
ketoconazole 1: 178-1
34)
ketones !11:21; 23, 24, 25, 29, 33, ~~
lethargy [11:8. 27. 67, 117, 129]
ketosis (u:321

leucine (u:34]
leucocytosis [II: 141)
kidney abnormalities 1:6, 13, 16 [r::41. 54, 63,
771
.
Jeuco!Tlalacia. periventricular.[11:39, 49)

cell

<

i9

ISS

'

'

-
..

'

ESSENTIAL QUESTIONS F.8R MRCPCH

leukaemia r:57, 68, 78, 149, 1'ss. ,59


leukocoria (white pupil) [n: 10 I: 102: W91
leukocyte adhesio[l d~ficiel)cy lYRe 1: .1. I 73,
18G-181
"
leukopaenia 1:164 [II: lA I 1
,
leukopheresis 1:1 76
leukotriene. receptor antagonist therapy
[11:121)
'
-\
' ' ..
"
levodopa therapy [n:971
, .,.,, ~
Leydig cells 1:H; 78, 80, 81,85, 69. 90
lichen planus 1:61 t
,:
likelihood ratio (statistics) [ri:I59] ,
limb
.
. . ','
ataxia [11:89] :

dystonia [n:97)
flaccidity [11:961
pain 1:27, 154[11:7, 9, 1361.
paralysis (II: I I 8) .

rigidity (11:88, 971


weakness [u:89, 143)
limp [II: 136] , , .,
lip anomoly [n:S41 .
Lisch nodulesJ:J43, 144(n:89!
.
listeriosis !Usteria rnonocywgenes infection)
1:178 [II: 15, 18, 38, 47, 551
lithium (maternal) 1: II (11:71
liver
displacement [II: I 211
enzymes (11:142)
function tests [11:18, 19, 125, 130)
glucorinidation r:45, 50, 51
metabolism r:40, 41, 43, 48, 50
pharmacokinetics 1:45, 51
storage [n:29)

live vaccines 1:14, 24, 32


loin pain i:112(11:76]
.
long-acting beta-agonist (LABA) therapy
[11;121)-- ' -- . "
'
'
long-chain hydroxy acyi~CoA dehydrogenase
(LCHADl deficiency [n:26, J.;]
loperamide, (1:2)

low birth weight 1:12 82(11:50; 741 .


developmental delay 1:28, 3.5-36
_
disease associations 1:113 [n: 101. I 091
dysmorpholoiD' [n:44)
,
very low birth weight(n:40, 56)
low density lipoproteins (LDLJ (:::69)
lower airway obstruction [i1: 11.8. 129)
Lowe syndrome (11:66, 70, 18. 79. 106] ... ,
lumbar lordosis 1:139

lumbar puncture [u:6. 11: 84, 9.;.)


~lung

alveolar development [11:124]


capacity [u: 130, 130) , . , .
compliance [II: 115, 126] .
development [11:114, 124)

186

diffusion capacity (u: j 30]


forced expiratory volume [n: 123)
...
function in clinical studies (II: 158-1591
growth (fetal) [11:461

hyperinflation [11:1271
1 '
hyper-luscency [II: ti'l f
peak expiratory flow [II: 130j
H
pressure-volume curve [II: 126)
residual volume [11:130]
shadows [n:l21, 131)
lungs. two morphological rightr:3
luteinising hormone (LH) 1:11; 78, 80, 81. 82,
89.90
.
'
Lyme disease {BOrrelia burgdorjeri. infection)
1:9,54,63 167;175 [11:12, 19-20,85,90,
94)
lymphadenopathy tlymph mide enlargement)
1:14.35.65,176,179[11:11,130, 139,.
141. 1431
childhood malignancy 1: 154, 164
differential diagnosis 1:27, 170 [1i:4, 85,
118. 136]
'. :; .
features (II: 15)
'

rheumatologic d-isease' [11: 1381


lymph node excision 1:170, 17':1
lymphoblasts 1: 158, 159
'
lymphocytesl:9, ll9, 140;173, 174,180
(11:101
..
:
. .
lymphocytic leukaemia 1:158
lymphocytosis (11:8! 1
lymphoedema 1:75, 87
lymphoma 1:68,.149, 154, 159, 164 [11:4,
15, 143]
.
lymphopaenia [n:20, 134, 141)
lymphoproliferative disease 1:174,175
lyonisation 1:142 [u:281
.
lysine loss [11:30]

.lyso5omal defects (n:30, '31; 78]
macrolide antibiotics [11: 1291
macrophages 1: 174; 177
macrosomia 1:91 [11:50)
macular inflammation [II: 102, II OJ
macular oedema (11:74)
'
magnesium metabolism t:4'
magnetic re5onarkeimagirig (MRI) 1:34 [u:88,
.
.
89, 90]
majority (age of} 1:95, 101, 102

malabsorption in cystic fibrosis (n: 130)


malabsorption syndrome 1:121
malaise 1:9. 10, 19~ us: 1291
malaria (11:3. 10, 14, 20]
malathion 1:68. 178
male T\lrrier syndrome 1: 17
maltase 1:119
maltose i: 119
mannose-binding le~tin 1: 17 7

...

Mann-Whitney U-test (statisti::S),(n:I64J


1

manometry[11:84, 941 .
maple syrup urine disease (MSUD) [11:21, 26,
28, 34, 37, 46)
. '.
.
Marcus-Gunn jaw winking syndrome [n:101,
109]
.
marfanoid habitus [11:291

Mar fan syndrome 1:8S, '142 [u~H}6J .


differential diagnosis 1:31 [li: .1071
disease. associations t73. 89, 103 [u:99)

features 1: I 30, I 41 [11:22. 29]


masculinisation 1: 140

,.
mastcellst148, 157[n:I09]
mastoiditis f1i!: 1 1J1
.

. " ~
matched pairs analysis (clinical studies) ')..
[11:146. 152, 155. 164!. :
* .;"!.t-;;..
maternal,..
.- .'"' .
~~~- . 1 -~ ,~
a?e 1:139
.
!'hsea~.l:l69 ,, r:.~ , . . '. ;.
.
drug use 1: I [n: 7, 11. 39, '49, 50] see ~/so

pregnancy
. " : . ., ,, , ~ ~
respo~sibility 1:95, . "\ , . , , _,, , , . ,
smokmg [1!,50] . _ ~ 1 1.
,
., .
McCune-AJibright syn~rom~" 71.~;~:. 80, 86,_
87" 89, 130. 142 ~; : . ' .
{7 ,,
mDNA depletion syndrome (n:31] ... , ,
mean ell volume (Mev') 1:124. i4 7, t's2; ''
mean'/statistical) [n:i.46J.147; 1561. ' .
measles (rubella) see a/So Mi-n{ vaccine i:ll
'
.
'" .
.
'1 '
[u:3. I 5, 16, 48. 49) ' . i...
. ,
differential diagnosis [n:'l8]

disease associations [n: 114. I 24]


features [n:l, 4. 7, II)
notifiability [II: 14]
. ;
Meckel-Gruber syndrome [n:52J
Meckel's diverticulum 1:93, 93. 97, 98, 105,
110, 118
. .
meconium'
aspiration syndrome [n:37,47)
ileus 1:122. t.=ntn:t27, 130]
umbilical[n:52]

.
median (statistical) [11:147, 150,156, i'62]
medium chain aceyi-CoAdehydrogenase
(MCAD) deficiency 1:28:-29, 88.[n:34]
medulla [11:96] .._ .
. .
. ,.
megalocephaly 1:84
Meibomian cyst (chalazion) [u: I 04; II 1]
meniory Joss [11:94]
menarche 1:8!
meningism [n:l I]
meningitis'(u:l3,'!4, 1JO)
aseptic 1:170, 175
aetiology 1: i is. 178 [n:3, 4, IS, 18, 19,
81, 92,)
.
neonatal1: I 70, 178
therapy 1:1 10. 178 (u:4, IS, 17).

tuberculous [n:l5; 90;d5J'' ' '. ; J'r'


meningococcaemia [lr:6,"1. 7) :
meningococcal. meningitis [li:3L~ '
meningoencephalitis 1: 11 1; 120; 180
,.
mental de\'elopment 1:23, 24, 26, 30,31
diarrhoea 1:130
'
'
' '
mental de\elopment disorders 1:25, 3.;, 62,
. 84, 141 (11:28, 33)
*" ' . . ''
differential diagnosis +1:! 30 '' ' "

metabolic acidosis 1116, 50 [11:35, 601


causes'of{u:21f
'
'
'
congenital J:I, U .
.
congenital heart disease 1:9, 20
drug~induced [ai:77]'
. .
neonatal I: I, II (u:1f( :hj
.r '
renal tub.ular [i,!25, 28, 33,.39; 58, 7o/78)
therapy [11:72, 73] "
.

' ,,.
metabolic"alkalo'sis 1:42, 'so [11:51. 59. 78t
metaboiic meclidne {n:21-:27. 28~35]'
metachrortiatic leukodystrophy (11:83]
metachronous. hernia t99
" '"
methacholine"Chlillenge (u:h3j
}' . .
methici II in-resistant Staphy/ocdccus aureus
(MRSA) (n:SS] . . .
..
_ . ..
,
methotrexate thera'py 1:57;66; ISO; 16<r
[,i:l33,135,,139.H2J
' -. '
methyldopa toxicit.(1~49 ',' . ' ....
metronidazole therapy r:t 12: 118,122
MHC aT)tigen compatibility 1_: 168; 176
micelle formation 1: I i 0' ' " '
microcephaly [u:is. 33, '44.s4. 91J.
microdeletion detection 1:.140
microphthalmia iu:54J

micturating.cystourethrogram (u:J 7.' 73, i4,


76] ..
mid-brain [11:95]

middl~ ce:rebral artery occlusion 'lu:84. 94]


middle-ear effusion t25, 34

mid-facial hypoplasia (Ji:54]


.
..
midgut malrotation (volvulus) 1:96,97. 104,
105, 106
.
mineralocorticoids 1:90
miscarriage [u:l53, !65_:167)
mitochondrial disorders t3 [n:78, 106. JOi;
109]
.. . . '
'
.
. ..
mitochondrial DNA (u:22, 30. 31}
mitral valve disease 1:15, 85 [ii: 72]
mixed connective tissue disease [II: 135, 142]
MMR vaccine t:23, 24, 30, 32, I 68, 175

Mobiu~ syndrome [11:38, 48, 94, 1o 1: 109]


modified Glenn operation 1: 13
molluscum contagiosum (mollusca
contagiosa) 1:22. 54; 63 (II: 121
mbnozygosity [u:S I}
Morgagni [u:47l
Moro reflex t:23, 30 [11:39.'50]

187

QUESTIONS FOR MRCPCH

.
.~;

.,

morphine metaboli;;m 1:49


Morquio syndrome [11:26, 35)
mosaicism 1:130, 140, 142
motor coordination.' Joss of [11:951
motor deficits (11:82, 921
motor nenie velocity (11:81. 89)
mouth
1
blisters 1:69
~.
inflammation [11:141) .
.. t
ulcers 1:57, 67; 173 (11:134, 140, 142)
movement disorders (jerkiness) J:2s:3s'.
mtDNA depletion syndrome (1r:23)
'
mtDNA point mutation (11:107]
1
mucociliaryele\:ator(n:127)
.
,. "
mucolytic drug therapy ju:124] -~
.
muc0 polysaccharidoses 1:136 (u:3S, 109, 124]
mucositis 1:14
'
'
~
mullerian inhibition factor [n:37, 47) '!
multicystic dysplastic; lddney (u:4t. 52] .. : :
multiphasic disseminated encephalomyelitis
(MDEl\t) [11:901
'
multiple sclerosis (MS) (n:9o-91] .
mumps {11: 14]

. ,
muscle
atr0 phy twasting) .[n:69. 941 '
..
pain [11:136]
, .
pain (myalgia) 1:27,35 (11:19, 32]
weakness [11:63, 85, 89, 94, 140, 143],
muscular d)'Strophy 1:15 (11:82, 931
.musculoskeletal pain (11:85] :
myasthenia [11:84, 93-94, 95)
myasthenia gra\is [n:93~ 1431
.
Myco!Jaccerium infection (atypical) 1:22. 179
[11:4, 12, 13]
mycophosphorylase deficiency (GSD VII)
(11:24, 32]
'

Mycoplasma pneumoniae infection 1:175 (u: 7,


18,118,129]
'
myelinated nerve fibres [II: 109]
myeloperoxidi!Sf!.t:.1.?7 .. __ ~ "'"~
myocarditis 1:168, 174
myopathy 1:33[u:48. 91)
myopia (II: I 08]
myositis (viral} (n:l43]
myotonia congenita (11:82]
.
myotonic dystrophy 1:121. 132, 134, 137, 139,
144 [11:46, 48, 95]
'
myxo\'irus [n:l]
naevi 1:54-55, 64, 87 [11:93}
nail-bed bogginess (11:123) .,
.
nail changes 1:54.55,61, 63, 65, 87[11:123,
134,136,140: 143]
nappy rash 1:53, 62, 65 c
narrow complex tachycardia 1:1 0, 20
nasal
anomolies [11:54) .

,.,

I
I

188

brushing [n: 127)


congestion [11:86, 123)
granulomata (11: 141]
'polyps [II: 114, 116, 1731
regurgitation 1: 140
ulceration (11: 123)
nasolacrimal duct obstruction (n: Ill]
nasopharyngeal aspirate (NPA) (II: I 08, 131 f"'
National Institute for Clinical Excellence
(NICE) t82'
- ~.
nausea Jn:86, 135: 142)
neck stiffness [u: 19)
neck webbing lt1:54)
,...,., .
Neisseria gonorrhoeae [11:102. I io)
Neisseria meningilidis tn:5. 4. 16]
nelfinavir treatment 1:,177
'
nemalin myopathy 1:132
neonatal alloimmune thrombocytopaenia'
(NAIT) 1:149. 157
"
neonatal lupus erythematosus 1:57, 59.
68,69
v
'
'
neonatal shunt 1:1 I
E
neonatology (n:37-45, 4~55] __..
nephritic syndrome (n:69. 76#. 78)
~.~
nephntisrn:57. 69, 72]''
:,
.
'.
nephrocalcinosis [n:58, 60, 71, 73-74]
nephrogenesis [n:58, 71]
'".
nephrogenic diabetes insipidus ~:74; 86 [u:391
nephrolithiasis [11:70, 76) ' .

nephrology (n:57-68, 69..:80]


nephronophthisis [11:59, 71, 74, 77)
nephropathy (reflux) (u:63, 74]
nephrotic syndrome r: 164 [11:57, 69. 76, 78]
nephrotoxicity [11:59)
nervous system cancer 1:143, 144, 1so, 158,
I~
.
.
Netherton S\'Tidrome t62
neuroblastoma 1:154,165 (u:l43]
neurodegenerative disorders [n:88]'

. ~el!r29~velopm<;pt~l al:i11orm~liti~ [n:S~)


neurofibromatosis type 1: I, I 28, 138, 143;
144, 154. 164 [11:81, 89, 99. 106)
neurofibromatosis type 1:2, 138
neurogenic bladder 1: I 03
neurological development ~:30
neurology [n:81-88, 89-971
neuroma (acoustic). r: I 38
neuromu5CUlar diseases [n:82]
neuronal ceroid lipfuscinosis [11:106]
neuropathy {u:5, I 7, 19, 89. 125]
drug-induced 1: I 50, 160 [11:1 7, 125. 130]
optic [11:101, 107)
peripheral!: 150, 160 [11: 17, 83, 91. 115,
125. 130]
.
polyneuropathy [n:94, 1291
sensory [!::92]

INDEX

neurotoxicity 1: ISO. 160


, ,,
.
1
neutropaenia 1:1 12. 122. 156, 16 I. 189 [11:23,
32]

..
.
-

neutroph~lst:9, .153.,_157~ 162~


18Q-181 (u:JO]'

'...

i73.'i.17 .. c.,,
.

nicotinic acid 1:118. . .


'"
Niemannc-Pick type C (ir~34J ' ''
night blindness [11:99. 106]
'
.
nipples (wide space a) r:87
nitric oxide 1:167, 174 [u: 126: J271
nitroblue tetrazoliurri test r: 180'
NMDA receptor antagonists (u:29J
_ :
nocturia r:72, 83
' - ,f ' . .
1
nonHodgkin's lymphoma r: 154.. 164
.
non-ketotic hyperglycinaemra (NKH) [:r:22,
.
~ ', :; ' :
28, 29.-37. 46]
non-penetrance (genetiC).r: 139 . .:
non-steroidal a'nti-inpamma'q.>ry drugs
(NSAIDs) [II: 1.4} ]."' .I . ,,
,. ~. "'
nonctuberculous mycobacterial infection
'
. .

' 1: I 70 119 ' 1


Noonan ~yndrome',:s,'i 7;u&;'t34, 135, 139.
143. 144, 145 (11:54]
.:
.
. .. .
normal distributip~ .cstatist,ics) {a 14 7. i 4,8, .,
155/156.157,160, 164), .
.. .
Norwalk virus, [11:2) , ; . ,:
.~.
Norwood procedure r: 13: , ,
notHiable.diseases (11:3: 14, 1'1-SJ
nuchal rdid 1:1; 139 .
:
.
nuclear DNA [11:311
nuclear medicine 'milk' scan r:l23 ..
null hypothesis [11:147. 149. J60. 161]
number needed to treat (clinical studies)
[11:149. 161]
. ..
.
nutritionand gastroenierology r:I09-116,
117~126
. .
.
nystagmus [11:107, U}J
obesity !:35--36, 73, 74, 83, 84,
86. 113
defin.ition 1:123

observational studies {II: 146, 155. 158- is9.


160}'
'
..
.

ss.

.obstructive uropathy [11:46,58, 711


octanotys,arnitine (C,8] lev~l [11:34]
oculomotor (Ill) nerve [11: 109]
odanset'ron therapy [II: 142)
oedema (swelling) 1:14, 176 [u:57. 69, 102,
128]
.
'
differential diagnosis [II: 136. 141)
disease associations r:33
in renal disease (u:65. 11:66]
oesophageai
atresia_J:96 [n:48]
fistula r:96. 104
reflux 1:104
oesophagitisi:l2l, 124
oesophogram ]11: 129!

oestradiol production 1:81


oestrogen production i:81, 84. 90,
oestrogen (theraPf!utic) 1:88, 100, 105,
.
.
oligoclonal bands.(n:90) 1
oligohydramnios (maternal) [11:741
.,
oligomenorrh-ea ,jj ' '
.
. .
1
oligonucleotide primers 1: l 76
..
oliguria [11:.58. 69. 71, 73, 74, 78],
' ,_
Omenn syndrome r:55, 65
,.
. , _,,
oncholysis 1:64
. .
,
oncology and haematology'I:H7-IS4, ,;,
'
c

'
'
.. :
. 155--165
ophthalmia ne.onatorum {u: 141 . .
ophthalmology (h:99-I05."!o6-1 L~J,,

ophthalmoplegia [11:103, 1101 1 .


opiates r:49 [u:4 7, 49. 50]
> ><t, , l
optic
.
.
,.
<l'
atroohv (u:89]'
'' '
chi as~ pressure [II: II OJ . : ' I" . I
'
gliomaJ:I38,143,144. 15(164[lr:89]"
nerve damage [11:94. 1061. . ~
nery('! hypoplasia 1:~1'.'88 'r:~ .,
~
nerve lesion [II: 102, ,I IOJ
,,, ..
. neuritis (11:911 .
'1
.
. , ..
oral challenge tes,!(food) irE 125] . , '
oral contraceptive pillt:54, 63, I 75 (u: 1221
oral glucose tolerance iest i;82
' '
orbital cellulitis [11:3,13, ioo: 108, 110(
orbit pain [II 95)
organomeg~Jy (11:34] ,
. ,. ,.
ornithine loss [11:30] .
. .
, ,
ornithine transcarbamylase deficiency. [n:.::i:,
. 22. 28. 29]

. '
orogenital ulceration 1: j 20
oropharyngeal inflammation 1:176
orotic acid [11:271 .

orthostatic pro-teinuria tu:70J .


Oslers nodes [11: I 9]

osmolality (11:58, 67, 68, 71. 751


osmotic fragility test 1:155
osteoarthropathy'(hypertrophic) (II: liS]
osteogenesis imperfecta [n: I 00, 107] .
osteomvelitis r:9, 171, 179(11;20]
osteoporosis r:33 [11:291
osteosarcoma 1:68 rn:20].
otitis media 1:73, 84 [u: I 201
ovarian cancer r:134, 144
ovarian dysfunction t:-75, 82, 85. 86, 87, 88
overdue delivery [11:41]
overwhelming bacterial sepsis [u: 19]
oxandrolone therapy 1:82
oxygen

absorption (skin) r:53. 61


index [ir:l.:;7, 156!
saturation (ll:8. 9. 24, 64. 67, 117, 121, 128]
therapy fu:73-74, 127]

"ESSENTil,!.. QUESTIONS FOR !:1RCPCH

l
I

I
I!

oxyhaemoglobin [u:49, 90, 113, 122]


p24 antigen unatern.iil) [u:49]
pachyonychia congenita t:53, 61
pacing. in-dwelling transvenous 1:16

packed red blood cell transfusion (u:42) .


pain scale [:1:86]
''

palate
.' 1 '
,. '
abnormalities 1:3. I 4, 85, 87 [11:88] ' ' '
high arched 1:85, 87 '
,;
.,
weakness (II: 140)
''
pa!ivizumab xantiRS\'l [u: 1271
>'
pallor !paleness) t27, 35. 152, 154 '(u:42. 531
paJmo~lantar r.<!ratoderm~ 1:64
,, I
1

pancreatic
'
fibrosis [1:: 12 i]
"
insufficiency 1:114. 122. 124 (11:127. do]
pancreatitis t: 112. 122 [n:32J
pancreozymin cholecystokinin 1: I 19
pan-cytopaenia.1: 154 [n:32]
.
pan-uveitis [n! 106]
papillo-oedema (n:63. 76. 84, 94, 95]
paracetamol t:40, 44; 48, 51, 57, 67, 161
parasitic diseases 1:148, 157
parathyroid gland t:14[n:54]
parathyroid hormone (PTH) t:84
parathyroidism (ti:74J
:; '
paratyphoid fever (II: 141
parental
'
'
consent 1; I 02. I 06
responsibili~ 1:96, I 0 I, 102, I 06
Parkinsons disease tu:95J
parotitis 1:33
parvovirus Bl9 (erythema infectiosum) r:63
(u:2, 12. 16. 18]
parvovirus infection [u: 143]
patent ductus venosus (n:22, 30]
patent omphalomesenteric duct [u:521
patent urachus (n:52J
paternal age 1:139
paternal responsibility 1: I 0 I
patient
confidentiality 1: I 01
consent t:95 (i!o 1541
peak bone mass 1:73,84
peak expiratory flow rate [u:119. 129, 130]
peanutallergyl:114,124 [tt:115.ll7,
125, 128]
pectus exca\arum 1:95, 103
pelviureteric junction obstruction [u:62. 76J
pemphigus foliaceus 1:69
penetrance ~genetic) Jol39
penicillin tl-.eraov 1:41. 46. 48, 49. 52. 63
(II: 118. 129. -142]
.
pentamidine therapy (u:131]
peptic ulceration 1:121
perianal warts ::63

190'

....

pericarditis (II: 141]


periodic paralysis [11:92)
periodontitis 1:32. 173
, .
perioral pigmentation diSorder~ 1:117
peritonitis 1:99
Perlman syndrome 1.:16.4
,.
permethrin treatment 1:178
peroxisomes [11:23, 32, 34]
persistent arterial duct 1:1.' 1 1. 12. 19
persistent hyperplastic primary vitreous
(11:101}
.
Perthes disease(n:1J~I37, 144] .
pertussis infection [11:3, 114, 116: 124]
petechiae 1: 154
"
_
:
1
Peutz-Jeugher syndrome 1: 10( 117.
phagocytes 1: IS 7
..
pharmacology and toxicology r:39-46. 47-52
phenobarbitone 1:118 (11:48)
phenylalanine [11:22. 30, 331
.. ;
phenylketonuria (PKU) 1:! [n'n: 25; 30. 33]
see .also ketqne production
'" .
phenyllactate [il:27, 35]
,. .
'

phenytoin [11:48]
.drug monitoring 1:4 (45. 49,
. maternal1:l, 11
,
_,:
metabolism t:4 1. 43 . 45; 50, 51
neonatal pharmocokinetics 1::Si 'protein binding tAO, 47. 49. 50,51
side effects 1:41. 49, 51, 52, I (.9, i .78 [i1:83,
931
.
therapeutic' range 1:43.
pH (gastrointestinal) t:39. 4 7. 114. 123
Philadelphia chromosome 1: 150, 159phosphate 1:159-160 [11:50, 63, 66. 76]
crystals 1: 150
-1
hyperphosphataemia {11:50]
hypophosphataemia !:80, 84
hypophosphatasia [II: 107]
therapy 1:84 .
.
phosphofructokinase deficiency (GSD \')
[11:24. 32]
phospholipids [11:124]
photophobia 1:69 [u:66, 78, 109; 110!
photopsias [11:86]
photosensitive rash 1:57, 59, 68, 69
photosen'siti\'ity 1:54.59,62, 68, 69. 144. 145
[11:134, 140]
phototherapy (reaction to) 1:68
physical development 1:23,24,26. 30. 31. 32,
34
physiotherapy [II: 124. 127]
PI BIDS 1:62
Pierre Robin sequence [11: t14, 1241

pigmentary retinopathy (11:34, 108, 109) see


also retinitis pigmentosa
pilocyUc astrocytoma [11:89]

si.

J
INDEX

pinna defect [11:44)


pointing skills 1:24, 31, 32
pipecolic acid oxidation (11:32)
point mutations (11:30)
poisoning [11:14)
pituitary
dsyfunction 1:78,79. 81, 88,90
polio 1:30, 175 [n:2)
1,
glucose metabolism 1:79,90 '
poliomyelitis[n:l4) .
,; . :

polyarteritis nodosa [n: 135, 141. 1421


hormones 1:71, 79, 80, 81, 86.90 (1!:1091
polycystic
" , . .
. ,
hormone testing 1:78-79,90
kidney disease [11:37, 46, 52, 59, 721.
hyperplasia 1:8!, 88
, ._,.
ovaries 1:86 [11:961. .

hypoplasia 1:8!
.,.
structure 1:8!, 83
polydactyly 1:8S.[u:52, 54)
pityriasis rubra pilaris 1:55, 64
polydypsia [11:66. 77J
placenta chorionicity (11:40,51) .
polyhydramnios 1:104. 127, 137 [11:37. 38. 42,
placental
48,61,66)
. '"
. '
'
''
polymerase chain reaction (PCR) r:3, 168,
disease transmission (11:49)
insufficiency (n:50J
I 69, 172. I 76, 177," 178, 180 [II: 17f
sulphatase deficiency 1:6S
polyostotic fibrous dysplasia 1:8 7
plague (II: 141
. . .. 0
....
polysplenia'l: 13 , .
plaque psoriasis 1:57.63, 66-67, ;
polyuria 1:72, 83 [u:50, 66, 71. 74. 77]
Pompe disease 1:8: 19
.
plasma disappearance cyrve (11:751
.
'pompholy,( 1:59. 69
plasmalogen synthesis by peroxisornes (11:32]
platelet

~ .-,,.
pons [11:95. 96]
activating-factor (11:1)51
porphyria 1:134 (11:28, 83] ,; .;;... , , ,
aggregating factor (n: 126)
port wine stains t64 . .
aggregation 1:167. 174
,
positive prediCtive value (statistics) (u: 148,"
differentiaLdiagnosis 1:162-163 . ; ,
,
15,3,159. 160,'167] :. '' '. .
posterior urethral valves (u:52, 60, 74]
haema!ological disease !:148. 152. 153. e.;
157, 162-163
ppstetior'uveitis [n:I06l .. , .
..
hyponairaemla (n:67J.
post -streptococcal glomer'onephritis (n:5 7]
potassium chloride (maintenance therapy}.
immunOdeficiency 1:173 ~
.
[11:68)
'
. -., -'
.
immunologic disease 1:175, 178
.
neonatal disease [u:42, 621
deficiency [u:77J .
non-imported UK infectious diseases 1:9.
ion 'exchange [n:72, 731- .,
20)
blood level I: I 59 [n:IOJ see also
renal disease [u:63)
hyperkalaemia; hypolcalaemia ..
rheumatologic disease (II: 138.143. 144] ,
hyponatraemia [11:67-68, 80]
tropicalinfectious diseases (n: 1OJ
levels (urine) [11:66, 67, 68. 78)
transfusion t:149, !57, IS8
liver diSease 1:41

pleulitis (n: 1411


,
renal disease [n:62, 63,--64. 77}
plexiform neurofibroma 1:138 (11:81. -89)
renal syndromes [n:65, 78]'
pneumatic reduction 1:99
.
tubular ,dysfunction [11:66] "
pneumococcal (Pneumococcus) infection
Potter disease [n:46) .
{11:57, 69. 113. 116]
power of study (statistical) [11: 149, 160-161)
Pneumocystis carinii infection (11:4. 121, 129,
poxvirus 1:63
Prader-Willi syndrome 1:85, 138. 140
131).
'
pneumocytes [n:ll3. 115, 126)
developmental delay 1:28,31
differential diagnosis 1:132
pn~umonia 1:32 [n:3, 13, 24) .
chlamydia! (11:55)

disease associations 1:24, 128


genetic factors-1:35-36
interstitial [11: 116. I27J
therapy 1:72. 82
.Mycoplasma 1:175 rll:7, 118, 129)
pneumococcal(ll: 1 13, 116]
precision of estimate (statistics) {11: 14 7, 156,
157, 162]
pneumocystic (n:l21,131)
streptococcal I: I 79(n: 18)
prednisolone 1:32, 64 [u:6. 18, 65, 78. 96. 121]
pneumonitis [n: 108) _

prednisone 1:150, 160


pneumothorax t:73. 85
pre-eclampsia [11:50]
poikiloderma (atrophic pigmentary
pregnancy 1:132, 143, I 75 [n:31, 37, 39, 40,
telangiectaisia) 1:68
41, 49, 51]
~

191

. ESSE.~liAL_ QUESTIONS. FOR MRt:;P.CH

premalignancy 1: I 17
.
premature rupture of membranes (PROM)
neonatal infection (n:8J

patient management (n:41]


' 1
respiratory disorders [11:37.'46] ''
viral infections
I 5, 19.~39)
prematurity (pre-term birth)
birthmarks 1:5~55, 64 , .,.,
'' '
cardiovascular disease 1: II, i2 '
,
endocrine dysfunction r:75, 84, 87 -" 'l
gro\vth disorders 1: t 11
'
..t ""'
~
inguiilal he'rni3 1:97# 'IOi
,, ~ , '~i' ~
neonatal disease (u:37, 38, ~9;4.7}49,
50, 53] . ~l ";: . - .
' '' ' ,.<t
neonatal pharmocokinetics 1:43. so. 5 I
ophthalmologic disease [11:101; 109] r..
renal disease [u:60, 71, 73-74] ~''- ', '"<' '
respiraiory disorders [rr: 126] " '
preputial cforeskln) disorClers {:96.'97. 100, '
104-105,.106
' '
'
'
pre-sacral mass r:95, 103 .
pre-septal cellulitis fu:I08)', .
pre-term gestation see prematunty '" 1
prevalence-'dependent sta'tistics (n:l 59)
'tl
primary cilia!) dyskinesia in: 1 i 6, 120. 12 7,
130] ' ' ' 1 .- ' ' "
'

primary ciliary dysplasia '(n: 113~ I 14;


' ~
124. 129] ' , J ..
,,:, .
primer types (PCRi i: 176 ,
'
PR interval (ECG) (n:7, 8, I 9, 142]
prions (disease-associated) 1:169. 177
probability (p-value) i11: 14 7, 149. I 57, I 58,
16), 164, 165)
;
'
' 'C
probio!ic therapy 1: I 18 .
progressive external ophthalmoplegia
[11:109)
'
proinfiarnmatory cytokim;s (11:4. I 6)

prolactin secrelion 1:11; 81,89


prolonged labour 1:55, 65
''
-prop-feeding (:1:331
.
propionyl-CoA carboxylase [n:32]
propranolol therapy 1:21
proprioception. loss of (11:89]
proptosis (u:89.' 1031
prostaglandin E therapy 1: J 3
prostaglandin synthetase inhibitor 1:74
protamine sulphate therapy .1: I 78
protein
'
absorption 1:1 19.
resistance 1: 169. I 77
binding (drug) !:40, 45, 4 7, 49. 51''
C (u:69]
'
'
.
levels (CSfl (u:81. 90}
levels (urine) see proteinuria
losing enteropathy 1:124 ~
pre-digestion (formula feeds) .:126

in:

restriction (dietary) (lt33)


, ' ' ' . I
to creatinine ratio (n~62, 70, 76, 77)
proteinuria lproteirl.in urine)
chronic (u:69] '

diiTerenlial diagnosis (n:62, 63]


'
features [n:58, 701 \:
'; ' '
persistent [11: 141]
renal syndromes (n:65!
proteus (n:30)
prothrombin time (?l) [11:8, 9,43, 48. 53]
prothrombosis 1:'1 50
Prototopic 1:64
.
proximal renal tubular leak [11:78)
pseudo-Banter syndrome [II: 77!
pseudohermaphridilism [11:4 7]
pseudohypoparathyroidism 1:71, 80, 85
pseudomacroglossia [llf124!
:..
Pseudomonas infection (u: 130]'
: '
pseudoprecocious puberty 1:75'
''"'
psoriasis
arthritic 1:54,63, 67(11:136, 143k
guttate 1:63
1 ~"- '
'
plaque 1:57,63,66, 67'.

Psuedomonas aeruginosa (i1:s; 16]"" .,
psychiilnj l:i3-29, J0.-;39 '

psychological
';
development (normal) i:23, 24. 30. 31 ..~.(
disorders 1:123

.
'
psychomotor slowing [11:83, 93]
''.J
,,
psychosis in StE (n:l4 IJ
'
ptosis[n:84/99, 101,109) '
puberty
boysl:72, 79,81,82,86'
constitutional delay 1:77;82, 88
delayedr:79,85,86,181 (11:60, 12i]
girls 1:75.81, 87, 88

precocious 1:75,86.87, 88, 89


pubettaJ gtoWt.h'1:8 I, 85, 88
pubertal growth spurt t:73,'83
pulmonary -:

''
artery 1:13, 14, 20
atresia 1:2

blood flow disorders 1:12. 16, I 7, 19 '


embolism [1!:57].
fibrosis [11:142] '
I
haemorrhage [n:37, 47, 130]
hypertension 1:5, 12, 17 [1t126]
hypoplasia 1:3{n:37, 38. 46, 52;'74]
plethora 1: II
pressureJ:Il(1t:l26]
'
stenosis 1:3, 5, 14, 144
'
cardiO\'ascular complications 1:6. I I. 16,
17. 19,20

genetic disorders 1:134, 139, 143


neonatal disease [n:54]

5 (11:69)

'

INDEX

tuberculosis (n:ll3, 120,130) ; .. ,. r. ,


vascularresistance1:U: 12,174. ,

vasoconstriction (11:115, ri6]


, ":,
veins, anomolousconnections 1:3, 13, 14
pulse
absent 1:2, 9, 13, 16, 20, 21 , .. . .
boun~ing 1: II

,_ , ,
_, .
brachial1:2; 7, 13, 16, 18
, ~'
'
femoral1:2, 13
_ "l " r.
jugular venous (i1:71)
rate 1: I
1 .. ,. _,,.,.
cardiovascular disease 1:2, 9; 18, 20, 21
infectious diseases (11:7, 10) '
; '
neonatall:l,ll '<"' -~
'
renaldiseasel:57f11:62,(i3,64,68)
pupillary
J' . ' '' ~. . . "': ~ ~
dilatiOni:SO(u:IIO)
,,, .:-
,,_c
fixation (11:86, 96]
' n , ,, ,
.:; .,
responses :(11:85, 95.. 96. 99. 102. 106, 110)
purine S)'l\thesis disorder (11:34],. , .
.,;
purpura (11:62) - :
. , ,1,

~~~o;h~~~~n1 :~8-~(11:~2~

.l,.

':

varicella (chickenpox) (11:12, 16]


vesicular(n:8. 1.9)' .. ,.

viral-1:68
. .
: , .J ", :;,./: ."
white scaling 1:57
,, _, . .>
Rasmussen's encephalitis (11:88, 97) ' _
reactive arthritis (II: 144]. ' '' ' '. 'reciprocal translocation (geriei lo_I3S, I '45 1
recogn!tion skills 1,:24, 31, .' . .''
'
rectal prolapse ~:93, 112, 122 (11!1'15;121; i30]
rectum
~ '.,,'' , ,.;, ' :r., -.- ~
bleeding 1:97. 118
.
dilated 1111
' i'
" 111 v.,
red blood ~ells (erythrocyteS) t:l48.
156-157,162,180(11:42,74');' .. >
redeye(n:99.106,138,141)" _._., ' '".'-~
reference ran~es in clinical siu'dies'illh si.-';
163]
,,,~;-'l''''"'' ,. .
referred pain (11: 144],. ' '. , , : -' ": ~,.,
1 "
reflexes 1:27; 35,,, , ..
,... T"
1
~reflexia (1_1:89; II_SJ , , 1 ,
asymmetncal tome neck .1:30 "<'r '<' .-.,,.

lsi:

:f '.

:;:,"'"r::-- ,.

~~~;~n~~~~~~~~1~:10i, I.I?J'' ; :;~: _.

P waves (ECG) 1:12


.
i ; , .,, .
deep tendon (11:96)
f' ' '"; ' <::"
pyloricstenosisl:55.65,93,94,99 _ ~ , "'
gag'(11>1~~9):~ .,... '""'" ,.::::..~.;~.,
pyrazinamide.therapy1:I79 (11:17,,125; 130].,
grasp (11:39_. :>0, 94t. ., ,_:, .... , ...
pyridoxine therapy.i:179 (11: 1'7] , . ~ ,;__
head-nghtmg 1:30

' .,
pyruvate (11:33]
.. '"' .I

hyper-reflexia (11:63, 74, {6('' 1 ' ' 1 '


pyuria 1:99 .,
'.i. }i<;-:<. , ;!'>
Moro 1:23, 30 (11:39, 50].' ,

~;!
QRS complex 1:8,19
_; 1 .,
parach,ute 1:30. .
,'
, ,,'_1.. ,,.
'QScomplei(J:19.
. ,
p!ipillaiy(11:99,'102,106,IIO].

QT intervall:4, 15
red (11:27, 34]
., ,
rabies (11:14]
tendon (deep) (11:63, 89, 96) . '
race factors (ethnicity) .1:84, 134. 137, I4J,
vestibulo-ocular (11:86, 95, 96)
144, 162
~
refleX tachycardia .(u: 1"25]
t.
l'adioallergoabsorbent assay (RASTJ (II: 12?1.
reflex times in clinka studies .(lr: 148, 158]

reflux nephropathy (n:63, i3. 74, 76; 77)
radiofemoral delay 1:16
radiofrequency ablation 1: I 0. 21
Refsum disease (11:32, I 06)
radioisotope clearance (11:61, 7S)
regression anaiysis (statisticah in: 151, 163]
.. radiotherapy .. .
, . ,. .. " ...........'" .: . __ !ehydration therapy 1:126, 150, 160 (11:2,} 2,
cancer 1;85,151,161.

.
59, 68, 7s:so.l27]seea/so fluid therapy
complications 1:32, 74, 85, 151, I6.1 (II: 18)
relative risk (statistical) (11: 161) ..
randomised clinical studies 111:145. 146, 149,
reliability (statistical) (11: 156-:-157)
150: 154, 155,161, 165]

renal
'
rash see also urticaria
.,
agenesis (1!:46]
arthritis !11:9, 19, 20, 134, 136, i39, ..,
aldosterone level1:86, 90 (11:72]
140, 143]
,.
angiomyolipoma (11:81, 90)
bacteriall:9, (11:'4, 19)
..
angiotensin (u:72]
blueberry muffil) lpetechiae) f11:7, 18)
arterv aneurYsm (II: 142]
'
exanthema (II: 4].
artery stenosis 1:129, 141 (1i:59; 76. 77) '"
itch\ 1:58
bicarbonate (11:58, 71)

biopsy (11:78]
drug-induced (11:93),
macular-papular 1:172 (II: I]
bone resorption (11:74]
malar 1:61 (11:140]
calculi (stones) 1:33 [11:17. 30, 93)
non- blanching (1!:6)
clearance 1:48, 51
'
roseaola 1: ISO , .
cysts i11:4 i. 52, 59, 71, 72] see also
l

'

193

ESSEt.iTIAL QUESTIONS FOR MRCPCH

,,

polycystic kidney diSease


development [n:58, 71] ,.
drug metabolism 1:40, 45. 51
dysplasia [n:59. 74, 77] ,
excretion (sodium) [11:58, 71] '
failure 1: 160, 1.62 [11:60, 90];
''
acute uso'[u:73] ' '
chronic 1:12. 82[11:59, 60, 74; 76, 78)
aialysis i:4o. 48 (11:60, 73]
' . ' !
end-stage (11:71, 74, 771,
intrinsic [11:58. 71]
I ~ :. ., '
pre-renal [11:58.71]
severe 1:40. 41. 48
'
glucorinidation 1:40
hamatoma [u:90}.
homeostasis [11:59) '
hyperk.alaemia (u:59]
hypertension (11:74. 76)
, ... ,
hypok.alaemic alk.alos~ (n:59] .
impairmentl:87(n:58, 70,76, 77. 78, 141]
insufficiency'(i1:57]
,
;
ion exchange [n:71] ' ;,

lesions (11:141)
' "
p
't . . '"~L11"'; r --~
mass [u:62]
...
metabolism (neonatal} 1:40,45. 51 '
pelvis dilatation [n:62] ,.
'' ,.. <'

perfu~ion, [II: l?J . ' r : ..


scamng (11:17. 76, 7?1 '" ~r

~~--~ . ,
threshold,;[u:33, 34)
.,.
. .
tubular acidosis t:lll [11:25. 28. 33, 35; 39...
50]'" l
'
,
. '

distai [n:SB,:70, 71,7.31 ~:


proximal [11:39. '58, 70, 73]
.
tubular damage (drug-induced) '[n: 127]
tubulardysfunction [u:66. 78-79]
tubular necrosis [11:69, 121
'
renin (n:64, 66, 72, 77]
'
renin-angiotensin system 1:86. 90
renography [u: 17, 60, 62, 73. 76]
.
repeat expansion (trinucleotide) 1:136
repetitive nerve stimulation (EMGJ [11:84. 85,
93)
.
'
reproductive function [u:37]
"
residual volume (lung} [11:130]
respiration (skin} 1:61
respiratory
acidosis [n:28]
alkalosis (11:24. 33)
chain complexes [11:22,31]
chain disorders [n:28, 31)
depression 1:49
disorders 1:6, 173 [n:l3)
distress syndrome [11:109. 126i
failure [11:41, 46. 52, 118)
,
medicine (n:tt3-121, 122-131}
rate [n:IO)
'

194

syncytial \irus (RSV) [u:l, 11, 127] tract clearance (u:l21)


reticular formation [11:95]
reticulocytes [11:42. 44; 52]
retinal
,(
degeneration (n: I 07]
detachment [n:54]
dystrophy [II: 107]
hamatoma (11:81. 106]
hyperoxia [n:109]
. . u~ '
oedema (u:74J
opaCity (11:109)
,.
pigment disor_d!'!r [II: 106] ~
pigment epithelium [II: 108]
retinitis pigmentosa-[u:99. 106]
retinoblastoma [u:IOO, 101, 107]
retinopathy (11:34, 108, I 09]
of prematurity 1:53-54. 99, 101, 106.
T ' . [n'43. 109)'' .,.

1
retrobulbar neuritis (drug-induced) [t:: 112] .
Rett syndrome 1:136 [11:82, 91) '- 1:1 .. ,.
reverse transcriptase (RT) PCR t:l76 ,.,..
revised Jones criteria for rheumatic fever
(II: 142]
-' ' : ' ,... ! . ;
rhesus factor (n:42r 44] .c.': n:..i
,
rhiisus haemolytic i:lisease[it:52J
. \
r;
rheumatic fever 1:63 [no 135; 141: 142]
rheumatoid'factor [11:133, 134, 139r
rheumatology (ml33-138,,'139-14~)
.
rhinitis [11:114. 120. 123, 125,127]
rhinorrhoea [11:123]
' 1
rhinovirus [II: 1]
"I .
ribavirin therapy 1:179
rickets.l:80. 84 [11:66, 74); "
rifabutin therapy 1: 179'
rifampicin

..,
side effects [11:5, 17;125)i~
. '"'
therapyt:l79 [n:I2,S, 130[- ' :-
right atrial appendage i:14,
"
right atrial isomerism 1:3
right bundle branch block' ('RBBB) 1:4. 9
righ-to-lefi shuntt': 12
right ventricular hypertrophy 1:9, 12, 16. 18
rigid bronchoscopy [11:12()-;121, 131]
ristocetin co-factor 1:148
Ritalin r:31
rituximab 1:1 74
RNA amplification (PCR) 1:168
Robertsonian chromosome translocation
1:129, 139, 140, 143
rockerbottom feet [II: 18, 54]
Romano-Ward syndrome 1:4, 15
roseola infant urn (enxanthern subitum) t:63,
180[11:4, 16)
rot<~virus infection 1:116, 125 [n:IJ
Rothmund-Thomson syndrome 1:54. 62. 68
'

.,
INDEX
:
.... '

- -. t~ ~-

Rubinstein-}lybi,_s~d~ome,)ii:S~J -~:IT'_:
h'

">\ ~

t r

t"'

f,r

:,'i~,~ .~r~,

salicylate 1:42, so [11:73) .,. , ... , "


salicyli~ acid l:t~ 68 [11:12.3(, .~ _ _
Salmonella infection 1: 118; 179 [II: I. 2. 122)
salmon patches'~:64 -, ' .., '
,f'l.

.....

~ ~ ~

~ ~;,..

salt-losing crisis 1:89


,
" ~. ~ _ :;
salt-poor tluid [11:67, 68, 80)
.
,.
sample size in clinical studies [11: 14 7; .149,
ISO, ISS, 156:157, 161, 162). ~ ,. 'i-~Sampler's triad [11:123)
'
. ,_ ,
Sand hoff disease [u:34J ~ _ l'
..... e. ,
sarcoidosis 1:168, 'I is (11:114,'f22). :.~--:
Sarcoptes scabiei human is infec-tio'n (scabies)
1:53. 56, sa. 62:66.68, T7o: /~.8,:. J79
scalp defects in trisomy 1:13 [11:54) :n; l
scarlet feyer [II: 14)
_
.
, .. ' , _
scatte'~plois (data anal)'sts> [11:'16~) _;' ..
Schwachman-Diamond syndrome 1:112.122
sclera disorcters [11:100: 1071 -

scleroderma [11:135, J42J
_. _:;~ ~ ._
sclerosingcfi'olangitis cintlainm'atory bowel
disease) 1:.119, 120, 121;'133:.173, 18!
[11:123. 134, 140)
.
. ,,
--~
scoliosis 1:77, 85 [11:94} see also spinill, .
disorders.
"""~
scotoma [11:112) _
.
screening tests 1:.132, 133.-1;43, [11:t48)sebaceous neVi 1:64
-
sedation [11:931
Segawa disease (dopa-responsive dystonia)
[11:88, 97)
seizures/convUlsions see also ~pilepsy_
afebrile [i1:24, 93)
'
Borrelia infection [11:94)
clonic movements 1:27 fetal [11:46]
focal [11:97)
infantile 1:28
inetaboiic disease [11:29)
myoclonic 1:35 [11:96)
_
neonatal , 1:43 5 I, [11:.3 7, 46)
neurodegenerative -disorders [11:88)
partial complex [11:96)
- ret! ex anonic 1:27, 35
in renal disease [11:68. 74)
in rheumatologic disease (II: 1411
Sturge;-Weber syndrome [11:83)
tonic-donie 1:35 [11:87, 96)
tuberose sclerosis [11:90)
self-harm 1:31

1i

~~ l

Rotor syndrome [11:48)


rt ~ .t~~ j lj't"
riJbella see also measles
,_ , _, , -r.
congenital [1!:54,'100, 106,.:1~~1
1
maternal [11:49, 54,I0_8) ,,. ''~"' --.
vaccination 1:23, 24, 30, 32. 175 [II: 108)
ll l

_~.

seminiferous tupule dysge_nesis 1:82. ,.


sensorineural hearing loss (11:108) .
sepsis 1:20, 5 I (u: 18, 45, 47,-.5 I, 55)
'.
septicaemia 1:161 r --:,1-..
.~, d ( h -septo-optic dysplasia (De Morsier syndrome)
1:72, 76, 80, 81; 88 .;_jJ , ' --- ;.1h..
septum
~...
~
defectsl:l43 ,1
'lil- ""''''-"'
pellucidum (absent) 1:8!,
!'
tiansversum [11:47)
- .!>
serositis [II: 1:411 ,
" . - "
Sertoli cells 1:90
_
, , '- </
severe combined immunodeficiency., '
syndrome !SCID) [11:129)
"'"
severity scales in clinical ;;tudies [II: I 52-153,
1?41 I C<' (.- ,(. ,-.' -,
-- . ); -
sex
1
..
chromosomes 1:89, i3S [u:47).
hormo.nes 1:8 7_ [u: 127)
steroids 1:80. 87,90
sexual abuse 1:63 ;>: '
Sheridan Gardner test [11:111),
shield~shaped chestl:87 _ _ .,.
Shigella infection 1:110, 118
shock'in typhoid fever [II: II)
'. ;,,.
short stature see also achondroplasia 1:76,
78-79, 87. 133, 144
' '
shoulder dystocia [11:50)
;
SIADH (SJndmme of inappropriate secretion
of ADH), 1:74, 86 [11:67, 93).
sickle cell disease 1:179 [11:39, 49, 82, 92)
sideroblastic anaemia 1:161
silver nitrate cauterisation [11:52)
Silver-Russell syndrome 1:128, 138 ,
sinopulmonary infections 1:164
sinus
r
.'
drainage [II: I I OJ
granulomata [11:135, 141).
pain [II: 123)
-~
,. 1 ,
thrombosis [1':103,-110).
X-ray [II: I 00, I 08)
sinusitis [11:13, 108, rro-rr.r)
sitting ability [11:25)

~situs invert us (Kartagener S)~drome) (II: 127,


130)
.
Sjrogen-Larsson syndrome ~:54, 62
-.
skeletal survey I: 133, 144
skin
dryness 1:33
.
lesions 1:63, I 73 [u:2)
m;;tlignancy 1:68
. ,.,
pigmentation disorder E:B7 (11:!17.1 08)
prick test I: I H. 124 [11:125)
structure and function,E:53, 61
skipping 1:30
skull table erosion 1:94

i:: ,.

... ) (

195

QUESTIONS FOR MRCPCH


'sla-pped-cheek ~dr;;me see parv~virus B I 9
infection
sleep apnoea~ 1:128 (11ill7, 128)
slit-lamp examination ,(11:35, 140) "
slow acetylators 1:41, 49'
'' '
siO\V \\'ave EEG activity [11:91]. ..
.~
,,, small,,
,' i' ,, ,,,.r

...

,,

"

I,

airway function [II: I 30)


bowel biopsy 1: II 7, 122
bowel cancer 1:109, I I 7
bowel malrotation 1:3 ,

for gestational age 1:72, 82 [11:47)


intestine brush border 1: I I o; I 19 ' ~'
lung fields (11:42)
'
smallpox [11: 141
Smith-Lemli-Opitz syndrome 1' I 29. 'I 40
I
(11:23, 28, 3Jj
. ' , J'
smoking (maternal)1:23. 30, 83 [11:50):
Snellen chart [11: II I J
snoring (11:117) "' ;
-
sodium: 119 see alsohypernatraemia '
absorption 1: I 19 "
'
benzoate therapy (11:291.
bicarbonate [ia:73)'
blood level (u: 10, 62.'63, 64, 65, 66, 67, 68.
80)
'
I
breast milk levels [11:4o]
chloride therapy (n:67, 68]' ,.
lung levei"[11:128J
'
phenylbutyrate therapy(n:fi. 29j h
potassium transporter [n:72; 78]'
sweat level [11: 126] .
urinary [11:58, 64, 66, 67; 68, 71, 72, 73. 7!.
78)

'
valproate [11:87, 93. 96)
somatostatin 1:80
sore mouth [u: 1 I)
sore throat 1:21 [11:7, I I, 65)
sotalol 1:20, 21
So to syndrome 1:73, 89
soya-based formula feed 1:126 [u:27)
spastic paralysis (11:97)
speech
development 1:24, 26, 28-29, 31, 36
disorders 1:26, 28, 31, 34,36 (u:93)
<
sperm
,
development t:8Q-81, 90
disorders 1:84,85 (u:l 15, 127)
sphenoid sinus (II: Ill]
spherocytes 1:156 (n:42, 52)
spherocytosisl:147, 155
sphingolipidosis [11:29)
spike wave discharges (EEG) (u:87. 9&-971
spinal.cord
compression 1: I 28
tumours 1:158-159

196

spinal
.
disorders 1:95, 102, 128, 139 [11:54]
injury [u:53)

,. , .
muscula~ atrophy (SMA) 1:35, 133, I 43.
[11:48.'53, 94.:.95]
.'
schwannoma 1:133,:143

stenosfs 1: 139 ' .


tumours!:l33:!43.'t58-J59
spirometry(;;: 117, 123,}29, l30L.. , ,1,"
spleen rupture '.:174. . ,
. . ,,
splenomegalv 1:9. 11; 20. .J64[1i:l. 85) ,.
splinter haemorrhages [n:I9J : :.:
spongiform encephalopathy!: 177 .. ,, ,
spoon feeding 1:32
. . _ . ..
sputum [n:l. li4]
. ', : 1 '''

sQuii'u '~:25.:'~3. 53-54t 89

v:

I
I

~'

SRYgene'[u:-47)
'
stair climbing 1:31
standard de\iation [n:l47, 150, 1'57, !63)
standard error [11:147, 157]
'standing on one foot 1:34
staphyiO<;oc~,al J\'fec\ions a:'! 78 fu; ~ 2, 1_1 0;
130). ..
.
"
,,,,
Staphylococcus au reus t5( 56, 63, 64, 66,

, 171.'! 79[11:! 6, 55}


~ ,<1
scaphylococcus epidermidis 1:66. [u:4 1, 45, 52)
statins (11:3 i I . , . ' ' '1> , 1
statistics [u:l45-153, 154-1681
status epilepius in sleep (u:96:;-91]' .
steatorrhoea 1:109. 118]u:ll7)
stem cell transplantation~: 168. 176
steroid'
'

resistance [II: 781


side effects 1:66 (11:60. 69. 73, 109, i31}
sulphataie deficiency tSS, 6s
therapy see also corticosteroid therapy;
speciiic drugs
airway swelling 1:174
endocrine dysfunction 1:83
Henoch-Schonlein purpura [11:76, 14 I 1
meningitis (u: !5)
orogenital ulceration 1:120
skin diseases 1:64, 65, 66, 67
vernal conjunctivitis [n:109)
use (maternal) [u:IOO, 107)
Still'smurm.Jrl:l7,19
Stokes-Adams attacks 1:15
stomatitis [:::93] ,
stool-reducing substances 1:116, 125
stools (faeces) (11:20, 95) II o. 1 18, 12 L 122.
124. 125
stork bites 1:64
strawberry hemangioma 1:64
strawberry naevi 1:54-55, 64
streptococcal infedions.l:51, 63, 67. 168,
175. liB (11:12. 16. 110, 122)

.r:. '

.I

'INDEX

acute post-streptococcal
glomerulonephritis [11:57, 69]
group A streptococci [II: 18, 135. 1421
group 8 streptococci 1:8, ! 5, 51 [n: 1~1 "l
Streptococcus pneumania 1: 179 [u: 181
streptomycin side effecJS [11:5, 17]
stridor [11:2, 117]
'
stroke [11:82, 84, 92, 94]
structural heart diseasel:4
Sturge-Weber syndrome 1:64 [11:83, 93, 106)
stye (hordeolum) [II: I 04, I JIJ
'
subacute sclerosing pan-eneephaliiis [u:90J
subarachnoid haemorrhage [II: 72. 81]
subclaVian artery i: 13; 18
'subcutaneous nodules [II: 135, 142]
.subsiantia nigra lesions (u:97]
succinylacetone [Ji:27, 35]
sucralfate toxicity 1A8
sucrase f: 119
SUCrase isomaltase deficiency I; 116, 125 .
sucrose 1:119,'125
.

. '.
sudden death 1:3, 14, 15
sudden infa~Jt death syndrome {SIDSI1:23: 30
suicidn:24,'31
. ..
,.
sulfasalazinethetapy[n:l34, i40J ,
sulfonamides 1:48, 49, 63, 175[11: 112]
superior obli'que muscle tendon complex
[11:111.)
.
supra~um~i.li~cJI hei'rii~ r:94, 100
~ ''~
. supraventricular tachycardia 1: i 0,'11' ''.
surfactant (lung) [11:37 . 47. I 14, 124. 126)
surVival motor neuron (SMN) gene [u:94, 95]
swallowing disorder [n:91]
sweating:l2i,l64,165[n:86, 115; 126.128]
sweat test 1: 122 [11: 115, 126, 128]

swelling see. oedema .


swinging pyrexia [u:ll]
symmetrical intrauterine growth restriction
[li:39.49]
.
- synehronised cardioversion 1: I o. 20. 21
synovitis [11:144]
syPhilis [u:48, 141]
systemic lupus erythematosus !SLEJ 1:61, 156
[11:74, 13( 140, 144]seea/soneonatal
lupus erythematosus
antibodies 1:1 i [il:l35, 138.' 141, 142, 143]
.,
'diagnosis [u: 140]
drug toxicity 1:41, 49
maternal!: 15 [u:50]
neonatall:57, 59
therapy 1:53
tachycardia !:10, 21,51 [11:125]
tachypnoea [u:8, 24, 27, 33, 67, 96. I 07)
. tacrolimus therapy ~:55, 65
talipes 1: I 3 7

tall stature 1:73, 77,' 84. 85, 89 [n:38)


TAPVC in right atrial isomerism 1:3
Taq polymerase (PCR) 1il 76
Tay-Sachs disease 1:24,31; 131. l42;143
[n:25. 341

t
,
TcellsJ:3, 14.155,177,180[n:l31]
tear discolouration [n: 125]
. ~.
tear duct. obstructed [u: I 04, I I I)
teeth disorders 1:14, 32; 57, 68. 84

telangiectasia 1:57, 68, 69
temperature of bedroom (eczema) 1:5 7, 67
temporal lobe enlargement 1:172. 180 tendonitis [u:f43J

'
Tensilon test [u,85, 95)
'
teratpgenicity [11:30; :33, 96)
teratoma !:'?5. 103 . .
terminal h:peraemia ~:64

lerminal ilf.um abSOrPtion l:lHt, i 19 ".... ~


,'
testes. uncescended 1:89
testicular
atrophy 1: I 07
. ,
developmentl:77, 81~ 82,85 [n:47i
ischaemia 1:93: 98, 107
;,
testosterone 1:78, 80, 81, 85, 86, 89, 9b'
therapy ::82

test sensitivity in clinical studies [n: 160)


1
tetanus [u: 141

tetralogy of Fallot 1:2, 4, 6. 14, 16, 18, 19


thalassaemia 1:147,152, 155, 161. 162
thalidomide 1:120
'
theophylline metabolism t48
thoracocolumbar gibbus 1:139
thoracotomy 1:4. 1 [n:7J
thrombin time ITT) 1:139, I 49, 158
thrombocytopaenia 1:25.33. 112. 122. 156,'
164, 175 [u:20, 141] see also neonatal
alloimmune thrombocytopaenia
drugind:..ced 1:169, 178 [11:93]
normal neonate 1: 162-163
throinbob:tosisl:ll4[n:l33,139,141]
thrombolYtiC therapy [u:92)
thrombophlebitis till, 120
thrombosis 1:14, 176 [11:29, 69)
thrush. oral [II: I OJ see also Candida
infection

thumb abnormalities (11:54]


thymus
absent 1:14
function ::3
hypoplasia [11:54]
thvroid
gland di-;ease r:20, 21, 35, 82, 83, 88, 117
hormon,;; !:90 [11:1241
stimula:ing hormone (TSH) 1:71, 78, 79, 81,
82. 55. 88, 89; 90 [11:44)
.
thyrotoxicosis 1:89

197

FOR MRCPCH

thyrotrophin-releasing hormone {TRH) test


trimethoprim-sulphamethoxazole therapy
, . . .
.,.
.
!:78, 90
[11:131}
. ' . '
.
thyroxlne 1:83, 86, 89 .. ,.. : , r
triple therapy 1: i 78
trisomyi:l3, 18, 21, 139, 143(n;39, 44, 46,
tibial periosteal eleVation In:9J."~ ~"'
tick (Ixodes) !:63, 175(11:12, 19. 941.,
49.54,107, 124}seealsoDown
tinea [11: 12]
. ". ~ ,
'
syndr~me.
.
toe abnormalities (n:54]
trochlear (IV) nerve [11:104, 110, III]
tongue
,.',.,
tropical infectious diseases (11: 10; 201
choreoathetosis {u~881
Hest (statistics) [11:149, 152. 160.,1641
enlargement (II: 11 ( 124]
tuberculin
skin test 1: I 79
~eduction surgery'(u: 124]
white,t:57, 67
testing (intradermal) (n: 130]
tonic-donie seizures (u:87, 96]
tuberculosis {TB) i:5, 18, 20, 118, 122, 123,
(u: 125] see also pulmonary ttlberculosis
tonsillitis [II: 1361
'""
neonatai~:J71, 179.
tonsil obstruction [u:128j
topiramate (11:83, 93].
,
notifiability'tli:l4, 1\5]
,
.. "
total lung capacity (11: 1301
. .
therapy 1:5. 17, 112, 115, 125, 171, 179
totally anomolous pUlmonary venous
(11:130]
'
connections 1:2, 3, 13

vaccination !BCG) 1:168, 175. 176, 179


tuberculous meningitis (n: 15, 90; 125]
drainage [II: 116} '"
., ,
.
total parenteral nutrition (TPN) [11:39, 50]
tuberose sclerosis 1:35 130 [11:59, 72. 81.' 90,
96]
'
'
'
'
toxic erythema i:63 ' ,
~ '
toxicology and pharmacology 1:39-46,
,,,tl!mour
,
.
. ...
lysis syndrome {TlS) r: 150, 159-160
47-52
i ,
I
,
.

necrosis factor (TNF) 1:84, 169, 177 (n:4,


toxocariasis (70xocara canis infection) (II: 101.
16] t.. .

.. '.'.:
.. '
,!Q2, I 06~,} JOJ':,.. "!i " (i'-'

toxoplasmosis 110xoplasma infection) [11:4,


1\Jrner syn!lr()me 1_: IT. 75. 76. 8~ . 86;-~8 . 139,
140;,145:(u:I07]

.15, 18, 48~ 4,91


,. .
, ' ,
twin pregnancy (u;37, 40, 51}.'' :

tracheal r
.::.
1
atresia [u:46}
twin-to-twin transfusion synd1bme (n:48. 50,
51)
.
...
compression [II: II 7]
tympanometl): 1:2s. 34
plugging [u:46J , . ,,
typhoid (fever) ~:30, 175 (n:l, 10, 11. 14, 20]
tracheo-oesophageal fistula 1:96. I 04 (11:54,
129)
.
tyrosinaemia [11:23, 27~ 31, 35, 38, 48; 70, 78]
tyrosine (n:29J
tram!ine calcification [11:93]
kinase 1:80
tranexamic acid r: 17B
ulcerative colitis 1: II I, 115, 120; 125; 1.68,
traniaminltis [n: 135}
~ .
174,175 (11:122]
trans-annular patch surgery 1:4 . '
ultraViolet light reaction. in neonatal lupus
transglutaminase (tissue) 1: 115, I 17, 125
!:68"' .....
.. .....
.
transient hyperammonaemia'o(the ne\\born.
umbilical
cord
(11:30}
blood (stem cell transplant) 1:168, 176
transient neonatal
prolapse (11:491
diabetes 1:1 28, 138
separation 1:173, 181
myasthenia [n:93]
umbilican
"
pustulosis 1:60, 70
hernia 1:79. 91, 94, 100,
transposition ofthe great arteries 1:2, 9. I I,
stump bleeding 1:153, 158,_163 (n:43, 48.
12, 15, 19,20
53]
transtentorial herniation (n:86, 95. 96]
umbilicus dis.::harge [11:41. 52]
tremor [11:93}
unconjugated bilirubinaemia [n:46J
trichothiodystrophy 1:54. 62
unconjugated hyperbilirubiriaemia [!1:31]
tricuspid
unidentified bright objects (UBOs) (n:89)
atresia 1:2
upper airway constriction [n:.I31J
valve anemolj 1:7, II
urate leve_ls (!!:24. 32, 34)
tric'yclic an~idepressant toxicity t:50
urea cycle defects [u:29, 30, 32]
trigeminal (V cranial) nerve [II: I 09}
~rea blood le\el(n:68}
triglycerides 1:110, 119. 125 [11:24, 32]

198

INDEX

'endocrine dysfunction 1:89


..
hyponatraemia 1:67'-68]
infectious diseases (11:8; 9; io, 17)
metabolic disease [u:30(' .-Y '
renal disease differential diagnosis [11:62,
. 63, 64. 71,''76) >:t. ' '
'
renal syndromes [n:6SJ ''.
.
''
tubular dysfunction tu:66)
. W
.,,
urea-splitting organisms [ii:30j \' '.''
urethral
."; :' ' i:. .
abnormalityl:77 [11:50; 74r'" 1 .-~
. ; 1
meatus 1:94, 100
' ' '
uriate oxidase 1: 160 .,
~- .
uric acid .production i: I so; 16() [rl:'33i
uridine diphospfiate (UDP) gi~coronyl .
transferase 1:118
' '
'
'' '
urinal-v
. dysfunction 1:94. I 02 i ' '
now obstruction (11:76]
incontinencet95 (n:94l: -r -
'
tract infection [11:6:22;'30, 73:76. 17]
urine pH [li:sS, 66, 70, 78) see also aciduria
urine discolouration [li: 125]
'
. .
urticarial rash 1:58;'58;68. 1(6. "124: li6
[II: 1 1'7]
~'
urticaria pigmentosa 1:62. 70, 126 Y
. Usher syndrome [11: IOI')J

_
uterine growth r:49-"SO, 73. 83 (11:39. 54]
restriction [n:39. 49"-SOJ .:.
~
uterus development [n:37, 47]

uveitis [n:I06. 140] '' '



VACTERL association iu:54J
vagal manouevres -1:20, 21
vaginal delivery [11:49]
valine [11:34]
Yalsalva manoeuvre 1:21
vancomycin 1:41. 49 [11:45, SSJ
variables (clinical studies) (11: 146. I 54. 156,

...

'T64j'

varicella (chickenpox) 1:6, 12, 68, I 70 [11:2, 16,


41. 52. 53]
varicella zoster immune globulin I\'ZIG) 1:6,
IS. 42. I 70, 178 (11: 52]
vascular resistance [II: 126]
vasculitis 1:14, 168. 175
vas deferens
aosent 1:137
obstruction (II: 127]
vasoconstriction (peripheral) [II: 118]
vasodilation (peripheral) [11:125)
vasopressin-neurophysin 1:86
vasopressin secretion ~:71, 81. 86
VATER complex 1: I 03
,
velocardiofacial syndrome [11:54)
velopharyngeal insufficiency 1:129
venepuncture

site bleeding 1:153, f63 "'


...
\vith tourniquet [n:21J
'.
venous sinus thrombosis [11:92)
' ' 1
venous stasis [n:69]
' '
ventricular
'" - .,, ,.,,.,,
fibrillation l: 10, 20 [n:73} ' ~- '~I .,. c
septal defect (VSD) 1:2, 6, 9. tl',' i 2. 14; 16;
17. 19
,' ' ''. ' . ..r- . ' ...
tachycardia 1:20,21 [11:73}
,,.. - ,,t;
vernal conjuricii\rius {u: 1ot 109] '"''' .
' ' "'' i ''-' \1J,,,.,
vernJca [ll:2. 12).
vertebral abnormalitieS see spin~! di$6rders
vertigo [11:93] .. '
' l 'i''"
very-long chain acyl-co:A dehydrogenase
deficiency(VLCAD) (n:21,28J ' ~~''' 4
very-long chain fatty acids [n:32. 34)
very lo\: btrt h wetght [1i:40. 58) '' -~
\'esJcopustular les1ons i:60, 69-70'
'
vesicouret<"ric reflux (V!JR) (u!J7, 60,73! 76)
'vibrio cho!er.:Je: 1:118 < ' 1 '
Vl cranial (abducens) nerve [n:94. 104. Ill)'
vigabal!in (u:87, 96. lOS, I 12] '""'
vincristine t 1SO, 160 : t v\- I
violaceous heliotrope [11:143]
..,_~
. vital infections 1:54; 172 [it:2J
"''
viral myositis [u:l43]
'
virilisation t:87 [11:471 '
\
vision defects (u:S9, 102. 105, JOJ:.,JoS:I 10.
;~
'.' ' : l': '
112]' ' , . '
visual
-' .,,. '
acuity (11:94. l03.,1t!J''
. ,:...
function 1:25fu:84, 106)
"
monitoring (II: 1 12)
vital capacity (limg) [11:118, 129,"130)' ''
vitamin
< "
A 1:65. 119 [11:23, 32]
absorption 1:119
8 12 (cobalamin) 110 (11:29]

...

,--~s~:-

1so

,. _. . .

'.'.x
,.
.

-'- ..

-~-~------"~-~-- --.--~ ---~-,

. D, 1:61. 66--67, 73, 80, 84 [11:50. 73, 150,


161. 174]
E (11:106)
K (11:38. 48. 53]
supplementation studies [u: 145. ISO, 161]
vitreous opacity [u: 109)
volume of distribution(drug) 1:39.43,45, 47.
48,50
vomiting [II: 1 I I]
differential diagnosis 1:97
drug side effect [n:93J
hyponatraemi~ [11:67]
IHPS 1:99
induced t:42. so. 51
liver failure [11:27)
. neonatal infection 1:8)
neonatal jaundice [11:44]

199

:f.r'l
. )(:t:~.(

'~~

c.:

ESS~NTIAL

,QUESTIONS FQ=>, i"'1RCPCH

tropical infectious diseases [u: IOj


lyphoid fever [u: II)
.'
, ,
viral infections 1:172, 180 ~
! 11
vomitus
~~ ' ,1 ' , :
,
,
bile-stained (green) J93,,94. 97. -10_1.,121
coffee-ground 1:93, 99
sodium [n:SOJ . . .
von \'llillebrand" _, ,
factor 1:148, 1'57. Iss
disease 1:148, 157-, .163 .
. ,: 1'".
Waardenburg synd.roine [n:iOSJ
WAGR'syndromeJ:I64'
.
..
walki_ng ability 1:26, 32 [u:85, 88 94-951
wardship (Children Act) 1:36

warfarin treatment 1:41, 49


' ,1 , .
warts
water ..
intake [11:75. 80).
,.
loss 1:119 [11:71, 73, 78)
weaning [1i:3Sj
" , ', .
wegener's granulomatosis {II: 123.. 135. 14 I]
weight disorders.
,
anorexia 1:35-36
childhood immunodeficiency 1:173 .. , 1
childhood malignancy 1: 154
drug-induced [u:93. 1251
Hodgkin's disease 1:164,
hypothyroidigln 1:88.
'
'.
liver failure 1:27
tropical infec.tious diseases [II: 10)
Werdnig-Hoffman disease [n:9(J
west syndrome [11:961 .

wheeziness[n:7,11,117,118,120. 121,126.
.
129, 130 [11:131].
white blood cells (leukocytes)
acute lymphqblastic leukemia 1: 150. 159

1;63 . '

'

childhood immunodeficiency 1:173


iminunefunction~:148,153, 157,162-163,
174
,
infectious diseases [11:8, 9. 10, 18) .
neonatal disease [11:42)
, . ~, .
renal disease [11:62, 63,
78(

rheumatologic disease [u: 136, 138, 142]


white matter lesions [n:9l, 97)
white pupil see leukocoria
whitlow 1:22 [II: 19]
whooping cough [n;l4i
.. , . ,
William's svndrome 1:3, 14 129. 140, 141, 154
[11:60, l3)
. ..
..

6i.

Wilms' tumour 1:154, 164


Wilson's disease 1:119 [n:35, 70, 78) ..
Wiskott-Aldrichsyndrome (WAS) 1::)4:153,
.
162, 168, 175
'
.
. .
Wood's light examination 1:28, 35[u:87)
wound healing 1:173

, ,
writing skills 1:30 ,
,
,.
v..'litten consent (parental) 1: I 06
xanthochromia [1_1:81, 90),
, :
xer()derma pigmentosum 1~6.8, 69
.. :
X-linked genetk disorders 1:65, 77, i!7, t:n
136,141. 144:162,163, 168,181 [n:2L

28. 77. 78. 91)


Y chromosome.[u:47)
yellow fe\er 1:30, 175 [n: 14)
,
Yersinia emerocolitica.infection [11: 122)
Zellweger syndrome [11:23,26, 31, 32:34)
zidovudine il\lT) 1:177 [u:22, 31) .
zinc deficiency 1:62, 65
zona glomerulosat:74, 78, 86, 89
zygosity [11:51)

r;

I
'

I
I

'l

200

Potrebbero piacerti anche